==================== THE RISING SEA Foundations of Algebraic Geometry 満ちてくる海 -- 代数幾何の源泉 -- Ravi Vakil (日本語訳の試み:searial) http://searial.web.fc2.com/sea/index.html 更新:2019/06/06 ==================== 内容: 序文 0.1 読者へ 0.2 熟練者へ 0.3 前提知識など 0.4(**) この本の目標 Part I 予備知識 1章 いくらかの圏論 1.1 動機 1.2 圏と関手 1.3 普遍性は対象を同型を除いて決定する 1.4 極限と余極限 1.5 随伴 1.6 アーベル圏の導入 1.7(*) スペクトル系列 2章 層 2.1 典型的な例 2.2 前層と層の定義 2.3 前層と層の準同型 2.4 茎で決まる性質、層化 2.5 開集合の基による層の復元 2.6 O_X加群 2.7 逆像 Part II スキーム 3章 アフィンスキームへ:底集合とその位相 3.1 スキームへ 3.2 アフィンスキームの底空間 3.3 スキームを見る(1):生成点 3.4 アフィンスキームの位相 3.5 ザリスキ位相の開集合の基:基本開集合 3.6 位相的(およびネーター的)な性質 3.7 SpecAの部分集合をAのイデアルに写す関数I(・) 4章 構造層と、一般のスキームの定義 4.1 アフィンスキームの構造層 4.2 スキームを見る(2):冪零元 4.3 スキームの定義 4.4 3つの例 4.5 射影スキーム:Proj構成 5章 スキームのいくつかの性質 5.1 位相的な性質 5.2 被約と整 5.3 アフィン局所的に確認できる性質 5.4 正規性と一意分解性 5.5 関数の台:スキームの随伴点 Part III 射 6章 スキームの射 6.1 導入 6.2 環つき空間の射 6.3 局所環つき空間とスキームの射 6.4 次数付き環と射影スキームの射 6.5 被約スキームからの有理写像 6.6(*) 表現可能関手と群スキーム 6.7(**) グラスマン多様体(最初の構成) 7章 都合の良い射のクラス 7.1 都合の良い射のクラスの例:開埋め込み 7.2 ちょっと代数を:Lying overと中山 7.3 無数にある有限性の性質 7.4 射の像:シュヴァレーの定理と消去理論 8章 閉埋め込みと関連する用語 8.1 閉埋め込みと閉部分スキーム 8.2 もう少し射影幾何を 8.3 〜を満たす最小の閉部分スキーム 8.4 効果的カルチエ因子、正則列と正則埋め込み 9章 ファイバー積と底変換 9.1 それは存在する 9.2 実際にファイバー積を計算する 9.3 解釈:引き戻した族、射のファイバー 9.4 底変換で保たれる性質 9.5(*) 底変換で保たれない性質と、その修正の仕方 9.6 射影スキームの積:セグレ埋め込み 9.7 正規化 10章 分離射、固有射、そして(ついに!)多様体 10.1 分離射(と準分離射) 10.2 分離スキームへの有理写像 10.3 固有射 Part IV 「幾何的」性質:次元と滑らかさ 11章 次元 11.1 次元と余次元 11.2 次元、超越次数、ネーターの正規化 11.3 余次元1の奇跡:クルルの補題とハルトークスの補題 11.4 11.5 12章 正則と滑らかさ 12.1 ザリスキ接空間 12.2 体上の正則性と滑らかさ 12.3 例 12.4 ベルチニの定理 12.5 もう1つの(余)次元1の奇跡:離散付値環 12.6 滑らか(でエタール)な射(最初の定義) 12.7(*) 分離性と固有性の付値による基準 12.8(*) 正則局所環に関するより高度な事実 12.9(*) フィルター付き環と加群、Artin-Reesの補題 Part V 準連接層 13章 準連接層と連接層 13.1 ベクトル束と局所自由層 13.2 準連接層 13.3 基本開アフィンを使った準連接層の特徴づけ 13.4 準連接層はアーベル圏をなす 13.5 加群様の構成 13.6 有限型層と連接層 13.7 有限型層と連接層の喜ばしい性質たち 13.8(**) 非ネーター環上の連接加群 14章 直線束:可逆層と因子 14.1 射影空間上のいくつかの直線束 14.2 直線束とヴェイユ因子 14.3(*) 効果的カルチエ因子"="可逆イデアル層 15章 準連接層と射影A-スキーム 15.1 次数付き環に対応する準連接層 15.2 射影A-スキーム上の可逆層(直線束) 15.3 大域的に生成された直線束、底点を持たない直線束 15.4 準連接層と次数付き加群 16章 準連接層の押し出しと引き戻し 16.1 導入 16.2 準連接層の押し出し 16.3 準連接層の引き戻し 16.4 直線束と射影スキームへの写像 16.5 曲線から射影の拡張定理/**/ 16.6 豊富な直線束、非常に豊富な直線束 16.7(*) モジュライ空間としてのグラスマン多様体 17章 ・・・ 21章 微分 Part VI その他 ・・・ 24章 平坦 24.1 導入 24.2 より簡単な事実 24.3 Torの視点での平坦 24.4 イデアル論的な視点での平坦 24.5 位相的な視点での平坦 24.6 局所的な視点での平坦 24.7 25章 滑らかとエタール射と平坦性 25.1 いくつかの動機 25.2 滑らかとエタール射のいくつかの特徴づけ 25.3 幾何的な滑らかさとクライマン-ベルチニの定理 26章 深さとコーエン・マコーレー ・・・ =========================== -- 私は..(物事を理解する)方法を..ナッツを開くイメージで描写できる。 最初に思い浮かんだ連想は、ナッツをやわらかい液体に沈めることであった。 時間を掛けて液体はナッツを侵食する。行き届かなければ、待つだけである。 殻は時間を掛けてより柔らかくなり、機が熟せば、手の力で十分である。 殻は完全に熟したアボガドのように開くだろう。 数週間前に、別のイメージが思い浮かんだ。 知るべき未知のことは、物を通過させない大地、泥灰土のように思えた。 海は静かに満ちて行き、何も起きないかのように見え、何も動かないが、 水ははるかかなたにあったが...ついには障壁を乗り越えるのである。 -- -- A.Grothendieck ============================ 序文 この本は、この分野の(将来の)専門家のためだけでない、代数幾何への堅苦しい体系的で入門を意図している。目標は狭く定め(0.4)、この分野の1つの理解の仕方を与えた。 40年前のことになる。David Mumfordが代数幾何を「他の数学を乗っ取ろうと企んでいる人による、難解で排他的で非常に抽象的なものだと評されるようである!この最後の点は、1つの側面では的を得ている...」と描写した。この分野の発展は、我々の純粋数学の多くの分野への見方を根本的に変えることとなった。多くの発展は、Alexander Grothendieck, Jean-Pirre Serreらにる洞察や発想に依存している。 多くの理由で、代数幾何は手の届かないものと評されている。抽象的な重い道具のせいもあるし、対象や方法に対する直観的な視点を見失いやすいのもある。近くの分野については、漠然とした考えしか与えない。代数幾何それ自身は多くの部分に分かれており、その中でも、新しい研究者はしばしば自分の部分以外についてよく知らない。 しかし、ここには別の楽観的視点がある。代数幾何が数学の複数の分野をつなぎあわせている方法は、根本的で、十分に動機を持っている。近くの分野の多くの人は、この根源的な所を、表面的だけではなくよく理解することが役に立つ。代数幾何それ自身でも、(少なくともこの特定の方向でアプローチする人にとっては)この分野の人みながよく理解しているべき聖典がある。スキーム理論は、独自の言葉で理解する必要があるのだが、その広がりは、この半世紀の間にはるか遠くまで広がった。 0.0.1 練習の重要性 この本には多くの練習がある。十分に考え通す問題がなければ、考え方は定着しない。いくつかの練習は自明である -- 挑発的と考える人もいるが、私は望ましいと考える。難しい練習は少ないが、読者はそれらを扱う基礎を与えられているべきである -- それらは単に難しい内容を本文から押し出す言い訳ではない。練習は章の終わりではなく本文に混ぜた。多くは実地試験のようなものである。これはKedlayaとPoonenと数学をどうやって学び、考えるかを模索したときの視点である。多くの人は、受動的に読むのではなく、手を動かすことで学ぶ。良い問題は、学習者に良い視点を与える一番の方法である。 0.0.2 構造 すべての内容には章、セクションに続く番号をつけた。練習問題にはアルファベットをつけて、本文に混ぜた。後で参照しやすいため、あるいは新しい話題であることを示すため、個別の段落にしばしば番号をつけた。定義は太字で示される。(このノートには反映していない)何かのついでな文脈で定義を書くことも時々ある。 0.0.3 謝辞 これは本当に難しいので後で書く。The Rising SeaはGrothendieckによる言葉で、Daniel Murfetの素晴らしいブログのタイトルにより知れ渡ったものである。 --- 0.1 読者へ -- これは最後のチャンスだ。これを逃すと戻れない。 青い薬を飲めば、物語は終わり、ベッドで起きる。信じたいものを信じれば良い。 赤い薬を飲めば、不思議の世界に留まる。兎の穴がどんなに深いかを見せてあげよう。 -- Morpheus この本の内容は、不完全なフィルターで蒸留された、公開された知恵の集まりとして意図されている。これの使い方について、二三の言葉を言いたい。読者がこれに従うか、あるいは従うべきかどうかは、私には分からない。 詳細の前に、外枠を明らかにしたい。現代の代数幾何の素晴らしい道具は、広い意味での幾何に関する基本的なことや純粋な疑問についてよく理解するために作られた。この本の目的は、この強力な方法に対する行き渡った説明を意図している。根拠のない抽象に心酔にすることに誘惑されないようにせよ!幾何的な動機に強くつかまり、証明された形式的な構造が基本的な疑問を調べるのにどれほど効果的かを学ぶようにせよ。新しい概念に出会ったら、いつもどうしてそれを考えるのかを気にせよ。すぐに答えを得ることは期待しないでほしいが、機を見つけては答えを求めよ。新しい抽象的な考えの具体的な例を考えてよく理解することを試みよ。 代数幾何の理解は難しいと思われている。大がかりな複雑な道具から作られるためである。実際には、逆である。狭く深いというより、浅く広いのである。Grothendieckの数学の視点に経てば、それはとても小さいパーツがたくさん集まってできている。その構造はとても入り組んでいるので、すべての構造を頭に把握するのは、難しい挑戦である。 話を始めるのに良いのは「アフィン複素多様体」である。これは多項式によって切り取られるC^nの部分空間である。幾何的直観がはたらくだろう--読者は既に次元、滑らかさ、RやQのような部分体上での解のようなことについて考えや疑問を持っているかもしれない。スキームを学ぶ前に複素多様体を理解するのに時間を掛けるのは賢いやり方かもしれない。しかし私は代わりに、アフィン複素多様体はその中心的な例として学びつつ、すぐにスキームを学ぶことを応援する。これは理想論ではなく、時間を節約し、実にうまくいくのである。もちろん、他で多様体を学んでから後で戻ってきても良い。 スキームに対する直観は、アフィン複素多様体に対する直観に基づいている。Allen KnutsonとTerry Taoが指摘したことには、これは数学において3つの一般化を同時にしている。(i)関数の環において冪零元を許す。これは近傍の様子も考慮するという意味で解析的な意味を持つ。(ii)微分幾何学でやるように、アフィンスキームを貼り合わせて一般のスキームを考える。これは座標ではなく多様体そのものを見る幾何的精神である。(iii)Cのような代数閉体上で考える代わりに、代数閉体でない体、あるいは体でない環でも考察する。これは、代数体やその整数環で方程式を解くという数論的な意味を持つ。 我々の目標は包括的な理解である。最初に知るべきことをすべて理解することを目標にしている。従って必然的に興味深い応用例に辿りつくまでは長くかかる。これらの応用例に至るまでにはそんなに険しいのかと思うかもしれない。--そうではないのである。読者は例を追い求めるべきである。ただしそれは時間と忍耐を要するだろう。 代数幾何を学ぶ際には、いくつかの超重要な踏み石に注意を払うべきであろう。もちろん、そのステップは遠くにいくほど大きくなる。 1章:圏論は言語にすぎないが、それは論理が埋め込まれた言語である。圏論は、それが既に知っていることの抽象化であると認識すれば、ずっと理解しやすい。最初の章は、物事をすっきり考える準備である。例えば、誰かが何かを「余核」とか「積」と名付けたなら、どうしてその名前がついているのか、何を真に意味するべきなのかが気になるだろう。このように概念を考えることの利点は先に進むにつれて明らかになる。米田の補題--より一般に、対象をそれへの射を通して理解する考え--は重要な役割を果たす。 2章:層の理論は、またもや、よく知っているものの抽象化である(典型的な例として2.1を見よ)。層(茎、層化、開集合の基での層)の情報をどのように扱うかを理解する所が難しい所である。 1章と2章は、微妙なバランスを目指した賭けである。代数幾何を説明する時には、著者はしばしばこのような背景知識を他の(読者が読まないであろう)出典に任せたり、遠く付録に押しこんだりするものである。この本では代わりに必要なことをすべて、しかしできるだけ最小限の内容を説明し、また、この基本的な考えをどう扱うか、またそれらがどのように自然に生まれてくる概念かを説明することを試みた。 3-5章:この背景知識により、幾何学で扱われる様々な「空間」(微分可能実多様体、位相空間、複素多様体など)について、局所的に特定の形をした環つき空間として、統一的に考えることができる。スキームは、単に別の種類の「幾何的空間」であり、我々は古典的な幾何学からこの新しい舞台に直観を翻訳する手段を身につけている。(・・・) スキームは幾何的直観と違うところがあるが、幾何的直観を拡張してスキームに合わせて行くことができるであろう。欲しくなるであろう概念や、後で重要なことと示される概念がたくさんある。これらはすべてその名前に値する価値がある。これらの概念と仲良くなるのに時間を掛けよ。 6-10章:圏論的に考えれば、スキームの射について考えるのは必然である。グロタンディークの基本的な教訓は、射が中心的であるということである。重要な幾何的性質は、射の性質として捉えられるべきである。射には様々なクラスがあって、名前が付いていて、そのつど読者はどうしてその名前が付いているのか考えるべきである。 11-12章:読者はスキームの基本的な幾何的性質--次元と滑らかさである--を考えるのに良い所に来た。これらが微妙な概念であることに驚くかもしれないが、数学のどこにおいてもこれらは微妙な概念であることを頭に入れておくと良いだろう。 13-21章:ベクトル束は幾何学では常套手段であり、代数幾何も例外ではない。環上の自由加群が一般の環上の加群に一般化されるように、この概念は、より一般な準連接層に一般化される。この性質を、次にはコホモロジーを含めて調べる。19章では、これらの考えを曲線を調べるのに応用する。この概念が実際に役に立つことが明らかになるだろう。 23-30章:これを手にしたら読者はこの分野でもっと進んだことを学ぶ準備が整っている。多くの例が与えられ、27章の、3次曲面上の27本の直線という古典的な理論は、多くの考えが使われるのを見る良い機会である。 ・・・ 我々はしばしば、特にその状況に技術的な制限があるときに、多様体に限った結果を主張する。しかし代数閉体に制限することはほとんど有用でない。幾何学者は算術的な例や代数的な例を恐れる必要はない:代数幾何の中心的な洞察は、全く同じ定式化が適応できることにある。 病的な例は有用である。山の高速道路では悪天候のときのために道路の両側に高いポールが立っている。冬に道路が良く見えない時、これは警告になる:この線を超えたら死ぬ。病的な例や反例は同様の意味を持つ。また、新しい主張、定理、予想などを疑う時に現実味を確認するのにも役立つ。 代数幾何の本を学ぶ際には特に、他の代数幾何の本も持っておくことが、異なるアプローチやものごとが難しくなったときに異なる視点をみるのに役に立つ。この本は2番目の本として良い役割を果たすだろう。もしこれが読者の1番目の本なら、私が同じ精神だと思う本が[Liu]と[GW]である。De Jongによるオンライン百科[Stacks]も素晴らしい。他にも有用な資源は多くあり、それぞれ独自のアプローチを持っているだろう。読者はそれらを見て回って共感できるものを探すと良い。 もし読者がこの分野の歴史を探しているなら場違いである。この本はそのような意図はしていなく、多くの重要な出典が私の無知と怠惰により見逃されている。 --- 0.2 熟練者へ もし読者がこの本を授業で使うなら、読者は自身の視点と興味に合ったようにして使うべきである。特に、授業の最後に到達したい応用や定理(それはこの本に無いかもしれない)と、そこへ至る道筋について考えておくべきである。最後まで全力疾走させる必要はない。私は、そうではなく、学生を目的の所まで導くために、もっと時間をかけるように勧める。 (章の関連性を示したおおまかな図が役に立つだろう) 私は曲線の理論(19章)と3次曲面の27本の直線(27章)の話題が、冬の終わりと春の学期でこの目的に適していると思っている。これは、いくらかの背景知識をブラックボックスとして省略した場合も同様である。 0.4に示した目的のためには乱暴な選択が必要であった。私は読者が反対するかもしれない多くの決断をした。そのうちいくつかを簡潔に書こう。 物事をどう描写するかの決断は、学習者のためになされる。2つの方法があって、片方がより進んだ視点からの"正しい"方法であり、もう片方がより直接的でありのままな視点であったとしたら、私は後者をとるように心掛けた。 一方で、(代数閉体上の)多様体の理論を、最初に別途扱うことはしなかった。この選択は私に涙をもたらした。しかし最後には私はこれは、正しい精神でなされたならば、うまくいく方法だと確信している。 教材の最初で多様体の理論に時間を使う代わりに、私は別のやり方で時間を使った。学生が、圏論や層について十分な慣れと経験を既に得ている、あるいはこれらを自習すべきだと想定することは現実的でないだろう。私はこの世界で生きることを好む。私はこれらの基本的な話題をいい加減に済ませないことをお勧めする。これらの講義は私自身には辛いほどペースが遅くても、多くの学生には目から鱗が落ちるもので、最も経験がある学生でも退屈しなく、時間を無駄にしないことに気がついた。この投資は、学生が物事をすっきり考察し、実際にこれらの道具を使うことを信頼できるときに、元がとれるのである。また、これらの道具について手を動かす経験をすることができれば、それだけでも価値がある。/*訳に自信が無い*/ 多くの部分で、我々は使う証明が正当化できる最大限一般化された形での主張をする。しかしより強いあるいはより一般的な主張をするための、より難しい証明は与えない。多くの読者は気にしないであろう一般化のためにより難しい考察が必要になることがいくつかある。例えば、単に射影射ではなく、固有射について成り立つ多くの定理を証明する。読者はより広い一般性のために要求される微妙な問題は無視することを勧める。 私は因子よりも、直線束(と射影空間への写像)を基本的な概念だと考えた。(効果的なカルチエ因子は本質的な役割を持つが)一般的なカルチエ因子については議論しない。 コホモロジーは(セールがやったように)最初にチェックコホモロジーを使って定義し、導来関手によるコホモロジーは後で紹介した。私は、グロタンディークがチェックコホモロジーと導来関手によるコホモロジーが一致することは「たまたまと考えるべき」であり「コホモロジーの定義としてチェックコホモロジーを使わないことが技術的には重要である」と考えていた[Gr4,p.108]という事実はよく知っている。しかし私は、多くの人が初めてこの種のコホモロジーをみるときには私の方法のほうが適していると確信している。(代数幾何の多くの話題では導来関手の立場で理解するのが良いのは確かである。しかしこれは山の上から見下ろす視点であり、森の中を探索するには最適なものではない。導来関手を正しく理解するには、ホモロジー代数をよく理解する必要があり、単なるブラックボックスとして使うべきではない。) 我々は必要な時、あるいは(稀に)それが労力をかなり減らす時だけ、ネーターな場合に制限した。こうすると、非ネーターな人はどこで気をつける必要があるかが良く分かり、ネーターな人は非ネーターな場合では物事がどう複雑になるのかを知ることができる。また、ネーターという仮定を必要な定理にだけつけることで、どこでこの性質が使われるのかを覚えておくことができる。 この方法で、ネーターな人が余計な苦労する側面もある。例えば、ネーターという仮定を使う代わりに、準分離性という言葉が早期からしばしば現れる。圧倒的な言葉の数の中で、余分な単語を覚えるのは1つの苦労である。しかし一度済んでしまえば、実際に遭遇するスキームが準分離的であることを覚えておくのは難しくない。しかも、"qcqs"(準コンパクトで準分離的)が仮定が登場したなら、読者はすぐに証明の鍵となる考えが分かる。別の例として、連接層と有限型準連接層はネーターな場合では一致するが、それでも定理の主張や練習問題では、それらを区別しておくことには、先と同じ理由で価値がある。何が証明に使われるのかがより明らかである。 多くの重要な話題が省略された。値による基準(valuative criteria)は証明されない(12.7)、その主張は任意な章として扱われた。完全に省略された話題としては:devissage、形式的スキーム、台を持つコホモロジーがある。すまない! --- 0.3 前提知識など -- 「代数学者か幾何学者かどちらかだけであるべきか」と尋ねるのは 「目が見えないか耳が聞こえないかを選びたいか」というようなものである -- M. Atiyah すべての環は特別に言及されない限り可換であり1と書かれる単位元を持つ。環準同型は1を1に送る。0≠1は要求しない:すなわち零環は環である。(あらゆる環から零環への環準同型が存在する。零環からは零環への環準同型しかない。)「整域」の定義には1≠0を含める:すなわち零環は整域でない。選択公理を認める。特に、環の真のイデアルは、ある極大イデアルに含まれる。(選択公理はA-加群の圏が十分多くの入射的対象を持つことの議論にも現れる。練習23.2.G。) /* 関連して、積閉集合は断らない限り0が属しているかもしれない。そのような積閉集合による局所化は零環である */ 読者は可換環論の基本的な用語に慣れているべきである。特にイデアルの言葉(素イデアル、極大イデアル)と局所化である。テンソル積とA-加群の完全列は重要である。 局所環(極大イデアルをただ1つ持つ環)の表記に(A,m)あるいは(A,m,k)の表記を使う:Aは環で、mは極大イデアルで、k=A/mは剰余体である。単項イデアル整域A上の有限生成加群の構造定理を使う:そのような加群はA/(a)の形をした加群の直和で表すことができる。体論のいくらかの経験も時に役に立つだろう。 0.3.1 基礎論について 基礎論に関する微妙な問題は気にしない(集合論、宇宙など)。これらの問題に注意しなければいけない人がいるのは確かである。読者はどの道を進みたいか?もし気にする稀な人であるなら文献[KS2]が良い入門である。 0.3.2 さらなる背景知識 各分野の事実についてより掘り下げる本を(予め読むのではなく)持っておくことは役に立つかもしれない。可換代数では[E]が良い。[AtM][Mat2]も人気である。[Al]は代数幾何の良い視点を与える。ホモロジー代数には[Weib]が詳しくかつ読みやすい。 他の数学の分野の背景(トポロジー、幾何、複素解析、数論・・)はもちろん洞察や動機に役立つ。いくらかのトポロジーへ触れておくことは実際本質的に重要である。 0.3.3 非数学的な記法 「重要でない」は、文脈において重要でないことを意味し、より広い視点で重要でないという意味ではない。他の言葉も同様である。 2回目あるいは3回目に読むときに知るに値するアスタリスクをつけた項目がある。(*)の項目は難しいとは限らないが、重要でない。(**)の項目は、本当に読みたいと望まない限り読まないべきである。存在を知っていれば良い。早速試そう。 --- 0.4(**) この本の目標 代数幾何の分野の入門には数多くの選択肢がある、リーマン面、複素幾何、多様体の理論、例に基づいた厳密でない入門、数論的代数幾何、抽象的な関手的アプローチ、等。それぞれにはそれぞれの価値があり、読者に合うもの合わないものがある。この本は1つの方法でしかない。 我々の意図は、十分な例と計算によりこの道具への直観を育てつつ、この聖典を完全に、そして厳密に網羅することである。これはしばしば代数幾何の二度目の講義の内容で、理想的な世界では、可換環論、代数トポロジー、微分幾何、複素解析、ホモロジー代数、数論、そしてフランス語を学んでから、この代数幾何を何年もかけて学ぶ。我々はそのような理想的な世界に住んでいない。そういうわけでこの本は一度目の入門として書かれているが、しかし挑戦的なものである。 この本はごくわずかな物事しか追求しないが、それらをよく追及するように試みた。我々の目標は次のように約束される: 中心的な事柄は、一年で消化できるべきである。 血の汗と涙の一年の後、読者はこの分野について広く精通し、セミナーに出席しより進んだ内容を学ぶ準備ができている。単にこの分野の概念に対するおぼろげな直観的な理解をしているだけではない。興味深い例を知り、なぜ興味深いかを知り、詳細を語れるべきである。数学の他の分野からの読者は、それぞれの興味ある分野に現れる代数幾何的な考えを理解するのに十分知っているべきである。 これは、この本は百科事典ではなく、辛い選択をせざるを得なかったことを意味する。(特に解析的な側面は本質的に無視しているか、せいぜい証明なしで触れられている程度である。この本は代数的な代数幾何の本である。) この本は独学が可能であるように作られている。しかし完全に独学することは多くの人には困難である。しかし理想的には集団で読むか、質問できる人を持っている(ばかげた質問から良い質問まで)ことが望ましい。 ここには1年分を超える内容があるが、どの話題が必須でどの話題がそうでないかを明らかにするように努めた。講義する者は、どの必須でない話題が彼らにとっては重要かを選ぶことになる。 (少なくとも今回とる代数幾何へのアプローチには)聖典がある。私は(真剣な)一年で学ぶべき内容について、多くの人が同意を持っていることに繰り返し驚かされた。(もちろん異なる分野にはそれぞれ異なる聖典がある。) 異なる教師はおまけの鐘や笛をつけて、学生を特定の分野に進める準備をするが、この分野の多様性と豊かさにも関わらず、中核は変わらない。目標を定めるには深刻で辛い妥協が必要である。 代数幾何は皆に開かれている(皆という言葉を適切に定義すれば)。代数幾何の講義は多くの前提知識を要求することが多いが、 ・・・ これらの理由から、この本はできるだけ前提知識を少なくした。前提知識は驚くほど少ない。理想的には可換環論を少しと、点と集合の位相、それから素イデアルと局所化へのいくらかの慣れである。もちろんこれは誤解を招くかもしれない -- より知っているほどより良い。より知識が少ないほど、読むのに苦労する。これは簡単に読めるものではない。 この本はできるだけ内部完結するように意図した。「もしそれを使うならそれを証明できなければいけない」の精神に従うように努めた。多くの学生は人間であることに気が付いている。もしある代数的な事実が本の可換代数の本の後ろの方の章にあるなら、彼らはすぐにそれを読みに行かない。驚くべきほどいつも、我々が必要なのは抜粋で、それを読まなくても何が関与しているかが分かる。その本はもっと分厚くて、高度な精神と忍耐力が要求される。 ・・・ 例として、クルルの単項イデアル定理(11.3.3)は重要な道具である。私は本質的に標準的な証明を含めた(11.5)。私は読者に読んで欲しくない(本当に本当に望まない限り)と思っていて、二重アスタリスクで印をつけた。代わりに、表層だけすくってそれを読めて、しかもそれほど難しいものではないことを認識してもらうことを望む。 何が正しいかを知るだけでなく、なぜそれが正しく、なにが難しく、なにが難しくないかを知ることは重要である。これにより、前提知識の数を減らし、最初の目標の達成に役立つ。 この本は代数幾何の恐ろしい道具に対して、直観を育てることを意図している。 ・・ ・・・ 以上の目標のバランスをとることだけで既に不可能である。我々はこれ以上望むことは諦めなければならない。他には目標は無いのである ==================== 1.圏 『0の導入や群の概念の導入はgeneral nonsenseである(が)、 数学は、そんな子供っぽい段階を経る余裕がなかったから何千年も停滞していた側面がある』 -- グロタンディーク 『私を殺さないものが、私を強くする』 -- ニーチェ 1.1 動機 面白い幾何学に至るには、物事を見やすく効率的にぎろんする言葉が必要である。それには圏の言葉が最も良い。知るべきことはそれほど多くない:それは単に便利な言葉である。多くの数学の言葉のように、圏論によって、我々が良く知っている状況での直観を、より一般的な状況に論理的に抽象化できる。 我々の動機は:今後、新しい数学的対象(スキームやある種の層)を作る。それが、今まで見たことがあるものと同じように振舞うことを期待したい。「同じように振舞う」を正確にして逐一確認することもできるだろう。しかし幸運なことにその必要はない。先人が、鍵となる用語を既に定義している。例えば、「アーベル圏」は環上の加群と同じように振舞う。 我々の方法は:私は何を知る必要があって、それ以上は知る必要がないことを伝えるように試みる。(これは約束する。もし私が「トポス」という言葉を使ったら私を撃って良い。)私はまず馴染みのあるものから始めて、より一般的な定義に抽象化できるような方法で、例を使って本質を描写するつもりである。それによりよりなじみの少ない状況での定義が見えて、問題を解いたり定理を証明したりするのを通して仲良くなっていくだろう。 例えば、我々はスキームの積を定義する。我々は単に定義を与えることもできるが、あなたはどうしてその定義が積という名前を冠しているのか知りたいだろう。動機として、我々はよく知っている状況での積の描写を振り返る:集合の圏である。UとVの集合の積を定義する1つの方法は、順序つき組{(u,v);u∈U,v∈V}の集合として定義する方法である。別の人は、{u@v;u∈U,v∈V}と表記するかもしれない。これらは明らかに同じである。言い換えると明らかな全単射がある。 このことは普遍性の言葉を使って積を定義することでより正確にできる。 2つの集合M,Nと積P、それに付随する写像μ:P→Mとν:P→Nが与えられた時、任意の集合P'と写像μ':P'→Mとν':P'→Nは、必ず、Pを、一意的な方法で経由する こうして、積とは、単にPという集合ではなく、 P→N ↓ M という図式であるととらえることができる。(μとνはしばしば省略されるが) /* 普遍性という状況は「この図式を満たす最も短い射」と描写すると良いと思う */ この定義はひとひねりによって古典的な定義と一致する。積とは、1つではない:2つの積があるとしたら、それらは一意的な方法で同型である。言い換えると、積は同型を除いて一意的である。なぜか。もし読者が積μ1:P1→M, ν1:P1→Nを持っていて、私が積μ2:P2→M, ν2:P2→Nを持っているとしたら、私の積が持つ普遍性によって図式を可換にするf:P1→P2が存在する(fとμ2の合成がμ1に一致してfとν2の合成がν1に一致する)。同様に、読者の積が持つ普遍性によってg:P2→P1が存在する。再度、私の積が持つ普遍性を考える。μ2:P2→M, ν2:P2→Nは、P2を一意的な方法で経由する。これは恒等射で経由することもできるが、今存在を示したgとfの合成によって経由することもできる。よって一意性によりgとfの合成は恒等射である。同様に逆の順番の合成も恒等射である。すなわちP1とP2は同型であり、このようにして積をM×Nと名付けることができる。 このような定義は多くの状況で利点がある。一旦圏を定義してしまえば、任意の圏で、積がもし存在すれば一意的であることが同じ議論で通用する。・・・ これは一旦理解してしまえば有用である。例えば古典的多様体MとNの積を定義する時に、局所座標系に分解して積を取って貼り合わせるという方法で定義した場合には、私がある方法で分解したときと、読者が別の方法で分解したときと、結果が同じであることがすぐには保証されない。代わりに、圏論的な積をとれば、一意性が保証されるのである。これについては1.3で議論する。 別の重要な用語は、環上の加群のように振舞うような圏である。我々は書くと余核を定義し、それらが期待通りに振舞うことを確認する。これはアーベル圏の定義に至る。アーベル圏はグロタンディークのTohokuで初めて定義されたものである。 この章では、これらの概念の略式な導入を行い、なじみを深めることを目標にする。 --- 1.2 圏と関手 -- 関手性を知らなかった頃の人々は洞穴に住んでいた -- B.Conrad 圏と関手の略式な定義をする。 1.2.1 圏とは対象と、対象の間の射の集まりである。(熟練者へ:技術的には、これは局所小な圏の定義である。正確な定義では射は集合をなすとは限らない。が、我々の立場0.3.1はこれらを気にしない。)射はしばしば略式には写像と呼ばれる。圏Cの対象の集まりはobj(C)と書かれるが、我々は単にCと書く。A,B∈CとするとAからBへの射の集まりはMor(A,B)とかかれ、射はしばしばf:A→Bと書かれ、Aを始域、Bを終域と呼ぶ。 射は、期待されるように合成される。合成は結合則を満たす。任意の対象A∈Cに対して恒等射A→Aが存在し、他の射との合成に対して恒等的に作用する。(恒等射は一意的であることを確認できるだろう。)これで圏の定義は終わる。 2つの対象が同型であるという用語を定義できる。f:A→Bとg:B→Aがあって合成f・gとg・fがともに恒等射であることを意味する。 1.2.2 典型的な例は集合の圏である。対象は集合で、射は集合の間の写像である。 1.2.3 別の良い例は与えられた体k上のベクトル空間の圏である。対象はk-ベクトル空間で、射は線形写像である。 1.2.A 重要でない練習 すべての射が同型射である圏はgroupoidと呼ばれる。(この用語は重要でない。単に圏の扱いの練習である。) (a) 「群とは対象が1つからなるgroupoidである」という主張を解釈せよ。 (b) 群でないgroupoidを描写せよ。 1.2.B Mor(A,A)の可逆元は群をなす ・・・ 1.2.4 アーベル群の圏 1.2.5 A-加群の圏(A=kとすると1.2.3, A=Zとすると1.2.4) 1.2.6 環の圏(我々の立場0.3は環準同型は1を1に送る) 1.2.7 位相空間の圏 これらの圏では対象は集合に何かの構造が入ったものであった(具体圏と呼ばれる)。 1.2.8 部分順序集合の圏 1.2.9 与えられた集合の部分集合からなる圏、与えられた位相空間の開集合からなる圏  射は包含写像である。 1.2.10 部分圏 1.2.11 関手 ・・・ 1.2.12 忘却関手 1.2.13 位相空間の例 位相空間の基本群をとる関手、ホモロジー群をとる関手 1.2.14 表現関手h^A : B→Mor(A,B) 1.2.15 関手の合成は明らかな方法で定義され、結合則を満たす。 ・忠実、充満の定義 ・充満部分圏 1.2.16 反対圏、反変関手 1.2.17 線形代数の例 双対空間 1.2.18 コホモロジーを知っている読者へ:位相空間の圏からアーベル圏への反変関手である 1.2.19 位相空間上の実関数をとる関手は、位相空間の圏から環の圏への反変関手である 1.2.20 表現関手h_A : B→Mor(B,A) 1.2.21(*) 自然変換、自然同型、圏同値 これらの概念は6章までは本質的には登場しない ・・・ --- 1.3 普遍性は対象を同型を除いて決定する 圏を与えられた時に、しばしば既知の対象から新しい対象を作る。良い状況では、それは普遍性を使って定義される。何かの性質をもつ対象の存在を示したいとする。それが存在するならばそれは同型を除いて一意的であることを先に示し、それから構成の例を示すのである。 明示的な構成を使った方が、普遍性を使うより扱いやすいこともある。しかし普遍性は物事をすばやく器用に証明するのに便利である。実際、物事を学ぶ時には人々はしばしば明示的な構成を魅力的に感じ、それを証明に使うが、経験を積むと普遍性を使った議論のほうがより上品で洞察的であることに気づいてくるのである。 1.3.1 我々は既に1.1で積という例を見た。読者は議論を振り返り、任意の圏において、積がもし存在するならそれは一意的な同型を除いて一意的であることを示すと良い。 1.3.2 始対称、終対象、零対象 これらは単純だが普遍性の議論を初めて練習するには便利な概念である。始対称とはすべての対象へ1つだけ射を持つ。すべての対象から1つだけ射を受けるのが終対象である。始対称かつ終対象な対象が零対象である。 1.3.A 練習 始対称(終対象)が2つあったら一意的に同型である。 1.3.B 集合の圏、環の圏、位相空間の圏、1.2.9の例では、始対称と終対象は(もし存在するなら)何か。 1.3.3 環と加群の局所化 普遍性を使う別の重要な例が環の局所化である。我々は最初に構成的な定義を与え、それから普遍性を使って解釈し直すことにする。 「Aの積閉集合」とは、1を含み、積について閉じているようなAの部分集合である。我々は環S^-1.Aを定義する。その元は、a/s(A∈A,s∈S)の形をしていて、あるs∈Sに対してs(s2a1-s1a2)=0 のときに限り、a1/s1=a2/s2 と定める。和と積を a1/s1+a2/s2=(s2a1+s1a2)/s1s2, a1/s1*a2/s2=a1a2/s1s2と定める。(読者は望むなら同値関係が実際に同値関係であることや、環の性質を満たすことを確認しても良い) そこには標準的準同型A→S^-1.A: a->a/1 がある。0∈SならばS^-1.Aは零環であることに注意せよ。 特に2つの重要な積閉集合がある。1つは{1,f,f^2,...,} (f∈A)である。これによる局所化を、A_fと書く。もう1つは、A-p (pは素イデアル)で、これによる局所化を、A_pと書く。 ((f)が素イデアルのとき、A_fとA_(f)が意味するものは違うので注意) 警告:局所化はAが整域でSが0を含まない場合を例にしてしばしば紹介される。そのようなときA→S^-1.Aは単射であるが、これは次の練習で示すように常にそうとは限らない。(しかし非単射性は病的ではなく、あとで(3.2.L)幾何的に理解することもできる。) 1.3.C A→S^-1.Aが単射 ⇔ Sが零因子を持たない /*a∈Sが零因子ならaは0に移る。aが0に移るならaは零因子である。*/ Aを整域とする。S=A-{0}によるS^-1.AはAの商体と呼ばれ、K(A)と書かれる。この場合、AはK(A)の部分環となる。以下また整域とは限らない場合に戻る。 1.3.D 標準的準同型 A→S^-1.Aは次の普遍性を満たす: 「A-代数Bのうち、Sの元がBでは可逆元であるものの始対象である」 (訳者はこの状況を、そのようなA→Bのうち最も"短いもの"と呼ぶ。またSの元を「ひっくり返す」(invert:invertibleな元にする)という表現を好む。) 解釈:A-代数Bを考えることは、環準同型A→Bを考えることと同じである。上記の性質を満たすA→Bは必ずS^-1.Aを経由する。別の解釈:S^-1.Aからの環準同型を考えることは、Aからの環準同型でSを可逆元に送るものを考えることと同等である。また、S^-1.A-加群を考えることは、A加群のうち、s倍写像が同型を与えるものを考えるのと同等である。 /* 条件を満たすf:A→Bに対して、S^-1.Aを経由した後の射 g:S^-1.A→Bを定めたい。 f(s)は可逆元だから g(a/s) = f(a)/f(s) と定めることができる。 これがwell-definedであることを直接的に確認できると思う。 */ 実際には、A→S^-1.Aを普遍性で「定義」して、それから存在を示すほうがすっきりしている。加群の局所化を普遍性を使って定義することでこれを練習しよう。MをA-加群とする。A-加群としての準同型 M→S^-1.Mを、M→Nのうち、すべてのs∈Sに対してs倍写像がN→Nの同型となるようなもののうち、最も短いものとして定義する。・・・ 1.3.E 加群の局所化 M→S^-1.Mは存在する。 (m/sの形をしているもので、s(s2m1-s1m2)=0の時に限りm1/s1=m2/s2と定める。加法とスカラー倍を自然に定める。あとは1.3.Dと同様の議論によると思う。) 1.3.F 練習 (a) 加群の局所化は有限個の積(直積)=有限個の余積(直和)と可換である  M,M'をA-加群とする。同型:S^-1.(M×M')→S^-1.M×S^-1.M'を記述せよ。 (b) 無限個の直和とも可換 (c) 無限個の直積とは可換と限らない /* (a) 準同型 (m,m')/s → (m/s,m'/s)を定めることができる。  「通分」によって逆向きに戻ることができる。 (b) 非零な要素は有限個なので(a)と同様に通分できる (c) Q = S^-1.Z (S=Z-{0}) の無限個の直積を考える。  準同型 S^-1(Z×Z×...)→Q×Q×... を先のように定めると、  この像は、無限個の分母が有限な最小公倍数を持つものだけである。 */ 1.3.4 指摘 局所化はHomと可換とは限らない(1.6.8)。しかし良い状況では(Homの第一引数が有限表現を持つならば)可換となる(1.6.G)。 1.3.5 テンソル積 普遍性で構成する別の重要な例がA-加群のテンソル積(M,N)→M {@_A} Nである。添え字_Aは文脈から明らかな時はしばしば省略される。テンソル積はしばしば次のように定義される: M@Aはm@n(m∈M,n∈N)の有限個のA-線形結合であり、分配法則と a(m@n)=(am)@n=m@(an)を満たす。あるいは、M×Nの元で生成された自由A-加群を、ある種の部分群で割った商加群として定義する。 Aが体である場合は、ベクトル空間のテンソル積を再現する。 1.3.G (もし読者がテンソル積を見たことが無いなら、手を動かす練習として) Z/(10)@Z/(12)がZ/(2)と同型であることを示せ。 1.3.H 重要な練習:テンソル積は「右完全関手」である: M'→M→M"→0が完全ならば(定義は1.6)、M'@N→M@N→M"@N→0も完全である。 (この練習は1.6.Fで繰り返すが、ここでしておくのがテンソル積の練習に良い) /* しかし訳者の考察は1.6.Fにまとめて書くことにした */ 一方で、テンソル積が左完全とは限らないことはすぐに確認できる。 0→Z→Z→Z/(2)→0 (Z→Zは2倍写像) を Z/(2) でテンソルしてみよ。 さて、上記のテンソル積の定義はややこしく、実際「適切でない」。より良いものに導くには、A-双線形写像 M×N→M@Nに注目する。双線形写像とは両方の引数に対して分配法則が成り立ち、f(am,n)=f(m,an)が成り立つものである。任意の双線形写像M×N→Pは、M@Nを一意的に経由するのである。(これをじっくり考えよ。) これをテンソル積の定義に使うことができる。テンソル積T=M@Nとは、双線形写像M×N→Tのうち最も短いものである。 1.3.I こうして定めたTは一意的な同型を除いて一意的である。 こうして、存在が分かれば一意的であることが分かる。ただし存在を示すには明示的な構成を要する。 1.3.J 先1.3.5で定めたテンソル積は確かにこの普遍性を満たす。 以下の3つの練習はテンソル積により慣れるための有用な事実である。 1.3.K (a) MがA-加群でA→Bが環準同型ならば、B {@_A} M はB-加群である。  これはA-加群の圏からB-加群の圏への関手をなす。 (b) A→Cを別の環準同型とすると、B {@_A} C は自然に環の構造を持つ。(1.3.Uでまた現れる) 1.3.L 加群の局所化S^-1.M は環の局所化とのテンソル積 S^-1.A {@_A} M として解釈できる。 1.3.M テンソル積は無限個かもしれない直和と可換である。 /* 同型 M@(N1+N2+..) → (M@N1)+(M@N2)+... を次のように定める。 m@(n1,n2,...) の像を (m@n1,m@n2,...), (m1@n1, m2@n2, ...) の逆像を m1@(n1,0,0,..) + m2@(0,n2,0,...) とすれば良い。 */ 1.3.6 ファイバー積 任意の圏での射の組α:X→Zとβ:Y→Zを考える。(Zによる)ファイバー積 X{×_Z} Y とは、 Xへの射とYへの射と、α,βがなす図式が可換となるようなもので、最も短いものである。 (警告:これが定義するものはX{×_Z} Yという表記には現れないが、αとβに依存する。) 普遍性の議論より、これも存在すれば一意的な同型を除いて一意的である(読者はこれが明らかだと思うまでじっくり考えるべきである)。 ファイバー積のなす四角形の可換図式をデカルト図式(別名:ファイバー図式、引き戻し図式、カルテシアン図式)と呼ぶ。 1.3.N 集合の圏ではX {×_Z} Y = {(x,y)∈X×Y|α(x)=β(y)}である。 1.3.O Xを位相空間とする。Xの開集合の圏でのファイバー積とは、部分集合の交わりである。 1.3.P 通常の積 X×Y は、終対象によるファイバー積として解釈できる。 1.3.Q デカルト図式の塔はデカルト図式である: U→V ↓↓ W→X ↓↓ Y→Z を考える。WがZによるXとYのファイバー積で、UがVとWによるファイバー積ならば、 UはZによるVとYのファイバー積である。 1.3.R X1→Y, X2→Y, Y→Zがあるとき  自然な射 X1{×_Y}X2 → X1{×_Z}X2 がある (それらのファイバー積が存在するとして) 1.3.S マジック図式: X1{×_Y}X2 → X1{×_Z}X2 ↓   ↓ Y → Y{×_Z}Y はデカルト図式である。 これは驚くほど有用なのでマジック図式と呼ぶ(訳者には使いこなせていない) 1.3.7 余積、ファイバー余積は、積と余積の射の向きを反対にして定義される。 1.3.T 集合の圏での余積は非交和である。 1.3.U 環の圏のファイバー余積は(環としての)テンソル積である。(1.3.K) 1.3.8 モノ射とエピ射 1.3.9 モノ射の定義: 任意のμ1,μ2:Z→Xに対して、π・μ1=π・μ2ならばμ1=μ2が要求されるときπ:X→Yはモノ射である。・・・ 実際、これは単射の概念を一般化したものである。しかしモノ射は単射とは限らない。Q→Q/Zはモノ射だが単射でない。 1.3.V モノ射の合成はモノ射である 1.3.W X→Yがモノ射⇔ ファイバー積X {×_Y} X が存在し、対角射 X→X{×_Y} X が同型である /* X→Yがモノ射ならば、Xはファイバー積が要求する普遍性を満たす 逆にX×XがXに同型ならファイバー積の普遍性からf=gが言える */ 1.3.X Y→Zがモノ射ならば、X {×_Y} X' → X {×_Z} X' は同型である エピ射は、モノ射の双対である。この概念はすべてのアーベル圏で現れるが、我々にはあまり中心的ではない。これは直観的には全射の圏論版である。しかしエピ射は全射とは限らない。例えば環の圏でZ→Qはエピ射だが全射でない。 1.3.10 表現可能関手と米田の補題 ・・・ --- 1.4 極限と余極限 (訳者は極限とは与えられた図式に降り注ぐ最も短い射、 余極限とは与えられた図式から降り注ぐ最も短い射、と描写する) 1.4.1 極限 対象と射が集合であるような圏を小さい圏と呼ぶ(この状況を限定する用語は熟練者向けである) Fは小さい圏で、Cを圏とする。関手F→CをFで添え字付けられた図式と呼ぶ。Fを添え字の圏と呼ぶ。 通常、添え字の圏は、2つの対象の間に射が1つしかない半順序集合であるが、そうでないことも有り得る。例えばFを ・→・ ↓ ↓ ・→・ という圏とすると、関手F→CはちょうどCの可換な正方形という情報を持つ。 Cの対象X=lim← Xiが図式の極限であるとは、 XからXiの射たちがあって、関手F→Cによる射の像と可換であるもの(訳者補足:錘という言葉がある)のうち、終対象である。 (逆極限とか射影極限とか呼ばれることもあるが我々はその言葉は使わない) 普遍性の議論により、それは存在するなら一意的な同型を除いて一意的である。 1.4.2 例   ・   ↓ ・→・ という図式の極限がファイバー積である。 ・ ・という図式の極限が、積である。 1.4.3 p進整数Z_pは環の圏での ..→Z/p^3→Z/p^2→Z/p という図式の極限である。 1.4.B 集合の射影極限 1.4.4 余極限 1.4.6 集合(1/p^∞)Z:分母がpの冪からなる集合は、 Z→(1/p)Z→(1/p^2)Z→.. という図式の余極限である。 1.4.C (a) Q は (1/n)Z の余極限と解釈できる (b) 部分集合の合併は余極限として解釈できる。(交わりは極限として解釈できる) 1.4.7 フィルター付けされた半順序集合:x,yに対してx≧zかつy≧zとなるzが存在する フィルター付けされた圏:上記に加えて、2つのx→yをcoequalizeするy→zが存在する 1.4.D Fをフィルター付けされた圏とする。 +が集合としての非交和を表す記号であることを思い出しておく。 Fで添え字づけられた集合の圏の図式は以下で描写される余極限を持つ。 { (ai,i)∈+Ai } / ((ai,i)と(aj,j)が共通する(ak,k)に送られる時同値とする関係) 群の圏やA-加群の圏でも同様の描写ができる。 ・・・ 1.4.E 確認 1.4.F 余極限としての局所化 1.4.G 1.4.8 要約 対象が"集合のような"圏では、極限の元は、一貫性のある元たちとして解釈できる。 余極限の元は、図式の中のある対象の元として解釈できる。 /* 訳者はwikipediaの射影極限=極限と帰納極限=余極限にある数列の例を紹介したい。 極限(制限写像は切り取り写像)では、無限でも良い数列の集合、 余極限(余制限写像は0パディング)では、有限項を除いて0となる数列の集合である */ --- 1.5 随伴: 普遍性が圏の対象を本質的に一意的に決定したように、随伴は関手を本質的に(またもや、もし存在するなら)一意的に決定する。共変関手F:C→DとG:D→Cが随伴であるとはすべてのX∈CとY∈Dに対して、 Mor_D(F(X),Y) と Mor_C(X,G(Y))の間に自然な全単射τ_XYがあることを言う。 このとき、Fは(Gの)左随伴、Gは(Fの)右随伴であると言う。 ここで自然な全単射という言葉は、f:X→X'に対して Mor_D(F(X'),Y)→Mor_D(F(X),Y) ↓        ↓ Mor_C(X',G(Y))→Mor_D(X,G(Y)) という可換図式があり、g:Y→Y'に対しても同様の可換図式があることを意図している。 (縦の矢印が自然な全単射τ_XY、横の矢印はF(f),fで誘導されるもの) 1.5.A gに対する同様の可換図式を書き下せ 1.5.B τ_XYは以下の性質を持つ: 任意のX∈Cに対して写像τ:X→G(F(X))があって、τ_XY(g)はX→G(F(X))→G(Y)の合成である(後者はG(g)によって誘導されるもの) 同様にε_B:F(G(Y))→Yがあって、τ^-1_AB(f)は F(X)→F(G(Y))→Yの合成である。 随伴の例として鍵となるものがある 1.5.C Aを環、M,N,PをA-加群とする。Hom_A(M@N,P)とHom(M,Hom(N,P))の間の全単射を描写せよ (ヒント:@の普遍性を使うことを試みよ) /* どちらもM×NからPへの双線形写像を表している */ 1.5.D (・)@N と Hom(N,・)は随伴関手である /* 1.5.Cの全単射が"自然"であることを確認する:例えば準同型M→M"があるとき Hom_A(M"@N,P)→Hom_A(M@N,P) ↓         ↓ Hom(M",Hom(N,P))→Hom(M,Hom(N,P)) が可換である。これは結局どちらの行もM"×NからPへの双線形写像からM×NからPへの双線形写像を表していることから可換である。 */ 1.5.E A←Bを環準同型とする。A-加群Mは自然にB-加群とみなすことができる。これはB-加群からA-加群を作る、・{@_B}Aという関手と随伴である。 (これは16章(準連接層の引き戻し)で鍵となる。矢印が反対なのは後でSpecA→SpecBと書くため。) 1.5.1 (*) unitとcounit 1.5.2 他の分野での例 表現論ではフロベニウス相互律がこの文脈で理解できる。 位相論の人はsuspention関手とloop関手の組がお気に入りかもしれない。 1.5.3 アーベル半群の群化 ・・・ 1.5.H 忘却関手の随伴 例えばS^-1.A-加群の圏は、A-加群の圏の忠実充満な部分圏であった。 (明らかな埋め込みMod[S^-1.A]→Mod[A]によって) Sによる局所化は、この忘却関手に対する左随伴関手となる。 我々に関係する随伴関手の例を挙げた: A-加群(1.5.D) --- ・@N --- Hom(N,・) 環準同型B→A(1.5.E) --- ・[@_B] A ("係数拡大") --- M→M_B("係数制限"、忘却) 前層と層(2.4.L) --- 層化 --- 忘却 半群(1.5.3) --- 群化 --- 忘却 アーベル群の層と開埋め込みπ:U→Y(2.7.G) --- π^-1(逆像) --- π_*(順像) 準連接層とπ:X→Y(16.3.6) --- π^*(押し出し) --- π_*(引き戻し) 環準同型B→A(30.3.A) --- M→M_B ("係数制限") --- N→Hom_B(A,N) 準連接層とアフィン射π:X→Y(30.3.B(b)) --- π_* --- π_sh^! 他の例もまた現れる。例えば射影スキーム上の準連接層と次数付き加群の間の随伴(15.4) 1.5.4 左随伴は右完全、右随伴は左完全、  左随伴は余極限と可換、右随伴は極限と可換  極限は極限と可換、余極限は余極限と可換(1.6.12) --- 1.6 アーベル圏の導入 -- あなたのホモロジー代数の論文は注意深く読まれ、Dieudorneを含め、皆をあなたの視点に引きこんだ -- J.P.Serre, A.Grothendieckへの手紙 線形代数を学んだ頃から、読者は核や余核の概念になじみ深いだろう。後に読者はアーベル群の圏、さらにはA-加群の圏でも核や余核を見ることになるだろう。 我々はすぐに環上の加群のようななじみのある振舞いをする圏を定義する。そこでは核、余核、像などの概念が意味をなし、我々の期待する通りに振舞う。これはアーベル圏という言葉で正確に述べられる。アーベル圏はホモロジー代数をするのにふさわしい一般的な圏であり、核や余核などを使って、複体や完全列を議論することができる。 モノ射、部分対象、エピ射、核、余核、像などを定義する。しかしこの本では一般のアーベル圏でそれらが我々が期待する通りに振舞うことを証明はしない。具体例が環上の加群の言葉で直接解釈できるからである。特に、アーベル圏の定義を記憶しておく価値はそれほどない。 アーベル圏の中心的な例は、アーベル群の圏AbとA-加群の圏Mod_Aである。前者はZ-加群の圏という意味で、後者の特殊な場合である。定義を与えれば、A-加群の圏がアーベル圏であることはすぐに確認できるだろう。 先に、加法的な圏という用語を定義する。我々はこの用語をアーベル圏を定義するためだけに使う。定義を読む際に思い浮かべると良い2つの例は、A-加群の圏と、実バナッハ空間の圏である。 1.6.1 定義 圏Cが加法的な圏であるとは、下記を満たすことである: ・Ad1:A,B∈Cに対して、Mor(A,B)はアーベル群であり、射の合成に関して分配的である。(読者はこれが何を意味するか考えるべきである--それは2つの主張に翻訳される。) ・Ad2:Cは0と書かれる零対象を持つ。これは始対象でありかつ終対象であることを意味する(1.3.2) ・Ad3:2つの対象は積を持つ。(従って任意の有限個の対象は積を持つ) 加法的な圏では射は準同型と呼ばれ、MorはHomと書かれる。実際、Homの表記は加法的な圏で考えていることを示す良い指標となる。加法的な圏のあいだの関手でHomの構造を保つものは加法的な関手と呼ばれる。 1.6.2 指摘 加法的な圏では、有限個の対象に対する積は余積でもあることは定義から従う。詳細は我々には重要ではない。+という記号がこれを表すのに用いられる。 また、加法的な関手は零対象を零対象に移すことが定義から従う(対象Zが零対象であることは、Zの恒等射が零元であることと同値であることを使う)。アーベル群Hom(A,B)の零元となる射は、零対象0によってA→0→Bの合成で表せる。このようにして加法的な圏では零射という概念が意味をなす。A-加群の圏は明らかに加法的な圏であるが、もっと構造を持っている。それをこれから描写する。 1.6.3 定義 Cを零対象を持つ圏(従って0-準同型を持つ圏)とする。射f:B→Cの核とは、射i:A→Bであって、f・i=0かつ、次の普遍性を満たすものである: 「fを含むZ→B→Cの合成が零射ならば、それはiによってAを経由してZ→A→B→Cと分解できる」 つまり核とは対象のことではなく、Bへの射のことである。普遍性により、核は同型を除いて一意的である。核は、kerf→Bと書かれる。cokerfと書かれる余核は、これの矢印を反対にしたものである。(読者の手で実行せよ。)核は、   0  ↓ B→C という図式の極限(1.4)と解釈でき、余核は同様に余極限と解釈できる。 i:A→Bがモノ射(1.3.8)のとき、AはBの部分対象であるという(iは省略される)。その双対は、商対象である。 アーベル圏とは、加法的な圏に加えて、次の性質を持つものである: ・すべての射は核と余核を持つ ・モノ射はその余核の核である ・エピ射はその核の余核である 核や余核について読者が期待する性質がすべてここから従うことは自明でない。(注意:文献によっては追加の定義が含まれることがある。) 射f:A→Bの像は、im(f)=ker(coker(f))と定義される(それが存在する時)。射f:A→Bについてim(f)が存在するとき、fはA→im(f)→Bに一意的に分解し、A→im(f)はエピ射であり、アーベル圏ではこれはcoker(ker(f))に一致する。読者はこれを(難しい)練習として確認することを望むかもしれない /* https://math.stackexchange.com/questions/1885954/image-as-kernel-of-cokernel-and-epimorphism-in-category-theory */ モノ射の余核は商と呼ばれる。モノ射A→Bの商はB/Aと書かれる(「標準的全射」に相当する射B→B/Aを省略して) 我々はアーベル圏の基礎的な理論は踏み込まない。覚えておくべきことは、核、余核、像などを環上の加群の圏で理解すれば、アーベル圏でもこれらを扱えることである。これは正確には1.6.4のFreyd-Mitchellの埋め込み定理で描写される。 しかしながら、我々が今後扱うアーベル圏は明らかに加群に関連したものであり、我々の期待する性質が成り立つことが明らかなものばかりであるから、難しい定理に訴える必要はない。例えば、2.6で位相空間上のアーベル群の層がアーベル圏をなることを示し、「一貫性のある芽」の言葉で、核、余核などの概念を描写することで、対応するアーベル群の核や余核の概念と結び付けて解釈することができる。 1.6.4 指摘 Freyd-Mitchellの埋め込み定理により、Hom(X,Y)が集合をなすようなアーベル圏Cに対して、(可換とは限らない)環Aがあって、Cから A加群の圏全体に埋め込む忠実充満関手が存在する。従ってアーベル圏について「図式追跡」で証明することが正当化される。この定理の証明は補完するための資料とともに[Weib,§1.6]に簡潔に書かれている。[KS1,§9.7]にも証明がある。 証明していない定理を使うことをためらうなら、Mac Laneの"elementary rules for chasing diagrams"が手短な代替となる。しかしまあ読者は先に進むために気が向くことだけをすれば良い。 1.6.5 複体、完全系列、ホモロジー(以下アーベル圏を想定する) 系列 ..→A→B→C→.. が複体であるとは、合成A→B→Cが零射であることを言い、Bにおいて完全であるとは、ker(B→C)=im(A→B) であることを言う(coker(A→B)=coim(B→C)とも同値である。)。系列が完全(列)であるとはすべての(端でない)項において完全であることを言う。短完全列とは両端が0な5つの項を持つ完全列を言う。 Bにおけるホモロジーとは、ker(B→C)/im(A→B)である(より正確には単射im(A→B)→ker(B→C)があってその余核がホモロジーである)。すなわちすべてのホモロジーが0のときに複体は完全である。ker(B→C)の元をサイクルと呼び、im(A→B)の元をバウンダリと呼ぶ(従ってホモロジーはサイクルをバウンダリで割った群である)。複体が添え字が降順のとき添え字は下付き、昇順のとき添え字は上付きで書かれることが多い。昇順のときのホモロジーH^iはしばしばコホモロジーと呼ばれる。 完全列A●:..→A^(i-1)→A^i→A^(i+1)→.. は短完全列に分解できる: 0→ker f^(i+0.5)→A^i→ker f^(i+1.5)→0 より一般にA●が完全とは限らないときは次のように分解される: 0→ker f^(i+0.5)→A^i→im f^(i+0.5)→0 0→im f^(i-0.5)→ker f^(i+0.5)→H^i(A●)→0 (訳者が射の添え字を勝手に半整数にした) 1.6.A 次の完全列を描写せよ: 0 → im f^(i+0.5) → A^(i+1) → coker f^(i+0.5)→0 0 → H^i(A●) → coker f^(i-0.5) → im f^(i+0.5)→0 (これらは上記の双対である。実際こちらもホモロジーの定義となり得る。) 1.6.B 重要な定義:次元の交代和 1.6.C 重要な練習:複体の圏 1.6.D 重要な練習:複体の射はホモロジーの写像を誘導する:ホモロジー関手 1.6.6 短完全列から、連結準同型によって長完全列を作れる 1.6.7 関手の完全性 1.6.E 関手Fが完全関手であるとき、Fは完全列を完全列に移す。 1.6.F (a) A-加群の局所化は完全関手である (b) A-加群のテンソル積 ・@N は右完全関手である(1.3.Hの繰り返し) (c) Hom(N,・)は左完全関手である (d) Hom(・,N)は左完全反変関手である  [N'→N→N"→0ならば0→Hom(N",M)→Hom(N,M)→Hom(N',M)が完全 という主張である] /* すべて完全性のうち真ん中のker⊂imの包含だけ議論する。(完全性の他の部分はより簡単だと思う。) (a) M'→M→M"を完全とする。 m/s∈S^-1.Mが0∈S^-1.M"に移るとする。m/sにうつるS^-1.M'の元の存在を示す。 仮定よりあるs"∈Sが存在してms"∈Mが0に移る。 M'→M→M"の完全性よりm'→ms"となるm'∈M'がある。 そうするとm'/ss"∈S^-1.M'がm/sに移るので目的を達成する。 (局所化はS^-1.Aによるテンソルであるので右完全性は(b)の結果の一部でもある。) (b) M'→M→M"→0 を完全とする。f:M'→M, g:M→M"と名付けておく。 具体的な構成を使う視点を書いてみることにした。テンソル積に関する直接証明で困ったら直積の自由加群に戻って考えるという視点である。 M×Nの元で生成された自由加群をM~とおく。(m,n)をm@nに送る標準全射M~→M@Nがあって、その核は<(m1,n)+(m2,n)-(m1+m2,n),(m,n1)+(m,n2)-(m,n1+n2),a(m,n)-(am,n),a(m,n)-(m,an)>の形の元で生成されるイデアルということであった。標準全射M'~→M'@N, M~→M@N, M"~→M"@N の核をそれぞれI',I,I"とおく。 f,gから自然にM'~→M~→M"~が誘導される。これをf~,g~とおく。例えばg~は(m,n)を(g(m),n)に送るものである。 ★={y∈M~ | x∈M'~,i∈Iが存在して y=f~(x)+i}とおく。 ・g~(y)∈I"ならばy∈★を示すのが目標である。 ・I"の任意の生成元zに対して、g~(y)=zとなる任意のyが★に属することを示せば良い。 ・g~(y)=0となる任意のy∈★ かつ I"の任意の生成元zに対して、g~(y)=zとなるyの1つが★に属することを示せば良い。 というふう目標を言い換える。zの形は4種類あるのでするべき議論は5つ: [1-1] g~(y)=0 ならば y∈★を示す。 自由加群の自由性により、y=Σa[k](m[k],n[k])とおくと、固定された(m",n)に対してg~((m[k],n[k]))=(m",n)となるような項の係数の和が0になることが要求される。 すなわち y=Σ[m",n] { Σ[k] a[k](m[k],n), kはg(m[k])=m"となるものを渡る } という和の順番にすると、それぞれのm",nに対して Σ[k] a[k] = 0 が要求される。 Σ[k] a[k](m[k],n)≡Σ[k] (a[k]m[k],n) (mod I) であり、g(Σa[k]m[k])=0となるからM'→M→M"の完全性によりf(m')=Σa[k]m[k]となるm'∈M'が存在する。よってx=(m',n)とおけばΣ[k] a[k](m[k],n)≡f~(x) (mod I)が言えるので目的を達する。 [1-2] z=(m1",n)+(m2",n)-(m1"+m2",n)に移る1つのyが★に属することを示す。 M→M"は全射なのでm1",m2"に移るm1,m2∈Mをとれる。y=(m1,n)+(m2,n)-(m1+m2,n)はzに移る元の1つであり、その形からIの生成元の1つなので★にも属する。 [1-3,4,5] 同様の議論である。 *他にもいくつか視点がある: 「双線形写像と普遍性を使う視点」 http://blog.livedoor.jp/ron1827-algebras/archives/77540295.html 「随伴によりHomの左完全性を利用する視点」 http://blog.livedoor.jp/ron1827-algebras/archives/77698566.html 随伴を使う1.6.15は、さらに強力な視点である。その証明をテンソル関手に特殊化して書いてみる: これにはHom(M@N,X) = Hom(M,Hom(N,X))という事実(1.5.C)と余核の普遍性だけ使えば良い(Homの左完全性も利用しない)。これを納得した時はちょっと感動した。 M'@N→M@Nの余核がM"@Nであることを示せば良い。 M'@N→M@N ↓ 0 という図式からXに降り注ぐ射があるときにそれがM"@Nを経由することを示したい。(1.5.C)を使うと、 M'→M ↓ 0 という図式からHom(N,X)に降り注ぐ射を得る。M'→Mの余核がM"なのでこれはM"を経由する。つまり M'→M ↓ ↓ 0→M" という図式からHom(N,X)に降り注ぐ射がある。そこで再び(1.5.C)を使うと M'@N→M@N ↓ ↓ 0→M"@N という図式からXに降り注ぐ射を得るので、目的が達成された。 (c) 0→M'→M→M" を完全とする。(f:M'→M, g:M→M") x∈Hom(N,M)が0∈Hom(N,M")に移るとする。これはg・x=0を意味する。すると核の普遍性により、y∈Hom(N,M')によってx=f・yと書ける。これが欲しいものである。 (より地に足がついた言葉:x∈Hom(N,M)が、任意のn∈Nに対してg(x(n))=0を満たすとする。仮定の完全性よりf(m')=x(n)となるm'∈M'が存在するが、M'→Mの単射性よりこのm'は一意的に定まる。この対応によってnに対してm'を出力するものが目的のyである。万全を期すなら準同型性を確認してみても良い。) (d) M'→M→M"→0 を完全とする。(f:M'→M, g:M→M") x∈Hom(M,N)が0∈Hom(M',N)に移るとする。これはx・f=0を意味する。余核の普遍性によりy∈Hom(M",N)によってx=y・gと書ける。これが欲しいものである。 (より地に足がついた言葉:x∈Hom(M,N)が任意のm'∈M'に対してx(f(m'))=0を満たすとする。m"∈M"を与えられた時、M→M"の全射性よりg(m)=m"となるmがある。この対応によってm"に対してx(m)を出力する関数yが目的のものである。m"に対するmの選び方は一意的でないが、その差はim(f)に属し、仮定よりxはim(f)を0に送るから、mの選び方の任意性が打ち消される仕組みである。) */ 1.6.G Mを有限表現を持つA-加群とする。 つまり A^q→A^p→M→0 が存在するとする。 S^-1.Hom_A(M,N) から Hom_{S^-1.A}(S^-1.M,S^-1.N) への同型を描写せよ。 /* f'= f/s∈S^-1.Hom_A(M,N) とする。[f∈Hom(M,N), s∈S] m/s"を f(n)/ss" に送るものとして、行き先となる写像を描写できる。 反対向きが難しい・・ ・生成元の行き先を考える ・v1,v2,..,vp → n1/s1,n2/s2,..,np/sp とおく(このni,siの取り方に任意性がある)  s=s1*s2*..sp とおいてs=s1*s1'=s2*s2'=... とおく。 MからNへの写像を、v1,v2,..,→n1*s1',n2*s2',.. で定める。 これが準同型になるには、 核A^qの生成元u1,u2,...の像u1=Σci*viに対して、Σci*ni*si'∈Nが0になる必要がある。 仮定よりΣci*ni/si∈S^-1.N は0である。 ということは、ci*ni*si'*si"=0 となるようなsi"が存在する。 si"たちの積をs"とおいて、 MからNへの写像を、v1,v2,..,→n1*s1'*s",n2*s2'*s",.. で定め直せば良い。 と思った。 https://math.stackexchange.com/questions/1466017/localization-commutes-with-hom-for-finitely-generated-modules を参考にした。 */ 1.6.8 Homは局所化と可換とは限らない 先の例で A=N=Z, M=Q, S=Z-{0} とおいてみよ。 /* 左辺S^-1.Hom_A(M,N)は S^-1.Hom_Z(Q,Z) = S^-1.0 = 0 右辺Hom_{S^-1.A}(S^-1.M,S^-1.N) は Hom_Q(Q,Q) = Q */ 1.6.9(*) ホモロジー代数の2つの有用な事実 我々はここで2つの(一連の)事実の所に来た。それらの事実は私が子供の時に学べていたら良かったと思うものである(そうすれば私は多くの辛さを感じずに済んだ)。それらは良い意味でも悪い意味でもアブストラクト・ナンセンスの頂点である。その主張はあまりにも一般的で、直観的にとらえづらい。証明は極めて短い。行き当たりばったりな方法で簡単に証明できる挙動を一般化する。読者が一度それらに慣れれば、多くの微妙な事実がその特殊な場合として明らかだと感じられるようになるだろう。そしてそれらは(明示的あるいは暗黙的に)繰り返し使われていることに気づくだろう。 1.6.10(*) ホモロジーと(左/右)完全関手の相互作用 読者はこの証明は18章でコホモロジーを学ぶときまで待っても良い。この事実はそこで初めて本質的に使われる。 1.6.H 重要な練習:FHHF定理 (fernbahnhof) (a) Fが右完全関手のとき自然変換 FH → HF がある (b) Fが左完全関手のとき自然変換 FH ← HF がある (c) Fが完全関手のとき自然同型 FH ←→ HF がある (a)のヒント: A^i→A^i+1→coker(f^(i+0.5))→0を使って F(coker(f^(i+0.5))) = coker F(f^(i+0.5)) を示せ。 次に1.6.5に書いた 0→ker f^(i+0.5)→A^i→im f^(i+0.5)→0 を使ってエピ射 Fim(f^i+0.5)→im Ff^(i+0.5) を得よ。 次に1.6.5の2つ目の 0→im f^(i-0.5)→ker f^(i+0.5)→H^i(A●)→0 を使って FH^iA●→H^iFA●を得よ。 読者は F ker → ker F も描写したくなるかもしれない。 /* */ ・・・ 1.6.12 随伴と極限、完全性の相互作用 極限は極限および右随伴と可換である。特にアーベル圏では核は極限なので、右随伴関手は左完全である。余極限についても同様のことが言える。1.5.4で紹介したこれらの事実を以下で証明する。 A-加群の圏では、フィルター付けされた圏(1.4.7)で添え字付けられた図式の余極限は完全である。 1.6.13(**) 注意 すべてのアーベル圏で、フィルター付けされた圏で添え字付けられた図式は余極限を持つわけではない。・・・ 1.6.I 核は極限と可換である Fで添え字付けられた図式 a:F→Cと b:F→Cを考える。簡単のためAi=a(i),Bi=b(i)とおく。自然変換a→bをhとする。ker hi: Ci→Ai はFで添え字付けられた別の図式をなす。標準的な同型 lim← (ker hi) = ker(lim← Ai → lim← Bi) を描写せよ。 /* A=lim← Ai, B=lim← Bi, C=lim← Ciとおく。 ・ker(A→B)をK→Aとおく。K→CとC→Kを描写すれば良い。 ・K→A→Bは零射だから、K→Ai→Biは零射で、従って、K→Ciが存在し、従ってK→Cが存在する。 ・錘C→Ci→Aiが存在するからC→Aが存在する。同様にC→Bも存在する。  各Ci→Ai→Biは零射だから、C→Bは0を経由する。 C→0 ↓↓ A→B があるので核の普遍性によりC→Kが存在する。 */ 1.6.J 極限は極限と可換であるという言葉を定式化し証明せよ。 (前の練習はこれの系であるべきだ。) /* F,Gで添え字付けられた図式を考える。 つまり Fのi番目の対象とGのj番目の対象に対するCの対象A[i,j]があって iを固定したGに関する極限を A[i,-] jを固定したFに関する極限を A[-,j] と表記すると A1 = lim← A[i,-] = lim← A[-,j] = A2 を要求する。 A2....A1→ A[i,-] → A[i',-] ↓     ↓    ↓ A[-,j] → A[i,j] → A[i',j]  ↓    ↓    ↓ A[-,j'] → A[i,j'] → A[i',j'] を使って説明しよう。右下の四角形の射は、FやGの射から誘導された射である。 降り注ぐ射は、極限の定義によるものである。 外側の射は、遠くにあるほうの普遍性による。 例えばA1からA[・,j]への錘ができるのでA1からA[-,j]への一意的な射がある。 同様にするとA[-,・]への錘を得るので、A1からA2への一意的な射がある。逆も同様。 */ 1.6.15 右随伴は極限と可換 Aを圏DでのAiの極限とする。G:D→CをF:C→Dの右随伴関手とする。 G(A)がG(Ai)の極限であることを示せば良い。 錘 X→ G(Ai)がG(A)を経由することを示せば良い。 随伴性によって錘F(X)→Aiが存在するので仮定よりこれはAを経由する。 従って随伴性によって X→G(A) が存在し、目的を達する。 1.6.K A-加群の圏でフィルター付き添え字圏による余極限は完全である: (1.6.Fで局所化が完全であることや、2.6.Dで茎をとるのが完全であることをみる。 読者がそれらを既にやったならこの練習は易しく、逆にこれを経験すればそれらは易しくなる。) /* 1.4.7で使った具体的な描写を利用すれば良いと思う。 */ --- 1.7(*) スペクトル系列 ・・・ =========== 2. 層 しばしば空間はその上の関数でよく理解できる、ということは驚くべきことかもしれない。例えば微分可能多様体はその上の微分可能関数を使って調べられる。「幾何的空間」は(全体で定義された)関数を少ししか持たないため、先の洞察をより精密にしたバージョンとして、空間のそれぞれの開集合で定義される関数をすべて考えることで、空間の構造をもっとよく理解できる。この情報は層と呼ばれるものに埋め込まれる。 層は1940年代にLerayによって導入され、Serreによって代数幾何に導入された。(名前の由来について2.4.4で少し触れる。)これから層を定義し、有用な事実を紹介する。まず、この概念がそれほどかけ離れたものではないことを見るための典型的な例を紹介する。 層が捉えづらいのは、膨大な情報を持っていて、局所と大域の情報の間に微妙な問題があるからである。それを結びつける方法には3つの方法がある。 ・開集合を使うもの(定義2.2) -- 直観的であるが役に立ちづらい ・茎を使うもの(2.4.1) ・開集合の基を使うもの(2.5) /* 開集合の基Sとは、開集合全体の部分族であって、任意の開集合はSに属する開集合の合併として表せるようなものである。位相空間R^1に対する「連結な開区間」が簡単な例である */ どれを使うのが良いかを知るには経験が必要である。従ってこの概念を真に理解するには層の異なった側面の練習を多く行うことが必須である。 (エタール空間という解釈(2.2.11)を強く好む人もいる。) 2.1 典型的な例:微分可能な関数のなす層 位相空間X=R^n(あるいはより一般の微分可能多様体)上の微分可能な関数を考える。X上の微分可能な関数の層とは、Xの各開集合における微分可能関数すべての情報を持っている。このデータをどう扱うのかを見、そのいくつかの性質を観察する。Xの各開集合Uに対し、U上での微分可能関数がなす環を、O(U)と表記する。 ある開集合上の微分可能関数を与えられたときに、それをより小さい開集合に制限することで、そこでの微分可能関数を得られる。つまり、U⊂Vを開集合の包含とすると、「制限写像」resV→U:O(V)→O(U)が存在する。 大きな開集合の関数を取って、中間な開集合に制限し、それから小さな開集合に制限する。この結果は、直接小さな開集合に制限したのと同じである。つまり、包含U⊂V⊂Wがあるとき、resW→V;resV→U:O(W)→O(V)→O(U)とresW→U:O(W)→O(U)は可換である。 次に、大きな開集合Uで2つの微分可能関数f1,f2をとって、Uをいくつかの開集合{Ui}で被覆する。f1とf2のすべてのUiへの制限が一致するなら、f1とf2はUで等しいはずである。つまり、{Ui}がUの開被覆で、f1,f2∈O(U)で、すべてのiに対してresU→Ui(f1)=resU→Ui(f2)ならば、f1=f2である。こうして開集合上の関数は、開被覆たちで見ることで決定できる。 最後に、それぞれの開被覆Ui上に関数fiが与えられ、交わりで一致すると仮定する。そのとき、それらは張り合わさって、U全体での微分可能関数を与える。つまり、fi∈O(Ui)がresUi→Ui∩Uj(fi)=resUj→Ui∩Uj(fj)を満たすように与えられたならば、f∈O(U)でresU→Ui(f)=fiとなるものが存在する。 この例は、微分可能関数の代わりに連続関数でも滑らかな関数でも、単に関数でも、同様である。このように、「都合が良い」性質を持った関数は、共通する性質を持つ。この性質を層の言葉でこれから形式化していく。 2.1.1 微分可能関数の芽 その前に、別の定義を与える。点pでの微分可能関数の「芽」である。直観的には、それは微分可能関数の点pにおける断片である。芽は、{(f,開集合U)|p∈U,f∈O(U)}を、p∈W⊂U,Vとなる開集合Wが存在して resU→W(f)=resV→W(g)が成り立つとき(f,U)〜(g,V)とする、という同値関係(f,U)〜(g,V)で割ったものである。つまり、2つの関数は、pの共通する近傍において一致する(その外では異なっていても良い)とき同一の芽を持っている。点pでの芽の集合を、点pにおける茎と呼び、O_pで表記する。茎は環をなすことに注意せよ:2つの芽を足して別の芽を得ることができる。fがUで定義され、gがVで定義されているなら、f+gはU∩Vで定義される。さらに、f+gはwell-definedである:f'がfと同じ芽を持ち、g'がgと同じ芽を持つなら、開集合Wでfとf'が一致し、開集合W'でgとg'が一致するとおけるので、U∩V∩W∩W'上でf'+g'はf+gと一致する。/*従って同じ芽を与える*/ p∈Uのとき、写像O(U)→O_pがあることにも注意せよ。熟練者は既に、芽を余極限として扱っていることに気がついているかもしれない。 次に、O_pが局所環(極大イデアルを1つだけ持つ環)であることをみる。pで消える芽の集合をm_p⊂O_pとおく。これは確かにイデアルをなす。和について閉じていて、何かを掛けてもpで消えるままである。これが極大であることは、剰余環が体であることで確認できる。 /* 芽から点pでの値をとる全射:O_p→R; f->f(p) を考えると、その核がm_pである。すなわちO_p/m_p=Rである。*/ 2.1.A これがO_pのただ1つの極大イデアルだということを示せ。 (ヒント:O_p-m_pがすべて可逆であることを示せ) /* 連続関数の性質でfが0にならないようなpの開近傍をとれる。1/fをとれば良い */ 関数、あるいは芽の、「点での値」を「局所環を極大イデアルで割った剰余環の値」として解釈できることに注意せよ。(これは一般の層ではうまくいくとは限らないが、関数の層に似た層ではうまくいく。6.3で見るように局所環つき空間という用語で形式化される。) 2.1.2 寄り道。m_p/m_p^2はO_p/m-加群(R-加群)であることに注目せよ。すなわち実ベクトル空間である。これは実は自然に点pでの余接空間であることが判明する。この洞察は、スキームの接空間と余接空間を定義するときに役に立つ。 2.1.B(*) 微分幾何の背景知識がある人への練習 上記のことを証明せよ。(熟練者への質問:微分可能な関数の層の代わりに連続関数の層を使ったら何がまずいか。) /* R^nの原点に帰着させたとする: [1]pで0となる任意の微分可能関数f(x1,x2,..,xn)に対して、 一次式 g=a1x1+a2x2+...+anxn で、f-g∈m_p^2 となるgが存在すること [2]g∈m_p^2となるのはa1=a2=...=an=0のときだけであること を示せば集合{a1x1+a2x2+...+anxn|a1,a2,..,an∈R}がm_p/m_p^2を代表していることが言える。 「f∈m_p^2 ⇔ fのすべての偏微分係数が0である」という事実が期待され、これをしめせば良い。素朴にやるなら、変数の個数に関する数学的帰納法かなと思う。 */ 2.2 前層と層の定義 さて、前層と層を定義して、先の状況を形式化しよう。前層はより定義が単純である:核や余核が直接的である。層の定義はよりややこしく、いくつかの概念、例えば余核についてより考えなければいけないことがある。しかし層はある漠然とした意味でより幾何的であり、より有用である:層の情報は、局所的に得ることができる。 2.2.1 定義について 具体性のため、まず集合の層を定義する。しかし、集合の代わりに任意の圏で置き換えることができる。重要な例はアーベル群、k-ベクトル空間、環、A-加群などである。層はしばしばcalligraphic fontで表記される。Fが位相空間上の層であることはしばしばXの上に縦線を引いてその上にFと書くことで表す /* この日本語訳ではフォントは再現しない */ 2.2.2 前層の定義 位相空間X上の前層Fとは以下のデータである: ・Xの各開集合に対する集合F(U) いくつかの表記がある。F(U)=Γ(U,F)=H^0(U,F)。この本ではこれらをすべて使う。 F(U)の元はU上のFの切断と呼ばれる ・開集合の包含U⊂Vに対して、制限写像res[V→U]:F(V)→F(U)がある これは以下を満たすことを要求する: res[U→U]は恒等写像 res[W→V]とres[V→U]の合成はres[W→U]に一致する (そして以上だけである) 2.2.A 圏が好きな人へ 「前層は反変関手と同じである」 (この定義は驚くほど有用である) 2.2.3 茎と芽の定義について 1つの方法は直接的で、もう1つは余極限を使う方法である。 2.2.4 茎 F_p の直接的な定義 pを含む開集合での切断で、制限写像で同じになるものを同一視する 2.2.5 余極限 lim→F(U) ということでもある。(1.4.D) f∈F(U)のとき、f_p∈F_pをfのpでの芽と呼ぶ。 (2.1.1の例のような「点での値」という概念は一般には成り立たない。) (茎が局所環であるような状況である必要がある) 2.2.6 層の定義:層とは前層でさらに次の2つの公理を満たすものである: 開集合の族U_iがUを被覆しているとする ・一致の公理:f1,f2∈F(U)のすべてのU_iへの制限が一致するならば、f1=f2である。 ・貼り合わせ公理:すべてのU_iに対してf_i∈F(U_i)が与えられていて、すべてのi,jに対してf_i∈F(U_i)とf_j∈F(U_j)のU_i∩U_jへの制限が一致するなら、それらを貼り合わせたf∈F(U)がある。"貼り合わせた"とは、fのU_iへの制限がf_iに一致するような、という意味である。 (熟練者と空集合好きな人へ:F(φ)が終対象であることを追加で要求する公理が追加されることがある。これは空積が適切に定義されていれば上記の公理から従う。) 例えばUとVが交わらない開集合ならば、F(U∪V)=F(U)×F(V)である。 層の茎や芽は前層の場合と同様に定義される。 2.2.B 層でない前層の例:複素平面Cに古典的な位相を入れる。 (a)有界な関数 (b)平方根を持つ正則関数=正則関数の2乗となる正則関数 は層をなさない。 /* (a) f(z)=zは有界な開集合上では有界だが全体では有界でない (b) 原点以外でのf(z)=zは偏角を分割すれば根号をとれるが、全体ではとれない */ 2.2.7 完全列の等化子としての解釈 (集合の)前層が層であることは、次の"等化子完全列"の完全性として解釈できる: ・→F(U)→ΠF(U_i)→ΠF(U_i∩U_j) (原文では右端の矢印は二重矢印) 私はこれを詳しく説明しないが(集合の完全列とは何だ?)、読者は文脈から意味を明らかにできるかもしれない。 /* F(U)からΠF(U_i)への写像をfと名付け、これは各開集合への制限で構成される。ΠF(U_i)からΠF(U_i∩U_j)への2つの写像g1,g2を考える: g1:1つ目の写像の像のF(U_i∩U_j)のij成分は、F(U_i)からの制限 g2:2つ目の写像の像のF(U_i∩U_j)のij成分は、F(U_j)からの制限 左側の完全性は「fが単射であること」、右側の完全性は「y∈ΠF(U_i)に対して、g1(y)=g2(y) ⇔ y=f(x) となる x∈F(U)が存在する」と解釈する。アーベル群の層の場合は、g1(y)-g2(y)=g(y)を考えれば通常の完全列の概念となる。その描写は、4章のアフィンスキームの構造層の性質、7章のqcqs補題、13章の準連接層の性質の議論などで使われる。 */ 2.2.C 層の公理は、F(∪U_i)に対する極限の言葉で描写できる。 /*  F(∪U_i)は制限写像で結ばれたF(U_i)の極限である。 一方で∪U_iは開埋め込みで結ばれた U_iの余極限であるから、 反変関手としての前層が層をなすとは「余極限を極限に移す」と描写できる。*/ 2.2.D (a) 2.1で扱った微分関数のなす層などは確かに層である (b) 位相空間上の実数値連続関数は層をなす 2.2.8 層の開集合への制限 (あとで2.7によって開集合とは限らない任意の部分集合への制限を考えられる。) 2.2.9 摩天楼層 2.2.10 定数前層 2.2.E 局所定数前層は層である:定数層 2.2.F 写像は貼り合わさる Yを位相空間とする。XからYへの連続写像はX上の層をなす。 (2.2.D(b), 2.2.Eはこれの特殊な場合である。) 2.2.G (a)μ:Y→Xを連続写像とする。μの切断は層をなす:  Xの開集合Uに対し、{連続写像s:U→Y| μ・s=id|U}を切断と定める /* Yをエタール空間とすると通常の層の切断の概念を復元する。 (というよりエタール空間の位相がそうなるように定義されている) */ (b) Yを位相群とする。Yへの連続写像は群の層をなす。 2.2.11 (*) エタール空間 読者の背景によっては層の視点としてこの描写を好むかもしれない。FをX上の前層あるいは層とする。位相空間F'と連続写像π:F'→Xを次のように構成する:F'は集合としてはFのすべての茎の非交和として、πを自然な射影とする。F(U)に由来する部分集合を開集合の基とすることでF'に位相を入れる。この視点の利点は:2.7.Cで言及するように引き戻しはこの視点で見通しが良いこと、2.4.8で言及するように層化をうまく解釈できること、それから考え方は多く知っていれば言っているほど良い。 (訳者補足:このエタールe'tale'は紛らわしいことに(この本では扱わない)エタール射などに登場するe'taleとは別の単語である。) 2.2.H 層の押し出し=順像 π:X→Yを連続写像とする。FをX上の前層あるいは層とする。Y上の前層あるいは層π_*(F)を次のように定義する:Yの開集合Vでの切断を、F(π^-1(V))で定義する。この前層あるいは層を(πによるFの)押し出し、あるいは順像と呼ぶ。層の押し出しは層となることを示せ。 2.2.12 摩天楼層は点上の定数層の押し出しと解釈できる。 後で層の圏を定義すれば、押し出しはX上の層の圏からY上の層の圏への関手であることを見る。(2.3.B) 2.2.I 押し出しは茎の射(π_*F)_q→F_pを誘導する。 (2.2.4の明示的な代表類を使う方法と、2.2.5の普遍性を使う方法がある。 読者が片方を好むならもう片方も試みると良い。) /* 連続写像π:X→Yは自然にYの開集合からなる圏Op(Y)からXの開集合からなる圏Op(X)への関手π^-1を誘導する。 FをOp(X)からCへの反変関手とする。Op(Y)からCへの反変関手としての押し出しπ_*Fは、π^-1とFの合成である。 π(p)=qとする。{pを含む開集合}⊃{π^-1(qを含む開集合)}に注意する。 pを含む開集合をFで移した系から余極限F_pへ降り注ぐ射は、qを含む開集合をπ^-1とFで移した系から降り注ぐ射を含むので、(π_*F)_qを経由する。 */ 2.2.13 環つき空間とO_X-加群 O_Xは位相空間上の環の層とする。このとき(X,O_X)は環つき空間と呼ばれる。層O_Xは、環つき空間(O,O_X)の構造層と呼ばれる。U上のO_Xの切断は、U上の関数と呼ばれる。(注意:他の本では正則関数と呼ばれることもある。また、5.5.6で有理関数という用語を定義する。こちらは実は正確には関数でなく部分関数である。) O_Xという記号は常に環つき空間の構造層を意味する。O_Xの点pでの茎はここではOX_pと表記することにする。(訳者補足:プレーンテキスト記載のため習慣と違っている) O_X-加群の定義にはただ1つの可能性しかない:アーベル群の層Fであり、開集合Uに対してF(U)はO_X(U)加群の構造を持っていて、制限写像と可換となる。(私が何も忘れていないことを確信せよ。) A-加群の用語は、アーベル群の層の用語を引き継ぐ。任意のアーベル群は、定数層Zを構造層としたときのZ-加群とみなせるという意味である。 2.2.J 練習 (X,O_X)を環つき空間としてFをO_X-加群とする。茎F_pがどのようにOX_p加群となっているかを描写せよ。 2.2.14 ベクトル束を知っている人へ 動機付けとなるO_X-加群の例はベクトル束の層である。・・・ --- 2.3 前層と層の準同型 2.3.1 新しいものを定義した時にはいつでもその間の射を考えよというのが圏論の教訓である。前層や層の間の準同型を考えよう。(すなわち、我々は前層の圏や層の圏を描写しようとしている。) 前層の準同型:F→Gは、各Uに対してF(U)→G(U)が定まっていて制限写像に関して一貫性のあるものである。層の準同型も同様に定義される。(つまり層の圏は前層の圏の充満部分圏である。) 層の準同型の例は、R上の微分可能関数の層から、R上の連続関数の層への「忘却写像」である。(我々はそれらの関数が微分可能であることを忘れ、連続関数であることだけを覚えるのである) 2.3.2 Sets_X, Ab_X などでX上の集合の圏、アーベル群の圏などを表記する。 2.3.3 前層を位相空間からの反変関手とみなすとき、前層の準同型は自然変換である。 2.3.A 前層の準同型は茎の準同型を誘導する 2.3.B π:X→Yによる押し出し(2.2.H)は、X上の層から、Y上の層への関手である。 /* 開集合の逆像は開集合なのでYの部分開集合の圏Op(Y)からOp(X)の圏への関手π^-1がある。 関手圏 {Op(X)→C} から関手圏 {Op(Y)→C} への関手が、π^-1との(水平的な)合成によって、定まると考えることができる。 */ 2.3.C 層のHOM(sheaf Hom) F,GをX上の層とする。(実際にはFは前層でも良い) HOM(F,G)という層を、HOM(F,G)(U) = Mor(F|U,G|U) で定義する。 これが層であることを確認せよ。 (よくある混乱:右辺はMor(F(U),G(U))ではない。) (厳密には、"sheaf Mor"と呼ぶべきだが"sheaf Hom"という用語はあまりに定着してしまった。) これは通常のHomと同じように第一引数に反変で、第二引数に共変な関手である。 (訳者補足:ここのHOMは原文ではHomの花文字フォント) /* ・この概念はよくかみくだいて考えると、群の層から群の層への層準同型がなす群がなす層、だから実は結構ややこしい。 ・右辺をMor(F(U),G(U))で定義してしまうと層にならないことを実感する例として、例えばF,Gが正則関数の層で、Uが穴のあいた円板とするとき、偏角を分割してUを被覆すればそれぞれの開集合ではlog(z)な元が存在するが、U全体ではlog(z)な元はMor(F(U),G(U))には存在しない。 ・[Stacks] ではこの概念にinternal homという言葉が使われている。 https://stacks.math.columbia.edu/tag/01CM この概念は、層の視点のうち、2.5.Dの部分層を貼り合わせる視点と最も関係深い。 */ 警告:層のHOMは茎を取る操作と可換でない。 HOM(F,G)_p は Hom(F_p,G_p)と同型でない。 (少なくとも片方からもう片方への写像は存在する。それはどちらか?) /* HOM(F,G)_pは{pを含む開集合UでのF(U)→G(U)}を、制限して同じになるもので割ったもの Hom(F_p,G_p)はF(U)の余極限からG(U)の余極限への射 Hom(F_p,G_p)ではpを含む開集合の情報を忘れてしまっている。 HOM(F,G)_p → Hom(F_p,G_p) への忘却的な写像が存在する。と考えた。 */ /* しかし1.6.G のように、FがO_X-加群として有限表現を持つO_X-加群の層のとき HOM(F,G)_p と Hom(F_p,G_p)は同型になる。 1.6.Gの後ろにある反例が(Specを使って)この問題に対する反例にもなる。 これはまた13.7の準連接層と連接層の喜ばしい性質の最初に登場する。 */ 2.3.D (現実味の確認) (a)F∈Set_Xのとき一点を値に持つ定数層{p}_からFへのHOMはFと同じである (b)F∈Ab_Xのとき定数層Z_からFへのHOMはFと同じである (c)F∈Mod_OXのときO_XからFへのHOMはFと同じである 2.3.5 アーベル群の前層はアーベル圏をなす。 ・・ 前層の核を、ker_preφ(U) = kerφ(U)で定める 2.3.E これは前層となっている 前層の余核を同様に定める 2.3.F これは前層の圏での余核の普遍性を満たす ・・ 2.3.G アーベル群の前層の切断をとる関手は完全関手である。 2.3.H 次の意味で逆が成り立つ: 前層の列 0→F1→F2→..→Fn→0 が完全であることは すべての開集合Uに対して アーベル群の列0→F1(U)→F2(U)→..→Fn(U)→0 が完全であることと同値である。 しかし我々は前層より層に興味がある。アーベル群の層もアーベル圏をなすことをすぐに見る。しかしそこには層化という概念が生じる。層化は多くの問題への答えを与える。我々はただ、どのような問題があるかまだ十分に認識していない。 2.3.I φ:F→Gを層の準同型とする。前層としての核は層でもある。 2.3.J 重要な練習:前層としての余核は層とは限らない。 Xを複素平面Cに古典的な位相を入れたものとして、Z_を定数層とする。Fをlog(F)が正則関数となるような正則関数からなる前層とする。前層としての「指数完全列」0→Z_→O_X→F→0 を描写し、Fが層ではないことを示せ。ここでZ_→O_Xは自然な埋め込みで、O_X→Fはf->exp(2πif)である。(完全であることを確実に確認せよ。) この例は2.4.10で再び現れる。 /* 右側の完全性:f∈F(U)とする。Fの定義より、exp(g)=f となるg∈O_Xが存在する。 真ん中の完全性:exp(g)=0とする。gは局所的に定数である。 層でないこと:原点を除いた集合全体で定義された正則関数fで、exp(g)=z となるgは存在しないが、2.2.Bと同様に偏角を分割すれば存在する。貼り合わせ公理が成り立たない。 */ --- 2.4 茎で決まる性質、層化 2.4.A 重要な簡単な練習 層の切断は、茎で決定される:F(U)→Π[p∈U]F_p は単射である (一致の公理は必要だが、貼り合わせ公理は不要である) /* f1,f2∈F(U)のすべての芽が一致するならば切断として等しいことを示す。 点pでの芽がともにf_pであるとする。芽の定義により、pの開近傍U[p]があってf1とf2のU[p]への制限は一致する。 pを動かしていけば開集合族U[p]たちは位相空間を被覆する。f1,f2はこの開集合属のすべての元で一致するから一致の公理によりf1=f2である。 */ 2.4.2 台supportの定義:芽が0でない所 2.4.B Supp(s)はXの閉集合である 2.4.3 重要な定義:一貫性のある芽 Π[p∈U]F_p の元 Π[p∈U]s_p が「一貫性のある芽からなる」とは: それぞれのs_pに対してpが属する開集合U_pがあってsの代表元s~∈F(U_p)をとって、U_pに属する他の点qに対して、s~のqでの芽s~_qが、s_qと一致するようにできることを言う。 言い換えると、Uの開被覆Uiと切断fi∈F(Ui)の族があって、Uiに属する点pについてfiのpでの芽fi_pがs_pと一致するようにできることを言う。 明らかに、切断F(U)の元自体は一貫性のある芽たちを与える。 2.4.C 重要な練習 一貫性のある芽によって、層の切断が定まる(ヒント:貼り合わせ公理を使う) すなわち一貫性のある芽が、2.4.Aの F(U)→Π[p∈U]F_p の像をなす /* 丁寧に書いてみる。 すべての点pでの芽がs_pと与えられていて一貫性のある芽であるとする。s_pたちが一貫性のある芽であることが意味することには、pの開近傍U[p]があって、s_U[p]∈F(U[p])があって、すべてのq∈U[p]に対して s_U[p]のqでの芽が与えられたs_qに一致するようにできるのであった。 pを動かしていけば開集合族U[p]たちは全体を被覆する。s_U[p]∈F(U[p])が貼り合わさって1つの大域切断を与えることを示したい。 貼り合わせ公理を適用するには、s_U[p]∈F(U[p])とs_U[p']∈F(U[p'])について、それらの交わりへの制限で一致することを示す必要がある。しかしこれは2.4.Aより従う:任意のq∈U[p]∩U[p']に対して、s_U[p]とs_U[p']のqでの芽は一致するからである。(どちらも与えられたs_qに一致するから。) */ 2.4.4 指摘 これは層(英語ではsheaf、藁束という意味)という言葉の由来となる側面である。すなわち、茎を適切に束ねたものである。 2.4.D 層の射は茎で定まる φ1とφ2を集合の前層Fから、集合の層Gへの準同型とする。 それらが誘導する茎の準同型がすべて等しいなら、φ1=φ2である。 ヒント:次の図式を考えよ: F(U)→G(U) ↓ ↓ ΠF_p→ΠG_p /* (水平な射はφ1,φ2に対してそれぞれ定義される) F(U)→ΠF_p→ΠG_pと進む経路は、仮定よりφ1とφ2で同じ結果になる。 可換性より、F(U)→G(U)→ΠG_pと進む経路も、φ1とφ2で同じ結果になる。 一方で、G(U)→ΠG_pは2.4.Aより単射であったことからφ1=φ2が従う。 */ 2.4.E 技巧的な練習:層の同型は茎で定まる 集合の層の間の準同型について、それが同型であることは、すべての茎で同型であることと同値である。 ヒント:2.4.Dのヒントの図式を使うと良い。単射性を示したら、全射性を示すのには2.4.C、あるいは何かしらの方法で貼り合わせ性を使うだろう。これはより微妙な問題となるだろう。 (注意:これは2つの層がすべての点で同型な茎を持つならば同型と主張しているわけではない。) /* φ:F→Gで誘導される茎の射φ_p:F_p→G_pがすべてのpで同型(全単射)だとする。 F(U)→ΠF_p→ΠG_pは1つ目が単射(2.4.A)で2つ目が同型(仮定)だから単射。可換性より、F(U)→G(U)→ΠG_pが単射だからF(U)→G(U)が単射だと言える。 次にt∈G(U)を与えられたとしてtに移るs∈F(U)の存在を示したい。Πt_p∈ΠG_pは一貫性のある芽である。ここで、茎の同型でs_p∈F_pがt_p∈G_pに移るとした時にΠs_pが一貫性のある芽であることを示したい。そうすれば2.4.CによりΠs_pがF(U)→ΠF_pの像に居る。すなわちs∈F(U)があってsがF(U)→ΠF_p→ΠG_pでΠt_pに移る。よって先の単射性よりsはtに移る。 切断の射はFからGに向かう方にしか定義されていないのが厄介な所である。 Πs_pが一貫性のある芽ならば、2.4.3の定義の代表元s~をφで移すことで、Πt_pも一貫性のある芽であることが言える。しかしG(U)からF(U)への射は定義されていないので逆には戻れない。そうしたら、「Πs_pが一貫性のある芽でなければ、Πt_pも一貫性のある芽でない」を示したら良いのではないか。 Πs_pが一貫性のある芽でないとする。ある点pがあって、点pでの芽がs_pとのような近傍Uの切断s_U∈F(U)をどのようにとっても、q∈Uがあって、s_Uのqでの芽が与えられたs_qと一致しない。 それなのにt_pが一貫性のある芽だとする:t_V∈G(V)があって、任意のq∈Vに対してt_Vのqでの芽が与えられたt_qに一致する。 φ(s_U)の芽がt_pに一致するとしたら、ある開集合W⊂Uがあって、任意のq∈Wに対してφ(s_U)の芽がt_qに一致する。ということはs_Uのqでの芽はφによってt_qに移るような元だから、全単射性よりs_qに定まる。ところがこれは先の状況に矛盾する。 */ 2.4.A,D,Eは前層では成り立たない (ヒント:この種の反例を見つけるヒント:離散した2点からなる位相空間を考えると良い) 2.4.5 層化 すべての層は前層でもある。1.5.3の群化で、半群から"最も近い"群を作ったように、層化は普遍性を使って、前層から"最も近い"層を作る。(2.2.Bや2.3.Jの例を思い浮かべ続けておくと良いかもしれない。) (心に留めておくのに良い例としては、古典的な位相での複素平面での平方根を持つ正則関数のなす前層(2.2.B)の層化かもしれない。2.4.10の例も良いかもしれない。) /**/ 2.4.6 定義 Fを前層とする。層化F→F^shは、任意の層Gと前層の準同型F→Gに対して、これがF→F^sh→Gと一意的に分解するようなものである。 我々はその存在を示す必要がある。次の2つの練習では(構成は使わずに)存在だけ仮定せよ。 2.4.G 層化は一意的な同型を除いて一意的である。特にFが層ならば層化は恒等写像である。 (読者はこの種の議論にはもう慣れていることだろう) /* φ:F→F^sh, φ':F→F^sh'を2つの層化とする。 F^shの普遍性よりφ'は F→F^sh→F^sh'と一意的に分解する F^sh'の普遍性よりφは F→F^sh'→F^shと一意的に分解する 2つの右側の矢印が一意的な同型を与える。 */ 2.4.H 簡単な練習:層化はX上の前層の圏からX上の層の圏への関手である。 /* F→G ↓↓ F^sh→G^sh が可換であることを確認するが、これは茎ごとに見ればすぐに分かる。 */ 2.4.7 構成 前層Fに対して、F^sh(U)を、U上での一貫性のある芽の集まりとして定義する: 2.4.I 簡単な練習:F^shは層をなす /* ・一致の公理はすぐに分かる。 ・F^sh(U)の元fは、U上の一貫性のある芽s_pの集まりである。 ・F^shのpでの茎は、F_pと一致し、上記のfのpにおける芽はfを構成するs_pそのものである。 ・U上の一貫性のある芽Πs_pと、V上の一貫性のある芽Πt_pがあって、p∈U∩Vでs_pと一致する時、それらを合わせたものはU∪V上の一貫性のある芽となる。(2つの場合で記述したが無限個の場合でも成り立つ) ・これが貼り合わせの公理の内容に相当する。 */ 2.4.J 簡単な練習 前層の準同型 f:F→F^sh を描写せよ /* 2.4.3にあるようにF(U)の元自体は一貫性のある芽たちを与える */ 2.4.K f:F→F^shが普遍性を満たすことを示せ(読者がおそれるより易しいと思う) /* φ:F→Gを前層から層への射として F→F^sh→G と分解したい。F(U)に由来するF^sh(U)の元の行き先をF(U)の行き先と定義しなければならない。残りの課題は2つある。 ・well-defined性の確認: x1,x2∈F(U) が同じ x'∈F^sh(U) を誘導するならば、φ(x1)=φ(x2)である。 なぜならx1とx2の各点での芽は等しく、従ってφ(x1)とφ(x2)の各点での芽は等しい。Gは層だから一致の公理よりφ(x1)=φ(x2)が言える(2.4.A)。 ・F(U)に由来しないx'∈F^sh(U)の元の行き先が一意的に定まること: F^shの定義より開被覆{Uk}があって、x_Uk∈F(Uk)に由来するx'_Ukを貼り合わせたものがx'とできる(2.4.3の後半の描写)。φによるx_Ukの像φ(x_Uk)∈G(Uk)は貼り合わさってG(U)の切断を定める。層準同型の定義(制限の可換性)よりx'の行き先はこれでなければならない。 */ 2.4.L 有用な練習:圏論好きだけのためではない:層化は、層を前層と見なす忘却関手の左随伴関手である。これは難しくない。おおまかには普遍性の言い換えである。 (そういうわけで核のような極限は忘却関手で保たれるので層化する必要が無かった) (逆に余核やあとで登場するテンソル積(2.6.J)は余極限なので層化する必要がある) 2.4.M 層化が誘導する茎の射は同型である。 (構成を見ても良いし、2.7.3と結び付けても良い。) 読者は現実味の確認として、定数前層の層化は定数層である(2.2.10)ことを見ても良い。 2.4.8(*) 指摘 2.2.11のエタール空間の視点は層化に関して別の視点を与え、読者によってはこの視点を好むかもしれない。中心的な考えは同じである。Fが前層のとき、F'をそのエタール空間とする。F'→Xの切断(2.2.G(a)の意味)がFの層化である。2.2.Eはこの例として解釈できる。 2.4.9 部分層と商層 2.4.N 部分層:集合の層の準同型φ:F→Gについて、次は同値である: (a) φは層の圏でのモノ射である (b) すべてのp∈Xでφ_p:F_p→G_pは単射である (c) すべての開集合U⊂Xでφ(U):F(U)→G(U)は単射である ヒント:(b)⇒(a)には2.4.Dが使える。(a)⇒(c)には"指示層"(Uに包含される開集合でのみ空でない切断を持つ)を考えると良い。 このときFはGの部分層であると言う。(φは時に省略される) 2.4.O 商層:集合の層の準同型φ:F→Gについて、次は同値である: (a) φは層の圏でのエピ射である (b) すべてのp∈Xでφ_p:F_p→G_pは全射である ヒント:摩天楼層を使うと良いかもしれない このときGはFの商層であると言う。 どちらの定義も集合の層以外の場合にも同様に定義される。2.4.Oのほうには2.4.Nの(c)に相当するものがないことに注意せよ。(開集合の基で全射なら層の射としてエピ射(2.5.E)という事実はある。) 2.4.10 (2.3.Jの例) 複素平面Cに古典的な位相で正則関数の層O_Xと可逆な正則関数の層O_X*を考える。0→Z_→O_X→O_X*→1 がある。 2.4.P exp:O_X→O_X*が、O_X*を商層として記述することを示せ。この写像が全射でない開集合を見つけよ。 /* 原点を除いた集合全体で定義された正則関数fで、exp(f)=z となるfは存在しない */ これは、層の準同型における"全射"が何を意味するかについての素晴らしい例である。原点以外ではどこでも消えない関数は局所的には対数関数を持つが、大域的に対数関数を持つとは限らないという状況である。 --- 2.5 開集合の基による層の復元 層は考える対象としては自然であるが、手を動かすには難しい。我々は一致の公理と貼り合わせ公理が成り立つことを好むが、それらのせいで物事を証明するのが前層の場合よりややこしい。我々は一貫性のある茎という視点をすでに議論した。ここでは、もう一つ別の視点を紹介する:「開集合の基での層」である。 警告:このようにして限られた情報から層全体を理解するという方法は混乱しやすい。この部分的な情報が、全体を決定するのに十分であるという洞察を常に持ち続けておくと良い。 まず、開集合の基という概念を説明しよう。位相空間Xを考える。つまりその部分集合のうちどれが開集合であるか:開集合の族{Ui}を知っているとする。位相の基とは、開集合の族の部分族{Bi}⊂{Ui}であって、任意のUiがBjの合併であるようなものである。読者が見たことがあると思う例は、X=R^nのとき、古典的な位相が、開球Br(x){y∈R^n;|y-x|yk の引き戻しを考えると良い気がする。 */ 3.1.B π(p)=qのとき、茎の写像π#:OY_q→OX_pが誘導される。π#(mY_q)⊂mX_pを示せ。 言い換えると、qで消える関数を引き戻すとpで消えることを示せ。 (6.3で局所環つき空間の射という言葉で形式化する) /* qの開近傍上の、qで消える関数で代表されるから、引き戻せばpで消える関数の芽を与える */ 3.1.2 寄り道(2.1.2の続き) πは接空間の射を誘導する:(mX_p/mX_p^2)の双対→(mY_q/mY_q^2)の双対。これは幾何的に期待されることである。 興味深いことに余接空間の射(mY_q/mY_q^2)→(mX_p/mX_p^2)のほうが代数的には自然である。(双対を取らなくて良い) 熟練者は微分幾何の他の概念を位相空間と層の射のみを用いて解釈することを試みることができるだろう。例えばπが多様体沈めこみや埋め込みであることをどう確認できるか?(これらの代数幾何バージョンは滑らかな射や不分岐射の言葉で表される。25章で触れる。定義自体はもっと早く登場する。) 3.1.4 他の例 トポロジーに興味があるなら位相空間間の連続関数を考える。複素幾何に興味があるなら複素(解析)多様体間の正則関数を考える。スキームはこの一連の中に自然に入ってくる。 --- 3.2 集合としてのアフィンスキーム 環Aに対して、SpecA(Aのスペクトラム)というものを定義する。まずは集合として定義するが、すぐに位相を導入し、後には環の層を定義する。これをアフィンスキームと呼ぶ。 後でSpecAという記号は位相空間や環つき空間を表すが、ここでは単に集合を表すものとする。 3.2.1 集合としてのSpecAは、Aの素イデアルである。素イデアルpをSpecAの元として考えることを強調するのにここでは[p]という表記を使う。a∈AはSpecA上の「関数」と呼び、a mod p∈A/pZを[p]での「値」と呼ぶ。これは妙である:関数は点によって終域が違う。SpecZでは5という関数は[(2)]において値 1 mod 2 をとり、[(3)]において値2 mod 3 をとる。素イデアルpに属する環の元とは、点[p]で消える関数、点[p]を零点に持つ関数、に翻訳される。2つの関数の和や積の値は、値の和や積と一致していることに注意せよ。つまりA→A/pは環準同型である。これらの翻訳は重要である。仲良くなろう。習慣となるだろう。 /* 補足:関数は、各点において値をとるが、各点における値で決定されるものではない。 そうではなくて、一般に層の切断のように、芽で決定される。(3.2.11, 4.3.5を見よ) /* 元のpdfでは4章の始めに書いてあったのをここに持ってきた */ Aが基礎体上でx1,x2,..xnで生成されているとき、x1,x2,...,はしばしば座標と呼ばれる。n次元の空間の一部として解釈できるからである(3.2.9, 6.2.D) 3.2.2 いくらかの将来への展望 4.1.3ではSpecAの関数を、構造層の大域切断と解釈する。2.2.13の意味で。2.2.13の注意を繰り返す。我々が関数と呼ぶものを、正則関数と呼ぶ人もいる。そして、後で我々は「有理関数」を定義するが、それはpartially-definedな関数である。「関数の値」という言葉は、局所環つき空間での関数の値として解釈される(4.3.6) 3.2.3 例 [例1]:A_C^1 = Spec C[x] 複素アフィン直線 まずC[x]の素イデアルを見つけよう。C[x]は整域だから、0は素である。また、(x-a)も素である。しかも、剰余環が体だから極大イデアルでもある: x=aでの値をとる全射C[x]→Cの核がイデアル(x-a)であるから C[x]/(x-a)=C である。(2.1.1の例を思い出すかもしれない) 他に素イデアルが存在しないことをしめそう。 C[x]は互除法アルゴリズムを持つので、一意分解整域であることを使う。pが素イデアルとする。p≠(0)なら、f(x)∈pを0でない元で次数が最低のものとする。イデアルは1を含まないのでfは定数ではない。Cが代数閉体であることを使う。fが2次以上ならf=ghと分解される。このときg∈pまたはh∈pで次数に関する仮定に矛盾する。・・・ このようにして、A_C^1の絵を得られるし、得るべきであるし、得なければいけないかもしれない。これはSophie Germainの視点による最初の描写である:『代数は幾何的に書かれ、幾何は代数に帰着する』 それぞれの複素数に対応する古典的な点のほかに、おまけの点[(0)]がある。我々はA_C^1をほとんどCと同様に描ける:x=aに相当する所に点[(x-a)]がある。点[(0)]はどこに描けば良いのか?これは禅のようなものである。それは複素直線のどこかにあるが、どこか特定の所にはない。(0)は他のすべての素イデアルに含まれているから、ある意味ですべての点を通る直線に対応させることもできる。[(0)]はこの直線の「生成点」と呼ばれる。それは「直線の上に全体的に存在する」がそれ以上の特定はできない。(「生成点」の形式的な定義は3.6で行う。)どこにも置き場所が無いのでどこか遠くに置くこともできる。読者はA_C^1を1次元的に描いたことに気がつく。これは後でこれが1次元空間であることを思い出させるためである。次元は11章で定義されるが、代数的(あるいは複素幾何的)な意味である。 この空間を実感するために、まだ用語が定義されていないが役に立つ主張をしてみる。A_C^1上の関数は多項式である。f(x)=x^2-3x+1は関数である。[(x-1)]での値は何か?f(1)である!(x-1)で割った剰余類と考えても良いが同じことである。[(0)]での値は何か?...f(x)である。f(x)を(0)で割った剰余類は単にf(x)である。 もう少し複雑な例を挙げる。g(x)=(x-3)^3/(x-2)は有理関数であり、x=2以外で定義されている。構造層のことを知った後なら、開集合A_C^1-{2}での構造層の切断であると言える。g(x)は3で3重の零点を持ち、2で1位の極を持つと言いたい。これは12.5で可能になる。 [例2]:A_k^1 = Spec k ̄[x], (k ̄はkの代数閉体) これはk上のアフィン直線と呼ばれる。すべての議論は前の例と同様である。 よって同じ絵を使うことができ、それは結局比喩的な意図なのである。 [例3]:A_Z^1 = Spec Z これは、前の例とよく似ているのは驚くべき事実である。整数は、k ̄[x]と同様に互除法アルゴリズムを持つ一意分解整域であり、素イデアルは(0)、(p) (pは素数)である。なので例1の絵を含むすべてがそのまま適応できる。直線上に点[(2)],[(3)],[(5)],..があり、[(0)]は特定のどこかではないがどこかあるいは直線全体にある。 この空間の関数について考えよう。100はSpecZ上の関数で、[(3)]での点は1 mod 3, [(2)]での値は 0 mod 2 であり、実際には2重の零点である。27/4はSpecZ上の(2)以外の点で定義された「有理関数」で、[(2)]では2位の極を持ち[(3)]では3重の零点であると言いたいと思っている。[(5)]での値は、27×4^-1≡2×(-1)≡3 mod 5 である。(このような議論を今後徐々に正確にしていく。) [例4]: Spec k ばからしいが重要な例、ドイツ語でのベーコン(Speck) kが体のときの集合 Spec kは退屈である。1点しかない。Spec 0(零環)は空集合である。零環は素イデアルを持たないからである。 3.2.A 小さなスキームの小さな練習 (a) Spec k[ε]/(ε^2)を描写せよ。この環は二重数の環とも呼ばれ、役に立つ。εはとても小さい数だと思うと良い。とても小さいので(それ自身は0ではないが)2乗すると0になる。0でない関数で、すべての点での値は0となる最初の例がここにある。この現象は3.2.11でさらに議論する。 /*(整域でない環のスペクトラムの扱いは4.2や5.5でもまた登場する。) この環は整域ではなく(0)は素イデアルではない。(0)はイデアルとして(ε)の二乗であり、(ε)が素イデアルの1つで、そして、すべてである。 0でない定数項を持つεの多項式は可逆である。実際(a+ε)(a-ε)/a^2=1 なので集合Spec k[ε]/(ε^2)は1点である。 εという関数の、点[(ε)]での値は、ε≡0 modεである。*/ (b) Spec k[x]_(x)を描写せよ。(局所化の議論については1.3.3を見よ。) このスキームは繰り返し登場する(3.2.8, 3.4.K)。これは特になめらかな曲線の原点での断片だと思うと良いだろう。 /* これは具体的には有理関数で分母がx=0で消えないものの集合{f(x)/g(x);g(0)≠0}である。(0)と(x)は素イデアルのままだが、(x)に含まれない元はすべて可逆なので、この2つがすべてである。そういう意味で、Spec k[x]の直線(各点に相当する(x-a)と直線全体な([0]))のうち、a=0での断片というわけである。 これは局所環の例である。一般に素イデアルによる局所化は局所環を与える。(4.3.F) */ [例5]:A_R^1 = Spec R[x] 代数閉体ではない場合 素イデアルは、(0),(x-a),(x^2+ax+b)[a,b∈R, x^2+ax+bはRで既約]で尽くされる。後ろ2つが極大イデアル、すなわち剰余環が体となる。 例えば R[x]/(x-3)はRに同型で、R[x]/(x^2+1)はCに同型である。 3.2.B 重要でない練習 (x^2+ax+b)の形の素イデアルによる剰余環はいつもCに同型であることを示せ /* 平方完成でも解の公式でも */ そういうわけで、実直線には通常期待するより多くの点がある:実数に対応する点に加えて、生成点[(0)]があり、さらに複素数の共役の組(二次式の根)と解釈されるような新しい点がある。この最後の種類の点は、生成点よりも、実数に対応する点のほうの仲間である。なのでA_R^1の絵を、複素数平面で、共役を実軸に沿って結びつけるような絵として描ける。ガロア共役な点が結び付けられているのが重要な点である。 この空間の関数を調べよう。関数f(x)=x^3-1を考える。[(x-2)]での値は7である。あるいは7 mod (x-2)である。[(x^2+1)]ではどうだろうか。x^3-1≡-x-1 mod (x^2+1) である。これは -i-1 と解釈すると良いかもしれない。 この例から分かる精神は、我々は望むなら代数閉体以外でも議論ができるということである。それはより複雑だが、多くの気になる情報を復元できる。 3.2.C 重要な練習 A_Q^1 = Spec Q[x]を描写せよ。 (これは絵を描くのはより難しい。しかし雑な落書きのようなもので良い。) /* 有理数点に加えて、すべての代数的な数をQ上共役で結びつけた点、それから生成点がある */ [例6]:A_F_p^1 = Spec F_p[x] 有限体上のアフィン直線 今までの例と同様にF_p[x]はユークリッド整域であり、素イデアルは(0)と、((f(x)) [f(x)はF_p係数既約多項式]である。f(x)の次数は何でも良い。既約多項式は、F_pの代数閉包のガロア共役を同一視したような集合に対応する。 Spec F_p[x]はF_pの元に相当するp個の点を持っているが、もっとたくさん(無限個ある。次の練習。)の点を持っていることに注意せよ。そのためこの空間は単にp個の点があるよりもずっと豊かである。例えば、多項式f(x)はF_p上のp個の点での値だけでは決定できないが、Spec F_p[x]のすべての点での値を使えば決定できる。 (3.1.1で言及し3.2.Aで見たように、これはすべてのスキームで正しいわけではない) /* f(x)の生成点[(0)]での値はf(x)自身だから、この点の情報だけでf(x)は決定できる。そうではなくて、生成点以外のすべての点の情報から、f(x)を決定できる、というのが言いたいことだと思う。すべての既約多項式で割った余りから、多項式を決定できるということである。*/ この例は幾何だけに興味がある読者も考慮に入れるべきである。この直観は後に幾何的な状況で現れる。また、代数閉体だけに興味がある読者も、代数閉体でない体を考えることが必要である。(例えば関数体C(x)は代数閉体ではない) 3.2.D kが体なら、Spec k[x]は無限個の点を持つことを示せ。ヒントはユークリッドによる素数の無限性の証明である。 /* k[x]は一意分解整域であり、素イデアルは、既約多項式に対応する。 有限個だと仮定して、すべての積をとって1を足した多項式を考えれば良い。*/ [例7]:A_C^2 = Spec C[x,y] (複素アフィン平面。[例2]のようにCは任意の代数閉体で置き換えても同様に議論できる。) 残念ながら、C[x,y]は単項イデアル整域ではない。(x,y)は単項イデアルではない。我々はいくつかの素イデアルを素早く指名できるだろう。(0)は今までの(0)と同様の意味合いを持つ。イデアル(x-2,y-3)は素でかつ極大である。剰余環C[x,y]/(x-2,y-3)は写像f(x,y)->f(2,3)によりCと同型である。より一般にイデアル(x-a,y-b)は任意の(a,b)∈C^2について素である。それから、f(x,y)が既約多項式ならばイデアル(f(x,y))も素である。(例えばy-x^2とかy^2-x^3とか) 3.2.E これでC[x,y]の素イデアルをすべて尽くしたことを示せ。 (ヒント:pが単項イデアルでない素イデアルとする。f(x,y),g(x,y)∈pで、共通因子を持たないものが存在する。ユークリッド整域C(x)[y]での互除法アルゴリズムを考えることで、h(x)∈(f(x,y),g(x,y))⊂pがとれることを示せ。単項性を使い、h(x)の1つの因子は一次式であることを示せ。同様にしてあるyの一次式がpに属することを示せ) /* Af+Bg=h'となるA,B∈C(x)[y],h∈C(x)が存在する。分母を払って af+bg=ch' となる a,b∈C[x,y], c∈C[x]が存在する。ch'がhとして使える。 Cが代数閉体であるから1次式の因子がとれる。*/ ではA_C^2の絵を描いてみよう。極大イデアルは古典的な点に対応する。つまり[(x-a,y-b)]は点(a,b)∈C^2に対応する。([0])は、すべての古典的な点の裏側に住んでいる。それはどこかにあるが特定の場所にはないやつである。例えば[(0)]は放物線y=x^2の上にあるわけではない。 点[(y-x^2)]は、放物線y=x^2の上にある。しかしその特定の点にはない。 この絵から、次元というものを既に考えることができる。(x-a,y-b)は次元0で、(f(x,y))は次元1で、(0)は次元2だと想像できる。(ここでは複素次元あるいは代数的次元である。実次元で考えると倍になってしまう。) 次元については11章まで正確には定義しないが、いつでも心に留めておいて良い。 極大イデアルが最小の点に対応することにも注意せよ。小さいイデアルほど、大きい点に対応する。あるイデアルが他のイデアルを包含することは、点においては逆の包含に対応する。これは後に位相の言葉でより正確に議論できる。包含関係が入れ替わるのはやや紛らわしく、これは今後用語を正確にしても、紛らわしい。 [例8]:A_C^n = Spec C[x1,..,xn] これは例7の明らかな一般化である。より一般には係数環Cを任意の環に変えることができる。(厳密な人へ:この記法はn個の座標関数の情報を暗黙のうちに定めている。)n=3の場合を考えてみよう。興味は純粋に代数的な質問に帰着する:C[x,y,z]の素イデアルは何か? 前の例のように、イデアル(x-a,y-b,z-c)は素でありかつ極大である。これ以外に極大イデアルは存在しないことを示す定理がある: 3.2.4 ヒルベルトの弱零点定理 kが代数閉体なら、k[x1,..,xn]の極大イデアルは、(x1-a1,..,xn-an)の形をしている。 もう少し強いバージョンがある 3.2.5 ヒルベルトの零点定理 kが体なら、k[x1,..,xn]の極大イデアルによる剰余体はkの有限次拡大である。 言い換えると、環として有限生成なkの拡大体は、体として有限次拡大である。 この定理は7.4.3で証明される。11.2.Bで別の証明が与えられる。 (消去理論を使う方法と、ネーターの正規化を使う方法) 3.2.F 練習 3.2.5が3.2.4を含意することを示せ。 /* k[x1,..,xn]のイデアルの剰余環は有限生成である。 kが代数閉体ならkの有限次拡大はk自身のみである。 よってpが極大イデアルなら、k[x1,..,xn]/pはkに同型である。 つまり x1≡a1,..,xn≡an (mod p) となるa1,..,an∈kが存在する。*/ 次の事実は零点定理の仲間で有用である 3.2.G 練習(ヒルベルトの零点定理を使わない) 有限なk-代数(k-ベクトル空間として有限次元)な整域Aは体である。 (ヒント:x∈A,x≠0によるx倍写像A→Aの同型を利用せよ。) /* x倍写像は(環準同型ではないが)k-ベクトル空間の線形写像である。 Aが整域ならば、この写像の核は0のみである。 線形写像の次元定理により像はA全体である。特にx倍して1になる元がある。*/ よって零点定理を組み合わせることで、k[x1,..,xn]の素イデアルで、有限(k-ベクトル空間として有限次元)な剰余環を伴うものは極大イデアルだと分かる。これは覚えておく価値がある。 例8に戻る。C[x,y,z]には他の素イデアルもある。(0)は「3次元的な」点である。既約な多項式f(x,y,z)で生成されるイデアルもある。これはf=0の超曲面に対応し、従って2次元的である。これですべてではない!1次元的なものを逃している。例えば(x,y)という素イデアルがある。これは絵で言うとx=y=0で表される場所、すなわちz軸である。剰余環C[x,y,z]/(x,y)は整域C[z]に同型である。(そしてこれはz軸上の関数として解釈されるべきである。)1次元的な点はたくさんあり、分類するのはもはや難しい。ある意味で、それは既約な曲線として考えることができる。こうして、代数的な質問は幾何的な内容となっている。 A_Q^1の、Q[x]の極大イデアルに対応する点(3.6.8で閉点という言葉を導入する)が、Q ̄でガロア共役が結び付けられた点として対応するという事実は、A_Q^nに一般化される。例えば、A_Q^2においては、点(√2,√2)は(-√2,-√2)と結び付けられる:(√2,-√2)ではない。これがどうなっているのかを次の練習で考察する。 3.2.H 練習 この点に対応するQ[x,y]の極大イデアルを描写せよ。点(√2,-√2)に対応するほうの極大イデアルも描写せよ。剰余体はそれぞれ何か。 /* それぞれ、(x^2-2,y^2-2,x-y)と(x^2-2,y^2-2,x+y)である。 あるいは、(x^2-2,x-y)と(x^2-2,x+y)と書くこともできる。 剰余体はどちらも Q[x]/(x^2-2)、Q[√2]と同型である */ ガロア軌道によるA_Q^2の「閉点」の描写はは閉点でない点にも適応される。 3.2.I 重要でないが技巧的で楽しい練習 φ:C^2→A_Q^2:(z1,z2)を点(z1,z2)で消える多項式からなるQ[x,y]の素イデアルに対応させる、という写像を考える。 (a) (π,π^2)の像は何か (b)(*) φは全射であることを示せ。(この問題を解くにはまだ議論していない考えを使う必要がある。ザリスキ位相を定義した後にはC^2に古典的な位相を入れたときにφが連続であることを確認できる。この例は任意の次元に一般化できる。) /* (a) (y-x^2)であろう。他の関係が有ったらπの超越性に反する。 (b) これは難しい [1]z1,z2がともに超越数で、代数的に無関係である場合(0次元な点) [2]z1,z2がともに超越数で、代数的に無関係でない場合(1次元な点) [2]' どちらか一方だけが超越数の場合(1次元な点) [3]両方とも代数的な場合(2次元な点) という対応であり、それぞれを考察することになると思う。*/ 3.2.6 剰余環と局所化 環から新しい環を作る代表的な2つの方法であるが、これはスペクトラムの言葉で解釈できる。 3.2.7 剰余環 Spec A/I は SpecAの部分集合である。 A/Iの素イデアルと、Aの素イデアルでIを含むものの間に全単射があるのは重要な事実である。 3.2.J 欠かせない代数の練習(もし経験していなければ必須である) Aを環として剰余環A→A/Iを考える。φ^-1は上記の全単射を与え、包含関係を保つことを示せ。 /* 素イデアルの逆像は素イデアルということ(定義に従って示される)、0は任意のイデアルに属するので、イデアルの逆像は0の逆像であるIを含むこと、包含関係は逆像で保たれること、による。 */ 特に重要で典型的な例はアフィン多様体である。 Aを有限生成C-代数として生成元をx1,..,xnとして関係式をf1,..,frとする。 Spec Aの古典的な点は、自然にA_C^nの部分集合と解釈できる。 どの点がSpec Aに属するかを知るには方程式f1=..=fr=0を解けば良い。 例えば Spec C[x,y,z]/(x^2+y^2-z^2)は絵にすると、円錐である。 x^2+y^2-z^2=0の解x=a,y=b,z=cに対して、(x-a,y-b,z-c)がAの素イデアルとなる。 /* そうでないa,b,cだとどうなるか。P(x-a)+Q(y-b)+R(z-c)+S(x^2+y^2-z^2)=1 となる多項式P,Q,R,Sが存在し、(x-a,y-b,z-c)はA全体を生成してしまう。*/ 3.2.8 局所化 Spec S^-1.Aは、(Spec A/Iとは別のタイプな)SpecAの部分集合である。次の練習は局所化で素イデアルがどう振舞うかを示すものである: 3.2.K 欠かせない代数の練習(もし経験していなければ必須である) SをAの積閉集合とする。局所化A→S^-1.Aの素イデアルと、Aの素イデアルでSと交わらないものの間に包含関係を保つ全単射があることを描写せよ。 /* やはり素イデアルの逆像をとる。Sの元は局所化で可逆になるので、逆像がSと交わるようなイデアルを作ろうとするとS^-1A全体になってしまう。 */ 1.3.3で触れた特に2つの重要な積閉集合による局所化を思い出そう:1つ目はS={1,f,f^2,...}であった。典型的な例としてA=C[x,y],f=y-x^2である。もう1つは、素イデアルpの補集合であった。典型的な例はA=C[x,y],p=(x,y)である。 S={1,f,f^2,...}のとき、S^-1.Aの素イデアルは、fを含まない素イデアルで、「fが消えない」点に対応する。(位相を入れた後、3.5でこれを基本開集合と呼ぶ。) 従ってSpec C[x,y]_fを描くには、アフィン平面を描き、放物線y=x^2 -- 具体的には古典的な点[(x-a,y-a^2)]、および新しい種類の点[(y-x^2)] -- を捨て去れば良い。 零因子を可逆化する局所化について考えるのは最初混乱しやすい。練習1.3.CのようにA→S^-1Aが単射でないからである。幾何的直観が役に立つかもしれない。A=C[x,y]/(xy)でf=xの場合を考えよう。局所化A_fは何か。Spec Aは、平面の2つの座標軸の合併に「一致」する。{1,f,f^2,..}による局所化は、関数xが消える所を捨て去ることを意味する。すなわちx軸から原点を除いたものが残り、それはSpec C[x]_xであることを期待する。 3.2.L 練習 同型 (C[x,y]/(xy))_x = C[x]_x を示せ。 (左辺のyが右辺では0にいくことが分かるだろう) /* ・丁寧に局所化の定義に戻る視点では: ad-bc=0 のとき (a,b)〜(c,d)を同値とみなし、(a,1)をaと同一視するのであった。形式的にはこれはa/b=c/d, a/1=aとイメージすれば良かった。 y=(y,1)〜(xy,x)=(0,x)〜(0,1)=0 である。 ・普遍性の視点、特に、xは局所化によって可逆元になる、という視点では: 局所化C[x,y]/(xy)→(C[x,y]/(xy))_xをφと名付けると、φ(x)は可逆元である。φ(x)φ(y)=0ならばφ(y)=0でなくてはならない。もちろんφは環準同型だからφ(x)φ(y)=φ(xy)=0である。 */ 次に、Sが素イデアルpの補集合のとき、S^-1.Aの素イデアルはpに含まれる素イデアルであった。我々の例A=C[x,y], p=(x,y)ではS^-1Aの素イデアルは原点を通るものたちに対応した。すなわち0次元的な点(x,y)、2次元的な点(0)、それから1次元的な点(f(x,y)), f(0,0)=0, すなわち原点を通る既約曲線。これらを、原点の近くの破片として考えると良い。すなわち原点で見えない点は捨てられている。 もう1つの例はA=k[x],p=(x), より一般的にpが極大イデアルのときである。A_pは2つの素イデアルしか持たないのであった(3.2.A(b))。これは、「滑らかな曲線」の芽として、1つの素イデアルは古典的な点、もう1つの点は曲線の生成点として見るべきものである。これは離散付値環の例であり、実際すべての離散付値環はこtのように視覚化されるべきである。そのことについては12.5で議論する。そのときまでには「滑らか」と「曲線」の用語の使用を正当化しておく。 (現実味の確認:SpecZの(2)と(0)での局所化の視覚化を試みよ。それらの視覚化はどう異なるか?) 3.2.9 重要な事実 環準同型は、スペクトラム間の写像を誘導する。 3.2.M 重要な練習 素イデアルの逆像は素イデアルとなる。 従って環準同型B→Aは、反対方向の写像SpecA→SpecBを誘導する。これは環の圏から集合の圏への反変関手である:2つの環準同型の合成が誘導する写像は、それぞれの誘導する写像の合成に一致する。 3.2.N 簡単な練習(現実味の確認) (a)I⊂Bを環のイデアルとする。写像Spec B/I→SpecB は3.2.7に示されたものである。 (b)S⊂Bを積閉集合とする。SpecS^-1.B→SpecBは3.2.8に示されたものである。 3.2.10 明示的な例 アフィン複素多様体の場合、式による写像と、環準同型の関係は極めて直接的であった。例えば放物線b=a^2からC^3のy=x^2とz=y^2で切り取られる曲線への写像を考える。点(a,b)が点(a,b,b^2)に送られるとする。我々の新しい言葉では、 環準同型 C[x,y,z]/(y-x^2,z-y^2)→C[a,b]/(b-a^2) [x→a, y→b, z→b^2] で与えられる写像 SpecC[a,b]/(b-a^2) → Spec C[x,y,z]/(y-x^2,z-y^2) を考えていることになる。この考えがよく分からないなら、次の2つの練習をする価値がある--最初は混乱しやすいが、何をやっているかが分かれば、それは視界を開く(やがて自明となる)ものである。 3.2.O 重要な練習(具体的な場合) 複素多様体の写像 C→C; x→y=x^2 を考える。定義域をx軸、終域をy軸と考える。そうすると、これはxy平面の放物線y=x^2のy軸への射影として描写できる。 対応する環準同型を解釈せよ。上記の定義に基づいて、a∈C上の逆像(ファイバー)は±√a∈Cであることを確認せよ。 (より高度な例は9.3.3に登場する。注意:そちらではxとyの役割を逆にしている) /* k[y]→k[x,y]/(y-x^2)→k[x] (y-a)の像が属する素イデアルは(x±√a)ということ */ 3.2.P 重要な練習(3.2.10の例の一般化) 環準同型φ:k[y1,..yn]→k[x1,..,xm]が多項式たちyi→fi(x1,..,xm)で与えられている。 (a)φは Spec k[x1,..xm]/I → Spec k[y1,..,yn]/J を誘導することを示せ。ただしイデアルI,Jはφ(J)⊂Iを満たす。(まずI=0,J=0の場合を最初に考える。実はここでは単にA/IとB/Jとして考えれば良い。) (b)この写像が、点(a1,..,am)∈k^mを点(f1,..,fn)に移すことを示せ。 /* (a) 環準同型B/J→A/φ(J)→A/Iが誘導されるから。 (b) 素イデアルp=(x1-a1,..,xm-am)の逆像が、q=(y1-f1(a1,..,am),..,yn-fn(a1,..,am))であることを主張している。 yiにfiを代入したときの多項式をF(x1,..,xn)とする。 G(y1,..,yn) ≡ F(x1,..,xn) (mod (y1-f1,..,yn-fn)) という状況である: G(y1,..,yn) ≡ F(a1,..,an) (mod q) という状況である: φ(G(y1,..,yn))∈pならばF(a1,..,an)=0なのでG(y1,..,yn)∈qということになる。 */ 3.2.Q 練習 A_Z^n の描写 環準同型Z→Z[x1,..,xn]に対応するスキームの射π:A_Z^n→SpecZを考える。 pを素数として、[p]上のファイバーπ^-1([p])と A_(F_p)^n の間に全単射があることを記述せよ。 (その集合全体を記述する必要はない!というよりそれは無理である。) ([(0)]上のファイバーをA_k^nとして解釈できるか?) /* 例えば(x^2+1,7)というZ[x]の素イデアルは、F_7[x]での素イデアル(x^2+1)に対応する。 例えば(x^2+1,5)はZ[x]の素イデアルではないし、そういうとき(x^2+1)はF_5[x]の素イデアルでもない。 [(0)]上のファイバーとは(x^2+1)のような、すなわち剰余体が標数0となるようなイデアルで、 これはQ[x]の素イデアルに対応する。 */ 3.2.11 関数は点での値では決まらない:冪零元 直観に反するとても重大なことがある。 すべての点で消える関数は0とは限らない 二重数の例(3.2.A(a)) k[ε]/(ε^2);ε≠0, ε^2=0, があった。 これを一般化する: 3.2.R 練習 (a) Iが冪零なイデアルならば、3.2.JのSpecB/I→SpecBは全単射である。 すなわち冪零元は底集合に影響しない。 (3.4.5で、位相にも影響しないことを見る。その違いは、構造層に現れる。) (b) 環の冪零元全体はイデアルをなす。 従って、冪零元はすべての素イデアルの交わりにある。 実は逆も成り立つ 3.2.12 定理 冪零元全体は、すべての素イデアルの交わりと一致する。 幾何的視点:SpecAの関数がすべての点での値が0になることは冪零であることと同値である。 3.2.S 練習 もし読者がこの定理を知らないなら、調べよ、あるいはより良いことには自身で証明せよ。 (ヒント:ツォルンの補題により、Aの任意の真のイデアルは極大イデアルに含まれる。 さらなるヒント:xが冪零でないとする。1≠0を示すことで局所化A_xが零環でないことを示せ。) 3.2.13 関数は点での値では決まらないことは戸惑う事実だが、我々はこの直観がいつ失敗するのかを正確に特定した:2つの関数は、差が冪零の場合に限り点での値が同一になる。従って、0以外の冪零元がなければ、関数の値は点での値で決定される。そのような状況を、我々は「被約」と呼ぶ。 3.2.T 楽しいが重要でない練習 f(x)∈k[x]を考える。k[x,ε]/(ε^2)で考える。f(x+ε)は「何」か?(例を観察し証明せよ) これは、冪零元は微分の情報を定義するのに役に立つという伏線である(21章) --- 3.3 スキームを見る(1):生成点 -- 絵は厳密ではないといつも警告されてきた:これははったりだと言われることなく、常に人々を恐れさせてきた。いくつかの絵はもちろん厳密でない。しかし、私はほとんどの絵は厳密であると言いたい(私自身は可能な限り絵を使う) -- J.E.Littlewood 人々は何年も前から平面上にx^2+y^2=1を描写してきた。そして今SpecZをどう描写すれば良いかが分かった。我々が環を(Specという関手を通して)幾何的対象として理解したいので、幾何的な視点を発達させたい。そのためには、既になじみがある幾何的な空間に対する直観を拡張するような絵を思い浮かべるのが重要である。我々はこれから、スキームがどう見えるべきか、考察を進める。このセクションはこれまでの考察の要約である。 数学者のなかには絵を描かずに完全に代数的に考えることを好む者もいる。それは芸術で、科学ではないという事実に悩む人もいる。そして、手を振ってごまかすような議論の仕方は、厳密な理論の発展には決して使われない。これらの理由で、このセクションを飛ばしたい人がいるかもしれない。しかし、正しい絵を思い浮かべることは、何が正しいくてどう証明するかを理解するのに大いに役に立つ。 出発点は、アフィン複素多様体の例である。(C上有限個の変数による方程式で切断されるものである。)もう少し一般的には、任意の代数閉体上の例である。まず直観的な言葉から始めて(「古典的な点」は期待通りに振舞う)、それから2つの新しい、ややこしい特徴を指摘する。(生成点と、非被約な振舞いである。)これによりSpecZのような古典的でない空間に描写を拡張できる。 ヒルベルトの弱零点定理により、「古典的な点」はスキームの点としても存在し、これは任意の代数閉体で成り立つ。代数閉体でない場合は古典的な点はガロア共役によりいくつか張りあわさる(3.2における例5の実アフィン直線、例6のF_p上のアフィン直線)。これがヒルベルトの零点定理の幾何的解釈である。 しかし、スキームにはもっと他の点がある。それらは「既約」な(代数的)「閉」集合に「対応する」ことを覚えておくと良い。複素アフィン平面(例7)を思い出そう。1つには、既約な多項式それぞれ、もう1つには、平面全体があった。「閉」の意味は、次のセクションでザリスキ位相を定義することで正確になる。「既約」の意味は、既に想像できるかもしれない:3.6で明らかにする。「対応する」の意味は、それぞれの既約閉集合を支配する生成点があるということである。これも3.6で定義される。生成点は、その閉集合の他のどの点とも異なる新しい点である。(この対応はアフィンスキームでは3.7.E, 一般のスキームでは5.1.Bで描写される) 我々は、生成点をどこに描けば良いのか正確には分からない:我々はそれをどこにくっつけても良いが、それは「ほとんど至る所」にいると考えるべきである。特に、その集合の他のどの点に対しても、近くにいる。 3.2.7で、Spec A/Iの点がどのようにSpecAの部分集合と解釈されるかを見た。例えば、Spec C[x,y]/(x+y)を見るときにはこれをただ直線として見るのではなく、xy平面に住んでいるものとして見るべきである。C-多元環C[x,y]としてx,yという生成元を選択したことで既にアフィン平面への埋め込みを指定しているのである。 3.2.8で、Spec S^-1.Aの点がどのようにSpecAの部分集合と解釈されるかを見た。最も重要な2つの場合が扱われた。Spec A_fはSpec A上のfが消えない点に対応するのであった。後でこれを基本開集合として解釈する(3.5)。 pが素イデアルのとき、Spec A_pは「SpecAのpに対応する閉集合の近傍の破片」と見るべきものであった。最も単純な例はp=(x)⊂Spec k[x]=A(3.2.Aで扱う。3.4.Kで再度登場する。)である。 -- 「もし誰かがそれを説明出来たら」アリスは言った。(彼女は数分前からとても大きくなったので彼を遮ることに恐れは無かった)「私は6ペンス払うわ。それに意味があるとは思えないけど」 「もしそれに意味がないなら」王は言った。「それは世界を救う、もう意味を探さなくて良いからだ。」 -- Lewis Carroll --- 3.4 位相空間としてのアフィンスキーム 次に環のスペクトラムにザリスキ位相を導入しよう。この定義が初めてなら、奇妙に見えるかもしれないが、すこし経験するとこれが理にかなったものだと感じるようになるだろう。典型的な例としてA_C^2=Spec C[x,y]を考えよう。代数幾何では代数的な関数だけが許されている。x,yの多項式である。多項式が消える点の集合は、閉集合であるのが理にかなっているが、ザリスキ位相は、閉集合として、まさに多項式が消える点の集合と、他に位相の公理で要求されるものだけを考えるように定義されている。言い換えると、多項式の零点集合が閉となる最も粗い位相である。 位相はしばしば開集合で描写されるが、ここでは閉集合を使って定義した方が便利である。SがAの部分集合のとき、V(S) Sの零点集合(Vanishing set)を V(S):={[p]∈Spec A| S⊂p} で定義する。 すなわちSの元の零点集合である。これだけが閉集合(そうでなければ閉集合でない)と宣言する。 例えばV(xy,yz)⊂Spec k[x,y,z]にはどの点が属するか。閉点についてはヒルベルトの零点定理によりxy=yz=0を解けば良い。y=0またはx=z=0である:xz平面とy軸である。閉でない点としては、xz平面の生成点[(y)]とy軸の生成点[(x,z)]がある。xz平面に含まれる1次元の点もたくさんあることが想像できるだろう。 3.4.A 簡単な練習 x軸もV(xy,yz)に属することを確認せよ。 /* p=(y,z)とおいたときに xy,yz∈pである */ 一般的な状況に戻ろう。 3.4.B 簡単な練習 集合Sで生成されるイデアルを(S)と書くとV(S)=V((S))である 従ってSとしてイデアルを考えれば十分である すべてのSに対するV(S)は閉集合とすることでザリスキ位相を定義する。 3.4.C 練習 位相の公理の確認 (a)φとSpecAはともに開集合であることを示せ (b)∩V(Ii) = V(ΣIi)を示せ:開集合の合併は開集合である。 (c)V(I)∪V(J)=V(IJ)を示せ:有限個の開集合の交わりは開集合である。 (イデアルI,Jの積は、Iの元とJの元の積の、A係数有限線形結合、Σi[k]*j[k] [k=1..n]の形で表せる元の集合である。IとJの生成元の積たちで生成されるイデアルということもできる。これがイデアルをなすことを確認し、積が結合的であることをすぐ確認できるだろう。) 3.4.1 関数V(・)は包含関係を逆にする。S1⊂S2ならばV(S2)⊂V(S1)である。 S1≠S2でV(S2)=V(S1)となる場合がある。 3.4.D 定義と練習 Iをイデアルとする。根基√Iは {r∈A|r^n∈Iとなる正整数nがある}で定義される。 例えば冪零元(3.2.R)は√0である。√Iはイデアルをなす(3.2.R(b))ことを示し、V(√I)=V(I)を示せ。 √I=IのときIを根基イデアルという。√√I=√Iを確認し、素イデアルは根基イデアルであることを示せ。 ここに有用な結果が2つある。(I∩J)^2⊂IJ⊂I∩J (イデアルの積は3.4.Cで定義した。)なので、V(IJ)=V(I∩J)である。また3.4.Bと合わせるとV(S)=V((S))=V(√(S))である。 3.4.E 根基は交わりと可換である I1,..Inを環Aのイデアルとする。√(∩Ii) = ∩√Iiであることを示せ。 このことは今後暗黙に繰り返し使う。 3.4.F 練習、後で使う √IはIを含むすべての素イデアルの交わりであることを示せ(ヒント:定理3.2.12) /* 剰余環A/Iを考えれば3.2.12に帰着する。 */ 3.4.2 例 A_C^1は集合としては古典的な点(極大イデアルに対応しCと全単射がある)ともう1つの新しい点[(0)]からなることを思い出そう。A_C^1のザリスキ位相は単純である:開集合は空集合か、A_C^1から高々有限個の極大イデアルを除いた集合である。(これはほとんど補有限位相である。開集合は、古典的な点との交わりで決定されることに注意せよ。新しい点[(0)]はただ乗りしてくる。これは無害な徴候である。古典的な点のみをみると、このザリスキ位相は古典的な位相よりも粗いことに注意せよ。) 3.4.G 位相空間A_k^1を描写せよ(3.2.G) ([0]はすべての点の近傍にある、すなわちすべての点の任意の近傍は[(0)]を含むという現象に気づくだろう。これはこの種の新しい点の典型的な挙動である。3.6.Nを見よ。) SpecZの場合も同様である。位相は同じようにして、ほとんど補有限位相である。開集合は空集合か、SpecZから高々有限個の「通常の点」[(p)]を除いた集合である。 3.4.3 A_C^2の閉集合 A_C^2の場合はより興味深い。1次元的素イデアルについてよく考えるべきである。練習3.2.Eで我々はC[x,y]のすべての素イデアルを決定した。0次元的点[(x-a,y-b)](a,b∈C)と1次元的点[f(x,y)](fは既約多項式)と2次元的点[(0)]である。 そうすると閉集合は次の形をしている (a)空間全体:[(0)]の閉包 (b)高々有限個の曲線(それぞれ1次元的点の閉包):「1次元的な点」とその上に「乗っている」0次元的な点:と高々有限個の0次元的点(3.6.8で閉点と名付ける) すぐ後でこれをより一般的な視点で正当化するが、読者は3.2.Eの考えを使ってこれをこの場で証明できる。(f(x,y)とg(x,y)が既約な多項式で一方が他方の倍数でないときに、それらの零点の交わりがどうして有限個であるかという点が鍵である。) 3.4.4 重要な事実:環準同型は位相空間の連続写像を誘導する 我々は3.2.9で環準同型φ:B→Aは集合としての写像π:SpecA→SpecBを誘導することを見た。 3.4.H 重要な練習 πによって閉集合は閉集合に引き戻される:すなわちπは連続写像であることを示せ。こうしてSpecは環の圏から位相空間の圏への反変関手である。 /* V(f)がV(φ(f))に引き戻されることを確認すれば良い。 環準同型が素イデアルを引き戻す時に包含関係を保つことによる。 */ すべての連続写像がこう表せるわけではない。A_C^1の間の連続写像:0と1、すなわち([x])と([x-1])だけ入れ替えてほかをそのままにする写像は連続写像である:しかしこんな挙動をする環準同型(多項式)は存在しない。 3.2.9で我々はSpec B/IとSpec S^-1BはともにSpecBの部分集合とみなした。そのザリスキ位相が包含に対して都合良く振舞うか確かめるのは自然なことであり、実際都合良く振舞う 3.4.I 重要な練習(cf.3.2.N) I,SをそれぞれBのイデアル、積閉集合とする。 (a)Spec B/IはSpecBの閉集合であることを示せ。  S={1,f,f^2,..}(f∈B)のとき、SpecS^-1Bは開集合であることを示せ。  任意のSに対してはSpecS^-1Bは開とも閉とも限らないことを示せ。 (ヒント:SpecQ⊂SpecZ、あるいは3.2.Lの後ろにある例:B=C[x,y],S=B-{(x,y)}) (b)SpecB/IあるいはSpecS^-1Bのザリスキ位相は、SpecBのザリスキ位相の部分位相であることを示せ(ヒント:閉集合を比較せよ) 3.4.5 特に、Iが冪零元のとき、SpecB/I→SpecBは位相同型である(3.2.R)。冪零元は位相空間にも影響を与えない。(それは構造層に影響する) 3.4.J あとで有用な練習 I⊂Bをイデアルとする。fがV(I)で消えることはf∈√Iと同値である:すなわちあるn≧1があってf^n∈Iとなることと同値であることを示せ(ヒント:3.4.F。もし行き詰まるなら3.5.Eを見たときに別のヒントを得るだろう。) /* fがV(I)で消えるとは、すぺてのp∈V(I)に対してf⊂pを意味する。より具体的にはすべての I⊂pとなるpに対してf⊂pとなる。素イデアルpに対してI⊂pと√I⊂pが同値であることを言えば良い。この主張はほとんど3.4.Fである。 */ 3.4.K 簡単な練習(cf.3.2.A) Spec k[x]_(x) を位相空間として描写せよ /* 底空間は[(0)]と[(x)]の2点で、φ,{[(x)]},{[(x)],[(0)]}が閉集合 */ --- 3.5 ザリスキ位相の開集合の基:「基本開集合」 f∈Aに対して、基本開集合D(f)を次のように定義する。 D(f) = {[p]∈SpecA ; f∈/ p} = {[p]∈SpecA ; f([p])≠0} これはfが消えない点の集まりである。(私は略式的にしばしばD(f≠0)と書いている。またこれを、V(f)がVanishing setであることに結びつけて、Doesn't-vanish setと呼んでいる。)この集合は、Spec A_fをSpecAの部分集合としてについて議論しているときにすでに見た。例えば、Spec Aのザリスキ位相から得られるD(f)⊂SpecA上の誘導位相は、Spec A_fのザリスキ位相に一致する(3.4.I)。 この集合が重要なのは、これがザリスキ位相の開集合の基をなす点である。 3.5.A 簡単な練習 基本開集合はザリスキ位相の開集合の基をなすことを示せ。 /* ザリスキ位相の定義による */ これを実感するのに良いいくつかの重要だが難しくない練習がある 3.5.B f_i(i∈J)∈Aを、Aの元の列とする。このとき開集合属D(f_i)がSpecA全体を被覆することは、f_iたちが生成するイデアルがA全体となる、あるいは言い換えると、Σa_i*f_i=1となるa_iが存在することと同値である(左辺は有限和)。 (同値のうち片方は、Aの真のイデアルがある極大イデアルに含まれるという事実を利用する。) /* f_iたちが生成するイデアルをIとする。IがA全体でないと仮定する。Iはある極大イデアルに含まれる。特にI⊂pとなるp∈SpecAが存在する。このときすべてのf_iについてf_i⊂pだからpはどのD(f_i)にも属さない。 どのD(fi)にも属さない素イデアルpが存在すると仮定する。つまりすべてのf_iについてf_i⊂p。このとき、Σa_i*f_i∈pとなるからそれは1にならない。*/ 3.5.C SpecAが無限個の開集合の族D(f_j)で被覆されるなら、そのうち有限個のもので被覆される。(ヒント:3.5.B) /* Σa_i*f_i=1となる有限和があるのでそれに使われるf_iを取り出せば良い */ /* これは後で登場する準コンパクトという概念である */ 3.5.D D(f)∩D(g) = D(fg) 3.5.E 重要な練習(cf.3.4.J) D(f)⊂D(g) ⇔ f^n∈(g)となるn≧1 がある ⇔ gはA_fで可逆 /* 具体例には例えばf=x^ay^b,g=x^cy^d (c,d≧1) D(f)⊂D(g)を仮定する。これはg⊂p ⇒ f⊂pを意味する。特に g⊂√(g) なので f⊂√(g) が成り立つ。逆にf⊂√(g)を仮定すると、f⊂g⊂√(g)なので g⊂p ならば f⊂p である。 また、局所化 S^-1A でgが可逆であるとは定義から (g,1)*(a,b)〜(1,1)、すなわちbg-a=0 となる a∈A,b∈Sが存在することを意味した。この場合 f^n=agとなるa∈Aが存在することと同値である。 */ これはしばしば使う。純粋的に代数的に考えても良いが、次のように幾何的に解釈することが役に立つ。(自身で絵を描くことを試みるべきである。)SpecAにおいて閉集合F(g)=Spec A/(g)とは、gの"零点集合"であり、D(g)はgが消えない集合である。fを閉集合V(g)上に制限した関数、あるいは正確にはSpec A/(g)上の関数、とみなすと、仮定によりfは閉集合全体で消える関数である。環のSpec全体で消える関数は冪零元に限るのであった(3.2.12)。すなわち剰余環A/(g)においてf^n=0(すなわちAではf^n∈g)となるnが存在する。 3.5.F 簡単な練習 D(f)=φ ⇔ f∈√0 /* g=0の場合。 */ --- 3.6 位相的性質とネーター性 多くの位相的な用語は、SpecA、後では一般のスキームの位相空間に適用すると有用である。 3.6.1 連結性、既約性、準コンパクト性を考える。 3.6.2 連結性 Xは、2つの空でない開集合の非交和に表せないとき連結と呼ぶ。 次の練習は連結でない例を与え、その後すべての例はこのような形をしていることを指摘する。 3.6.A 練習 A=A1×A2×..×Anのとき、位相同型 SpecA1+SpecA2+..+SpecAn→SpecA があり SpecAiはSpecAの基本開集合D(fi)に写されることを示せ。 (ヒント:n=2の場合で確信を得よ。f1=(1,0)とf2=(0,1)を考えよ) /* D(g1)+D(g2)+..+D(gn)=D((g1,g2,..,gn)) と対応することになる。 gi=1のときSpecAi成分は全体で、gi=0のときSpecAi成分は消える。 fiとしてはi番目の成分だけが1で他の成分が0のものを使えば良い /* 3.6.3 指摘 SpecAが連結でないことは、Aが零環でない2つの環の直積と同型に書けることと同値である。これを示す鍵は、どちらも次の条件と同値だと示すことである:自明でない(0でも1でもない)冪等元a1,a2=1-a1が存在すること。これは練習9.5.Jでも登場する。 3.6.4 既約性 位相空間は空でなく、2つの真な閉集合の和集合で表せないとき、既約と呼ぶ。言い換えると、Xが空でなく、X=Y∪ZでY,Zが閉集合ならばY=XまたはZ=Xが成り立つときである。これは古典的な幾何学ではあまり役に立たない。例えばC^2は既約でない。しかし、アフィンスキームA_C^2は既約である。(3.6.C) 3.6.B 簡単な練習 (a)既約な位相空間では空でない開集合は稠密である (そういうわけでザリスキ位相では開集合は「でかい」) (b)Xが位相空間で部分集合Zが(誘導された位相で)既約な閉集合ならば、Zの(Xにおける)閉包もまた既約である /* (a) 補集合を考えれば良い。 (b) 閉集合Z1~,Z2~によってZ~=Z1~∪Z2~とおく。Z1=Z~∩Z1~, Z2=Z~∩Z2~とおく。 Z1,Z2は閉集合でZ=Z1∩Z2である。Zは既約なのでZ=Z1またはZ=Z2である。 Z=Z1とする。Z1~はZ=Z1を含む閉集合でかつ閉包Z~の部分集合だからZ~に一致する。*/ 3.6.C 簡単な練習 Aが整域ならばAは既約である(生成点[(0)]に注目せよ)。3.7.Fで一般化される。 /* 任意の空でない開集合は[(0)]を含む */ 3.6.D 練習 既約な位相空間は連結である /* 既約ならば、交わらない空でない開集合の組が存在しない。 従って交わらない空でない開集合で、和集合が全体になるような組も存在しない。*/ 3.6.E 練習 SpecAが連結だが既約でない例を挙げてそうだと証明せよ。 /* 例えばSpec k[x,y]/(xy) */ 3.6.F 技巧的な練習 (a) I=(wz-xy,wy-x^2,xz-y^2)⊂k[w,x,y,z]を考える。k[w,x,y,z]/Iが整域であることを示し、Spec k[w,x,y,z]/Iは既約であることを示せ。 (これは難しいのでヒントを示す:この剰余環は、k[a,b]の次数が3の倍数な単項式で生成される部分環に同型である。これはイデアルが素であるかどうかをどう調べれば良いのかという難しい問題の例である。)後でこれがねじれ3次曲線であることを見て(8.2.A)、さらにVeronese埋め込みの特殊な場合であることを見る(8.2.6)。 (b)(*) (a)の生成元は rank (w,x,y ; x,y,z) ≦1 で表せることに注目せよ。(a)を2×nの行列の場合に一般化せよ。これは次数nの有理曲線上の錘というものに相当する(練習8.2.J)。 /* (a) w=aaa,x=aab,y=abb,z=bbb とおくと k[w,x,y,z]/Iからk[a,b]への環準同型を得る。像は整域であるから、環準同型が単射であることを確認したい:多項式F(w,x,y,z)があってF(aaa,aab,abb,bbb)=0ならばF∈Iであることを確認する必要がある。 私のうまくいかなかった方針 z=xy/w, y=xx/w を代入することで、F=A*(wz-xy)+B*(wy-xx)+C(x,w) A,B∈k[w,x,y,z,1/w], C∈k[w,x,1/w] を得る。aaaとaabは代数的に独立なのでC=0が要求される。分母を払って改めてF={A*(wz-xy)+B*(wy-xx)}/w^mとおける。 しかしこれではF∈Iは示せない。 答え的なもの: https://math.stackexchange.com/questions/167321/krull-dimension-of-mathbbcx-1-x-2-x-3-x-4-left-x-1x-3-x-22-x-2-x-4-x/227255#227255 辞書式順序で割り算を行う。wyをx^2に置き変え、xzをy^2に置き変え、wzをxyに置き変えていく。残る項はw^i*x^j(i≧1),x^k*y^l(k≧1),z^u*w^vである。 a,bの式にした結果はそれぞれa^(3i+2j)*b^j, a^(2k+l)*b^(k+2l), a^(u)*b^(2u+3v) であり一致しないことが確認できる */ 3.6.5 準コンパクト 位相空間Xが準コンパクトであるとは、開被覆が与えられたときに有限個による開被覆を抽出できることである。あるいは、すべての開集合は有限の開被覆を持つ。我々は無限を恐れているのでこの状況を好む。(コンパクトの定義によるが)これはコンパクトからハウスドルフ性を抜いた条件である。しかしこれはコンパクトの代数幾何バージョンではない。(我々はA_C^1がコンパクトであって欲しくない。その代わりは固有性(10.3)である。) 3.6.G 練習 (a) Spec Aは準コンパクトである(ヒント:3.5.C) /* 1∈(f1,f2,...,)であることは、いくつかの有限のfkの線形結合で1を表せることを意味する */ (b)(*) Spec Aは準コンパクトでない部分集合を持ち得ることを示せ。 ヒント:A=k[x1,x2,...], m=(x1,x2,...)⊂A として V(m)の補集合を考えよ。 これは他の反例の構成にも役に立つ。7.1.C, 5.1.Jなど。このような奇妙な振る舞いは、ネーター環では起こらないことを後で見る(3.6.T)。 3.6.H 練習 (a) Xが有限個の準コンパクト空間の合併ならばXは準コンパクトである。 (b) 準コンパクトな位相空間の閉部分集合は準コンパクトである。 3.6.6(**) 楽しいが関係ない注意 3.6.Aは、+(Spec Ai)=Spec ΠAiを示したが、無限個の場合でAiがすべて零環でない場合には正しくない。3.6.Gより右辺は準コンパクトであるが左辺はそうではないからである。 右辺は左辺より真に大きい。左辺から右辺への自然な写像があるが、右辺にはその像にない素イデアルが存在する。例えば、右辺の元を(a1,a2,..)と書いたとき、「あるMが存在してi≧Mではai=0となるようなもの」が例となる。これは超フィルターという概念に至り、便利であるが、現在の議論には関係ない。 3.6.7 Spec A上の位相空間 3.6.8 位相空間上の点pは、{p}が閉集合のとき閉点と呼ぶ。 古典的な位相ではすべての点は閉点である。 Spec ZやSpec k[t]では[(0)]以外のすべての点は閉点である。 3.6.I 練習 Spec Aの閉点はAの極大イデアルに対応する (特にすべての空でないアフィンスキームは閉点を持つ(0.3)) 3.6.9 古典的多様体との関係 ヒルベルトの零点定理より、古典的な点とはすなわち閉点である。 A_k^nの議論でおまけの点と呼んでいたものは、すなわち閉でない点である。 3.6.J 練習 (a)kを体としてAは有限生成k-代数とする。Aの閉点は稠密である。すなわちf∈Aに対して、D(f)は空でなければ閉点を持つことを示せ。(5.3.Fで再び登場する) ヒント:A_fもまた有限生成k-代数である。零点定理を使い、剰余体が有限次拡大であることで閉点を特徴づけよ。 (b) 有限生成とは限らない場合、閉点は稠密とは限らないことを示せ(ヒント:3.4.K) /* (a)D(f)=Spec A_fであった。 Aは極大イデアルを持つのであった。選択公理(0.3)により環は極大イデアルを持つが、ここでの注意は、A_fの極大イデアルmをAに引き戻したものは極大イデアルとは限らないことである[(b)が具体例]。 そこで、零点定理より A_f/mがkの有限次拡大であると言い換える。 https://math.stackexchange.com/questions/1611450/closed-points-of-speca-are-dense (3.2.Gを使っている) (b)3.4.Kのk[x]_(x)では閉点が1個と有限のために生成点が開集合で反例となっている。閉点がないという、より病的な例(3.5.G)も反例になるだろう。 */ 3.6.K kは代数閉体としてA=k[x1,..,xn]/Iは有限生成k-代数で、 Aは0以外に冪零元を持たないとする。(3.2.13の議論が適用する) X = SpecA を A_k^nの部分集合とみなす。A_k^nはk^nに相当する古典的な点を持つ。X上の関数はこれらの閉点での値で決定されることを示せ。(弱零点定理3.2.4により、古典的な点はk^n∩SpecAである。)ヒント:f,gがX上の異なる関数だとすると、f-gはX上のある開集合で恒等的に0でない。練習3.6.J(a)を使うと良い。 /* h=f-g≠0とする。開集合D(h)を考える。仮定よりhは冪零でないからD(h)は空でない(3.5.F)。 */ 多様体を定義した後(10.1.7)なら、「代数閉体上の多様体の関数は、古典的な点での値で決定される」ことがすぐに従う。(これは古典的多様体をスキームの特殊な一面として考えることが無害であることを示す) 3.6.10 特殊化と一般化 (一般の)位相空間Xの2点x,yに対して、x∈y ̄の関係があるとき、「xはyの特殊化である」「yはxの一般化である」と呼ぶ。これ(と3.6.L)によってある点が他の点を含むという怪しかった概念を正確にできる。すなわち、ある点が他の点を含むと言う主張は無意味であるが、ある点の閉包が他の点を含むという主張は意味を持つ。具体的には、A=Spec C[x,y]においては、[(y-x^2)]は[(x-2,y-4)]の一般化であり、[(x-2),(y-4)]は[(y-x^2)]の特殊化である。図形的には、放物線とその上の点である。 3.6.L X=SpecAとする。[q]が[p]の特殊化であることは、p⊂qと同値である。  従って、V(p)は[p]の閉包と等しい。 この重要な用語はグロタンディークにさかのぼる。20世紀初めのイタリアの代数幾何学者は多様体の生成点という用語を使っていて、それは特殊な性質を持たない点だから、生成点で正しいことは多様体のほとんどすべての点で正しいと議論していた。現代的な生成点は、同様の意味合いを持つ。既約閉集合上の点に対して、何かが(稠密な開集合上で正しいという意味で)一般的に、あるいはほとんど正しいならば、それは生成点上でも正しく、逆もそうである。 これは原理を主張していて、事実を主張しているわけではない。興味深い例は、被約性であり、この原理が一般にはあてはまらない(5.2.2)。ただし、ある種の都合のよい性質を持つスキーム(例えば多様体10.1.7)ではあてはまる。 また、例えば整スキームにおいては、関数の生成点での値が0であることは、すべての点での値が0であることと同値である(5.5.C)。他にもこのような例には注目するべきである。 「一般的な点」(general point)は「生成点」(generic point)と異なる意味で使われることに注意せよ。「Xの一般的な点で性質Pが成り立つ」という記述は、「Xのある稠密な開集合のすべての点で性質Pが成り立つ」ということを意味する。しかし、用語がいつもこの習慣に従うわけではない。例えば: 生成点的自由 generic freeness(7.4.4) 生成点的平坦 generic flatness(24.5.9) 生成点的滑らか generic smoothness(25.3) 3.6.M (y-x^2)がV(y-x^2)の生成点であることを確認せよ。 生成点という用語の他の動機:先の例で、[(y-x^2)]を放物線上のどことも特定しない点ととらえることができる。すなわち(x-a,y-a^2) (a∈C) という不定点である。特殊な点で考える時に、特殊化をするわけである。 3.6.N 簡単な練習 pを閉集合Kの生成点とする。pはKに属する点qの近傍にある(具体的にはqのすべての開近傍はpを含む)が、Kに属さない点rの近傍には無い(具体的にはrの開近傍でpを含まないものがある)ことを示せ。(この考えは3.2.3の例1と3.4.Gで言及された。) /* 前半:p⊂qのとき、qを含む開集合はpを含む。D(f)がqを含むことはf∈qでないことを意味し、包含関係からそのときf∈pでないこと、すなわちD(f)はpを含むことが従う。 後半:p⊂rでないとき、pに属するがrに属さない元fがあるのでそれによる開集合D(f)を取れば良い。*/ 我々はすぐにSpecAの点の集合から、SpecAの既約閉集合の集合への全単射があることを見る(3.7.E)。具体的には各点から、その閉包をとるという写像である。これは既約閉集合は(一意的な)生成点を持つということを意味する。読者はこれを今証明できるが、我々はいくらかの便利な用語を先に定義する。 3.6.12 既約成分と連結成分、ネーター性 既約成分とは、既約な部分集合で極大なものである。既約成分は閉集合である(3.6.B(b))から、Xの既約成分をXの既約閉部分集合で極大なものと考えることは時に役立つ。これをXのパーツと考えることができる。例えばXが2つの曲線(交わっていても良い)と4つの点からなるような絵であれば、既約成分は6個である。 同様に、連結成分とは連結な部分集合で極大なものである。 3.6.O 練習 すべての位相空間は既約成分の和集合である。位相空間の点xは、Xのどれかの既約成分に属する。 (ヒント:ツォルンの補題) 3.6.13 指摘:連結成分もいつも閉集合であり、これを今証明することもできるが、これは9.5.Hの(*)つきセクションで必要になるときまで保留する。連結成分は開とは限らない。例えばSpec +[∞]F_2 3.6.14 我々が考察した例では空間は自然に有限個の既約成分に分解できた。例えばxy=0は2つの既約成分を持っている。この有限性は、位相のネーター性に由来する。位相空間Xがネーターであるとは、閉集合について降鎖律を満たすことである:つまり閉集合の列Z1⊃Z2⊃...⊃Zn⊃...はいずれ停留する:あるrがあってZr=Z[r+1]=..となる。例を与えよう。 3.6.P 練習 A_C^2の位相空間はネーターであることを示せ。上記のrはいくらでも大きくできることに注意せよ。(ネーター環の性質を引用せずに明示的に議論するのが望ましい。) また、C^2の古典的な位相はネーターではないことを示せ。 /* ・明示的な議論 A_C^2の閉集合は、平面全体、代数曲線、点の有限個の和集合である。(3.4.3) 真に部分閉集合をとるには、平面から有限個の代数曲線や点に制限する、代数曲線から有限個の点に制限する、いくつかの成分を消す、という操作しか許されない。そこにある有限性によって、この操作は有限回しかできないことが分かる。 ・後の3.6.Rと3.6.17の内容を使った一般的な視点: A_C^2の閉集合は多項式{f1,f2,....}の零点集合の形をしている。 列 Z1⊃Z2⊃...⊃Zn⊃ に対して、それらの交わりをZ_allとする。 Zも多項式{f1,f2,....}の零点集合の形をしている。 ここで多項式環k[x,y]のイデアルは有限生成であるという事実がある。(ヒルベルトの基底定理) なのでZ_allは多項式{f1,f2,..,fn}の零点集合の形をしている。 Z_allはV(f1)に包まれるから、あるkがあってZ_kから先はV(f1)に包まれるとおける。 これをf2,...,fnにも適用して使ったkのうち最大のものを取れば良い。*/ /* 後半は例えば半径rの開円盤でrを少しずつ刻めば良い */ 3.6.15 命題 Xはネーター位相空間とする。空でない任意の閉集合Zは、一意的に、互いに包含関係がない有限個の既約閉集合の和集合Z1∪Z2∪..∪Znの形で書ける。3.6.12の言葉を使うと、閉集合はいくつかのパーツに分解される。 [証明] この技法は「ネーター帰納法」と呼ばれる。名前の由来はやがて分かるだろう。我々は今後これを何度も使う。 Xの閉集合で、有限個の既約閉集合の合併で表せないようなものを族として考える。これが空であることを示したい。そうでないと仮定してその族から閉集合Y1をとる。Y1が他の閉集合を真に包むならそれをY2として、Y2が他の閉集合を真に包むならそれをY3とし、これを繰り返す。降鎖律によりこれはいずれ停留してYrに至る。Yr自身は既約でないのでYr=Y'∪Y",[Y',Y"はYrに真に含まれる閉集合]と書ける。仮定よりどちらも既約閉集合の合併で表せるので、矛盾を引き起こす。これで、閉集合は有限個の既約閉集合の合併で書けることが示せた。 いくつかを捨てれば、互いに包含関係がないようにできる。 次に一意性を示す。Z1∪Z2∪..∪Zn = Z1'∪Z2'∪..∪Zn'と仮定する。Z1'は既約なので、Z1'∩Zkのどれかに包まれる。そのようなZkをZ1と名付け直すと、Z1'⊂Z1である。同様に、Z1⊂Zj'となるjが存在するが、Z1'⊂Z1⊂Zj'であり互いに包含関係が無いことからj=1でなければならない。 3.6.Q 練習 位相空間の連結成分は既約成分の和集合である。 開かつ閉な部分集合は、いくつかの連結成分の和集合である。 XがネーターならXの連結成分は常に開かつ閉である。 (ネーターと限らなければ正しくない:3.6.13) /* 1つ目:3.6.Dによる。 2つ目:Uを開かつ閉な部分集合とする。p∈Uを含む連結成分をWとおく。W∩UとW∩(Uの補集合)はともに開集合でありWを覆う組なので、連結の定義よりどちらかが空である。W∩Uはpがあるので空でないのでもう片方が空である。 3つ目:命題3.6.15より既約成分は有限個なので、連結成分も有限個である。 pを含む開かつ閉な部分集合で最小のものをWとする。Wを覆う連結成分をU1,U2,..,Unとする。p1∈U1としておく。連結成分の定義にある極大性よりWは連結でない。従ってWには開かつ閉な自明でない部分集合Yがある。Wは開かつ閉なのでこれはXの位相で開かつ閉な部分集合でもある。するとU1の連結性よりY∩U1は空またはU1全体である。同様にしてYはいくつかのUkの和集合である。YがU1を含むならWの定義よりYはW全体であるのでYはU1を含まない。従ってW-U1に包まれる開かつ閉な空でない部分集合が存在することになる。p2∈U2を含む開かつ閉な部分集合のうち最小のものをW2としてやり直すと、やがて矛盾する。 (もっとうまく議論する方法をどこかで見た気がする・・)*/ 3.6.16 ネーター環 今まで考えたアフィンスキームは(3.6.G(b)(*)以外)すべてネーターであったことが判明する。しかしすべての環のスペクトラムがネーターであるわけではない。我々の例に共通する環の性質はネーター環である。ネーター環とはすべてのイデアルが昇鎖律を満たすことを意味する:I1⊂I2⊂....がいずれ停留する。 いくつかの事実をすばやく紹介する。読者はこれを証明できるべきである。 ・体はネーター環である。Zはネーター環である。 ・Aがネーター環でIがイデアルならA/Iはネーター環である。 ・Aがネーター環でSが積閉集合ならS^-1.Aはネーター環である。 3.6.R 重要な練習 Aがネーター環であることは、すべてのイデアルが有限生成であることと同値。 /* 3.6.Pの議論の一部 */ 3.6.17 ヒルベルトの基底定理 Aがネーター環ならば、A[x]もそうである。 ヒルベルトはこれを文献[Hil]で示した。その文献ではヒルベルトのシチジー定理(15.3.2)も証明され、ヒルベルト関数が定義され、それが多項式であること(18.6)も証明された。 3.6.18 これにより、体上の多項式環あるいは整数環上の多項式環はネーター環である。その剰余環や局所化もである。従ってkやZ上の有限生成-代数やその局所化はすべてネーター環である。ほとんどの素姓の良い環はネーター環である。しかしすべての環がネーター環ではない。例えばk[x1,x2,...]はそうではない(3.6.G(b))。 3.6.19 ヒルベルトの基底定理の証明 イデアルI⊂A[x]が有限生成であることを示す。 I≠(f1,..,f[n-1])と仮定して、Iに属する他の元で最も次数が低い元f[n]をとる。すなわちI≠0ならば、f1はIのうち最も次数が低い元である。この操作が終了しないとする。anをfnの最高次係数とする。仮定より(a1,..)で生成されるイデアルは、(a1,..,aN)で生成されるイデアルに等しくなるようなNが存在する。すなわちa[N+1]はaiの線形結合Σb[i]a[i]と書ける。 そうすると f[N+1] - Σb[i]f[i] x^(deg f[N+1]-deg f[i]) を考えるとIに含まれない元でf[N+1]より次数が低いものだからf[N+1]の設定に矛盾する。 ここで、ネーター環とネーター位相空間を結びつけよう。 3.6.S 練習 Aがネーター環ならばSpecAはネーター位相空間であることを示せ。 また、Spec Aがネーター位相空間でない環Aの例を挙げよ。 (Spec Aがネーター位相空間でも、Aがネーター環とは限らない。 例えば A=k[x1,x2,x3,...]/(x1,x2^2,x3^3,...)のときSpec Aは1つの点しか持たないがAはネーター環ではない。) /* 止まらない閉集合の列があったら、止まらないイデアルの列がある。 逆が成り立たないのは、閉集合が異なるならば対応するイデアルは異なるが、異なるイデアルに同じ閉集合が対応することがあるから。 */ 3.6.T 練習(3.6.G(b)で予告した) ネーターな位相空間では開集合は準コンパクトである /* 開被覆U1,U2,...をとると、閉集合の列 X-U1, X-U1-U2,.. はどこかで止まる */ 3.6.20 将来のために /*9.7.3*/ ネーターA-加群とは、部分加群について昇鎖律を満たすことを意味する 例えばA自身がネーターA-加群であることは、Aがネーター環であることと同値である。 3.6.U (cf.3.6.R) MがネーターA-加群ならばMの部分加群は有限生成 3.6.V 0→M'→M→M"→0が完全列なら、 Mがネーター ⇔ M'とM"がネーター /* ・M'に終わらない部分加群列があるなら、その像の列をとる ・M"に終わらない部分加群列があるなら、その逆像の列をとる ・Mの部分加群m1のM"への像をm1"として、その核をm1'とする  M'をMの部分加群とみなすと、m1' = m1∩Mの関係にある  部分加群の列 M⊃m1⊃m2... に対して、  M'⊃m1'⊃m2',... およびM"⊃m1"⊃m2"... が定まる*/ 3.6.W Aがネーター環ならばAの有限個の直和は加群としてネーターA加群である 3.6.X Aがネーター環でMが有限生成A-加群ならば、MはネーターA加群である。 従って3.6.Uよりネーター環上の有限生成加群の部分加群は有限生成である。 3.6.21 ネーター性について悩む価値 ・ネータースキームのファイバー積はネーターとは限らない(9.1.4) ・ネーター環の正規化はネーターとは限らない(9.7.4) ・ネーター性がどのように議論に使われるかを知る ・アデールはネーターではない --- 3.7 SpecAの部分集合からAのイデアルを得る関数I(・) 零点集合V(・)の逆関数に近いものを導入する:I(S) = ∩[p∈S] p (少なくともSが空でないとき) と定義する(右辺はAの部分集合としての交わり) ・I(S)はAのイデアルをなす ・S1⊂S2ならばI(S2)⊃I(S1) ・I(S ̄) = I(S) 3.7.A 練習 A=k[x,y], S={(x),(x-1,y)}とする I(S)はy軸と点(1,0)で消える多項式からなる。その生成元を与えよ。 /* {x(x-1), xy} */ 3.7.B 練習 S⊂A^3をxyz軸とする。I(S)の生成元を与え(確実に証明せ)よ。練習12.1.Fにおいてこのイデアルが3つより少ない元では生成できないことを見るだろう。 /* {xy,yz,zx} x^aのような1変数の単項式は不適であり、残りの単項式はこれで生成されている */ 3.7.C V(I(S)) = S ̄ (3.7.Dと比較せよ) /* q∈V(I(S)) ⇔∩[p∈S]p ⊂ q ⇔任意のp∈Sに対してp⊂q ⇔任意のp∈Sに対してf∈pならば、f∈q ⇔基本閉集合V(f)がSのすべての点を保有するなら、V(f)はqも保有する ⇔q∈S ̄ */ I(S)は常に根基イデアルとなる。f∈√I(S)、すなわちあるnがあってf^nはSで消えるならば、fもSで消える、すなわちf∈I(S)であることに注意せよ。 3.7.D I(V(J)) = √J (ヒント:3.4.J) /* 3.4.Jあるいは3.4.Fの通り。 左辺=「Jを含むすべての素イデアルの交わり」=√J */ これらの練習は、VとIは「ほとんど」逆写像であることを示す。正確には: 3.7.1 定理 V(・)とI(・)はSpecAの閉集合とAの根基イデアルの全単射を与える。 /* Sが閉集合ならV(I(S))は恒等写像で、I(S)は根基イデアル Jが根基イデアルならI(V(J))も恒等写像で、V(J)は閉集合 */ これはヒルベルトの零点定理と呼ばれることがあるが、我々はその名前は3.2.5に使った。 3.7.E 重要な練習 V(・)とI(・)はSpecAの既約閉集合と、Aの素イデアルの全単射を与える。従って SpecAの点と、SpecAの既約閉集合には全単射がある(閉包をとる)。従って SpecAの各既約閉集合Zにはただ1つの生成点zがある(Z={z} ̄となるz) /* これは、3.7.1の全単射を制限したものである。次を示したい: ・Jが素イデアルなら、V(J)は既約(3.6.C) ・Jが素イデアルでない根基イデアルなら、V(J)は既約でない: 冪零でないx,yでx,y∈Jでないがxy∈Jとなるものがある。D(x)∩D(y)=D(xy)はV(J)と交わらないかつ、D(x)∩V(J)やD(y)∩V(J)は空でないのでこれらがV(J)が既約でないことを示す。 */ 3.7.F 練習 極小イデアルについて Aの素イデアルは包含関係で極小なとき極小イデアルと呼ばれる。例えばk[x,y]の極小イデアルは(0)である。Aが任意の環のとき、SpecAの既約成分と、Aの極小イデアルと全単射があることを示せ。特にAが既約であることは、Aがただ1つの極小イデアルを持つことと同値である(3.6.Cの一般化)。 /* 既約成分とは、既約な部分集合で極大なものであった。VとIは包含関係を逆順にするのであった。 なお後半の条件は、√0が素イデアル、つまりAが冪零でない零因子を持たないことと同値であると認識している。*/ 命題3.6.15, 練習3.6.Rから、ネーター環は有限個の極小イデアルを持つ。いま、これが幾何的な事実となった。 /* つまりネーター環の既約成分は有限個である */ 3.7.G 練習 k[x,y]/(xy)の極小イデアルは何か /* (x)と(y)である */ ========== 4. 構造層と、一般のスキームの定義 アフィンスキームを定義する最後の成分は、「構造層」である。これは代数関数がなす層と思うことができる。よく理解するにはC^n上の代数関数の例をいつも心に留めておくのが良い。例えばA^2では、D(xy)上(座標軸以外)では、(3x^4+y+4)/x^7y^3は代数関数のはずである。 スキームでは関数は点で値をとるが、点での値で関数が決定されるとは限らないのであった。そうではなく、層の切断のように、芽で決定されるのである(4.3.5を見よ)。 ここで、2.5.1(層は開集合の基で定めれば決定される)と3.5.A(基本開集合は開集合の基をなす)により、基本開集合での構造層を描写すれば十分である。 4.1.1 SpecAの構造層の基本開集合D(f)での切断O_SpecA(D(f))を、D(f)で消えないような関数という積閉集合(具体的にはV(g)⊂V(f)、あるいはD(f)⊂D(g)となるようなgの集合である。3.5.E)によるAの局所化と定義する。これはD(f)にのみ依存し、fに依存しない。(空集合好きな人は3.5.Fより、空集合での切断は{0}であることに気づくだろう。) 4.1.A 偉大な練習 自然な環準同型 A_f→O_SpecA(D(f))は同型であることを示せ(3.5.Eがヒントかもしれない) /* D(f)⊂D(g)⇔f∈√(g)であった(3.5.E)。 積閉集合S={1,f,f^2,...}, T={g|D(f)⊂D(G)}={g|f∈√(g)}に対して 「sa=0となるs∈Sが存在する ⇔ ta=0となるt∈Tが存在する」が主張である。 ta=0となるt∈Tが存在する ⇔ga=0 となる gでf∈√(g)が存在する ⇔ga=0, f^n=bgが存在する ⇒f^n*a=0 (逆方向は明らか) */ D(f')⊂D(f)のとき、制限写像 O_SpecA(D(f))→O_SpecA(D(f')) は自然に定義される:後者は前者のさらなる局所化である。制限写像は明らかに可換である。従ってこれによりO_SpecAが前層であることが言えた。 4.1.2 定理 このデータはSpecAの基本開集合上で層を与え、従って位相空間SpecA上の層を定める。 4.1.3 これを「構造層」と呼びO_SpecAと表記する。あるいは文脈から明らかな時は単にOと表記する。このような構造層つきの位相空間をアフィンスキームと呼ぶ(4.3.1)。SpecAは今後、この構造層つきの位相空間のことをさすものとする。定理4.1.2の重要な教訓としてはO_SpecAが単に層であるということだけでなく、基本開集合を考えることが構造層を考察するのに良い方法であることをも示している。また、Aの元をSpecA上の関数として解釈することが正当化されたことにも注意せよ。 [証明] 我々は一致の公理と貼り合わせ公理を確認する必要がある(2.5)。まずSpecA全体について確認し、開集合に拡張することを読者の練習にする(4.1.B,4.1.C)。 SpecA=∪D(fi)と仮定する。3.5.Bよりこれはf_iの生成するイデアルが環A全体となることと同値であった。 (2.2.7の等化子完全列になじみがある熟練者なら、我々が今 0→A→Π[i]A_fi→Π[i≠j]A_fifj の完全性を示そうとしていることが分かるだろう。一致の公理は左側の完全性、貼り合わせ公理は右側の完全性に対応する。) 一致の公理を開集合の基で確認する。まず、3.5.Cより、被覆を有限個に絞れるのであった。つまり線形結合c1*f1+..+cn*fn=1が存在する。s∈Aが与えられて、各D(fi)への制限が0であると仮定し、s=0であることを示したい。各D(fi)への制限が0であることは、あるmがあってfi^m*s=0となることを意味する。「貼り合わせに関する重要なテクニック」の使いどころである。:c1*f1+..+cn*fn=1の両辺を十分に冪乗すればC1*f1^m+..+Cn*fn^m=1を得られることから、両辺にsを掛けてs=0が従う。 4.1.B 上記の議論を改変して一致の公理が開集合D(f)でも成り立つことを示せ。 (ヒント:AをA_fに置き変えていけば良い) 次に貼り合わせ公理を確認する。(セールはこれを「1の分割」議論と呼んだ。これは正しく見れば、その洞察はとても視界を開くものである。) 再びSpecA=∪D(fi)と仮定する。A_fiの元の組を持っていて、重なりA_fifjでは一致すると想定する。 族fiが有限である場合を考える。関数ai/fi^m∈A_fiたちを持っているという状況である。まず、m=1に帰着できる。なぜならD(fi)をD(fi^m)で置き換えても開集合は同じだから、fi^m=giとおいて族giで考えれば良い。 次に、交わりでA_gigjが一致するという条件は、またあるnがあって(gigj)^n*(gj*ai-gi*aj)=0 が成り立つことを意味する。i,jごとにnは異なるが、族の有限性より最大のnを採用し、a_iとa_jを調整すれば、nは共通にできる。 ここでn=0に帰着するテクニックがある。bi=ai*gi^n, hi=gi^(n+1)と置き変えることで、やはりD(gi)とD(hi)は開集合として変わらず、交わりで一致する条件の式がhj*bi-hi*bj=0に簡略化できる。 さて線形結合c1*h1+..+cn*hn=1が存在する。この係数を使ってr=c1*b1+..cn*bnとおく。これが求める大域的な関数である。 実際 r*hj=Σci*bi*hj=Σci*hi*bj=bjの関係がある /* 真ん中の等号で、交わりで一致する条件を使った */ 族が無限の場合を考える。Aを被覆する有限個のD(fi)を選ぶ。rを上記のように構成する。選ばれなかったfjに対してもrのD(fj)への制限が、持っている関数aj/fj^mに合致することを示したい。 fjを追加して同様の構成を行いr'を得る。そうすると一致の公理によりr=r'である。r'の制限はaj/fj^mに合致するからrの制限も合致する。(一致の公理を利用して貼り合わせ公理を示したことに注目せよ。このように、一致の公理は貼り合わせ公理に先立つ側面がある。) /* 何が起きているのか具体例でいくつか観察しておく: ・A=Speck[x], f1=x,f2=x-1のような場合 a1/xとa2/(x-1)が重なりで一致するとは、あるnがあって、(x*(x-1))^n*{a1/x-a2/(x-1)}=0 しかしAは整域なので(x*(x-1))^nは役に立たない。結局 a1/xとa2/(x-1) そのものが一致するという要求となる。a1=wx,a2=w(x-1)みたいな形である。 線形結合 x-(x-1)=1 が存在し、r=wx-w(x-1)=w という計算でwという大域関数表示を得る。 (これは、1という関数をxという関数とx-1という関数に分割して、それぞれの分割に対応する開集合の関数を掛けて足し合わせることで、関数の貼り合わせを実現している、ということである。) ・A=Spec k[x]/(x(x-1)),f1=x,f2=x-1の場合 a1/xとa2/(x-1)が重なりで一致する条件は、n=1とおけば常に満たされる。実際SpecAは2点集合で、D(x)とD(x-1)は重なりがないからどんな関数同士でも貼り合わさる。 ・A=Spec k[x]/(xy),f1=x,f2=x-1の場合 重なりで一致する条件は、a1/x-a2/(x-1)の分子がyという因子を持つことである。開集合D(x)ではyという関数は0に消えるから、yの違いがあっても貼り合わさるのである。*/ 4.1.C 貼り合わせ公理が開集合D(f)上でも成り立つことを示せ (とにかくAをA_fに置き変えれば良い) これで定理4.1.2が証明された。次の一般化は、13章で準連接層を定義する時に本質的である。 4.1.D 重要な練習、定義 MをA-加群とする。次のM~という構成を考える。 M~(D(f))をD(f)で消えない関数という積閉集合によるMの局所化と定める。制限写像は構造層と同じように定める。これが層を定め、特にO_SpecA-加群であることを示せ。 /* 上記の証明で、Aの元a1,a2などをMの元m1,m2に置き変えていけば良いと思う。 */ 4.1.4 指摘 0→M→Π[i]M_fi→Π[i≠j]M_fifj が完全であることを見ただろう。特に、MはM_f1×..×M_frの部分加群とみなせる。M→M_fiは埋め込みでないかもしれないが、M→M_f1×..×M_frは埋め込みとなる。これは後で役に立つ:Mが何かの性質を持つ時にM_fもそうであることや、(f1,f2,..,fn)=Aで各M_fiが何かの性質を持つ時にMもそうであることを示したいときに役に立つ。この考えはアフィン交通補題(5.3.2)で登場するだろう。 4.1.5 指摘 定義4.1.1と定理4.1.2は、(基本開集合とは限らない)任意の開集合での構造層の切断を記述するときの罠の存在をほのめかしている。O_Spec(U)はUで消えない関数によるAの局所化とは限らない。Aを、2つのA^2のコピーを原点で張り合わせたアフィンスキームとする。(正確には24.4.Iの式 Spec k[w,x,y,z]/(wy,wz,xy,xz)、つまりwx平面とyz平面を合わせたもの。)Uを原点を除いた開集合とする。wx平面の原点以外では1、yz平面の原点以外では0、という関数は前述の局所化の形で表示することはできない。 (どうしてこの議論は2つのA^1を貼り合わせたものでは成立しないか?) /* A = Spec k[x,y]/(xy)では、"y/y"という関数がそれを実現する? */ 4.1.6 繰り返し登場する(反)例 ・二次曲面(上の錘):A=k[w,x,y,z]/(wz-xy)のときのSpecAあるいはProjA 4.4.12, 5.4.D, 5.4.I, 5.4.L, 6.5.H, 8.2.2, 8.2.9, 9.6.2, 11.2.D, 12.1.D, 12.1.E, 12.2.14, 12.3.D, 12.5.13, 14.2.O, 14.2.R, 14.2.U, 19.8.B, 19.10.J, 19.10.7, 22.4.N, 22.4.11 ・1点で交わる2つの平面:4.1.5, 18.5.B, 18.6.U, 24.4.I, 24.4.10, 24.6.E, 26.1.G, 26.1.6, 26.2.2, 26.2.12, 26.3.2, 26.3.4 ・アフィン空間A^nから原点を除いたものや、原点を二重にしたもの、n=1,2,∞:3.6.G, 3.6.18, 4.4.1, 4.4.3, 4.4.5, 4.4.B, 4.4.C, 5.1.J, 7.3.9, 10.1.D, 10.1.8, 10.2.C, 10.2.D, 12.7.B, 12.7.C, 18.3.A ・無限個のスキームの非交和、とくに+Spec k[x]/(x^n):4.3.E, 4.4.4, 5.2.E, 8.3.2の例4, 8.3.11, 13.3.I, 29.2.7 ・Spec Q ̄→Spec Q:7.2.2の後のコメント, 7.3.10, 7.3.14, 9.1.4, 9.2.E, 9.4.C --- 4.2 スキームを見る(2):冪零元 比喩の代償は永遠の警戒である。 --A.Rosenbluth, N.Wiener 3.3で、古典的な点(閉点)に生成点を加えた時のスキームの底集合をどう視覚化するかを議論した。ザリスキ位相はその描写によく合った。アフィンスキームの定義では、冪零元は底空間では区別できない(3.2.11)のであった。適切な定義をした今、これをどう視覚化するかを考える。ただし直観を優先するためには、厳密には描写できない。 動機として、次の全単射対応を思い出す: Aの極大イデアル <-> SpecAの閉点 (3.6.I) Aの素イデアル <-> SpecAの既約閉集合 (3.7.E) Aの根基イデアル <-> SpecAの閉集合 (3.7.1) 我々の視覚化の目標は根基でないイデアルに対しても Aのイデアル <-> SpecAの? を明らかにすることである。(厳密にはこれは8.1で登場する閉部分スキームが対応するが、ここでの目標は視覚化である。) Spec C[x]/(x(x-1)(x-2)) は、A^1の閉集合{0,1,2}に対応した。特に、環準同型C[x]→C[x]/(x(x-1)(x-2)) は、A^1の関数を0,1,2での値に制限すると解釈することができた。 根基でないイデアルの最も単純な例は、Spec C[x]/(x^2) (3.2.A(a))である。集合としては単に原点であり、むしろC[x]/(x)として考えてきた。C[x]/(x^2)を描写する時には、x^2で割った剰余の情報を覚えていたい。すなわちf(x)の0での値と、1階微分係数である(3.2.T)。これを、点と、点の周囲の微小なもやもやとして描写する。・・・ 同様に、C[x]/(x^3)はより多くの情報を持つ:2階微分係数もである。そういうわけでこれはより多くのもやもやを持つ点として描写する。 2次元ではより微妙な状況が現れる。Spec k[x,y]/(x,y)^2 は「全方向の」1階の微分係数の情報を持っている。これを、円の形をしたもやもやとして描写する。同様に、Spec k[x,y]/(x,y)^3はより大きいもやもやを持った点である。 次に、イデアル(x^2,y)を考える。これはx軸方向のみの偏微分係数の情報を持っている。これを、x軸方向のみに広がるもやもやを持った点として描写する。 こうして、この種の視覚化の手段を得るが、これは科学より芸術に近い。例えばSpec C[x,y]/(x^2,y^2)に対してはこれは原点周囲の正方形状のもやもやと描写するかもしれない。(この正方形はC[x,y]/(x,y)^3には取り囲まれていて、一方でこの正方形はC[x,y]/(x,y)^2を包含していることを想像できるだろうか?) この例の特徴は、I,Jを2つのイデアルとしたときに、SpecA/(I,J)はSpecA/IとSpecA/Jの「交わり」となるべきだということである。(8.1.Jでこれを正確にできる。)例えばSpec C[x,y]/(x^2,y^2)は、2つの微小な幅を持った軸の交わりである。(Spec C[x,y]/(x^5,y^3), Spec C[x,y,z]/(x^3,y^4,z^5,(x+y+z)^2), Spec C[x,y]/((x,y)^5,y^3)などをどう描写するか?) 我々の最後の例は、5.5で登場するSpec C[x,y]/(yy,xy)である。C[x,y]の多項式を(yy,xy)で割った剰余を知ることは、x軸上での値と、原点におけるy軸方向の1階微分係数の情報を知ることに同等である。これは注意深く考える価値がある:x軸の原点をもやもやさせた図でこの情報がどう捉えられるか分かるだろうか? 我々の視覚化は、読者が既に直観を持っている有用な情報を捉えている。例えばy=x^2とx軸の交わりを考える。この交わりは重複度2を持っていることを読者は知っているだろう。視覚化のために、交わりを次のように解釈する: Spec C[x,y]/(y-x^2) ∩ Spec C[x,y]/(y) = Spec C[x,y]/(y-x^2,y) = Spec C[x,y]/(y,x^2) ここから重複度2という部分は、x軸方向であることも解釈できる。これはまた8.1.Jで正確にされる。 4.2.1 我々は後で随伴点について議論する時(5.5)に、もやもやの場所をある意味でより正確に描写する。また、病的でない状況ではもやもやは閉集合に集まっていることを見るだろう(13.7.2)。 幾何学は、間違った絵から正しく推論する芸術であるとよく言われる。しかし、それらの絵は、我々を裏切る意図で描かれたものでなければ、ある条件を満たしていなければいけない:割合は大きくずれているかもしれないが、相対的な位置関係はひっくり返っていてはいけない。 -- H.Poincare --- 4.3 スキームの定義 4.3.1 定義 一般のスキームを定義しよう。まず、環つき空間(X,O_X)と(Y,O_Y)の同型を定義する。これは位相同型π:X→Yがあって、πによって同一の空間とみなしたときO_XとO_Yが層として同型であることとする。後半はより正確には、O_Y→π_*O_XがY上の層として同型であること、あるいは随伴をよって π^-1(O_Y)→O_XがX上の層として同型であること、と記述できる。言い換えると、集合・位相・構造層がちょうど対応していることを意味する。 環つき空間(X,O_X)がアフィンスキームであるとは、ある環Aによる(SpecA,O_SpecA)に同型であることを言う。 環つき空間(X,O_X)がスキームであるとは、Xの任意の点がアフィンスキームであるような開近傍(U,O_X|U)を持つことを言う。スキームの位相はまたザリスキ位相と呼ばれる。スキームは(X,O_X)と書かれるが、しばしば単にXと書かれる。 スキームの同型は、環つき空間の同型を意味する。2.2.2のΓ(・,・)の定義を思い出すと、Γ(U,O_X)の元はU上の関数と呼ばれる。これはアフィンスキーム上の関数(3.2.1)の概念の自然な拡張である。 4.3.2 指摘 アフィンスキームの構造層の定義からいくつかのことが分かる。まず、(X,O_X)がアフィンスキームのとき、その環(X=SpecAとなる環A)は大域切断から復元できた:すなわちX=D(1)が成り立つのであったから: Γ(X,O_X) = Γ(D(1),O_SpecA) = A (読者は「XをSpecAとして認識する」ことができるだろう。すなわちπ:(SpecΓ(X,O_X),O_SpecΓ(X,O_X))→(X,O_X)という同型が存在する。例えばmがΓ(X,O_X)の極大イデアルならば、{π([m])}=V(m)である。次の練習はこの考えをより厳密にする。これは見た目よりも微妙な問題である。) 4.3.A 意外にも混乱する、視界を開く練習 スキームの同型SpecA→SpecA'と環同型A'→Aの間の全単射を描写せよ。 ヒント:一番難しいのは、スキームの同型π:SpecA→SpecA'が環同型π#:A'→Aを誘導し、環準同型π#:A'→Aがスキームの同型ρ:SpecA→SpecA'を誘導するならば、π=ρであるのを示す所である。まずこれを集合として確認せよ(実はここが技巧を要する部分である)。それから位相空間としての同型を確認し、最後に構造層としての同型を、開集合の基を使って確認せよ。後のセクションの洞察を自由に使って良いが、循環論法にならないように注意せよ。この練習に取り組むことは失敗しても役に立つだろう。 /* この問題を考察するにはスキームの同型の定義、環つき空間の同型を良く考える必要がある。それは位相空間としての同型:同相写像で、開集合における構造層の切断が同型で、制限写像と可換であることを意味すると理解している。 大域切断の環準同型が固定されたπ#となるような、環つき空間の同型が1つしかないことを示す課題となる。 π#を与えれば、πの集合としての作用が1つに決まることを示す。f'∈A'がf∈Aにうつるとき、位相空間の部分集合としてD(f')がD(f)に移ることを示せば十分だろう。 (例えばx,yをそれぞれx',y'に移す環準同型による Spec k[x',y'] → Spec k[x,y] で例えばx軸がy軸に移ったとする。補集合での構造層の切断の同型 k[x',y',1/x'] → k[x,y,1/y] は存在するけどこれはどうやって定めても大域切断での環準同型の制限と可換にならない。) これ、つまりD(f')≠D(f)ならば、A'_f'とA_fに同型があったとしてもπ#:A'→Aの同型の制限と可換にならないことを証明するのがこの問題の鍵だと考えた。 すなわち図式: A'→A ↓ ↓ S'^-1.A' → S^-1.A (上の行がπ#で下の行がなんらかの同型)が可換ならば、{S'と交わらない素イデアル} = π#{(Sと交わらない素イデアル)}であることを示せば良い。 ・pがSと交わらない ⇔ pはS^-1.Aの素イデアルを誘導する ・S'^-1.A' → S^-1.A は同型なので素イデアルは素イデアルに移る。 を考えれば2つの集合は等しくなければならないことが言えると思う。 */ より一般に、f∈Aに対してΓ(D(f),O_SpecA)はA_fに同型である。SpecA_f→SpecAは集合として埋め込みであり、SpecAのザリスキ位相の制限はSpecA_fのザリスキ位相に一致し(3.4.I)、そしてSpecAの構造層の制限は、SpecA_fの構造層に一致することを次の練習で示そう。 4.3.B 重要だが簡単な練習 SpecAのD(f)をSpecA_fと同一視し(3.5)、環つき空間の同型 (D(f),O_SpecA|D(f))=(SpecA_f,O_SpecA_f)があることを示せ。 ヒント:A_fの基本開集合は既にAの基本開集合でもある /* Aの開集合D(g)をD(f)∩D(g)に制限したとき、これはgをA_fの元とみたしたときのSpecA_f上の開集合D(g)に一致する。SpecAの構造層のD(f)への制限のD(g)での切断と、SpecA_f上の構造層のD(g)での切断は、ともにf,gで生成される積閉集合によるAの局所化である。 */ 4.3.C 簡単な練習 XをスキームとしてUをXの開集合とする。(U,O_X|U)もスキームである。 4.3.3 これをXの開部分スキームと呼ぶ。それがさらにアフィンスキームならば、開部分アフィンスキーム、あるいは単に開アフィンと呼ぶ。例えばD(f)はSpecAの開アフィンである。(重要でない指摘:SpecAの開アフィンが常にD(f)の形で書けるとは限らない。19.11.10) 4.3.D 簡単な練習 Xの開アフィンは、Xのザリスキ位相の開集合の基となる 4.3.E 簡単な練習 スキームの非交和は、読者が想像する通りの定義である。集合としての非交和で、非交和として定まる位相を持ち、貼り合わされた構造層を持つ。スキームの射が何であるかを知った後なら、集合の圏や位相空間の圏と同じように、スキームの非交和はスキームの圏での余積であることがすぐに分かるだろう(9.1.A)。 (a)有限個のアフィンスキームの非交和はアフィンスキームである(ヒント:3.6.A) (b)(アフィンでないスキームの最初の例) 無限個の空でないアフィンスキームの非交和はアフィンスキームでない。(ヒント:3.6.G(a)よりアフィンスキームは準コンパクトであった。指摘3.6.6もこれに答えるものであった。) 4.3.4 指摘 閉部分スキームについて スキームの開集合は自然に部分スキームと見なせた。一方で閉集合は、複数のスキームの構造を持ち得る。8.1でまた扱うが、Xの閉部分スキームは、非正式には、Xの閉集合上のある種のスキーム構造と思っておけば良い。例えばIをAのイデアルとすると、SpecA/IはV(I)という閉集合上のスキーム構造を持つ。しかし同じ集合で消える異なるイデアルが存在したことに注意せよ:k[x]における(x)と(x^2)のような例。 4.3.5 構造層の茎、芽、点での値と剰余体 構造層は層であるから茎を持ち、代数的な視点でも重要である。 4.3.F 重要で簡単な練習 O_SpecAの点[p]での茎は局所環A_pである 同様に4.1.Dで定義した層M~の[p]での茎はM_pである。2.4.Aのように切断は茎で定まるのであった。これは、M→Π[p]M_pが単射であることに翻訳される。従って、例えばA-加群Mが自明な加群0であることは、すべての素イデアルによる局所化が自明であることと同値である。 4.3.6 すべての茎が局所環であるような環つき空間を、局所環つき空間と呼ぶ。4.3.Fよりスキームは局所環つき空間である。2.1.1の古典的多様体は別の例である。どちらの場合も、極大イデアルによる剰余をとる操作は、その点での値をとる操作と解釈できた。 局所環OX_pの極大イデアルと剰余体はそれぞれm_p, κ(p)と表記される。点pでの値はκ(p)に値をとる。関数が点pで消えるとは、pでの値が0であることを意味する。これは3.2.1での定義の拡張である。局所環つき空間とは限らない環つき空間では、「関数が消える点」という表現自体が意味を持たないことに注意せよ。 /* 局所環つき空間ではない環つき空間の例には、例えば連続関数とは限らない関数の層を考えた場合がある:https://detail.chiebukuro.yahoo.co.jp/qa/question_detail/q12159537842 */ 4.3.G 有用な練習 (a) fが局所環つき空間X上の関数のとき、fが消える点は閉集合である。 (ヒント:fの芽が可逆である点の集合は開集合である) (b) 大域関数fがどこでも消えないならfは可逆である /* 仮定がとても抽象的なものだけであることに驚く。 (a) f_pが可逆 ⇔ p∈Uでfg=1となるg∈F(U)が存在する ⇒ そのUの他の点q∈Uに対してもf_qは可逆 (b) 各pに対してpの近傍でfg=1となるgが存在する。g=1/fは近傍の重なりで一致するから貼り合わさる。*/ スキームの話に戻る。 点pでの局所化と剰余環をとる操作は可換であることを指摘する。すなわちA→A_p→A_p/pA_p と、A→A/p→K(A/p) は同じ結果を与える。 /* 1つ目は {(f,g)|f,g∈A, g≠0 (mod p)} を fg'-g'f∈p ならば(f,g)〜(f',g')という同値で割ったもの 2つ目は {(f,g)|f,g∈A/p,g≠0} を fg'-g'f=0 ならば(f,g)〜(f',g')という同値で割ったもの */ ・例えば (x^2+y^2) / x(y^2-x^5) は y軸と曲線(y^2-x^5)から離れた所で定義される関数である。 点(2,4)での値は (2^2+4^2) / 2(4^2-2^5) である。 剰余環の言葉では (x^2+y^2) / x(y^2-x^5) ≡ (2^2+4^2) / 2(4^2-2^5) (mod (x-2,y-4)) である。 これが不思議に見えるなら確認せよ。 x軸の生成点[(y)]での値は、yを0とおいて得る x^2/(-x^6) = -1/x^4である。これは実際剰余環k[x,y]/(y)の商体すなわちk(x)の元である。 ・27/4はSpecZ-{[(2)]}上の関数である。 [(5)]での値は何か?2/(-1)≡-2 (mod 5)である。 生成点[(0)]での値は何か?27/4である。 どこで消えるか?[(3)]においてである。 4.3.7 FがO_X加群のとき、Fの点pでのファイバーを F|p = F_p {@_OX_p} κ(p) で定義する。これは13.7で登場する。 --- 4.4 3つの例 ここでの短期的な目標は、実際に構造層を使って作業し、アフィンスキームの基本開集合とは限らない開集合での切断を計算できることを見ることである。長期的な目標は、将来繰り返し現れる興味深い例に会っておくことである。 4.4.1 <例1> 平面から原点を除いたもの この例は読者に、基本開集合が作業するのに良いことを知らせる例となる。A^2=Spec k[x,y]とおく。開集合U=A^2-{(x,y)}を考える。これが基本開集合かどうかはすぐには明らかでない。(読者は数段落後で、そうではないことを知るだろう。(x,y)が単項イデアルではないことを示せば十分である。)しかしいずれにせよUを基本開集合の和集合で表すことができる:U=D(x)∪D(y)。我々はU上の関数を、これらの基本開集合上の関数を貼り合わせることで見つけることができる。 D(x)上の関数は、定義より、k[x,y,1/x]である。同様に、D(y)上の関数はk[x,y,1/y]である。(整域では局所化への写像は単射であった(1.3.C)。さらにk[x,y]_xとk[x,y]_yは全商環K(A)の部分環とみなせる。)U上の関数は、D(x)∩D(y)=D(xy)で一致するようなものである。分母にxしか現れないかつ分母にyしか現れないものは、多項式しかない。すなわちUでの構造層の切断はk[x,y]である。言い換えると、原点を除いても我々は新しい関数を1つも得られなかったわけである。また、計算がいかに簡単であったかにも注目せよ。 4.4.2 A^2-{(0,0)}の関数は平面全体に拡張できることに注意せよ。これは複素幾何でのハルトークスの補題の類似である:余次元2以上を取り除いた複素多様体で定義された正則関数は、取り除かれた集合上に延長できる。これはもっと代数的な拡張がある。滑らかでなくても、穏やかに特異的であっても良い:我々は正規と呼ぶ。これは11.3.11で正確にする。この事実はとても便利なものとなるだろう。 4.4.3 上記のUはスキームであるがアフィンスキームではない。さもなければ、大域切断からU=SpecRとなる環Rを復元できるので、4.4.1の結果からUとA^2が同型ということになってしまう。 --- 素イデアルIを与えられたら、閉集合V(I)によって切り取られる素イデアルを復元できた。また点pを与えられたら、pで消える関数のなすイデアルI(p)を復元できた。特に、素イデアル(x,y)はA^2上の点である。ところが、Uのほうでは、V(x)∩V(y)=φである。従ってUはA^2と同型でない。(読者が今後アフォンでないスキームによる反例を探すときにはこの例を思い出すと良い。) --- 4.4.4 これから2つのA^1を異なる2つの方法で貼り合わせる。それらはまた別な、アフィンでないスキームの例となる。1つ目は病的で、2つ目は中心的な例となる。その前にスキームをどうやって貼り合わせるかを読者に伝える必要がある。読者は先に、位相空間を同型に沿って貼り合わせる方法を思い出すと良い。2つの位相空間X,Yとそれぞれの開集合U⊂X,V⊂Yがあって同型U〜Vがあるとき、XとYを貼り合わせた位相空間Wを構成することができた:それはXとYの非交和をU〜Vによる同値で割ったものである。そうするとXとYは自然にWの開部分集合と同一視できる。読者はこれを有限とは限らない任意個の位相空間の場合に適切に記述できるだろうか。 4.4.A 本質的な練習(2.5.D):以下の情報を与えられたとする: ・Xを被覆する(1組の)開被覆Xi ・開部分スキームXij⊂Xi ・同型写像fij:Xij→Xjiの組で「コサイクル条件」を満たすようなもの (コサイクル条件:fik|Xij∩Xik =  fjk|Xji∩Xjk・fij|Xij∩Xik) これらの情報からXを一意的な同型を除いて復元できる。 2つのA^1を異なる2つの方法で貼り合わせる。X=Spec k[t], Y=Spec k[u]とおく。U=D(t)=Spec k[t,1/t]⊂X, V=D(u)=Spec k[u,1/u]⊂Yとおく。 4.4.5 <例2> 2重の原点を持つアフィン直線 tとuを対応させることでUとVを同型にする。(読者は4.3.Aを思い出しても良い。)できあがるスキームは2重の原点を持つアフィン直線である。これはハウスドルフ性の欠如を思い起こさせる。しかしザリスキ位相ではA^1自体がハウスドルフでないのでハウスドルフ性の欠如とは言えない。代わりの概念が分離性であり、10章で扱う。分離的なスキームではザリスキ位相は都合良く振舞う。例えば開アフィンの交わりは再びアフィンとなる(10.1.8)。 4.4.B このスキームはアフィンでない。ヒント:大域切断を計算せよ。 4.4.C A^1の代わりにA^2で同様の構成をせよ。 4.4.6 <例3> 射影直線 今度はtと1/uを対応させることでUとVを同型にする。これが射影直線と呼ばれ、P^1と表記されるものである。点がどのように貼り合わさるかに注意せよ。簡単のため一旦kは代数閉体とする。1つ目のアフィン直線ではt軸上の通常の閉点(t-a)[a∈k]と、生成点(0)がある。2つ目のアフィン直線では、u軸上の点と生成点がある。t軸上の"点a"はu軸上の"点1/a"に貼り合わさる(もちろんa≠0ならば)。また生成点は生成点に貼り合わさる。 4.4.7 この場合、P^1上の閉点は定数倍を同一視した[a:b]の形で解釈できる。 4.4.8 P^1はアフィンではない 大域切断を計算する。それはtの多項式とuの多項式の組で、t=1/uによって一致するようなものである。それは定数しかない。従ってP^1がアフィンだとしたらそれはSpec kである。しかしP^1はSpec kよりも多く点がある。 複素多様体でもCP^1上の正則関数は定数関数しかなかった。今我々が証明した重要な事実はこれの類似である。(10.3.7で拡張する。) 4.4.9 射影空間P^n これはn+1個のA^nを貼り合わせて作る。・・・ 4.4.D 4.4.10 任意の環上の射影空間 4.4.E 射影空間上の関数は定数しかない 4.4.F (4.4.7の拡張) 4.4.11 中国剰余定理の幾何的解釈 4.4.12(*) 例 X = Spec k[w,x,y,z]/(wz-xy)上の開集合D(y)∪D(w)を考える。(1点(y,w)を除いた開集合である。)D(y)上の関数z/yとD(w)上の関数x/wは明らかに交わりD(y)∩D(w)で一致する。従ってそれらは張り合わさってD(y)∪D(w)上の1つの切断を定める。読者は複素幾何の解析接続でこの現象を見たことがあるかもしれない。ある開集合ではz/yと描写される"正則関数"は、他の場所ではこの記述は崩壊するが、それでも解析接続することで他の場所では他の定義を使って関数を与えることができる。 好奇心がある専門家へ:この関数は、"1つの式"では描写できない!ここには興味深い幾何的現象がたくさんあり、このスキームは我々が反例を探すのに常に良い源泉となる。まだ定義していない言葉を使うが、ちらっと見ておこう。Spec k[w,x,y,z]は4次元である。我々はその部分集合である超平面、すなわち3次元なXを考えている。これはP^3における2次曲面(8.2.M)上の錘である。開集合D(y)⊂XはXからさらに超平面を除いたもので、すなわち捨てられたV(y)は余次元1である。ということは読者はV(y)∩V(w)は余次元2であるので、上記の例はハルトークスの補題の反例のように思うかもしれない。しかしそうではない。V(y)∩V(w)は実は余次元1なのである。 V(y)は2つの直線L1,M1の和集合上の錘であり、V(w)は直線L1,M2の和集合上の錘である。それらの交わりは直線L1上の錘であり、余次元1である。このL1上の錘は純粋に余次元1であるにも関わらず、集合論的にでさえ1つの方程式で切り取られることができない(14.2.R)。すなわちどんな有理式f/gも我々のD(y)∪D(w)上の関数を記述できない:どこかで0/0になってしまう。理由:我々の関数はV(y)∩V(w)で定義できないので分母gはここで消える。しかしgはL上の錘だけで消えることはできない。必ず別の場所でも消えなければならないのである。 --- 4.5 射影スキーム、Proj構成 射影スキームは多くの理由で重要である。例えば、幾何学の古典的な興味の対象や、スキームについて知る前に気にしていた対象は、おそらくすべて射影的か、あるいは準射影的(4.5.9)である。実際多くの興味のあるスキームは射影的か準射影的である。むしろ、基本的な性質を持った(例えば体上の有限型で分離的な)スキームで、準射影的でない例を与える方が難しい。 幾何学者へ:コンパクトな複素多様体で射影的でないものを見つけるのは難しく(10.3.6)、射影多様体の開部分多様体でないコンパクト複素多様体を見つけることすら難しい。だから射影的スキームは本当にありふれている。 また、射影的なk-スキームは、コンパクト性の代数幾何バージョンである固有性(10.3)に近い概念でもある。 最後に、射影スキームはアフィンスキームを貼り合わせて得られるが、貼り合わせを追跡するのは面倒である。実は貼り合わせを気にしない方法がある。環とアフィンスキームに対応があったように、次数付き環と射影スキームに(もう少しゆるい)対応がある。アフィンスキームの代わりに環で考察できたように、射影スキームの代わりに次数付き環で考察することができる。 4.5.1 古典的な幾何学からの動機 幾何的な直観に従って、古典的な実射影空間をどう考えたかを思い出そう。P^nはR^(n+1)の原点を通る直線として解釈できた。このようにしてP^nの部分集合は、R^(n+1)の原点を通る直線の集まりとして解釈できて、それが閉集合となるような集まりは、P^nの閉集合に対応した(閉集合を開集合に置き換えると同様のことは成り立たない!) またP^nをR^(n+1)の無限遠点として描写することもある。ここで、ある向きの無限遠点は、反対向きの無限遠点と同一である。この描写は、非交和分解 P^(n+1) = R^(n+1)+P^n として描写できる。すなわち、P^(n+1)は、R^(n+1)と同一視できる部分(射影座標の最終成分が0でない部分)と、その補集合となる閉集合でP^nと同一視できる部分(射影座標の最終成分が0な点)に分解できる。(射影座標は4.5.8で定義できるが、4.5.Aや4.5.Bでも使う。) こうして、P^nの部分集合Vを切り出す多項式は、R^(n+1)の部分集合も切り出すことになり、それは直線の集まりである。これをVの「アフィン錘」と呼ぶ。また、同様にP^(n+1)に含まれるP^1の部分集合を切りだすのを「射影錘」と呼ぶ。 例えば、P^2でx^2+y^2=z^2で切り取られる部分集合のアフィン錘は、R^3のx^2+y^2=z^2で切り取られる集合(2つの円錐)で、射影錘は、その無限遠点に対応する円である。 この類推を8.2.12で正確にする。 4.5.2 射影スキームの最初の描写 Proj構成の動機付けとなる、射影スキームの構成を描写する。まずは錘の解釈から始める。 4.5.A (このセクションの残りを読む前にする価値がある)練習 射影座標x0,x1,x2を持つP^2を考える。P^2でx0^2+x1^2-x2^2=0と解釈されるスキームをどう定義できるかをよく考察せよ。 ヒント:x2≠0に対応する開アフィン上では、(x0/x2)^2+(x1/x2)^2-1=0、すなわち Spec k[x02,x12]/(x02^2+x12^2-1)の形をしているはずである。ほかの開アフィン上でも同様であり、それらを貼り合わせよ。 /* 例えば Spec k[x02,x12]/(x02^2+x12^2-1)とSpec k[x01,x21]/(x01^2+1-x21^2)を貼り合わせる関係式は、(x01,x21) = (x02/x12,1/x12)である。 */ 4.5.3 指摘:P^nの次数dの超曲面 (8.2.K, 14.1.C) (n+1)変数の次数dの多項式は、C(n+d,d)次元のベクトル空間をなす。2つの多項式がP^nの同じ部分集合を切り取るなら、ほとんど、一方が他方の非零倍である。ただし、xxy=0とxyy=0のような例がある。そこで、2つの多項式が同じ「閉部分スキーム」を切り取るなら、一方が他方の非零倍である。零多項式は超曲面を切り取らない。こうして、「P^nの超曲面は、P^(C(n+d,d)-1)でパラメータ付けされる」。この直観(の特殊な場合)は繰り返し現れる。その正確な意味は28.3.5で与える。(超曲面などの用語は8.2.2で定義する。) 4.5.B より一般にP_A^nで斉次多項式f_iたちによって切り取られるスキームを描写せよ。 これを射影A-スキームの定義としてもよいが、よりよく描写するために4.5.9まで待つ。 4.5.4 次数付き環 Z-次数付き環S●は、+[n∈Z]S_nであり、乗法が次数と対応する:S_m×S_nはS_[n+m]に送られるようなものである。S_0は部分環であり、各S_nはS_0-加群であり、S●はS_0-代数である。S_nの元はS●の斉次元と呼ばれ、非零な斉次元は次数を持つ。斉次元で生成されるイデアルを斉次イデアルと呼ぶ。 4.5.C (a) Iが斉次イデアルであることは、任意のf∈Iとnに対してfのn次成分f_nがIに属することと同値である。 すなわちIはn次成分I_nの直和に分解され、自然にS●/IはZ-次数付き構造を持つ。 (b) 斉次イデアルの和、積、交わり、根基は再び斉次イデアルとなる (c) 斉次イデアルが素イデアルとなることは、I≠S●で、任意の斉次元a,bに対してab∈Iならばa∈Iかb∈Iであることと同値である。 /* (a) 左から右:Iの元は常にその形をしている。右から左:生成元を常に斉次成分に分解できる。 (b) 和と積は生成元による記述を見れば良い。交わりは(a)による。  冪根:f^n∈I として最高次の項は最高次のみに由来することからなど。 (c) 左から右は明らか。右を仮定する。非斉次元x,yに対してxy∈Iとする。  x,yの最高次をm,nとする。(a)よりxy_[m+n]∈Iなのでx_m∈Iまたはy_n∈I  例えばx_m∈Iならxからx_mを差し引いたものを改めてxとするのを繰り返せば良い。 (もしy_n∈Iでない場合はこの作業でxのすべての次数がIに属することが要求される。)*/ TがS●の斉次元のみからなる積閉集合のとき、T^-1.S●は自然にZ-次数付き環となる。 4.5.6 Z≧0-次数付き環、A上の次数付き環、有限生成次数付き環 Z≧0-次数付き環とは、負の次数の元を持たないZ-次数付き環である。 【重要】 この本の残りでは断りがない限り「次数付き環」はZ≧0-次数付き環を指すことにする(この慣習は標準的ではない)。 基礎環と呼ぶAを固定する。S0=Aのとき、S●はA上の次数付き環であると呼ぶ。鍵となる例はA[x0,..,xn]あるいはA[x0,..,xn]/I, Iは斉次イデアルでI∩S0={0}となるようなものである。ここでA[x0,..,xn]の次数は各xiが次数1を持つという古典的な次数である。 部分集合S+ = +[i>0]Si ⊂ S●はイデアルをなす。これを無縁イデアルと呼ぶ。この名前の由来はこの後明らかになるだろう。S+が有限生成イデアルのとき、S●はA上の有限生成次数付き環と呼ぶ。S●がA-代数としてS_1で生成されるとき、「S●は次数1で生成される」と呼ぶ。(これは、Sym^●S_1→S●が全射であると解釈することもできる。Symは13.5.3で議論される。) 4.5.D (a) A上の次数付き環S●が有限生成であることは、S●がA-代数として有限生成であることと同値である。 (ヒント:S+のイデアルとしての生成元は、S●の-代数としての生成元でもあることを確認せよ) (b) S●がネーター環であることは、A=S0がネーター環でかつS●が有限生成次数付き環であることと同値である 4.5.7 Proj構成 (Z≧0-)次数付き環S●に対して、ProjS●というスキームを定義する。2つの例が、このトンネルの出口を照らす。S●=A[x0,..,xn]のとき、P_A^nが得られる。S●=A[x0,..,xn]/(f), fが斉次式, のとき、P_A^nからf=0で切り取られるような何かが得られる。 環のスペクトラムで行ったように、ProjS●をまず集合として構成し、次に位相を入れ、それから環つき空間にする。・・・ 各斉次元f∈S+に対して、局所化S●_fはZ-次数環をなす。ただしdeg(1/f)=-deg(f)と考える。その0次成分 Spec(S●_f)0 を考える。(この表記はややこしい。1番目と3番目の添え字は次数付き環を表し、2番目の添え字は局所化を表している。)特にS●=k[x0,..,xn]でf=xiとおいた場合、通常の射影空間のアフィンな部品となる環を得られる。 ProjS●の点は、S●の斉次な素イデアルで、無縁イデアルS+を含まないもの、の集合と定める。 4.5.E 重要で技巧的な練習 f∈S+を斉次とする。 (a) (S●_f)_0の素イデアルと、S●_fの斉次な素イデアルの間の全単射を示せ。 ヒント:表記を単純にするためS●_f=Aとおく。環準同型A_0→AはAの斉次素イデアルをA0の素イデアルに引き戻す。逆はより難しい。P_0⊂A_0を素イデアルとして、P⊂Aをまずは集合として次のように定める:Pのi次の成分 = {a | a^deg(f)/f^i∈P_0} これがAの斉次イデアルをなすことを示し、それからPが素イデアルとなることを確認せよ。 (b) (S●_f)_0の素イデアルをProjS●の部分集合と解釈せよ。 /* 例えばf=x^2+y^2, A=(x,y)のとき、P0は (x^2/(x^2+y^2), xy/(x^2+y^2),y^2/(x^2+y^2)) となる。ここからAを復元するには、P0*f^iの形をした元だけでは不十分であり、その根基を取る必要があることが分かる(4.5.C(b))。ヒントにある構成はこれを具体的に書いたものである。 */ ProjS●の点が、斉次な素イデアルに対応するという視点は、ProjS●の視覚化に役に立つ。例えば斉次イデアル(z^2-x^2-y^2)をまずは集合として描写する:それは円錐である。4.5.1のようにP^2を"無限遠点平面"と描くことができる。そしてこのイデアルを円錐による無限遠点平面の切り取りと描くことができる。我々はこの直観を8.2.12でもう少し正確にする。 アフィンスキームの場合にならってS●の「正の次数を持つ」斉次元の集合Tに対して、零点集合V(T)を、{S●の斉次な素イデアル | Tを含むが、S+を含まない}と定義する。斉次元fや斉次イデアルIに対してV(f)やV(I)は同様に定義される。D(f)は補集合ProjS●-V(f)と定義する。後には、D(f)は単に開集合を表すだけでなく、開部分スキームをも表すことになる。・・・ (「正の次数を持つ」という制約は、定義には必要ないが、後の都合のためである。例えば我々はD(f)が開アフィンになることを望むが、f=0に対するD(f)は開アフィンにならない。) 4.5.F D(f)は4.5.E(b)で描写したSpec (S●_f)_0に対応する /* D(f)=S●の斉次な素イデアルのうち「fを含むがS+を含まないS●の斉次な素イデアル」以外 (S●_f)_0の素イデアル S●_fの斉次な素イデアル S●の積閉集合{1,f,f^2,..}と交わらない斉次な素イデアル */ アフィンの場合のように、V(I)は閉集合の公理を満たし、ProjS●上の位相を定める。これをまたザリスキ位相と呼ぶ。(他の文献の定義は表面上は異なるかもしれないが、結果は同等だろう。)アフィンの場合の多くの結果はそのまま成り立つ。例えばD(f)∩D(g)=D(fg)など。 4.5.G D(f)はザリスキ位相の開集合の基をなす 4.5.H S●を次数付き環とする。 (a) fがV(I)で消える⇔f^n∈Iとなるnが存在する。  特にD(f)⊂D(g)ならばf^n∈(g)となるnが存在する。 (b) 部分集合Z⊂ProjS●に対して、I(Z)を定義せよ。I(Z∪Z')=I(Z)∩I(Z')が成り立つ。 (c) V(I(Z))=Z ̄が成り立つ。 4.5.I 次は同値である(S+が無縁イデアルと呼ばれる由来) (a) V(I)=φ (b) Iの生成元fiに対して、∪D(fi)=ProjS● (c) √I⊃S+ 4.5.J Proj S●の位相 埋め込み D(f)=Spec (S●_f)0 → Proj S● を考える(4.5.F)。 右辺のザリスキ位相の制限位相は左辺のザリスキ位相と一致する。 4.5.K Spec (S●_fg)0 と、Spec(S●_f)0上の開集合D(g^deg(f)/f^deg(g))は同型である。 同様にSpec(S●_g)0上の開集合D(f^deg(g)/g^deg(f))とも同型である。こうしてD(f)とD(g)はスキームとして貼り合わさる。 4.5.L 3つの重なりに対してうまく振舞うことを確認せよ(2.5.D, コサイクル条件) 4.5.M ProjS●を「一貫性のある芽」の視点で再解釈せよ 4.5.8 射影空間をP_A^n = Proj A[x0,..,xn]と定義する。x0,..,xnを射影座標と呼び、これらは定義の一部をなす。 4.5.N これが以前のP_A^n(4.4.9)と合致することを確認せよ。 (D(xi)がProjA[x0,..,xn]を被覆することをどう確認する?) /* 無縁イデアルを生成するから */ D(x^2+y^2-z^2)⊂P^2がアフィンであることはこの新しい定義を使えば明らかである。それはSpec (k[x,y,z](x^2+y^2-z^2))0 である。 4.5.O kを代数閉体とする。P_k^nの閉点は古典的な点に対応した。古典的な点[a0:,,:an]に対応する斉次な素イデアルは何か? /* (a1x0-a0x1,a2x0-a0x2,..,anx0-a0xn)ということだと思う */ 4.5.P S●が次数1で生成され、f∈S+が斉次元であるとする。fをV(f)をスキームとして定義せよ。(fはProjS●の関数とは限らない。後で近いものを定義する:直線束の切断である:14.1.2) 同様に、V(I)も定義せよ(13.1.Iでより一般的な場合を扱う) /* Proj S●/f ということだろうか。 */ 4.5.9 射影スキームと準射影スキーム A上の「有限生成な」次数付き環S●に対するProjS●として書けるスキームを射影A-スキームと呼ぶ。準射影A-スキームは、射影A-スキームの準コンパクトな開部分スキームと定義する。 4.5.10 重要でない指摘 ・ProjS●という記号自体はS●が有限生成でなくても意味を持つ ・準コンパクトという要請はAがネーターな場合には不要である 4.5.11 自明な例 SpecAは射影A-スキームである。(ProjA[T]と書ける) 4.5.12 ベクトル空間の射影化PV VをA上の(n+1)次元のベクトル空間とする。Sym●V^ = k+V^+Sym^2 V^+... と定義する。 (双対^を取る理由は次の練習が示す。Symの定義は13.5.3を見よ。) この言葉で、x0,..,xnをベクトル空間Vの1次形式として解釈できる。 4.5.Q 重要でない練習 kを代数閉体とする。Vの1次元部分空間とPVの閉点の間の全単射を記述せよ。 重要でない指摘:・・・ 4.5.13 グラスマン多様体 ========== 5.スキームのいくつかの性質 連結、既約、準コンパクト、閉点、特殊化と一般化、生成点は3.6で与えた 5.1 位相的な性質(特に準コンパクトと準分離) 5.1.A P^nは既約である 5.1.B 既約閉集合と点には全単射がある(3.7.Eの一般化) 5.1.C Xが有限個のネーター環のSpecで覆われるならXは位相空間としてネーター  P_k^n, P_Z^nはネーターである。 5.1.D Xは準コンパクト⇔Xは有限個の開アフィンの合併  従ってP^nは準コンパクト 5.1.E 重要な練習 準コンパクトなスキームXは閉点を持つ (警告:準コンパクトと限らなければ閉点を持たないスキームも存在する。従って読者は議論のどこかで準コンパクト性を使うだろう。) /* SpecA⊂XをとるとSpecAには閉点pがあるが、これはXの閉点とは限らない。 Xの開アフィンによる被覆Ui=SpecAiをとる。準コンパクトなのでi=1..nと有限にできる。もし、SpecA1の閉点pがあって、点pが他のU2からUnに属していなければ、(U1-p)∪U2∪..∪Unは開集合なので、pは閉点であることが言える。 そこでU1のみで覆われる部分をVとおく。Vは空でない(さもなければU1は被覆に不要である)。Vの点qをとる(まだ閉点とは言えない)。Vは、U1の閉集合ではある(Xの閉集合とは言えない)。{q}のU1の位相における閉包q ̄をとる。従ってq ̄⊂V。3.7で定義したIを使ってA1のイデアルI(q)を考えて、I(q ̄)⊂p となる素イデアルをとればp∈q ̄ である。*/ 5.1.1 準分離スキーム: 準分離はちょっと変わった概念である。後で、これが実は射の性質であることを見る(7.3.1)。実際出会う大抵のスキームは準分離的である。 位相空間が準分離的であるとは、2つの準コンパクト開集合の交わりがまた準コンパクトであることを意味する。これは"分離的"(2つの開アフィンの交わりがアフィンである)を弱めたバージョンである。 5.1.F 準分離的であることは、  2つの開アフィンの交わりが有限個の開アフィンの合併で書けることと言い換えられる 5.1.G アフィンスキームは準分離的である 5.1.H Xが準コンパクトで準分離的(qcqs)とは  Xが有限個の開アフィンで覆えて、その2つの交わりが有限個の開アフィンの合併で書ける 5.1.I 射影Aスキームはqcqsである 5.1.J 準分離的でない例:Spec k[x1,x2,...] の原点を二重化したスキーム 5.1.2 次元:これはより難しいので11章にゆずる --- 5.2 被約と整 関数は点での値で決定されないことがスキームについての注意点の1つであったことを思い出そう。それは、冪零元の存在のせいであった(3.2.11)。 5.2.1 定義 環は非零な冪零元を持たないとき被約と呼ぶ。 スキームXはすべての開集合での構造層O_X(U)が被約であるときに被約と呼ぶ。 5.2.A 被約性は茎局所的である。 スキームが被約である⇔すべての茎が非零な冪零元を持たない 従って被約スキームにおいてが2つの関数(O_Xの大域切断)f,gがすべての点で一致するならばf=gである。 (2つのヒント:2.4.A, 3.2.12) 5.2.B 環Aが被約ならばSpecAは被約である。A_k^nとP_k^nは被約である。 4.2で紹介したスキームSpec k[x,y]/(yy,xy)は被約でない。 5.2.C ( k[x,y]/(yy,xy) )_x は非零な冪零元を持たない。 (ヒント:別の環と同型であることを示せ。3.2.Lが参考になるかもしれない。) またSpec k[x,y]/(yy,xy)の非零な冪零元を持つ茎は原点の茎のみである。 /* 前半:非零な冪零元の候補はyしかないが、xを含む積閉集合での局所化でyは0に移る[y/1 = xy/x]。 後半:ある素イデアルでの茎がxを含まない積閉集合での局所化となるには、xがその素イデアルに属する必要がありそのような素イデアルは(x,y)だけである。*/ 5.2.D 重要でない練習 Xが準コンパクトスキームなら、被約性は閉点だけ確認すれば良い。 ヒント:被約性が開集合ごとに確認できることを示そうとするな(次の指摘を見よ)。代わりに、被約でない点があったとすると、その閉包に被約でない閉点が存在することを示せ(5.1.Eが利用できる)。この結果は興味深いが、我々は使わない。 /* A/pに非零な冪零元fが存在するとしたら、D(f)すなわち局所化(A/p)_fを考えると良いだろう。 */ 5.2.2(*) 指摘 X = Spec C[x,y1,y2,...] /(y1y1,y2y2,...,(x-1)y1,(x-2)y2,...)を考える。Xの底空間はSpecC[x]の底空間と同じで、Xの被約でない点は正の整数に対応する点である。この集合の補集合はザリスキ開集合でない(この環は5.5の性質(B)の段落の後で再度登場する)。 一方で5.5.5で(13.7.2で再び)Xが局所ネーターならば被約な点の集まりは開集合であることを見るだろう。 5.2.3(*) 熟練者への注意 Xが被約ならば大域切断の環は被約である。しかし逆は正しくない。例えばP^2からx^2=0で切り出されるスキームなど。(この例は18.1.6で再度登場する。) 5.2.E Xが準コンパクトでfがXのすべての点で消える関数とする。f^n=0となるnが存在する。 また、Xが準コンパクトでない場合には成り立つとは限らない。(こちらはより重要でないが、なぜ我々が準コンパクトを好むのかを示す。) ヒント:Spec k[ε]/(ε^n) の無限個の非交和を考えよ。この例は8.3.2で再度登場する。 5.2.4 定義 スキームXが整であるとは、Xが空でなく、すべての開集合Uに対してO_X(U)が整域であることを言う。 5.2.F 重要な練習 整である ⇔ 既約かつ被約 (すなわち、「1つのパーツからなるもやもやを持たないスキーム」) ヒントかもしれない:4.3.G /* 次のような特徴づけができる 既約⇔√0が素イデアル、被約⇔√0={0} */ 5.2.G SpecAが整 ⇔ Aが整域 5.2.H Xを整スキームとする。μをXの生成点、SpecAをXの空でない開アフィンとする。 生成点での茎OX_μは自然にK(A)と同一視できることを示せ。 こうして、整スキームの「関数体」が定義でき、それはXの任意の空でない開集合を使って計算できる。名前の由来:X上の有理関数と思うことができる(5.5.6, 5.5.Q) /* 既約閉集合上の開集合は常にその生成点を含む。 */ 5.2.I Xを整スキームとする。構造層の制限写像O_X(U)→O_X(V)はV≠φならば埋め込みである。 U≠φならば生成点の茎への写像O_X(U)→OX_μ=K(A)も埋め込みである。 /* 局所化は積閉集合が零因子を持たなければ埋め込みであった */ こうして整スキーム、特に既約な多様体(後で定義される)は異なる開集合上の関数を、同じ環の元として扱うことができるという便利な性質を持つ。特に、制限写像はいつも埋め込みなので、貼り合わせが簡単である。開集合属Ui上のfiは、それがK(X)上の同じ元を与える時のみ貼り合わさる。これが、既約な多様体が通常スキームより前に紹介される理由である。 5.2.5 整という性質も茎局所的ではない。Spec(A×B)を考えよ(3.6.A)。しかし茎局所的に近い性質はある。5.3.Cを見よ。 --- 5.3 アフィン局所的に確認できる性質 この章はスキームの定義に現れる微妙な問題に対するものである。我々はスキームをアフィンスキームで被覆される環つき空間として定義した。従って被覆となるアフィンスキームは知っているが、2つの被覆の間の交通については知らない。例えば、私がアフィン被覆を持っていて、読者が別のアフィン被覆を持っている時、私が私の被覆を使って何かを計算したときに、それを読者の被覆を使った計算に翻訳する方法が存在するべきだ。アフィン交通補題5.3.2がこの問題に対する便利な方法を与える。 この補題のおかげでスキームに関する多くの重要な性質を定義できる。それらは開被覆ごとに確認することができる。これを"アフィン局所的"な性質であると呼ぶ。(後でより正式に述べる。)我々はこの性質を好む。例えば対象のスキームが準コンパクトなら有限個の開アフィンで確認すれば良い。 5.3.1 SpecAとSpecBがスキームXの開アフィンだとする。SpecA∩SpecBは、それらの同時基本開集合の和集合で表せる。 p∈SpecA∩SpecBを考える。pを含むAの基本開集合Spec A_fで、SpecA∩SpecBに包まれるようなものをとる。(SpecA∩SpecBはSpecAの開集合だから、SpecAの基本開集合の和集合である。そのうちpを含むものを選ぶ。)次に、Spec B_gとして、pを含む基本開集合で、SpecA_f∩SpecBに包まれるようなものをとる。そうすると、切断g∈B=Γ(SpecB,O_X)は、SpecA_f上の切断g'に制限される。g'=g"/f^n とおけば、Spec B_g = Spec A_fg" であり従ってこれはSpecAの基本開集合でもある。 5.3.2 アフィン交通補題 スキームXの開アフィンに関する性質Pが (1) SpecA が性質Pを持つならば、SpecA_fも性質Pを持つ (2) Spec A_fi (i=1,2,..,n)たちがすべて性質Pを持ち、Spec A_fiがSpec Aを被覆するならば、SpecAも性質Pを持つ を満たすとする。このときXを被覆する(一組の)開アフィンが性質Pを持てば、Xの任意の開アフィンは性質Pを持つ。 このような性質を、アフィン局所的な性質と呼ぶ。(例えば被約性はアフィン局所的である。なぜだか分かるか?)UがXの開部分スキームのとき、UはXのアフィン局所的な性質を引き継ぐことに注意しよう。茎局所的な性質は常にアフィン局所的であることにも注意しよう。しかし開アフィンの性質に相当する茎の性質が明らかでない時もある。(例えば次の章で現れる正規性) [5.3.2の証明] Xの開アフィンによる被覆 Spec X_iが性質Pを満たすことが分かっているとする。 SpecAを任意の開アフィンとする。 p∈SpecAに対して、AとどれかのSpecX_iの同時基本開集合U_pをとる(5.3.1)。 仮定の(1)によりU_pは性質Pを持つ。 SpecAはU_pで被覆されるが、SpecAの準コンパクト性によりそのうち有限個のもので被覆される。 従って仮定の(2)によりSpecAは性質Pを持つ。 性質Pを適切に選ぶことで、スキームに関する重要な性質を定義する。 5.3.3 Aを環として、(f1,f2,..,fn)=Aとする。 (a) Aがネーター環 ⇔ それぞれのA_fiがネーター環 (b) Bを環としてAをB-代数とする。  Aが有限生成B-代数 ⇔ A_fiが有限生成B-代数 これらはあと5.3.9で示す。先にこれを前提としていくつかの興味深い定義と結果を紹介する。(訳者補足:先に持ってくることにした。) 5.3.9 [5.3.3の証明] アフィン交通補題5.3.2の(1)(2)に対応する主張をそれぞれ示す: (a)(1) I1⊂I2⊂..をA_fのイデアルの包含列とすると、局所化の標準的準同型A→A_fの逆像によって、Aのイデアルの包含列 J1⊂J2⊂.. を復元する。 Im≠InならばJm≠Jnとなる。 (a)(2) J1⊂J2⊂.. をAのイデアルの包含列とする。それぞれのA_fiに対応する包含列 I1i⊂I2i⊂.. が定まる。ここで、Jm≠Jnならば、どれかのfiに対してImi≠Iniとなることが鍵となる。4.1.2の構造層の性質を使うことになる。 /* a≠b ならばどれかの局所化への像は異なる。これは構造層が一致の公理を満たすことを言っている。*/ (b)(1) これは明らかである (b)(2) 1 = Σci*fi とおくことができる。 A_fiの生成元を fij/fi^kj とおく。私は{fi}∪{ci}∪{rij}がAを生成することを主張する。説明する。r∈Aをとる。A_fiへの像で、rをrijとfiの多項式を、fiの(大きな)冪で割ったものとして表せる。ここでも4.1.3で使った「貼り合わせに関する重要なテクニック」によって、この分母を消去することができる。 --- 5.3.4 Xがネーター環によるアフィンスキームで被覆できる時、Xを局所ネータースキームと呼ぶ。さらにXが準コンパクトであるとき、つまり有限個のネーター環によるアフィンスキームでXを被覆できる時、Xをネータースキームと呼ぶ。(我々は今後、Xが性質Pを持つときに局所Qと呼び、さらにXが準コンパクトであるときにQと呼ぶ、という例を多く見るだろう。) 5.1.Cにより、ネータースキームの底空間はネーター空間である。従って3.6.Tにより、ネータースキームの任意の開集合は準コンパクトである。 5.3.A 局所ネータースキームは準分離的である。 /* SpecA∩SpecBは一般には無限かもしれないアフィンスキームで被覆されるが、ネーター性によって被覆は有限である。*/ 5.3.B ネータースキームの既約成分は有限個である(ヒント3.6.15)。 従ってネータースキームの連結成分は有限個であり、それぞれは有限個の既約成分からなる(3.6.Q)。 /* Xはネーター位相空間であるから3.6.Qの議論が使える。 */ 5.3.C ネータースキームXが整であることは、Xが空でなく、連結であり、すべての茎が整域であること同値である。 ヒント:整=既約+被約であった。茎が整域なら、茎は被約であり、被約は茎局所的な性質であった(5.2.A)からXは被約である。ネータースキームは有限個の既約成分からなるのであった。もし既約成分が複数あるとすると、連結性によりそれらは交わるが、交わりにおいて茎は整域でなくなる。 (空でない連結なスキームが既約であることは、開集合な既約成分で被覆されることと同値であることを使って局所ネータースキームに対する結果に拡張しても良い 。) 5.3.4 局所ネータースキーム:ネーター環のアフィンスキームで覆える 5.3.5 重要でない注意 ネータースキームの切断はネーター環とは限らない(19.11.H) 5.3.6 A-スキーム 特定の体上で考察したい時がある。そこでA-スキームという概念を定義する。A-スキームとは、すべての構造層がA-代数であり、制限写像がA-代数の準同型になっているようなものと定義する。(準分離性などと同様に、この性質は6.3.Gで相対的な射の性質として見直される。) XをA-スキームとする。XがSpecBiで被覆されて、各Biが有限生成A-代数のとき、XはA上局所有限型であるとか、局所有限型A-スキームであるとか言う(7.3.12でまた扱う)。さらにXが準コンパクトであるとき、Xを有限型A-スキームと呼ぶ。ネーター環上の局所有限型スキームは局所ネーターであり、ネーター環上の有限型スキームはネーターである。幾何的な例は: ・Spec A[x1,..,xn]/Iは有限型A-スキームである ・P_A^nは有限型A-スキームである 5.3.D ・Xが準射影A-スキーム(4.5.9)ならばXはA上有限型である・・・ ・射影A-スキームの開部分スキームで、準コンパクトではない例を挙げよ (ヒント:4.5.11) 5.3.7 k-アフィン多様体、k-射影多様体:被約有限型 5.3.8 局所有限型k-スキームの閉点の次数:剰余体の拡大次数 5.3.G(**) 複素多様体の解析化 --- 5.4 正規性と分解性 ・・・ Xはすべての茎が整閉整域のとき、正規と呼ぶ。 5.4.A 整閉整域性は局所化に対してよく振舞う。  Aが整閉整域で、Sが0を含まない積閉集合なら、S^-1は整閉整域である。 /**/ 従って、SpecAが正規であることは、Aが整閉整域であることと同値である。 正規なスキームは整とは限らない。 例えば、Speck+Speck = Spec(k×k)=Spec k[x]/x(x-1) 5.4.2 逆にSpecAのすべての茎が整閉整域ならAは整閉整域  すべての閉点の茎が整閉整域ならAは整閉整域 /* 後者を示せば十分である。Aが整閉整域でないと仮定する。 X^n+a[n-1]*x^(n-1)+...+a0 = 0 の解sがK(A)-Aにあるとする。 sの分母によるイデアル I={r∈A|rs∈A}を考える。 仮定により、I≠Aである。 このIを含む極大イデアルmをとれば、解決する。*/ ・・・ --- 5.5 関数が支えられている場所:随伴点 随伴点とはスキーム上の関数の本質的な情報を捉える鍵となる点である。これを描写する方法はいくつかあるが、多くは代数的である。我々は、幾何的動機から始める標準的でない視点をとることにする。随伴点は冪零元と生成元両方と関係する。どちらも古典的な幾何的直観にはないのでそれを幾何的に捉えるには時間がかかるかもしれない。まず、2つの方法で動機づける例を提示する。それから鍵となる性質を議論する。最後に、適切な代数的な定義と証明を与える。数学でよくあるように、証明を覚えるより性質を覚える方がずっと重要である。 随伴点には他の視点もある。代数的に重要な視点は、5.5.13で軽くのみ言及した準素分解の視点である。 5.5.1 もやもやの中心としての随伴点 被約でないスキームSpec[x,y]/(yy,xy)のもやもやした絵(4.2)を思い出そう。我々は原点にもやもやがあって被約でない振舞いを示していると紹介した。これは練習5.2.Cで正当化された。原点は構造層の茎が被約でない唯一の点である。こうして、被約度合いの違いは、既約閉集合の単位で分布する。先の例では原点とx軸全体である。既約閉集合は素イデアルと全単射の関係があったから、被約度合いの違いはx軸の生成点[(y)]と原点[(x,y)]で分布しているとも言える。この2つの点がこのスキームの随伴点である。 5.5.2 切断の台となる既約成分の生成点としての随伴点 我々はこれから上記のスキームに関する一見関係ない練習問題を与える。 スキーム上の関数の台(定義は2.4.2、芽が0でない所)は閉集合であることを思い出そう。 5.5.A Spec k[x,y]/(yy,xy)の関数fを考える。fの台は空集合か、原点か、全体のどれかである。 /* (芽が0でない所と値が0でない所の違いに注意せよ。例えば関数(x-1)はp=(x-1)で値としては消えるが芽としては消えない。) 原点での茎は{f(x)/g(x)+b*y|g(0)≠0,xy=yy=0}とおける。特にこの局所化は単射となっているから、原点で芽として消える関数は0のみである。 一方で5.2.Cでコメントしたようにxを含む積閉集合での局所化でyは0に移るから、開集合D(x)では関数yが芽としても消える。 そういうわけでf=0のとき台は空集合、0≠f∈(y)のとき台は原点のみ、それ以外のとき台は全体である。 */ 2つの視点から同じ閉集合が現れたのは偶然ではない。5.5.Aに現れた既約成分の生成点が、Spec k[x,y]/(yy,xy)の随伴点なのである。 我々はまずアフィンの場合(SpecA)について議論する。【この章を通してAはネーター環とする。】より一般に、ネーター環A上の有限生成A-加群Mの随伴点を考える。環の言葉を使うときには随伴点というより随伴素イデアルということになる。これらはより一般的に定義されるものであることについては5.5.Qで触れる。 我々はこれから随伴点に関して3つの重要な性質を述べる。最初のが最も重要である: (A) Mの随伴点とは、あるMの元の台となる既約成分の生成元となるような点である 例えば、5.5.Aにより、Spec k[x,y]/(yy,xy)は2つの随伴点を持っていた。別の例を与えよう: 5.5.B (性質(A)を前提とする) Aを整域とする。生成点がSpecAの唯一の随伴点である。 /* すべての局所化は単射だから */ ・・・ 次の性質は(A)をもっと衝撃的なものにする: (B) Mの随伴点は有限個である 例えばSpecAが有限個の既約閉集合しか持たなければ、随伴点の候補はそれらしかない。読者はこれを意外に思うかもしれない。読者はこれをネーター性の要求する有限性として解釈すると良い。(我々はこれを、3.6.15:ネーター環のスキームには既約成分が有限個しかないことの拡張であるとみることができる。ネーター性を仮定しない場合に何が起きるかを見るには5.2.2の例が良い。) そういうわけで随伴点はスキームにとって重要な点なのである。 我々は後でSpecAのどの部分集合がMの元の台、すなわち随伴点の閉包となり得るかを、完全に描写できることを見る(5.5.O)。 5.5.3 我々は性質(A)から、既約成分で1となるような関数を考えることで、M=AならばSpecAの既約成分の生成点が随伴点となることが分かる。それ以外の随伴点は、「埋め込まれた点」と呼ばれる。従って、Spec k[x,y]/(yy,xy)の原点は埋め込まれた点である。 5.5.C (性質(A)を前提とする) Aが被約ならばSpecAには埋め込まれた点は存在しない /* ・f∈Aとする。Aが被約ならばfの台はD(f)の閉包であることを示す。 fが点pで芽として消える ⇔fがA-pによる局所化で消える ⇔fs=0 となる s∈A-pが存在する ⇔開集合D(s)があって、p∈D(s), D(s)∩D(f)=D(fs)=φ ⇔開集合Uがあって、p∈U, U∩D(f)=φ [開集合は基本開集合の和集合である] ⇔pはD(f)の閉包の外にいる (fs=0 と D(fs)=φ の同値性で被約という仮定が使われている) ・次にD(f)の閉包の既約成分Wへの制限は、空かW全体のどちらかである。 D(f)は開集合で、既約成分では空でない開集合は稠密だから。 */ さらに、M → ΠM_p は単射である。(これは重要な性質である。ここでも随伴点は物事が起きる点を表している。)なぜならm∈Mがすべての随伴点での芽が消えるとすると、mの台は随伴点を含まないので(A)よりmの台は空集合である。すなわちm=0である。 5.5.4 Xの開部分スキームU⊂Xは、任意の開集合V上の任意の関数がV∩Uへ制限すると0になるようなとき、スキーム論的稠密であると言う(我々はこの用語は今後使わない)。Xが局所ネーターならば、U⊂Xがスキーム論的稠密であることは、Uがすべての随伴点を含むことと同値である。 5.5.D (性質(A)を前提とする) m∈Mとする。mの台はmが非零な芽を持つ随伴点の閉包である。 5.5.E (性質(A)(B)を前提とする) A_pが被約でないような点pの集合は、SpecAの随伴点のうち茎が被約でないものの閉包である。 /* A_pに0でない冪零元tが存在することを考える。 関数tが芽として消えない所では、tは局所化で消えないので引き続き0でない冪零元となる。 茎が被約な随伴点の閉包の和集合=茎が被約な随伴点の閉包(ここで(B)を使う) */ 5.5.5 上記の練習5.5.Eが、随伴点ともやもやの関係を部分的に説明する。(5.5.13の準素イデアル分解がより正確な視点だが我々はその議論はしない。)将来の参照のために述べておくと、Yが局所ネーターならば、Yの被約な点はYの開集合となる。 (C) Aの元fがMの零因子である(fm=0,m≠0が存在する)⇔fがMのある随伴点で消える (つまりfがaある随伴素イデアルに属する) 5.5.F 重要な練習 (性質(A)を前提とする) (A)による随伴点の定義は、局所化と整合的に振舞う。Sを積閉集合とすると、S^-1.Mの随伴点は、Mの随伴点でSと交わらないものと対応する。 /* pを、Sと交わらないAの素イデアルとする。定義を追う。 ・pがMの随伴点 ⇔0≠m∈M, x∈pがあってxm=0 かつ pの元でないy∈Aに対してはym≠0 ・pに対応するS^-1.Aの素イデアルp'がS^-1.Mの随伴点 ⇔0≠m/s∈S^-1.M, x∈p'があって xm=0 かつ p'の元でないyに対しては ym≠0 ⇔0≠m/s∈S^-1.M, x∈pがあって xm=0 かつ pの元でないy∈A, s∈Sに対して sym≠0 pがSと交わらないことからこれは先と同値である。 */ こうして、随伴点は"局所的に決定"できる。例えば、Aの随伴点は構造層の茎を見ることで確認できる(茎局所的)。別の例として、Mの随伴イデアルはSpecAの開被覆で考察することができる。5.5.Fのおかげで、局所ネータースキームXに対する随伴点を定義できる:pを含む開アフィンSpecAでの随伴点かどうかで定義できる。これはSpecAの取り方によらない。 (ここで我々はM=Aの場合だけ一般化した。準連接層を使った一般化は13.6.5で扱う。) 以上の性質を組み合わせることで、重要な事実をいくつか得る。 ・ネータースキームでは随伴点は有限個である。 ・局所ネータースキームでの関数の台(の既約成分)は、随伴点の閉包の和集合の形である。 ・局所ネータースキームの既約成分の生成点は随伴点である。(そうでない随伴点は「埋め込まれた点と呼ばれる」) ・被約な局所ネータースキームは埋め込まれた点を持たない。 ・局所ネータースキームの被約でない所は、被約でない茎の閉包である。 整スキームではΓ(U,O_X)をK(X)の部分環とみなせた。随伴点を使ってこの概念を一般化することができる。 5.5.G (性質(A)(B)(C)を前提とする) Xを局所ネータースキームとする。 Γ(U,O_X) → Π[pはUにある随伴点] OX_p は単射である。 ("関数は随伴点での振る舞いで決まる") /* Uを開部分スキームとみなすと、5.5.Fより、Uの随伴点は、UにあるXの随伴点である。 従って5.5.Cの後ろに書いてある議論の通りとなる。 */ 我々はこの性質を使ってスキームの視覚化をより数学的な情報を正確に捉えた方法に改善することができる。 5.5.H もやもやした絵について (性質(A)(B)(C)を前提とする) X=Spec C[x,y]/I の随伴点が (y-x^2),(x-1,y-1),(x-2,y-2)であるとする。 (a) Xをもやもやをつかってスケッチせよ。 (b) Xが被約であるかどうか分かるか (c) x+y-2が零因子かどうか分かるか。x+y-3やy-x^2はどうか。 (後で練習5.5.RがこのようなXの具体的な例を示す) /* (b) 絵からは(x-1,y-1)で被約ではない。 底空間より、√I = (y-x^2)∩(x-1,y-1)∩(x-2,y-2)である。 Xが被約だと仮定すると、I=(y-x^2)∩(x-1,y-1)∩(x-2,y-2)である。 この場合 (y-x^2)⊂(x-1,y-1)なので I=(y-x^2)∩(x-2,y-2)である。 具体的には I=( (y-x^2)(x-2),(y-x^2)(y-2)) しかし(x-1,y-1)の閉包を台に持つ関数f≠0の存在が要求される: (x-1,y-1)に属するが(y-x^2)には属さない元f'がfを零化する。 しかしIが上記の形のときそのような零因子は存在しない (c) x+y-3は零因子ではない。y-x^2は零因子である。x+y-2は分からない。 [(y-x^2)の閉包を台にもつ関数g≠0があってg'∈(y-x^2)がgを零化する] */ 次の練習は、超曲面は埋め込まれた点を持たないことを示す。(もちろん5.5.Cによって超曲面が被約でない場合に興味がある。) 5.5.I (性質(A)(B)(C)を前提とする) 0≠f∈k[x1,..,xn]に対して、A:=k[x1,..,xn]/(f)は埋め込まれた点を持たない。 ヒント:gを零因子とするとgはfと共通因子を持つことを示せ。 (読者はk[x1,..,xn]がネーターな一意分解整域であることを使うだろう。この結果は18.6.3で使い、26.2.7で拡張する。) /* 因数分解をf=f1^c1*..*fn^cnとおいてしまえば、V(f1)等が既約成分。 gの台は、gとfkが共通因子を持たないようなV(fk)の和集合と描写される */ 5.5.6 定義:有理関数 局所ネータースキーム上の有理関数とは、随伴点をすべて含む開集合Uに対する、5.5.Gの単射の像となる元である。 ・・・ 5.5.8 最後に随伴点の定義をして局所ネータースキームでは性質(A)(B)(C)を満たすことを示す。 (D) 素イデアルp⊂AがMに随伴するとは、pがMのある元の零化イデアルとなることを言う。  すなわち p={a∈A|am=0} となるようなmが存在することを言う。 5.5.9 あるいは、MがA/pと同型な部分加群を持つと言い換えられる。  Mに随伴する素イデアルの集合はAss_A(M)と表記される。ややこしいことに、AのイデアルIに対しては、A-加群A/Iに随伴する素イデアルを、Iの随伴素イデアルと言う。このややこしさは私のせいではない。 5.5.10 定理(随伴素イデアルの性質) Aをネーター環としてMを有限生成A-加群とする。 (a) Ass(M)は空でない有限集合。 (b) 自然な準同型 M→Π[p∈Ass(M)] M_p は単射 (c) Mの零因子は∪[p∈Ass(M)] p (d) Ass_{S^-1.A} S^-1.M = Ass_A(M)∩Spec S^-1.A 以下の一連の練習でこれを証明する。 5.5.J 重要な練習 M≠0をA-加群とする。I⊂Aが零化イデアルとなる真のイデアルのうち極大ならば、Iは素イデアルで、従ってI⊂Ass(M)である。従ってAがネーター環ならばAss(M)は空でない。 (これは「なんらかの性質を持つ真のイデアルで極大なものは素である」という精神の例である) /* xy∈I だが x,yはIの元でないと仮定する。xym=0とおく。 ym≠0なら(ymをmとして採用し直せば)x∈IとすることができるのでIはまだ極大でない ym=0ならy∈IとすることができるのでIはまだ極大でない。 */ 5.5.K Mはネーター環A上の加群とする。 m=0 ⇔ Mの随伴点での局所化がすべて0 (ヒント:前の練習を利用せよ) /* m≠0とする。mが生成する部分加群をM'として前の練習を利用する。 M'は随伴点pを持つ。すなわちM'はA/pAと同型な部分群を持つ。 従ってMもA/pAと同型な部分群を持ち、m_pは0でない。 (pと素なAの元はA/pAの生成元に対応する元を零化しない) */ 5.5.L 0→M'→M→M"→0をA-加群の短完全列とする。 AssM' ⊂ AssM ⊂ AssM'∪AssM" (参考:mが零化イデアルpを持つならAmはA/pと同型である) /* m≠0, am=0 ならば am'=0となる0≠m'∈M'あるいはam"=0となる0≠m"∈M"が存在することを示す。 mのM"への像が0でないならば、それがm"として適する。 mのM"への像が0ならば、m'→mと映るm'が適する。 */ 5.5.M Mをネーター環A上の有限生成加群とする。 (a) 0=M0⊂M1⊂...⊂Mn=M となる列で、M[i+1]/M[i]がある素イデアルp[i]を使ったA/p[i]に同型となるような列が存在する。 (b) 随伴素イデアルは(a)の列に現れる素イデアルに含まれる。 (c) Supp A/p[i] は Supp(M) に含まれる。言い換えると、すべてのp[i]は随伴素イデアルを含む。 (ヒント:そうでないとするとpiはMを消してしまう) (注意:p[i]自体は随伴素イデアルとは限らない。例えばM=A=Z, 0⊂2Z⊂Z) 5.5.N 定理5.5.10の(d)について: (a) Ass(M)∩Spec S^-1.A ⊂ Ass_{S^-1.A} S^-1.M を示せ (b) q∈ Ass_{S^-1.A} S^-1.M に対して、pを対応するAの素イデアルとする。  q=Ann{S^-1.A}m は Hom_{S^-1.A} (S^-1.A/q, S^-1.M)の自明でない元を生成する。  ここから1.6.Gを使って Hom_A(A/p,M)≠0を導け。 /* (a) M加群の元としてm≠0, Aの素イデアルIがあって Im=0とする。 ・s∈Sに対して常にsm≠0ならば、S^-1.A加群としても m≠0, Im=0 ・ある s∈Sに対して sm=0だとする。Iとsは互いに素だからm=0になってしまう。 (b) S^-1.M には S^-1.A/q と同型な部分加群がある。という視点。*/ これで定理5.5.10の証明が終わる。残りは、随伴点を台の言葉で理解するための重要なおまけである。 5.5.O SpecAの部分集合で、Mの元の台となるようなものは、随伴点の閉包である。 (ヒント:随伴点pに対して、pの閉包を台とする切断があることを示せ) (これを次の5.5.Pに使うことができるが、先に5.5.Pをすることを好む人もいる) 5.5.P 重要な練習 Aをネーター環、Mを有限生成A-加群とする。Mの随伴点が性質(A)を満たすことについて: (a) 随伴点はあるm∈Mに対するSupp(m)の既約成分の生成点である (b) m∈Mに対してSupp(m)はmが0でない芽を持つ随伴点の閉包である。 (ヒント:) /* (a) pが随伴点とする。mの零化イデアルとする。  そうするとSupp(m)とはpに包まれる素イデアル、すなわちpの閉包である。 (b) mの零化イデアルをIとおく。Supp(m)とはIに包まれる素イデアル。  Iに包まれる素イデアルのうち極大なもの の閉包 と言い換える(5.5.J)。 */ 5.5.11 おまけ:ネーター以外の場合 5.5.Q ネーターとは限らない整スキームでの随伴点 (A)を定義として使い、(B)と(C)を確立せよ ・・ 5.5.R 5.5.12 指摘 5.5.3と(C)を組み合わせると、Aがネーター環ならば任意の極小素イデアルpは零因子であることが言えるが、これはネーター環という仮説なしに成り立つ。 ・・・ この事実は11.1.Gで使う 5.5.13 準素イデアル この用語は重要であるが、我々は使わない。(I⊂Aが準素イデアルとはxy∈Iならばx∈Iまたはy^n∈Iとなるn>0が存在することを言う。) 随伴素イデアルは・・・ ========== 6 スキームの射 6.1 導入 スキームの間の射を描写する。射に関するいくつかの簡単に述べられる性質を定義するが、より微妙な性質は後回しにする。 スキームは(1)集合であり、(2)位相空間であり、(3)構造層を持つことを思い出そう。同様に、スキームの射の定義も三段階に行う: (1)集合としての写像、(2)位相空間としての写像、(3)それから構造層に関する情報 である。 アフィンスキームでは、集合としての写像の定義と、それが連続であることを既にみた(3.2.9, 3.4.H) ここではスキームの射が振舞うべき2つの姿を提示する。1つ目は代数的な姿で、2つ目は幾何的な姿である。 6.1.1 代数的な姿 アフィンスキームの射SpecA→SpecBは、環準同型B→Aであってほしい。 我々は既に環準同型B→Aが逆方向の位相空間の写像を引き起こすことを見た(3.4.H) 関数のなす構造層の情報をどのように射に組み込むかを見たときに、スキームの射は、"アフィンにおいてはこのような姿をしている"べきである。 6.1.2 幾何的な姿 3.1.1での古典的多様体の理論に基づき、スキームの射は最低でも環つき空間の射であってほしい。 π:X→Yを微分可能多様体の間の微分可能写像とすると、Y上の微分可能関数はX上の微分可能関数に引き戻される。より正確には開集合U⊂Yに対して自然な写像Γ(U,O_Y)→Γ(π^-1(U),O_X)がある。これは制限写像と可換である。すなわち我々は層準同型O_Y→π_* O_X を考えたことになる。スキームの射π:X→Yについても同様にO_Y→π_* O_Xを誘導するべきである。しかし微分可能多様体の圏では連続写像π:X→Yが微分可能多様体の間の微分可能写像であるとは、ちょうどY上の微分可能関数はX上の微分可能関数に引き戻されることを意味する。つまり層準同型O_Y→π_* O_X こそが微分可能多様体の圏での射を特徴づけているのである。そういうわけで我々はこれを微分可能多様体の圏での射の定義として使うこともできる(3.1.A)。 スキームの圏でこれを適用するにはどうしたら良いか。微分可能多様体の圏では連続写像は関数の引き戻しを"誘導"し、我々はそれが微分可能な関数に引き戻されるかを考えることができた。しかしスキームの圏では関数の振る舞いはより奇妙であり、位相空間の写像だけから関数の引き戻しを復元することができない。これを定義に組み込むことが正解である:射を定義するには、位相空間としての写像π:X→Yと構造層の引き戻しπ#:O_Y→π_* O_X を同時に定義するというわけである。この考察が環付き空間という圏の定義に至る。 読者はスキームの射を環付き空間の射として定義したいかもしれない。しかしこれは実は正しくない。6.1.1が満たされないのである。スキームの射A→Bに対してSpecB→SpecAが誘導されるが、この方法で現れない環付き空間の射SpecB→SpecAが存在するのである(6.2.E)。改善するには、局所的にみることになるが、これは実際に扱うには適していない視点なので、我々は別の視点を使う。その結果が同等であることを6.3.Cで見る。 まずは環付き空間の射を正式に定義するところから始めよう。 --- 6.2 環つき空間の射 6.2.1 定義 環つき空間の射 π:X→Y とは位相空間としての連続写像(残念ながらこの写像のこともπと呼ぶ)と、"引き戻し写像" O_Y→π_*O_Xの組のことである。2.7.1の随伴性から、これはπ^-1 O_Y→O_Xを考えるのと同じである。 射の合成の明らかであり、これにより環付き空間の圏を得る。従って自己準同型と同型も描写することができる。読者は環付き空間の同型が以前4.3.1で描写したものと同じものだと確認できるだろう。 U⊂Yが開集合なら、開埋め込みという自然な環付き空間の射U→Yがある(2.7.G)。 6.2.A 環つき空間の射は貼り合わさる (X,O_X),(Y,O_Y)を環つき空間とする。X=∪Uiとして、環つき空間の射たちUi→Yは交わりで一致するとする。環つき空間の射X→Yが誘導される。(2.2.Fが本質的にこれの位相空間の場合を示している) /* Xの開集合Uに対して、UからYへの環付き空間の射を返す前層を考える。被覆Uiに関してコサイクル条件を満たすことから2.5.Dの方法で層をなす。なので交わりで一致している切断たち(=それぞれの開集合からYへの射たち)は貼り合わさる。 */ 6.2.B 簡単で重要な練習:O-加群の押し出し(2.3.B) 環付き空間の射π:X→Yにより押し出しは関手Mod_O_X→Mod_O_Yを誘導する。 /* f:F→GをO_X-加群の層の準同型とする。VをYの開集合とするとπ_*F(V)→π_*G(V)は、Vの逆像をUとしてF(U)→G(U)で定まる。 F(U),G(U)には、O_X(U)の元によるスカラー倍の構造があった。環付き空間の射の定義により環準同型O_Y(V)→O_X(U)が定まっているので、これと合成することで、π_*F(V),π_*G(V)にO_Y(V)の元によるスカラー倍の構造を入れることができる。 */ 6.2.C 簡単で重要な練習:茎の準同型(2.3.A) 環付き空間の射π:X→Yでπ(p)=qとおくと茎の準同型OY_q→OX_pが誘導される。 /* OY_qの代表類(s,V),(s',V')が同じ元を定めるなら、π#(s)とπ#(s')はOX_pの同じ芽を与える。 なぜなら、(s,V),(s',V')は共通する代表類(s",V")に制限され、層準同型π#は制限と可換だから、π#(s)とπ#(s')はどちらもπ#(s")に制限される。 */ 6.2.D 鍵となる練習 環準同型π#:B→Aが先に与えられたとする。環付き空間の射π:SpecA→SpecBを次のように定義する: 位相空間としての写像は素イデアルの引き戻しで定める(3.4.H)。層準同型O_SpecB→O_SpecAを記述するには基本開集合での準同型を記述すれば良い。D(g)⊂SpecBにおいてO_SpecB(D(g))→O_SpecA(π^-1(D(g)))=O_SpecA(D(π#(g)))をB_g→A_π#(g)によって定める。これがgの取り方によらないことを確認し、確かに層の準同型を与えることを確認せよ。(これが3.2.9, 3.4.Hとともに、6.1の冒頭に述べた三段階を完成させるものである。) /* D(g1)=D(g2)ならばB_g1とB_g2は同型である。 地に足がついた説明:B_g1の元b1/g1^n1に対応するB_g2の元b2/g2^n2が存在することを示す:D(g2)⊂D(g1)なので3.5.Eよりg2^m=c*g1となるm,cが存在する。b1*g2^(m*n1)=b1*c^n1*g1^n1 なのでb2=b1*c^n1,n2=m*n1とおけばよい。 このとき、A_π#(g1)とA_π#(g2)は同型で、先のb1/g1^n1とb2/g2^n2はこの同型で対応する組に送られる。 確かに層の準同型を与えること:制限写像と可換であることは環準同型と局所化が可換であることによる。 */ 練習6.2.Dで描写された環付き空間の写像はまったくややこしくない。C[y]→C[x]:y->x^2を考える。これはx軸からy軸への写像で、閉点(x-a)を(y-a^2)に送る。(x-3)の像は何か?(y-9)である。(y-4)の逆像は何か?(x-2)と(x+2)である。これは3.2.Oで既に扱った。次は関数の引き戻しを考えよう。D(y-4)=A^1-{4}における関数3/(y-4)の引き戻しは何か?当然A^1-{±2}における関数3/(x^2-4)である。 練習6.2.Dの構成はすぐにスキームの射の例となる。その前に、次の練習によって、アフィンスキームの間の環付き空間の射がすべてこの方法で得られるものというわけではないことを指摘しておこう。(6.3の言葉では、これは環付き空間の射だが局所環付き空間の射ではない例である。) 6.2.E 重要でない練習 Y=Spec k[y]_yは2つの点を持つことを思い出そう(3.4.K):生成点(0)と閉点(y)である。環付き空間の射Spec k(x)→Spec k[y]_y で、(0)を(y)に送り、構造層の準同型は、yをxに送る写像なものを描写せよ。これが6.2.Dの方法で与えられる形ではないことを示せ。 /* 構造層の開集合(0)への制限写像はk[y]_y→k(y)であり、層準同型の要請する可換性: k[y]_y → k(x) ↓  ↓ k(y) → k(x) を成り立たせるような環準同型k(y)→k(x)が必要であるがこれはyをxに送る写像で実現される。素イデアル(0)∈k(x)の逆像は素イデアル(0)∈k[y]_yだからこれは3.4.Hの形をしていない、つまり6.2.Dの形をしていない。 */ 6.2.2 今後は使わない暫定的な定義(6.1.1参照) スキームの射π:(X,O_X)→(Y,O_Y)とは、局所的に6.2.Dの形をしているような環付き空間の射である。正確には、任意のSpecA⊂X, π(SpecA)⊂SpecB⊂Yに対して、環付き空間の射で定まる構造層の射のSpecBへの制限が6.2.Dの形をしていることを要求する。 我々はこの定義が任意のアフィン被覆で確認できることを望むので、アフィン交通補題を使って議論をしていきたいかもしれない。これはうまくいくのであるが、しかしある技巧を使う便利な視点がある:次の章で局所環付き空間の言葉を使う。一旦その言葉を手にしたら、前の段落にある定義にはもう用はなくなる。 --- 6.3 局所環つき空間からスキームの射へ 射が我々が望むような振る舞いをすることを証明するために、局所環つき空間という用語を使う。この用語は、スキームの射を定義し、それが理にかなった性質を持つことを示すためだけに使う。しかしこれは他の幾何学でも有用である。この言葉は古典的多様体に動機づけられている(3.1.B)。π:X→Yを古典的多様体の射としてπ(p)=qとすると、Y上の関数fがqで消えるならば引き戻したX上の関数π#(f)はpで消える。言い換えると、qで消える関数の芽は、pで消える関数の芽に引き戻される。 6.3.1 定義 局所環付き空間とは、茎が局所環であるような環付き空間と定義された(4.3.6)。局所環付き空間の射π:X→Yは、環付き空間の射であって、茎の準同型(6.2.C)OY_q→OX_pが極大イデアルを極大イデアルに送る(「局所環の準同型」)ようなものである。これはまさに「gがqで消える関数ならば、その引き戻しはpで消える」という具体的で直観的な意味を持つ。(補足:読者は、gがqで消えない関数ならば、その引き戻しはpで消えないことも望むかもしれない。これは実は我々の定義から従う。なぜか?) /* OX_pの極大イデアルの逆像はOY_qの素イデアルでかつOY_qの極大イデアルを含むから、極大性よりOY_qの極大イデアル自体である。 */ こうして局所環付き空間は圏をなす。 局所環つき空間について覚えておくべきことは: (1)関数は点において値をとる (2)関数の零点の引き戻しは、引き戻した関数の零点に一致する。 である。 6.3.A 局所環つき空間の射は貼り合わさる(6.2.A) (X,O_X),(Y,O_Y)を局所環つき空間とする。X=∪Uiとして、環つき空間の射たちUi→Yは交わりで一致するとする。局所環つき空間の射X→Yが誘導される。 6.3.B 簡単で重要な練習 (a) SpecAは局所環つき空間である(4.3.F) (b) 6.2.Dで環つき空間の射として定義した環準同型π#:B→Aで誘導される環付き空間の射π:SpecA→SpecBは局所環つき空間の射である。 /* π(p)=qとして、茎の準同型B_q→A_pを考える。g∈qならばπ#(g)∈pを確認したい。これはπ(p)=qであることはqのπ#による逆像がpであることを意味するから従う。 */ 6.3.2 鍵となる命題 π:SpecA→SpecBが局所環つき空間の射ならば、それは、環準同型π#:Γ(SpecB, O_SpecB)→Γ(SpecA, O_SpecA)から6.3.B(b)のように誘導されたものである。(4.3.Aはこれの特殊な場合である) [証明] π:SpecA→SpecBを局所環つき空間の射とする。これが環準同型π#から誘導されるものに一致することを示したい。まず集合の写像として一致することを示す必要がある。 p∈SpecAは、大域関数という環において、点pにおいて消える関数のなすイデアルと同一視される。像π(b)は、SpecBの点で、点qで消える関数がちょうど「点pで消える関数を引き戻した関数」に一致するような点だと解釈される。(ここでπが局所環つき空間の射であるという事実を用いた。)そこでこれは、π#から誘導される射のpの行き先を素イデアルの逆像として定義(3.2.9)したのとまったく同じ意味となる。 特に、局所環付き空間の射という仮定により、b∈Bに対してπ^-1(D(b))=D(π#b)となることも言える。すなわち位相空間の写像として一致する。 最後に、π#:O_SpecB→π_* O_SpecAが6.2.Dで与えた層の準同型であることを示す必要がある。これは基本開集合で確認すれば良い(2.5.C)。すなわち、開集合D(b)での写像が大域切断B→Aの写像によって定まることを見る: 次の可換図式を考える。 B=O_SpecB(SpecB)→O_SpecA(SpecA)=A ↓ ↓ B_b=O_SpecB(D(b))→O_SpecA(D(π#b))=A_π#b=A {@_B} B_b 縦の矢印は制限写像、すなわちbによる局所化である。従って、「環準同型は局所化の準同型を誘導する」によって下の段の矢印が定まる。 /* 層の準同型をなすことは6.2.Dでも考察した通りの内容である。 */ これで次の定義の準備ができた。 6.3.3 X,Yをスキームとする。局所環つき空間の射π:X→Yをスキームの射と定義する。こうして、スキームの圏Schが定義された。(終域Yは時に底スキームと呼ばれる。これはπをYでパラメータ付けられたスキームの族と解釈するときの言葉である。これは9.3.2で射のファイバーを定義するとより明らかになるかもしれない。) この局所環付き空間の言葉での定義は、6.2.2の暫定的な定義を含意することはすぐにわかる: 6.3.C 重要な練習 スキームの射X→Yは、局所的にはアフィンスキームの射のように振る舞う環つき空間の射である。すなわち、SpecAの像がSpecBに含まれるとき、誘導される射はアフィンスキームの射である。 (W⊂XとY⊂Zが環つき空間の開埋め込みのとき、X→YはW→Zを誘導する。SpecAiがXを被覆し、SpecBiがYを被覆するとして、SpecAi→SpecBjがアフィンスキームの射であることを確認することに帰着せよ。) /* SpecAへの制限も局所環付き空間の射だから6.3.2を適用すれば良いと思った */ こうして実用的にはこのアフィンスキームの解釈を使い、局所環つき空間の概念はもう忘れて良い。特に: 6.3.D アフィンスキームの圏は、環の圏の反対圏である。 特に、1つの点からの射が複数有り得る。例えばSpecCからSpecCの射として、恒等写像と複素共役写像に対応する2つの射がある(もっとたくさんある)。 射は貼り合わさるのであった(6.3.A)。射の同型は4.3.6で定義した通りである。 6.3.E 視界を開く練習(スキームの射を開アフィンに分解して考える) 次の文章を解釈せよ: 「A^(n+1)-{0}からP^kへの写像(x0,x1,..,xn) -> [x0:x1:..:xn]はスキームの射である」 注意:点がどこに行くのかを言うだけでは不十分である。関数がどこに行くのかを言う必要がある。従ってアフィンに分解して描写し、それらを貼り合わせる必要があるかもしれない。(基礎体kを一般の環Bに一般化できるか?6.3.Mで扱う。) /* P^kの関数は1つの視点では、開アフィンに分解すると、例えばU0(x0≠0)上の関数は、(x00=1,x10,..,xn0)の多項式で、U1(x1≠0)上の関数は(x01,x11=1,..,xn1)の多項式で、その間の関係式は、xj0/x10 = xj1 による。A^(n+1)に引き戻す時には、xj0はxj/x0, xj1はxj/x1に引き戻せば、上記の関係式の貼り合わせはうまく振舞う。 例えばU0∩U1上の関数 x01^2+1+x21 = 1/x10^2+1+x20/x10 は、A^(n+1)上の関数 x0^2/x1^2+1+x2/x1 に引き戻されるがこれはU0上の表記でもU1上の表記でも同じ結果を与える。*/ 6.3.4 アフィンスキームの射 次の結果は、アフィンスキームの射を記述するには、開アフィンでの被覆を一生懸命考える必要はないことを示す 6.3.F 欠かせない練習 射X→SpecAの集まりは、自然に環準同型A→Γ(X,O_X)の集まりと全単射に対応する。ヒント:まずXがアフィンの場合を確認せよ。それから6.3.Aの、射は貼り合わさるという事実を使うと良い。(この性質は局所環つき空間の圏でも成り立つが、スキームの圏で考えた方が易しい。) /* XをSpecXiで被覆する。 ・X→SpecAが与えられたとき 局所環付き空間の定義より構造層の引き戻し準同型A→Γ(X,O_X)がある。 ・A→Γ(X,O_X)が与えられたとき 制限写像Γ(X,O_X)→XiによりA→Xiを得るのでXi→SpecAを得る。これを貼りあわせてX→SpecAを得る。 ・6.3.2によりこれらのそれぞれのSpecXi上への制限は互いに逆像なので全体としても互いに逆像である。 */ /* 環準同型A→Γ(X,O_X) はスキームの射 SpecΓ(X,O_X)→SpecA と対応するから、X→SpecAは、標準的にX→SpecΓ(X,O_X)→SpecA と分解することが従う。この結果は、アフィン射がアフィン局所的であることの証明7.3.7に使われる。 */ 特に、スキームから自身の大域切断のスペクトラムへの標準的な射が存在する。(警告:Xが有限型k-スキームでも、大域切断の環は有限生成とは限らない。19.11.13、ヒルベルトの第14問題と関係する。) この標準的な射が同型であることが、Xがアフィンスキームであることと同値である(4.3.2)。 6.3.G 簡単な練習:6.3.7の定義と5.3.6の定義が一致することを示せ。 6.3.H 簡単な練習:S●が有限生成次数付きA-代数であるとき構造射ProjS●→SpecAを描写せよ 6.3.I 簡単な練習:SpecZはスキームの圏での終対象である。  kが体の時、Spec kはk-スキームの圏の終対象である。 6.3.5 スキームの圏での積について 練習6.3.Fから、任意のスキームに対して積が存在することを示すのは小さな一歩である。しかし我々は急がないので練習9.1.Bまで待つ。 6.3.6(**) 熟練者へ:6.3.Fの主張はΓとSpecは局所環付き空間の圏と環の圏の間の随伴関手であることを示している。 (面白いが重要でない質問:環付き空間の圏と環の圏の間でもΓは右随伴関手を持つ。それは何か?) /* もしかして極大イデアルSpecmかな・・? */ 6.3.7 A-スキーム、あるいはS-スキーム Sスキームの圏は、スキームの射X→Sを対象として、可換図式 X→Y ↓↓ S=S をX→SからY→Sへの射とする圏として定義される。このX→Sは構造射と呼ばれる。特にS=SpecAのとき、5.3.6で考察したA-スキームの概念となる。 6.3.8 (いくつかの)アフィンスキームからの射 アフィンスキームからの射は、アフィンスキームへの射ほど単純でないが、いくつか注目するに値することがある。 6.3.J 点からの射 (a)pをスキームXの点とする。局所環からの標準的な射 Spec OX_p→Xを描写せよ。(まずXがアフィンの場合を考察せよ。一般の場合に、アフィンの取り方に依存しないことを確認せよ。) (b)剰余体からの標準的な射κ(p)→Xを定義せよ。(これはしばしば単にp→Xと書かれる。) 6.3.K 局所環からの射 Xをスキームとし、(A,m)を局所環とする。π:SpecA→Xが、[m]をpに送るとする。pを含む開集合はπの像を含む。Mor(SpecA,X)と、{p∈Xと局所環の準同型OX_p→Aの組}の集まりの間の全単射を描写せよ。(6.3.J(a)) これらの練習は次の概念を導く: 6.3.9 点関手functor of points、Z値点(A値点、k値点) Zをスキームとする。Z値点はスキームの射Z→Xを意味し、X(Z)と表記される。Aが環の時、A値点は(SpecA)値点として定義され、X(A)と表記される。(この概念を使う時のAはたいてい体である。)B-スキームの圏で考察している時には、構造射Z→Bを伴う。 (k値点は時々Xのk-有理点などと呼ばれることがあるが、これは誤解しやすいので我々は使わない。) Z値点は、残念ながら、点とは限らない。しかしこの用語はこの分野でよく広まっている。(この言葉の一貫性の無さを見るには(原文の)索引のpointの所を見ると良い) 6.3.L (a) X→Yは、Z値点の写像X(Z)→Y(Z)を誘導する (b) X→Yは点の行き先だけでは決定されないのであった。(どんな例があっただろうか?)一方でX→YはあらゆるZを考えることで、Z値点の行き先で決定される。(ヒント:Z=Xを考えよ。これはおおまかには、米田の補題(9.1.C)である。) /* X=SpecC, Y=SpecR とか */ 6.3.10 「Z値点の積」は期待通りに振舞う(9.1.3)。背景として、X(Z)は、スキームの圏から集合の圏への反変関手と見なせて、スキームXの点関手と呼ばれる(1.2.20)。 A値点が有用であることを示す例: アフィンスキーム Z[x1,..,xn]/(f1,..,fr)のA値点は、f1=..=fr=0の環Aにおける解である。x^2+y^2=16の有理数解は、SpecZ[x,y]/(x^2+y^2-16)のQ値点である。従ってアフィンスキームのA値点は単純な解釈を持つ。 一方で、射影スキームのZ値点の解釈は微妙である。 6.3.M 簡単だが視界を開く練習(cf.6.3.E) (a) Bを環としてXをB-スキームとして、f0,..,fnをX上の共通零点を持たない関数とする。[f0,..,fr]はB-スキームの射X→P_B^nを与える。 (b) gはXのどこでも消えない関数とする。[g*f0,..,g*fr]は(a)と同じ射を与える。 6.3.11 具体例(6.3.E再訪) A^(n+1)の(n+1)個の関数x0,..,xn(あるいはn+1個の自明束の切断)を考える。A^(n+1)-{0}でそれらは共通零点を持たない。従ってそれは射A^(n+1)-{0}→P^nを定める。 6.3.12 もしかしたら射影空間への射はすべて6.3.Mの形で書けると期待したかもしれない。それは正しくない。例えば恒等射P^1→P^1はこの形でない。なぜならP^1は定数関数以外の大域関数を持たないからである。 しかし、修正ができる(16.4.1)。おおまか言うと、fiは関数ではなく、直線束の切断とすれば良い。射影スキームへの射を良く理解するのは、直線束を理解する1つの重要な動機となる。 別の一般化が15.3.Fで与えられ、それは普遍的な描写であることが16.4.1で判明する。 グロタンディーク以前には、環上の射影空間の点を座標を使って解釈することは実際的な問題であった。この難しさは関手的な視点が欠けていたことによる。正しい答えへの鍵は、すでに存在していた(多様体からRP^nへの写像について同様の問題があった。)。このような幾何的な疑問の答えは(射影空間は幾何的なものである)、純粋に代数的なものではなく幾何的なものであるべきである。 6.3.13 スキームを見る(3):冪零元がある時のスキームの射 読者はスキームの点を視覚化し(3.3)、冪零元を視覚化した(4.2, 5.5)。次のやや正確でない練習は、冪零元が関与するスキームの射をどう視覚化するかを示す。a∈Cとする。 環準同型C[x]→C[ε]/(ε^2), x→aεを考える。a≠0のとき、この射をどう描くか?a=0のときはどう変わるか?(その場合は接ベクトルが押しつぶされるであろう。) 練習12.1.Iはこれを拡張する。(「Spec k[ε]/(ε^2)からXへの射は、剰余体kを持つXの点pとその点での接ベクトルの組の情報を持つ」) 6.3.N(**) 解析化関手 ・・・ --- 6.4 次数付き環と射影スキームの射 環準同型はアフィンスキームの反対向きの射に対応した。次数付き環の準同型は、射影スキームの反対向きの射にいくらか対応する。これは完全的ではない。すべての次数付き環の準同型が射影スキームの射を与えるわけではなく(6.4.2)、すべての射影スキームの射が次数付き環の準同型に由来するわけでもない(19.11.9)。そして異なる次数付き環の準同型が同じ射影スキームの射を与えることがある(6.4.C)。 6.4.1, 6.4.2の例をよく考え通すと役に立つだろう。 6.4.A 欠かせない練習 φ:S●→R●を(Z≧0-)次数付き環の準同型とする。(すなわちあるdがあってφはすべてのnに対してS_nをR_dnに移す。)これは射影スキームの射 ProjR●-V(φ(S+))→ProjS● を誘導することを示せ。 (ヒント:fをS+の斉次元とする。D(φ(f))→D(f)を定めよ。異なるfに対してそれら貼り合わさることを示し、結論を示せ。) 特に、V(φ(S+))=φならば、ProjR●→ProjS●を得る。 /* D(g=φ(f))は次数付き環S●のφ(f)による局所化の0次成分のSpecと定義されるのだった。 環準同型 S●_φ(f)_0 → R●_f_0 が誘導されるので D(φ(f))→D(f) を得る。 ・・・(作業中) */ 6.4.1 例 実際に6.4.Aの様子を見よう。例えばP_C^1→P_C^2: [s:t]→[s^20:s^9t^11:t^20]という写像を考える。これは斉次的なので定義として成立している。 代数的には、C[x,y,z]からC[s,t]へ [x,y,z]->[s^20,s^9t^11,t^20]と送る環準同型が対応する。これを6.4.Aの考察と比較し、3.2.10のアフィンの場合と比較すると良い。 6.4.B φ:S●→R●が、√φ(S+)=R+を満たすなら6.4.Aの仮定は満たされる。 6.4.C 重要でない練習・・・ 6.4.3 重要でない指摘・・・ 6.4.4 ベロネーゼ部分環 S●は有限生成次数付き環とする。Sの次数nの倍数の成分を取り出したものをS●のn次ベロネーゼ部分環Sn●と呼ぶ。 6.4.D Sn●→Sは同型ProjSn●→Proj●を引き起こす。 (ヒント:f∈S+が次数がnの倍数の斉次元のとき、ProjS●でのD(f)とProjSn●でのD(f)を同一視できる。それらの基本開集合はProjS●を被覆するとなぜ言えるか?)/* √Sn+ = S+ */ 6.4.E S●が次数1で生成されるならSn●もそうである。 6.4.F R●とS●が0以外の有限個の次数でのみ異なるなら(これを正確に述べよ)、ProjR●とProjS●は同型である。 6.4.G SはS0上f1,..,fnで生成されるとする。Sd●が次数1で生成されるようなdを求めよ。(これは技巧を要する。ヒント:x1,..,xnの次数をd1,..,dnとする。次数nd1..dn以上の単項式x1^a1*..*xn^anに対して、あるiがあってai*di≧Π[j]djとなることを示せ。) これらの結果より、次数1で生成される場合だけを考えてもそれほど実害は無いことが分かる。後で8.2.Gで任意のProjをある射影空間に埋め込めることを示す。 6.2.H より重要でない練習・・・ --- 6.5 被約なスキームからの有理写像 有理写像 X --> Y とは 稠密な開集合で定義された射の集合を Uで定義されたfと、Vで定義されたgが、U∩Vの稠密な開集合で一致するとき f〜g とみなす同値で割ったものである。 Yが分離的なときは、少し定義を改善できることを後で示す(10.2.3) 像が稠密のとき、支配的という。 6.5.A 既約スキーム間の有理写像が支配的 ⇔ 生成点を生成点に送る  アフィンの場合:環準同型の核が冪零であることを意味する 6.5.B 整スキームの支配的な有理写像は、逆向きの関数体の準同型を与える 関数体の準同型がいつも支配的な有理写像を与えるわけではない。Spec k[x] --> Spec k(x) となる有理写像は存在しない。(なぜならk(x)からk[x]_f へ送る環準同型は存在しない)しかしながら、有限型整スキームの場合は、関数体の写像は支配的な有理写像を与える。(特に既約多様体の場合。6.5.7) 注意:SpecB→SpecAが支配的であるなら、環準同型A→Bの核は冪零である。 6.5.4 有理写像が双有理的とは、逆な有理写像が存在し、合成が恒等になることである。これは整スキームと支配的な射からなる圏において同型ということである。 ・・・ 6.5.5 命題 XとYを被約なスキームとする。XとYが双有理的であることは、Xの稠密な開集合UとYの稠密な開集合Vで互いに同型となるものが存在することと同値である。 ・・・ 6.5.6 既約な多様体間の有理写像 6.5.7 命題 Xは整なk-スキームでYは整で有限型なk-スキームとする。 関数体の拡大(単射) K(Y)→K(X) でkを保つ準同型が与えられたとする。 そのとき、対応するkスキームの支配的な射 X→Y が存在する。 ============ 7 いくつかの有用なスキームの射のクラス 我々はこれから射に関する膨大な定義をする。いくつか(特に有限性に関する性質)は、"普通に現れる"射はだいたいその性質を持ち、証明での使い方が明らかであるという理由で有用である。別のいくつかは、幾何的な振舞いに関係し、読者はその性質の意味について絵を持っておくべきである。 グロタンディークの教訓の1つは、対象の性質をよりよく理解するには、射の性質として理解した方が良いというものであった。(このような言葉は2.3.1でも現れた。)Pをスキームに関する性質としたとき、我々は(常にではないが)しばしば次のようにして射の性質に読み変える:射π:X→Yについて、すべての開アフィンU⊂Yに対してπ^-1(U)が性質Pを持つ。良い状況ではPはアフィン交通補題5.3.2を満たし、我々はすべての開アフィンについて確認する必要はない。略式には、読者はすべてのファイバーが性質Pを持つと考えることができる。(読者は、位相空間の射としてのファイバーを定義できるだろう。しかし我々はあとでファイバー積を定義した後にスキーム論的なファイバーを定義することで、この議論を正当化する。) しかしPであるという主張は、ファイバーごとに成り立つことより強い主張である。 (比較のため:滑らかな古典的多様体での滑らかな写像は、ファイバーが滑らかであることよりも強い主張である。) 7.1 都合のよい射のクラスの例:開埋め込み 7.1.1 都合のよい射のクラスに何を期待するか: (1) 終域に関してアフィン局所的 (2) 射の合成で保たれる (3) 底変換で保たれる(9.1) 誰かが読者に新しい射のクラスを教えた際には、読者はすぐに自分自身あるいはその人にこれら3つの性質が成り立つかを問うべきである。ここに最初の例がある: スキームの開埋め込みとは、環つき空間としての開埋め込み(6.2.1)である。すなわちYのある開集合Uがあってπ(X,O_X)→(U,O_Y|U)→(Y,O_Y)と分解し、最初の矢印が同型となるようなものである。同型射は開埋め込みである。Xが実際にYの開部分集合であるときXはYの開部分スキームであると言う。この区別は少し混乱しやすく、それほど重要でない。開部分スキームは開部分集合のスキーム論的な類似である。閉部分スキームというのもあるが、少し様相が異なる(8.1)。 7.1.A,B 開埋め込みは7.1.1の性質(1)(2)(3)を満たす 7.1.C X→Yを開埋め込みとする。Yが(局所)ネーターならXもそうである。しかしYが準コンパクトでも、Xは準コンパクトとは限らない。(ヒント:3.6.G(b)) 7.1.2 始域に関してアフィン局所的という概念を考えることができる。 7.1.D 開埋め込みは始域に関してアフィン局所的ではない /*閉埋め込みでも、始域の位相に制限したら開埋め込みになる */ --- 7.2 Lying over と Nakayama -- 代数学は悪魔から数学者への提供である。悪魔は言う:私はおまえにこの強力な機械を与えよう。それはおまえの好きに選んだ質問に答えを出してくれよう。おまえに必要なのは私に魂をよこすことだ:幾何を諦めるのだ。そうしたらこの素晴らしい機械を与えよう。 -- M. Atiyah, (しかし0.3の最初の引用も見よ。) 整射に関する次の章の議論を確立するために、我々は代数的な背景知識を築いておく必要がある。その時に使う賢い技巧は、中山の補題を示すのにも使われるので、我々はその議論もここで行うことにする。 φ:B→Aを環準同型とする:a∈がB上整であるとはφ(B)係数モニック多項式の零点となることを言う。φが整であるとは、Aの任意の元がB上整であることを言う。さらにφが環の埋め込みのとき、φを整拡大と呼ぶ。読者はこれを体の拡大の類似と考えて良い。 7.2.A 整射は貼り合わさる B→Aを環準同型として(b1,..,bn)=1とする。それぞれのB_bi→A_φ(bi)が整ならば、φは整である。 ヒント:「貼り合わせに関する重要なテクニック」 7.2.B (a) φ:B→Aが整射であるとき、(a1) T^-1.B→φ(T)^-1.A, (a2) B/J → A/φ(J)A, (a3)B → A/I は整射であるが、(a4) B→S^-1.A は整射とは限らない (b) φ:B→A が整拡大であるとき (b1) T^-1.B → φ(T)^-1.A は整拡大であるが(b2) B→S^-1.A, (b3) B→A/I は整拡大とは限らない。(k[t] -> k[t]/(t) ) (c) (c1) B/J→A/φ(J)A も整拡大とは限らない。(c2)しかしJ⊂BがAのあるイデアルのBへの制限となるような場合はこれは整拡大となる。 (特にJがBの素イデアルなら、後のLying overにより(c2)の状況に当てはまる。) (しかしLying overの証明でこの(c2)の形を使うから循環論法にならないことに注意。) /* (a1) φ(T)^-1.Aの元a'=a/tがT^-1.B上整であることを示す。 aはb上整だから a^n+...+b1*a+b0=0 そこでbk' = bk/t^(n-k) とおけば a'^n+...+b1'*a'+b0'=0 が成り立つ (a2) 標準的全射B→B/J, A→A/φ(J)A をπ,τとおく。 a'=π(a)がB/J上整であることを示す。 上記と同様に a^n+...+b1*a+b0=0 を考えて左辺にτを作用させる。 bk' = π(bk)とおけば良い。 Bの元をAに移してからτを作用させる結果は、Bの元にπを作用させてからA/φ(J)Aに移すのと同じ結果であることによる。 (a3) a^n+...+b1*a+b0=0 に標準的全射を作用させれば良い。 (同じ係数を使えば良い) (a4) k[t] -> k[t]_t は整でない。 (b1) 単射は底変換で保たれる (b2) (a4)による (b3) bの像とIが交われば単射でない。 (c1) 元の本に書いてあった例: B=k[x,y]/(y^2) → A=k[x,y,z]/(z^2,xz-y), J=(x)を考える k[x,y]/(x,y^2) → k[x,y,z]/(z^2,xz-y,x) は単射ではない。(yが0に移る) (c2) B→Aが単射で、J以外のBの元がφ(J)Aと交わらなければB/J→A/φ(J)Aは単射になる。 (*)Jが素イデアルのとき https://stacks.math.columbia.edu/tag/00GH */ 次の補題は、有用だが独特の技巧を使う。/*「行列式のトリック」*/ 7.2.1 整と有限生成 a∈AはB上整 ⇔ B[a] がB加群として有限生成 [証明] 左から右は易しい。右から左: 生成元m1,m2,..,mnをとる。a*mi = Σbij*mj とおく。 縦ベクトルの関係式 ( a*Id - {bij} ) (m1;..;mn) = (0;..;0) を得る。 随伴行列を掛けることで、det(a*Id-{bi})倍は任意の縦ベクトルを0に移すことが言える。すなわちB加群の元として0である。そこで行列式を展開すればaを零点とするモニックな多項式を得る。 7.2.2 系:A全体がB加群として有限生成ならば、AはB上整である /*注意:AがB上整でもA全体がB加群として有限生成とは限らない。例えばQの代数閉包。*/ 7.2.C AがB上整でBがC上整ならばAはC上整 7.2.D B→Aを環準同型とする。B上整なAの元は、Aの部分環をなす。 /* B[x],B[y]がB上有限生成ならB[x+y],B[xy]もB上有限生成、を示せば良い。 x^n = b0+b1*x+...+b[n-1]*x^(n-1) y^m = c0+c1*y+...+c[n-1]*y^(n-1) とおける。 B[x+y]もB[xy]も x^u*y^v (uf(t), n次多項式 例2: 閉埋め込み 例3: 正規化(9.7)  Spec k[t]→Spec k[x,y]/(yy-xx-xxx), x->tt-1, y->ttt-t  特異点(x,y)の逆像は、(t-1),(t+1)の2点に分かれている 7.3.H 例4: X→Speckが有限射なら、Xは有限個の点からなり、各点はkを剰余体とする有限次拡大である。(3.2.Gより体上の有限生成-代数は整域ならば体である) /* このように、有限射≒閉射+ファイバーが有限、と言える。 ちなみに12章の概念と関連させると 有限射=アフィン射 + 固有射(有限型+分離的+絶対閉射) 整射=アフィン射 + 絶対閉射 https://stacks.math.columbia.edu/tag/01WG, 18.1.8, 29.6.2 */ 7.3.I 簡単な練習(cf.7.2.C) 有限射の合成は有限射である 7.3.J SpecAへの有限射は射影射である RをA-代数として、次数付き環S●をS0=A, Sn=R(n>0)で定める(乗法構造は何か?ヒント:Rの元同士の積と、Aの元によるRの元のスカラー倍が定まっている。)同型ProjS●=SpecRを描写せよ。Rが有限A-代数(A-加群として有限生成)ならば、S●はA上の有限生成次数付き環であり、従ってSpecRは射影A-スキームである(4.5.9)。 /* こうして、射影スキームの概念はある意味で有限性を含んでいる */ 7.3.K 重要な練習 有限射はファイバーが有限であることを示せ。(これは有用な練習である。読者は射のファイバーをどう調べるかを考えなくてはいけない。9.3.4の議論のあとではより簡単だろう。) ヒント:X=SpecA, Y=SpecB, q∈Yのとき、mod qを取ることでq ̄の外の情報を無視することができる。その逆像はSpec(A/π#qA)であることを示せ。そうすれば、Bが整域で、qが(0)の場合に帰着する。今度は素イデアル(0)で局所化すれば、残りの点を無視できる。その逆像はAをπ#(B-{0})で局所化した環のスペクトラムであることを示せ。有限性はこれらの操作で保たれることを示せば、7.3.Hに帰着する。 有限射であるという性質は、ファイバーが有限であるという性質より強い。 7.3.9 開埋め込みA^2-{0,0}→A^2は有限なファイバーを持つが、アフィン射でない(4.4.1)ので有限射でもない。 7.3.L 開埋め込みA^1-{0}→A^1は有限なファイバーを持ち、アフィン射であるが、有限射ではない。 /* k[x]_x=k[x,1/x]はk[x]-加群として有限でない */ 7.3.10 定義 X→Yが整射であるとは、アフィン射でかつ、SpecBの逆像をSpecAとおくと対応する環準同型B→Aが整であることを意味する。これは7.2.A, 7.2.B, アフィン交通補題(5.3.2)により、affine-local on targetな性質である。また、7.2.Cにより合成に関して閉じている。有限射は整射である(7.2.2)(逆の反例はSpec Q ̄→Spec Q)。 /*整射かつ対応する環の射が単射のとき、Lying overが成り立つ(7.2.4) (スキームの底空間が全射) また、整射では、Going-upが成り立つ(7.2.F) (スキームの底空間が閉射)*/ 7.3.M 整射は閉射である。従って有限射も閉射である。 ヒント:アフィンの場合に帰着せよ。Lying over(あるいはGoing-up)を利用せよ。8.2.5で別の視点を与える。 7.3.N 重要でない練習 B→Aを整とする。B→Cに対して、C→A {@_B} C が整であることを示せ。 (9章の言葉で「整射は底変換で保たれる」) ヒント:Σai@ciがC上整であることを示したい。aiはB上整であることを利用し、7.2.Dを利用せよ。/* 整と有限生成の言い換え */ 7.3.11 整射のファイバー 有限射と異なり、整射のファイバーは有限とは限らない。(例を思いつくだろうか?) ところが9.3.2でファイバーを位相空間としてみなす方法を得た後なら、このファイバーは、どの点の閉包も他の点を含まない性質を持つことが確認できる(11.1.D)。 7.3.12 (局所)有限型射 π:X→Yが局所有限型射であるとは、SpecB⊂YとSpecA⊂π^-1(SpecB)に対して、対応する環準同型B→AがAを有限生成B-代数として表すことを意味する。(5.3.3(b))より、これはSpecBがSpecAiで被覆されて、各Aiが有限生成B-代数であることと同等である。 これに加えて準コンパクト射であるとき、有限型射と呼ぶ。これはSpecBが有限個のSpecAiで被覆されるという条件を追加したものである。 7.3.O 局所有限型射と有限型射は終域上アフィン局所的な性質である。 例:構造射P^n→SpecAは有限型である。 5.3.6でk上有限型と定義した性質は、構造射X→Speckが有限型射であることと言い換えられる。 7.3.P 有限射 = 整射 + 有限型射 7.3.Q 難しくないが重要な練習 (a) 開埋め込みは局所有限型射である。・・・ (b) 局所有限型射の合成は局所有限型射である。 (c) X→Yが局所有限型でYが局所ネーターならXも局所ネーター  X→Yが有限型でYがネーターならXもネーター 7.3.14 準有限射とは、有限型射でかつファイバーが有限集合であることを言う。主要な部分はファイバーが有限集合というところにある。有限型射という性質は、底変換で保たれる(9.4.C)ことを保証するために付いている。 有限射は準有限射である。逆の反例は7.3.9があった。しかしすぐに、Spec kへの準有限射は有限射であることを見る(7.4.D)。ファイバーが有限だが準有限射でない例はSpecC(t)→SpecCとかSpecQ ̄→SpecQがある。(9.1.4) 7.3.15 準有限射の描写 ・・・ 7.3.16 フロベニウス ・・・ --- 7.4 射の像:シュヴァレーの定理と消去理論 このセクションでは、読者が代数幾何という言葉を聞くずっと以前に考えたことがあるかもしれない疑問に答える。いくつかの不定係数の多変数の多項式方程式があるときに、共通解が存在するための係数の条件を考えたい。この問題の代数的背景を考えると、答えは純粋に代数的であるべきであろう。それはランダムではないし、指数関数やコサインのような関数を含まないだろう。つまり代数的な条件を期待する。これは実際正しく、多様体あるいはスキームの射の像として問題を解釈できて、シュヴァレーの定理によって応えられる。また、シュヴァレーの定理は零点定理(3.2.5)の証明をも与える。(7.4.3) いくつかの場合では像はうまく振舞う。例えば、有限射は閉射であることを見た。我々は、消去理論の基本定理(7.4.7)という古典的な結果を証明する。それにより、有限射は閉射であることが、射影空間からの写像に一般化される(8.2.5で説明する)。我々はこれを繰り返し使う。別の方向では、遠い将来、良い条件(平坦+α)では射は開写像であることを見る(24.5.G)。例えば、A_B^n→SpecBが1つの例である。(直接示すことを試みても良い) 7.4.1 シュヴァレーの定理 π:X→Yをスキームの射とする。集合としてのπの像という用語は明らかである。単に、Xの点の像にあるYの点のことである。我々は、像は開でありえるし、閉でも有り得ることを知っている。また、局所閉であるときもあった。しかしより奇妙なことがありえる。A^2→A^2の射 (x,y)->(x,xy)を考えよ。像は平面からy軸を取り除き、しかし原点を取り戻したものである。これを捉える用語を作る。 ネーター位相空間において「構成可能な集合」とは、次を満たす最小の族である: (1)すべての開集合はこの族に属する (2)この族の有限個の交わりはこの族に属する (3)この族の補集合もこの族に属する 例えば、(x,y)->(x,xy)の像は構成可能である。より一般の位相空間の場合には9.3.1で触れる。 7.4.A 練習 構成可能な集合は、有限個の局所閉部分集合の非交和として表せる。 その結果、X→Yがネーター位相空間の連続写像ならば、構成可能な集合の逆像は構成可能である。局所閉部分集合は、開集合の閉集合で表せる集合=開集合と閉集合の交わりで表せる集合のことである。 (スキームの局所閉部分スキームについては8.1.2で扱う) /* 有限個の局所閉部分集合の非交和は(1)(2)(3)の方法で表せ、「有限個の局所閉部分集合の非交和で表せる部分集合」の族は(1)(2)(3)を満たす。*/ 7.4.B 練習 (7.4.3で使われる) kを体とする。A_k^1の生成点は構成可能な集合でないことを示せ /* 有限個の閉点の合併、その補集合は(1)(2)(3)の方法で表せ、しかもその族は(1)(2)(3)を満たしている。A_k^1には閉点が無限個あった(3.2.D)。従って生成点はこの族に属さない。*/ 7.4.C 練習 (24.5.Gで使われる) (a) ネータースキームの構成可能な集合は、特殊化について安定であるときに限り閉である。より正確には、ZをネータースキームXの構成可能な集合とすると、Zが閉であることは次と同値である:「y1∈y2 ̄となるXの点y1,y2に対して、y2∈Zならばy1∈Zである。」 (b) 同様に、一般化について安定であるときに限り開である。 /* (a)閉集合であるためには特殊化について安定である必要があるのはすぐに分かる。それで十分であることを検討する。 Zを有限個の局所閉部分集合W1,W2,...,Wnの非交和とおく。そうするとそれぞれの局所閉部分集合について検討すれば十分である。 W1=U∩V, Uは開集合、Vは閉集合とおく。ネーター性からVは既約閉集合V1,V2,..,Vmの合併で表せる(3.6.15)。既約な閉集合は生成点を持つ(5.1.B)。Vkの生成点をy2とおく。U∩VkはVkの開集合だから、y2∈U∩Vkである。ところがVkの点はすべてy2 ̄に属するので、U∩Vk=Vkでなければならない。よってU∩Vkは閉集合でその結果W1も閉集合でその結果Zも閉集合である。*/ スキームの射の像は構成可能な集合より奇妙である。実際、スキームのどんな部分集合Sもスキームの射の像になり得る。Sすべての点での剰余体の非交和から写像すれば良い。これは病的であり、より理にかなった状況では像は(x,y)->(x,xy)の場合より振る舞いが悪くなることは無い。すなわち: 7.4.2 シュヴァレーの定理 π:X→Yをネータースキームの有限型射とする。構成可能な集合の像は、構成可能な集合である。特にπの像は構成可能である。 (気にする人へ:局所有限表現射への拡張は9.3.7を見よ) 証明の前に、これを使って得られる結果を紹介する。幾何的で位相的に見える定理から代数的な系を得られることには驚くかもしれない。最初の例は零点定理である。 7.4.3 零点定理(3.2.5)の証明 我々はKがkの拡大体でk-代数として有限生成ならば有限次拡大体であることを示したい。Kの生成元をx1,..,xnとおいて各xiがk上代数的であることを言えば良い。 そうでないと仮定する。包含k[xi]→Kは支配的な有限射π:SpecK→A_k^1を引き起こす。当然SpecKは1点なので、πの像は1点である。シュヴァレーの定理と練習7.4.Bによりπの像は_k^1の生成点でない。従って、im(π)は閉点であり、従ってπは支配的でない。 /**/ 同様の考えが使える例として: 7.4.D 練習 体への準有限射は有限 X→Speckが準有限射なら有限射である (ヒント:まずX=SpecAの場合を考察せよ。Aがk上代数的でない元xを持つなら、埋め込みk[x]→Aが存在する。7.3.Hが役に立つかもしれない。) /* Spec k[x]は無限個の点を持つことを3.2.Dで示した。 SpecA→Speck[x]→Speck の分解を考えることで、 SpecA→Speck[x]の像が無限個あることを示せば良い。像は生成点を含むことと、生成点を含む構成可能な集合は無限集合であることから言える */ 7.4.E アフィンk-多様体では閉点に関して全射ならば全射である /* 像は閉点をすべて含む構成可能な集合である。 */ シュヴァレーの定理の証明には、グロタンディークの次の補題が有用である。 B-代数Aについての次の条件を一時的に†と名付ける: 「すべての有限生成A-加群Mに対して、0でないf∈BでM_fが自由B_f加群となるものがある」 7.4.4 グロタンディークの生成点的自由補題 Bをネーター整域とする。有限生成B-代数は†を満たす これを一連の練習で示す。7.4.KまでBはネーター整域とする。 7.4.F B自身はB-代数として†を満たす /* */ 7.4.G 補題7.4.4を次の命題に帰着せよ: 「Aが有限生成B-代数で†を満たすならばA[T]もそれを満たす」 (ヒント:B-代数としての生成元の個数で帰納法) /* https://math.stackexchange.com/questions/1048496/question-on-generic-freeness-ref-matsumura-page-185 難しい */ Aが†を満たすとする。Mを有限集合Sで生成された、有限生成A[T]-加群とする。 M[0]=0として、M[1]をSで生成された部分A-加群とする。 n>0に対して、M[n+1]=M[n]+TM[n] と定義する。 M=∩M[n]であることに注目せよ。 7.4.H T倍写像は全射φ[n]: M[n]/M[n-1] → M[n+1]/M[n] を引き起こす 7.4.I nが十分大きければφ[n]は同型写像である  ヒント:M1の昇鎖律を利用せよ 7.4.J 0≠f∈Bが存在して、(M[i+1]/M[i])_fはB_f加群として自由である  ヒント:iを変数とするとM[i+1]/M[i]は有限種類である 7.4.K (ここではネーター性は必要ない)  M[0]=0でM[i+1]/M[i]がすべて自由ならばM=∩M[n]は自由である  ヒント:nに関する帰納法と余極限を利用せよ これで補題7.4.4が示され、定理7.4.2の証明の準備が整った 7.4.L π:X→Yをネータースキームの有限射としてYを既約とする。 Yの稠密開集合Uがあって、πの像はUを包含するかUと交わらないかどちらかである (ヒント:SpecA→SpecBで考える。補題7.4.4により、A_fがB_f加群として自由なf∈Bが存在する。開集合D(f)を考えると良い。) このヒントを示す方法は他にもたくさんある。多様体の場合には、定理11.4.1の証明で現れる別の証明がある。補題7.4.4は将来再び使う(24.5.9)意味もあってここで紹介した。 7.4.M 定理7.4.2を示すにはπの像が構成可能であることを示せば十分である 7.4.N Yがアフィンである場合に帰着せよ。さらにXがアフィンである場合に帰着せよ。 残りは手を振るように議論し、正当化を読者に任せる。方針は、ネーター帰納法によって7.4.Lに帰着することである。 Yの既約成分ごとに考えれば良い、Yが既約である場合を考えれば良い。Bは整域として良い。7.4.Lの開集合Uをとる。U=Yであれば解決する。そうでなければUの補集合を取る。これを改めてYとして、再帰する。 7.4.O 上記の議論を正当化せよ --- 7.4.5(*) 量化の消去 連立方程式がいつ解を持つかという基本的な疑問はどこでも現れる。・・・ k ̄^nのザリスキ位相を自然な方法で定義する:方程式で切り取られる集合を閉集合と宣言することで定義する(11.2.Iで一般化する)。 7.4.P 代数閉体での量子化の消去 ・・・ 7.4.6 消去理論の基本定理 射影空間ではシュヴァレーの定理より強い結果が言える 7.4.7 定理(消去理論の基本定理) P_A^n → SpecA は閉写像である (ネーター性は仮定しなくて良いことにも注目せよ) ・・・ 7.4.8 定理7.4.7の証明 Z⊂P_A^nを閉集合とする。π(Z)が閉集合であることを示したい。 Zは斉次元f1,f2,..∈A[x0,..xn]によって切り取られたものである。 f1,f2,..がProjκ(p)[x0,..xn]で共通零点を持つようなpの集合⊂SpecAが閉であることを示したい。 例えばまずA=kの場合を考える。 P^nにおいて斉次元g1,g2,..∈k[x0,..,xn]が共通零点を持つかどうかは、 A^(n+1)=SpecS●においてそれらが原点以外の共通零点を持つかどうかに対応し V(g1,g2,...)がV(x0,..,xn)に包含されない、 すなわち根基イデアル√(g1,g2,..)が無縁イデアルを包含しないことに対応する ・・・ これは任意のNに対して(g1,g2,..)がSのN次成分を埋め尽くさないことに対応する これはk線形写像 g1*S[N-deg(g1)](+)g2*S[N-deg(g2)](+)・・・が全射でないと言い換えられ、その条件は、行列式が0であることを確認することに帰着する。こうして、条件はザリスキ閉な条件となる。 7.4.Q A=k以外の場合を正当化せよ。 射影性は証明の鍵であった。我々は次数付き環を本質的な方法で扱った。また、この証明の本質は線形代数であったことにも注目せよ /* モデル理論的視点 http://www.math.uchicago.edu/~may/VIGRE/VIGRE2007/REUPapers/FINALFULL/Ford.pdf */ =========== 8 閉埋め込みと、関連する概念 8.1 閉埋め込みと閉部分スキーム 閉集合のスキーム論的類似は、開埋め込みの場合に比べてだいぶ様子が違った。しかし開埋め込み(スキーム論的な開集合)が底集合の開集合に由来するように、閉集合も何かに由来している。これは、SpecBの部分集合を大雑把にイデアルに結びつけるという理解が予告している。I⊂Bをイデアルとすると、SpecB/I→SpecBはスキームの射で、底空間においてはBの閉集合に埋め込まれるのであった(3.4.I)。これが我々の閉埋め込みの局所からの由来となる。 8.1.1 定義 π:X→Yが閉埋め込みであるとは、それがアフィン射でありYのすべての開アフィンSpecB⊂Yに対してπ^-1(SpecB)=SpecAとおくと環準同型がB→Aが全射である:すなわちB→B/Iの形をしていることを言う。XをYの部分集合と見なす時、閉部分スキームと言う。(閉埋め込みとは閉部分スキームと同型なものである。) 8.1.A 閉埋め込みは位相空間としてYの閉集合に埋め込まれる。 (注意:Spec k[x]/(x)→Spec k[x]と、Spec k[x]/(x)^2→Spec k[x]を考える。異なる閉部分スキームが同じ閉集合に埋め込まれることがある。"微小"な情報は失われるのである。) /* Spec k[x]_(x)の場合は底空間の像は閉点と生成点の2点(像の閉包は全体になる)*/ 8.1.B 閉埋め込みは有限射であり、従って有限型射でもある。 /* B/IはB-代数として有限生成 */ 8.1.C 閉埋め込みの合成は閉埋め込み /* 全射と全射の合成は全射 */ 8.1.D 閉埋め込みである性質は終域上アフィン局所的である。 /* アフィン交通補題を思い出す。 ・A = B/I とおく。 g∈B, π(g)=f に対して誘導される A_f→B_gは全射である。 ・Bのイデアル(g1,g2,..,gn)がB全体であり, A_fk→B_gkが全射だと仮定する。 任意のb∈Bに対して、b*(gkの冪)はA→Bの像に居るので、 「貼り合わせに関する重要なテクニック」によりbはA→Bの像に居る。 */ 8.1.E 環準同型π:B→Aが全射ならばSpecA→SpecBは閉埋め込みである (これが成り立たなかったら我々の定義はひどいものである) /* 8.1.D による */ 閉埋め込みX→Yは、Yの「イデアル層」を定める: Yのイデアル層I=I[X/Y]は、O_Y-加群の部分加群なO_Y-加群である。それぞれの開集合でI(U)はO_Y(U)のイデアルを与える。従って、0→I[X/Y]→O_Y→π_*(X)→0 という層の完全列が存在する。SpecB⊂Yにおいて全射O_Y→π_*(O_X)は定義8.1.1における環準同型B→Aに他ならない。 こうしてすべての開アフィンSpecBに対してイデアルI(B)が定まっていて、逆にこの情報からXを復元することができる:開集合の基による層の復元である。一方で、このようなI(B)の情報が閉部分スキームを定めるとは限らない(次の例8.1.F)。我々の道は簡単ではない。 8.1.F 重要でない例 X = Spec k[x]_(x), アフィン直線の原点における芽とする。これは閉点と生成点ηを持つ。I(X)={0}⊂O_X(X)=k[x]_(x), I(η)=k(x)=O_X(η) とする。この層はイデアルを貼り合わせた層であるが、閉部分スキームを定めない。(次の練習を先にした方が良いかもしれない) 次の練習は必要条件を与える: 8.1.G I=I[X/Y]を閉埋め込みX→Yに対応するイデアル層とする。B→Yを開アフィンとしてf∈Bとする。自然な準同型I(B)_f→I(B_f)は同型である。(まず自然な準同型が何かを述べよ!) /* 自然な準同型は局所化の普遍性によるものである。この話題は準連接層の所でまた登場する(13.2.A)。この練習はイデアル層は準連接層であることを主張している(13.5.4)。8.1.Fの例は準連接層でない例でもある(従ってイデアル層でもない)*/ 8.1.H 本質的で難しい練習:イデアルがいつ閉部分スキームを定めるか。 任意の開アフィンSpecB⊂Yに対してI(B)がBのイデアルで、B→B_fが誘導するI(B)_f→I(B_f)が同型だとする。このデータは一意的な閉部分スキームX→Yに由来する。 これは難しい。そこでヒントとして3つの方法が挙げる。 <方法1>SpecA, SpecBが共通部分で一致することを示す(同時基本開集合で被覆せよ)。 次に4.4.Aあるいはそこでの考え方を使う:コサイクル条件によって貼り合わせる。 <方法2>イデアルI(B)の情報を使ってイデアル層Jを定義することを考える。 開アフィンSpecBでの切断が実際にI(B)であることを確認し、商層O/JがB/I(B)であることを確認せよ。 /* 開アフィンが開集合の基をなすことから、層を定義できるという視点 */ <方法3>まずXをYの閉集合として描写する。一貫性のある芽を使ってX上の関数の層を定義する。これが目的の閉部分スキームであることを示す。 8.1.I 重要な練習:切り取られるスキーム (a) Yをスキームとしてsを大域切断とする。sによって切り取られる閉部分スキームを定義せよ。我々はこれをV(s)と表記する。これは3.4で定義した集合としてのV(s)を拡張するものである。(ヒント:開アフィンSpecB上ではSpec B/s|B をとれば良い。8.1.Hを使ってこれが閉部分スキームを定めることを示せ。) (b) uが可逆な関数ならばV(s)=V(us) (c) 関数の集合Sに対してV(S)を定義せよ /* (a) s'∈B の定めるイデアルをI(B)とする。s'のB_fへの制限がs_fと同型であることは、アフィンスキームの構造層の性質による。 (b) 各アフィンで等しい (c) Sが生成するイデアルをI(B)とする */ 8.1.J (a) 有限個の閉部分スキームのスキーム論的和集合と、有限とは限らない閉部分スキームのスキーム論的交わりを定義せよ。読者はアフィンでは、I1とI2に対応する閉部分スキームの和集合は、イデアルの交わりI1∩I2あるいは積I1I2に対応すると期待するだろう。自身との和集合は自身であって欲しいことから、正解はI1∩I2である。 (b) A^2におけるV(y-x^2)とV(y)のスキーム論的な交わりを記述せよ。/* V(y,x^2) */  対応する曲線が重複度2で交わることに注目し、答えにどう表れるかに注意せよ。  スキーム論的和集合も記述せよ。/* V(y(y-x^2)) */ (c) スキーム論的和集合の底空間は底空間の和集合であり、交わりについても同様である。 (d) A^2におけるV(y^2-x^2)とV(y)のスキーム論的交わりを記述せよ。  閉部分スキームX,Y,Zに対して(X∩Z)∪(Y∩Z)=(X∪Y)∩Zが一般には成り立たない例である。 /* X=V(y-x^2), Y=V(y^2-x^2), Z=V(y) とおくと 左辺=V(y,x^2), 右辺=V( ((y-x^2)(y^2-x^2))∪(y) ) = V(y,x^4) */ 8.1.K 難しい練習(今後使わない) 他の文献では閉部分スキームπ:X→Yは、底空間が閉集合で、構造層の射π#:O_Y→π_*(O_X)が全射であるものとして定義されることがある。(ここで全射とは層の準同型として全射、つまり茎ごとに全射であることを意図している。)この定義が我々のものと一致することを示せ。 (2.5.Eにより全射性は開集合の基で確認できることを使うと良い。) これでスキーム論的な開集合と閉集合を定義できた。これらの定義は通常の位相空間の場合に比べてより"対称的"でない。閉部分スキームの補集合となる開部分スキームは1つしかないが、開部分スキームの補集合となる閉部分スキームはたくさんある。(我々はすぐにその中から被約なものという性質で標準的なものを選べることを見る。) 8.1.2 局所閉埋め込みと局所閉部分スキーム これは局所閉集合に相当する。通常の位相空間で局所閉集合は開集合と閉集合の交わりとして定義された。従ってそれは開集合のなかの閉集合であり、閉集合の中の開集合でもある。スキーム論的にはここには微妙な問題が生まれる。 X→Yが局所閉埋め込みであるとは、X→Z→YでX→Zが閉埋め込み、Z→Yが開埋め込み、と分解できるようなものであることを言う。例えばSpec k[t,1/t]→Spec k[x,y], (x,y)->(t,0)は局所閉埋め込みである。[原点を除いた直線を平面に埋め込む] XをYの部分集合と同一視するとき、XをYの局所閉部分スキームと言う。 8.1.I 簡単な練習 局所閉埋め込みは局所有限型である この時点で読者は2つの局所閉部分スキームの交わりを定義できて、それは再び局所閉部分スキームとなる。しかし面倒なことに、その構成が局所閉埋め込みの定義における分解に依存しないことを示す必要がある。代わりに、9.2.Cで交わりをファイバー積として定義すると議論はもっと簡単にできるのでそれまで待とう。 Xの閉部分スキームVの開部分スキームUは、明らかに開部分スキームの閉部分スキームとも解釈できる:Xの開集合U'があってU=U'∩Vとおける。U→U'は閉埋め込みで、U'→Xは開埋め込みである。 しかし開部分スキームUの閉部分スキームVが、閉部分スキームの開部分スキームとして表現できるかどうかは明らかではない。位相空間の圏では、Vの閉包V'を使うことができた。これに類似する構成を考えたい。これは後で8.3に登場するいくつかの考え方に関連し、8.3.Cで部分的に解決する(例えばXがネーターな場合)。 8.1.M V→Xをスキームの射として3つの条件を考える (1) VはXの開部分スキームと閉部分スキームの交わりである。 (2) VはXの閉部分スキームの開部分スキームである (3) VはXの開部分スキームの閉部分スキームである=Xの局所閉部分スキームである。 ここで(1)⇔(2)⇒(3)を示せ。(逆は成り立つとは限らない:https://stacks.math.columbia.edu/tag/01QW) ヒント:次のファイバー図式(縦が閉埋め込み、横が開埋め込み)を考えたくなるかもしれない。 V→K ↓↓ U→X (1) このような図式が存在する (2) 左下以外が存在する (3) 右上以外が存在する /* (2)⇒(1)は本文にある議論で構成できる。*/ 8.1.N 局所閉埋め込みの合成は局所閉埋め込みである /* 閉→開→閉→開 を 閉→閉→開→開 にできる */ 8.1.3 重要でない指摘 ここの非対称性には類似がある: 部分群の商群の部分群の商群は、部分群の商群であるが、 商群の部分群の商群の部分群は、商群の部分群であるとは言えない --- 8.2 もう少し射影幾何を 次数付き環での閉埋め込みの解釈を考える。心配しなくて良い、次数付き環の最もややこしい議論はすでに済んでいる。以前の議論をちょっと広げるだけである。 8.2.1 例:P^nへの閉埋め込み 4.4.10での定義と用語を思い出そう。x0,..,xnの斉次多項式fは閉部分スキームを定める。(fは関数としての意味を持たないが、fの零点集合は意味を持つ。)基本開集合Uiではfの定める閉部分スキームはV(f(x0i,..,xni))であり、略式にはV(f(x0,..,xn)/xi^degf)と考えることができる。交わりUi∩Ujでは、fは関数として正確には一致しないが、零点を持たないスカラー倍で一致するので、同じ閉部分スキームを切り取る(8.1.I(b))。 同様に、A[x0,..,xn]の複数の斉次多項式の組も、P_A^nの閉部分スキームを切り取る。(8.2.Cで、すべての閉部分スキームはこの形をしていることを見る。) 8.2.2 1つの方程式で切り取られる閉部分スキームを(P^nの)超曲面と呼ぶ。ただし、それは1つの大域関数で切り取られるわけではない。(P^nには大域関数は存在しない。)/* 直線束の切断ということになる。 */ 超曲面の次数はその方程式の次数である。(これが閉部分スキーム自体に依存することは18.6.Hで正確に示す。)次数1の超曲面を超平面と呼ぶ。n=2の場合、超曲面は曲線と呼ばれ、超平面は直線と呼ばれ、次数2の超曲面は円錐曲線と呼ばれる。n=3の場合、超曲面は曲面と呼ばれる。(11章で次元を使って正当化する。) 8.2.A (a)wz=xy,xx=wy,yy=xzはP^3の既約な閉部分スキームを定める。 実は、次元が何かを知った後ならこれは曲線であることが分かる(11章)。この曲線はねじれ3次曲線と呼ばれる。(この3.6.Fでも登場した曲線はいろいろな非自明なことを起こすので仲良くなっておくと良い。) (b)ねじれ3次曲線はP^1と同型であることを示せ。 /* (w,x,y,z)=(aaa,aab,abb,bbb)とパラメータ付けできる */ 8.2.B S●→R●を次数付き環の全射とする。対応する射影スキームの射の定義域はProjR●全体であり(6.4.A)、それは閉埋め込みである。 8.2.C (8.2.Bの逆) X→ProjS●を閉埋め込みとする。XはProj(S●/I)と表せる。(cf.15.4.H) 8.2.D kベクトル空間の線形写像V→Wは閉埋め込みPV→PWを誘導する。 /**/ ・・・ 8.2.12 アフィン錘と射影錘 S●を有限生成次数付き環とする。ProjS●のアフィン錘とはSpecS●である。注意:これはProjS●だけでは定まらず、S●に依存する。S●=C[x,y,z]/(xx+yy-zz)のとき、アフィン錘は(0,0,0)を頂点とする通常の円錐である。直観的には、原点をアフィン錘の原点を捨てればProjS●へ射影できる何かを得られるという関係である。 8.2.O ProjS●が体k上の射影スキームのとき 自然な射 SpecS● - V(S+) → ProjS● を描写せよ。 (V(S+)を原点と呼べる理由が分かるだろうか?) 8.2.P 簡単な練習 S●が環A上の有限生成次数付き環のとき 自然な射 SpecS● - V(S+) → ProjS● を描写せよ。 実際、ProjS●はアフィン錘から原点を除いたものの商集合と見なすことができる。 8.2.13 ProjS●の射影錘とはProjS●[T]とする。Tは新しい変数で次数1とする。例えばS●=C[x,y,z]/(xx+yy-zz)のとき、射影錘はProj C[x,y,z,T]/(xx+yy-zz) である。これはSpecS●の射影補完と呼ばれることもある。これもProjS●だけでは定まらず、S●に依存する。 8.2.Q より重要でない練習(cf.4.5.1) 射影錘はProjS●と同型な閉部分スキームを持ち、(略式にはT=0に相当する。)その補集合はアフィン錘SpecS●に同型である。 --- 8.3 スキーム論的な像 我々はこれから「何かを満たす最小の閉部分スキーム」という形をした 一連の用語を定義する。 1つの例は、スキーム論的な局所閉埋め込みの閉包であり、 これによって我々は局所閉埋め込みを3つの方法で解釈することができる: ・開部分スキームと閉部分スキームの交わり ・閉部分スキームの開部分スキーム ・開部分スキームの閉部分スキーム (cf.8.1.M) 8.3.1 スキーム論的な像 集合論的な像は、振る舞いが悪いことを7.4.1で見た。十分に良い条件のもとでも、振る舞いが悪いことを7.4.2で見た。例えば、A^2→A^2:(x,y)->(x,xy)の像にはスキームの構造は合理的な方法で入れられない。そこで、8.3.Cで、"像を最も良く近似する閉部分スキーム"を定義する。そこでは、被約な構造の像は被約な構造を持つ。ただしいつものように、この概念がうまくいくような仮定をおく必要がある(定理8.3.4と系8.3.5)。 8.3.2 定義 i:Z→Yが閉部分スキームで、0→J[Z/Y]→O_Y→i*O_Z→0 の完全系列が対応しているとする。 「π:X→Y の像がZに居る」とは、合成 J[Z/Y]→O_Y→π_*(O_X) が零射となることによって定義する。 略式には、Zで消える関数はXに引き戻すと局所的に零関数になることを言っている。 特にπの像がZの部分スキームに居るならば、πの像はZに居る。 次に、「πのスキーム論的な像」を、上記の性質を満たす最小のZとして定義する。 特にYがアフィンのとき、像は、Xに引き戻すと零になる関数で切り取られる閉部分スキームである。 /* この定義の解釈がなかなか分からなかったが、次のように認識した。J[Z/Y]→O_Y→π_*(O_X) が零射ということは、J[Z/Y]→O_Yの余核がi_*(O_Z)だから、それを経由する:つまりJ[Z/Y]→O_Y→i_*(O_Z)→π_*(O_X)がある。ということは X→Z→Y という分解で、Z→Yが閉部分埋め込みとなるようなものが存在すると認識した。この認識は役に立った。 */ <例1> Spec k[ε]/(ε^2) → Spec k[x], [x->ε] のスキーム論的な像は k[x]/(x^2)である。  こうして、もやもやした点は引き続きもやもやしている。 <例2> Spec k[ε]/(ε^2) → Spec k[x], [x->0] ではスキーム論的な像は k[x]/(x)である。  この例では、もやもやは準同型によって"つぶされて"いる。 <例3> Spec k[t,1/t] → Spec k[u], [u->t] ではスキーム論的な像は全体である。  一方で、集合論的な像は点(u)を除外した開集合となる。これはそれほど奇妙な例ではない。スキーム論的な像は閉集合でなくてはならず、今回の場合、集合論的な像の閉包となっている。我々は十分に良い状況ではいつもこのような振舞いであることを想像し、それは実際正しい(8.3.6)。 しかし残念ながら、十分に良い状況でなければ、奇妙なことが起き得る。 <例4>(8.3.11) 非交和 X = +Spec k[ε]/(ε^n) (5.2.Eのヒントとして現れた) → Y = Spec k[x], [x->すべての成分がx] を考える。集合論的な像は原点だけだが、スキーム論的な像は全体である。 8.3.3 スキーム論的な像がアフィン局所的に計算できるための条件 Y=SpecBがアフィンならば、π:X→Yのスキーム論的な像は、Xに引き戻すと0になるようなY上の関数のなすイデアルによって切り取られる(閉部分)スキームである。これを使ってスキーム論的な像をアフィン局所的に計算できたら素敵であろう。すなわち正確には SpecB⊂Y上で、I(B) = ker(B→Γ(SpecB,π_*(O_X))) と定義する。そうすると、もしI(B)_g → I(B_g) が常に同型であったとしたら、このIはイデアル層となり、閉部分スキームを得られるのであった(8.1.H)。引き戻すと0になるようなSpecB上の関数のD(g)への制限は、引き戻すと0になるようなD(g)上の関数である。問題は、引き戻すと0になるようなD(g)上の関数r/g^nがあったときに、引き戻すと0になるようなSpecB上の関数でr*g^mの形をしたものが存在するかどうかである。いくつかの状況ではうまくいくことを示す。(私は絵を描きたいが、読者を混乱させずに視界を開くような描き方を思いつかない。読者が自身に合ったものを試みると良い。) (1) π^-1(SpecB) が被約であれば、常にm=1ととれる。 (2) π^-1(SpecB) がアフィンな場合もうまく行く。 r,gの引き戻しをr',g'とする。r'がD(g')上で0ならば、r'(g')^m=0となるmが存在する。アフィンスキームの構造層の性質によりD(g')上の関数はA_g'だからである。 (2)' より一般にπ^-1(SpecB) が準コンパクトであればうまくいく。それぞれのアフィン被覆でr*g^m_i=0となるm_iが存在するのでそれらの最大値をとれば良い。(ここでも準コンパクトは我々の友であるのだ。) 以上より、次の(ちょっと微妙な)定理を得た: 8.3.4 定理 π:X→Yについて、Xが被約であるか、πが準コンパクト射ならば、πのスキーム論的な像はアフィン局所的に計算できる:Spec B⊂Yでは引き戻して0になる関数で切り取られる閉部分スキームである。 8.3.5 系:上記の仮定で、スキーム論的な像は集合論的な像の閉包である。 8.3.6 上記の仮定で、集合論的な像が閉集合ならば、スキーム論的な像は集合論的な像に一致する [8.3.7 系の証明] 集合論的な像はスキーム論的な像に含まれる。(確認せよ!)/* 8.3.2の段落に書いた私の認識 */ スキーム論的な像は閉集合なので、集合論的な像の閉包を包んでいる。一方で、Uを集合論的な像の補集合とすると、π^-1(U)=φである。現在の仮定ではスキーム論的な像はアフィン局所的に計算できて、従ってスキーム論的な像のUへの制限は空集合である。 8.3.A 簡単な練習 Xが被約ならX→Yのスキーム論的な像も被約である 8.3.B(*) 重要でない練習 π:X→Yを局所ネータースキーム間の準コンパクト射とする。スキーム論的な像の随伴点は、集合としてのXの随伴点の像の部分集合となる。(準コンパクト性を仮定しないと+Spec C[t]/(t-a)→Spec C[t]という反例がある。始域の被約性は役に立たないことに注意せよ。)ヒント:XとYがアフィンな場合に帰着せよ。(読者はこの主張がもっともらしく思えるような幾何的直観を持つことができるだろうか?) /* 問題文の「部分集合」という言葉は不可欠である。例えば環準同型k[y]→k[x,y]/(xy)に対応するスキームの射で、(x)という随伴点は(0)∈k[y]という随伴点に移るが(y)という随伴点の行き先(y)∈k[y]は随伴点ではない。 SpecA→SpecBの場合を考える。さらにYをスキーム論的な像に制限して考えれば良い。すなわち環準同型π#:B→Aが単射な場合に帰着する。つまりBをAの部分環とみなせる。 qをBの随伴点とする。Aの随伴点pでπ(p)=p∩B=qであるものを示したい。仮定よりa∈Aの零化イデアルIがI∩B=qを満たすようなものが存在する。5.5.Jの精神で、イデアルの昇鎖列を作れば良いと思った。 (追加考察:https://detail.chiebukuro.yahoo.co.jp/qa/question_detail/q13207368353) (我々は5.5(の最後以外)で随伴点を考える時にはネーターを仮定する立場であった)*/ 8.3.8 スキーム論的な局所閉埋め込みの閉包 局所閉埋め込みπ:X→Yの閉包を、πのスキーム論的な像によって定義する。 8.3.C 局所閉埋め込みV→Xが準コンパクトまたはVが被約ならば、練習8.1.Mの逆が成り立つ(Vは定義から開部分スキームの閉部分スキームであるが、閉部分スキームの開部分スキームとしても表せる)。 ヒント:系8.3.5 8.3.D 重要でない練習、直観に良い 局所ネータースキームへの局所閉埋め込みπ:X→Yの閉包の随伴点は、Xの随伴点と自然に同一視できる。略式には、被約でない(もやもやした)構造は、Xから"引き継がされた"ものだけである。 /* 閉埋め込みと開埋め込みそれぞれで示せば良いと思う(5.5.F) */ 8.3.9 閉集合における被約な部分スキーム構造 X^setをYの底空間の閉集合とする。X^setに、被約なスキーム構造Xを定義することができる。我々はこれを、X^setのすべての点での値が0になる関数で切り取られるものと記述することができた。SpecB⊂Yで、X^setがI=√I(X^set)とおけるなら(3.7の記号)XはSpec B/Iに対応する。読者はこれが任意の基本開集合の包含に対して都合良く振舞うことをすぐに確認できるだろう。我々はまた、次のような変わった視点で構成することもできる:WをX^setのすべての点の非交和からなるスキームとする。そのp∈X^setでの構造層はOY_pとする。ρ:W→Yを標準的な射影によって定める。ρのスキーム論的な像が求めるXである。3つ目の方法:底空間がX^setを包むような閉部分スキームのうち最小のもの。 8.3.E これら3つの定義が同じであることを確認せよ。 これをX^setに誘導された被約な部分スキーム構造と呼ぶ。 (曖昧な練習:読者が満足するまでこの記述を正確にせよ。) 8.3.F 被約な部分スキーム構造の底空間は閉集合X^setに一致することを確認せよ。Xが被約であることを確認せよ。 8.3.10 特にX^setがY全体とすると、Yの被約化Y^red→Yが定義される。開アフィンでは、√0で切り取られる閉部分スキームに相当する。略式には、もやもやを取り除いたものということになる。 別の同値な定義は:開アフィンで√0で切り取られることを定義とすると8.1.Hによりこれが閉部分スキームを定める。 8.3.G 実際に同値であることを確認せよ 8.3.11(*) 注意 すべての開集合U⊂Yに対してΓ(U,O_Y^red)がΓ(U,O_Y)を√0で割ったものとは限らない。8.3.2の<例4>で登場したスキームY=+Spec k[x]/(x^n)では、xは冪零でないが、Y^redでは0となる。これはxが"局所的に冪零"だからである。 8.3.12 スキーム論的な準連接層の台 同様の考えが18.9.Bで登場する --- 8.4 効果的カルチエ因子、正則列、正則埋め込み 正則埋め込みという概念を導入する。これは局所閉埋め込みの重要なクラスである。正則スキームの正則スキームへの局所閉埋め込みは、正則埋め込みの1つの重要な例である(12.2.L)。効果的カルチエ因子は、基本的に余次元1の場合、繰り返し有用であることが分かるだろう -- それがどれだけ有用かをみるには、索引にどれだけ現れているか見てみよ。まずこの場合から始める。 8.4.1 局所単項閉部分スキームとカルチエ因子 閉部分スキームはが局所単項であるとは、十分に小さい開被覆の各開集合で、1つの等式で切り出されることを意味する。より具体的には、局所単項閉部分スキームは、閉埋め込みπ:X→YでYの開被覆{Ui}があって、それぞれのπ^-1(Ui)→Uiが、si∈Γ(Ui,O_X)の零点集合V(si)となっているということである。特に、X→Yが局所単項閉埋め込みのとき、Yの開被覆は開アフィンなものを選択できる。単に各Uiを開アフィンで覆ってsiを開アフィンに制限すれば良い。 例えば、P^nの超曲面(8.2.2)は局所単項である。それぞれの(n+1)変数同次多項式はP^nの局所単項閉部分スキームを定める。(注意:局所単項性と違って、単項性はアフィン局所的な性質ではない。19.11.10を見よ。) イデアル層が局所的に零因子でない関数で生成されるとき、その閉部分スキームを効果的カルチエ因子と呼ぶ。より正確には、X→Yが閉埋め込みで、SpecAi⊂YによるYの被覆と零因子でないti∈Aiがあって、V(ti)=X|SpecAiとなっているとき、XはYの効果的カルチエ因子と呼ぶ。(この言葉の由来は我々の視点では関係がないので説明しない。) 8.4.A t∈Aは零因子でないとする。 任意の素イデアルpに対してtはA_pの零因子でもない。 /* tu=0となるuが無ければ、tus=0となるu,sも存在しない[sは局所化に使う積閉集合] */ 8.4.2 注意 DをSpecAの効果的カルチエ因子とする。D=V(t)と書けるとは限らない(14.2.M, 19.11.10)。すなわち閉部分スキームが効果的カルチエ因子となるかどうかは特定のアフィン被覆で確認できるが任意のアフィン被覆で確認できるわけではない。 8.4.B Xを局所ネータースキームとしてtを大域切断とする。V(t)がカルチエ因子となることは、tがXのすべての随伴点で消えないことと同値である。 /* 零因子でなければ局所化で消えない。 随伴点の性質より、どこかで消えれば、その近傍にある随伴点で消える。*/ 8.4.C V(t)=V(t')を効果的カルチエ因子とする。tはt'に可逆な関数を掛けたものである。 /* 局所ごとに存在し、貼り合わさる */ 効果的カルチエ因子の概念は、正則列の概念に一般化される。(この議論は都合の良いスキームでは8.4.Hで効果的カルチエ因子を余次元1の正則埋め込みと解釈する所で終わる。) 8.4.3 正則列 正則列という概念は、セールによるのだが、次の幾何的発想の代数的バージョンである: 局所的に、効果的なカルチエ因子をとり、その上にある効果的なカルチエ因子をとり、という操作を有限回繰り返す。ちょっとした注意が必要である。順番が問題になることがある。 8.4.4 定義 MをA-加群とする。x1,..xr∈Aは次の2つの条件を満たすときM-正則列と呼ぶ: (i) xiはM/(x1,..,x[i-1])の零因子でない。 (i=1の場合はx1はMの零因子でない、と解釈する。零因子は1.3.Cで定義した。) (ii) (x1,..,xr)M ⊂ M は真の包含である。 特にM=Aのとき、これは「完全交差」の理にかなった近似となるはずである。 実際、これを定義に使う(8.4.7)。A-正則列は単に正則列と呼ぶ。 8.4.D 練習 MをA-加群とする。M-正則列は任意の局所化を行っても正則列の定義の条件(i)を保つ。 /* i=1を考えれば良い。 xがMの零因子 ⇔ xy=0 となる y≠0 がある xがS^-1Mの零因子 ⇔ xy=0 となる 任意のs∈Sに対してsy≠0なyがある [(i)を保たなかったのが局所化で保つようになることはあり得る] */ 8.4.E x,yがM-正則列ならば、x^N,yもM-正則列である ヒント:・・・ 8.4.5 興味深い例 (文献[E]の例17.3) 正則列では順番が問題になりえる。・・・ 8.4.6 定理 (A,m)はネーター局所環でMは有限生成A-加群とする。M-正則列は順番を並べ替えても正則列を保つ。 (文献[Di]でネーターという仮定が必要であることが半ページの議論で示されている) まず最初の自明でないr=2の場合を示そう。これはひそかにKoszul複体の議論である。 x,y∈mがM-正則列だとする。つまりx∈mはMの零因子でなくyはM/xMの零因子でないとする。 二重複体: M→M ↑↑ M→M を考える:上矢印はy倍で、右矢印は上が(-x)倍、下がx倍。 スペクトル列を使って、全複体のコホモロジーを計算する。 最初のページでは、 (0:x) M/xM ↑  ↑ (0:x) M/xM を得る(上矢印はy倍)。ホモロジーをとって2枚目のページを得ると、 0 M/(x,y)M 0 0 を得る。ここでyがM/xMの零因子でないことを使った。ここで収束する。 次にスペクトル列を縦横入れ替えて走らせる。 ・・・(いまいちわからない) 8.4.F 8.4.7 正則埋め込み π:X→Yを局所閉埋め込みとする。πが点pで(余次元r)の正則埋め込みであるとは、局所環OY_pにおいてXに対応するイデアルが(長さrの)正則列で生成されることを言う。πがすべての点で(余次元r)の正則埋め込みであるとき、πは(余次元r)の正則埋め込みであると言う。 余次元という言葉自体については11.3で明らかになる。 8.4.G 局所閉埋め込みが正則埋め込みである条件は開な条件である: 局所ネータースキーム間の閉埋め込みπ:X→Yが点pで正則埋め込みならば、pを含むある開集合でも正則埋め込みである。 ヒント:πが閉埋め込みでYがアフィンな場合に帰着せよ。Y=SpecB, X=Spec B/I, (f1,f2,..,fr)がI_pを生成する正則列であるときにこれを開近傍に延長したい。次の代数的な事実を証明せよ: 「I,Jがネーター環AのイデアルでI_p=J_pならば、I_α=J_αとなるようなα∈A-pが存在する。」 I∩Jを考えることで、I⊂Jな場合に帰着せよ。K=J/IとおいてK_p=0ならばK_α=0の形に帰着せよ。 従ってXが準コンパクトならば閉埋め込みが正則埋め込みであるかどうかはすべての閉点で調べれば良い。 12.1.F(b)で、正則埋め込みでない閉埋め込みの例が登場する。 8.4.H 閉埋め込みX→Yが余次元1の正則埋め込みであることは、XがYの効果的カルチエ因子であることと同値である。 重要でない指摘:ネーターという仮定は、O_Yが連接層であるという仮定に置き換えられる。興味深いことに、効果的カルチエ因子ならば正則埋め込みというのは常に正しいが、逆はネーターという仮定が必要である。 8.4.8 余次元rの完全交叉とは、r個の効果的カルチエ因子D1,..,Drのスキーム論的な交わりで、すべての点においてD1,..,Drが正則列をなすものを言う。 ============= 9 ファイバー積と底変換 9.1 それは存在する 9.1.A X,Yをスキームとする。 貼り合わせ X+Y は余積である 9.1.2 注意:積の底空間は、底空間の積ではない。  A^1×A^1=A^2には軸に平行でない曲線に対応する素イデアルがある 9.1.3 S-valued pointは直積と可換である /**/ ・・・ [証明の精神] まず、我々は次の2つの場合のファイバー積を知っている。 ・X,Y,Zがアフィンのとき:環のテンソル積(6.3.6,6.3.F) ・Y→Zが開埋め込みのとき:X→ZのYへの制限(7.1.B) スキームはアフィンスキームを開集合で貼り合わせて作るので、 これらの場合が基本的であり、必要なすべてである。 議論は同じアイデアを繰り返し使う: 「スキームは張り合わさる(4.4.A)」「スキームの射は張り合わさる(6.3.A)」 この現象の背景について、9.1.6で熟練者への説明をする。 [証明] Step 1; X,Y',Zがアフィンで、YがY'の開部分スキームのとき  W'=X×Y', W=Y×W' とおくと W=X×Yである。 (練習1.3.Qで扱ったが、この場で示せるべきである)  開埋め込みは底変換で保たれる(7.1.B)ことを思い出す。  すなわちWはW'の開部分スキームである。後で使う。 Step 2(Key); X,Zがアフィンで、Yが任意のスキームのとき  Yが2つのアフィンの貼り合わせのときを説明する。  Y=Y1∪Y2, Y1∩Y2=Y12とおく。  W1=X×Y1とW2=X×Y2が存在する。Step1よりW12=X×Y12も存在する。  W12はW1,W2の開部分スキームなので、  W1,W2をW12にそって貼り合わせることができる。これをWとする。  普遍性を確認する。・・ --- 9.2 実際にファイバー積を計算する 特にファイバー積をとりやすい4つの例があり、すべての射はこれらの部品を組み立てて作ることができる。依り正確には、(1)はファイバー積を局所的に計算できることを示している。なので開アフィンに帰着すれば、A {@_B} Cを計算することに帰着する。環準同型B→AはB→B[t1,...]/Iの形で表現できるので、(2)変数を追加する、(3)剰余環をとる、の操作で理論上すべてのファイバー積を計算できる。(4)は局所化に対応し、これはまた具体的な理解に役立つ。 (1) 開埋め込み これは7.1.Bで既に行い、ファイバー積が存在することの証明でも使った (2) 変数の追加 9.2.A 簡単な代数の練習 同型 A {@_B} B[t] = A[t] を示せ。/* ヒント:普遍性が役に立つ */ 9.2.1 スキーム上のアフィン空間 Xを任意のスキームとする。A_X^n = X {×_Z} Spec Z[x1,..,xn] と定義する。これは3.2.3の[例8]で定義したようなアフィン空間A_C^nの拡張である。 (3) 剰余環をとる=閉埋め込みによる底変換 9.2.B φ:B→Aを環準同型とする。BのイデアルIのAへの拡張I^eを、φ(I)で生成されるイデアルとして定義する。同型 A/I^e=A {@_B} (B/I) を描写せよ。 (ヒント:テンソル積の右完全性(1.3.H)を利用せよ) 9.2.2 従って閉埋め込みは底変換で保たれる 9.2.C (a) Xへの2つの閉埋め込みの交わりをファイバー積として解釈せよ(cf.8.1.J)。 (b) 局所閉埋め込みも底変換で保たれる (c) 有限個の局所閉埋め込みの交わりをファイバー積を使って定義せよ。それはまた局所閉埋め込みとなる。 9.2.D X,Yを(局所)有限型A-スキームとする。X {×_A} Y も(局所)有限型A-スキームとなる。 9.2.3 応用として、k上有限生成な-代数のテンソル積を計算できる。 C @_R C = C @_R R[x]/(x^2+1) =(C@R[x])/(x^2+1) (3)より =(C[x])/(x^2+1) (2)より =C[x]/((x-i)(x+i)) =C[x]/(x-i)×C[x]/(x+i) 中国剰余定理 =C×C すなわち SpecC {×_R} SpecC = SpecC+SpecCである。これはスキームの射に関する多くの特徴的な振舞いの例として現れる(9.5)。2つの点がガロア群に対応することに注意せよ。 9.2.4(*) 指摘 L/Kが有限ガロア拡大でそのガロア群をGとすると、L@Lは |G|個のLのコピーに同型となる。これは指標の線形独立性という古典的な結果を示している!別の言葉では、SpecL×SpecLはガロア群G上のtorsorをなす。ただしtorsorという言葉はこの本では定義しない。 9.2.E 難しいがGal(Q ̄/Q)を知っている読者に魅力的な問題 Q上のスキーム SpecQ ̄@Q ̄ の点は、自然にGal(Q ̄/Q)と全単射にあり、そのザリスキ位相はガロア群の副有限位相と一致する。 ヒント:まず有限ガロア拡大を考察せよ。位相を逆極限と結びつけよ。ガロア群の単位元に対応する点はどの点か分かるだろうか?) (4) 局所化による底変換 9.2.F φ:B→Aを環準同型としてS⊂Bを積閉集合とする。φ(S)はAの積閉集合である。自然同型φ(S)^-1 A=A@(S^-1B)がある。 おおまかに「局所化のファイバー積はファイバー積の局所化である」「局所化は底変換で保たれる」ということである。基本開集合をとる操作B→B_fや生成点をとる操作B→K(B)などのときに有用である。 9.2.5 例 以上の4つの事実を使って計算する 9.2.G 重要なモノ射3つ 開埋め込み、閉埋め込み、局所化はモノ射である。 モノ射は合成について閉じている(1.3.V)ので、上記の合成である局所閉埋め込みや、Xの芽のスペクトラムからXへの埋め込み(6.3.K)もモノ射である。 /* 1.3.Wが利用できる。 */ 9.2.H Spec A[x1,..,xn] = Spec Z[x1,..,xn] {×_Z} SpecAである Proj A[x0,x1,..,xn] = Proj Z[x0,x1,..,xn] {×_Z} SpecAである 9.2.6 基礎体の拡大 XがK-スキームでL/Kが体の拡大のとき(例えばL=K ̄)、X ×SpecK SpecL (時にX ×K L とかX_Lと略記される)はL-スキームとなる。Xの性質はX_Lの性質をしばしば引き継ぐ。これは降下descentという概念である。例えば整閉整域であることや、2つの射が同一であること、アフィン射が閉埋め込みであることなどが、降下する(5.4.M, 9.2.I, 9.2.J)。代数閉体でない体でのスキームに興味がある人はこれを繰り返し使うだろう。 9.2.I K-スキームの射πとρ:X→Yがあるとき、上記のSpecLでファイバー積をとることでL-スキームの射π_Lとρ_L:X_L→Y_Lが誘導される(この意味を確信せよ)。π_L=ρ_Lならばπ=ρである。 (ヒント:9.4.Dで示すことにX_L→Xが全射であることを使っても良い。) /* X→Y_Lが同一であることは X_L→X→Y と X_L→L が同一 であることを意味する */ 9.2.J 同じ設定で、πが開埋め込み⇔π_Lが開埋め込み、πが閉埋め込み⇔π_Lが閉埋め込み (簡単のためπがアフィン射の場合に限っても良い。それが最もよく使われる場合でもある。) /* イデアルを係数拡大することに相当する */ 9.2.K 重要でないが楽しい練習 Spec Q(t) @_Q C の点はCの超越数に対応する。似た考えが11.2.16で現れる。 --- 9.3 解釈:引き戻した族、射のファイバー 9.3.1 ファイバー積は略式的に次のように幾何的に考えることができる。 Y→Zを射とする。これを、「基底スキームZによってパラメータ付けされたスキームの族」と解釈する。そうすると、別の射φ:X→Zに対して、X {×_Z} Y→Zは引き戻した族と解釈される。 X {×_Z} Yを、Yのスキーム論的引き戻しとか、スキーム論的逆像とか、Yの逆像スキームtか呼ぶ。こうして、ファイバー積は底変換とか引き戻しとか呼ばれる。1.3.6で紹介したデカルト図式に対する様々な名前に加えて、代数幾何では底変換図式とか引き戻し図式とか呼ばれ、射X {×_Z} Y→ZはYのφに沿った引き戻しとか呼ばれる。 X {×_Z} Yについて議論する時、特にXがZの局所閉スキームの時、しばしばX上のという言葉を使う。(・・・) 典型的な例を挙げよう。tでパラメータ付けされたxy平面の曲線の族y^2=x^3+txを考える。翻訳すると、Spec k[x,y,t]/(y^2-x^3-tx)→Spec k[t]を考える。これをuv=tによってuv平面の族に引き戻す:Spec k[u,v]→Spec k[t] (t→uv)。そうするとy^2=x^3+uvxを得る。すなわちSpec k[x,y,u,v]/(y^2-x^3-uvx)→Spec k[u,v]。もしtを3に設定する、すなわちSpec k[t]/(t-3)→Spec k[t]に沿って引き戻せば、y^2=x^3+3xを得る:Spec k[x,y]/(y^2-x^3-3x)→Spec k。これは曲線の族のt=3の上にあるファイバーと解釈される。これからこれをファイバー積の言葉で構成する様子を解釈する。 9.3.2 射のファイバー もし練習7.3.Kで既に有限射のファイバーが有限であることを見ているなら、次の議論は驚くべきものではないだろう。引き戻しの特殊な場合は、射のファイバーという視点である。我々はこれを位相空間の圏でのファイバーの視点から動機づける。 9.3.A Y→Zが位相空間の連続写像で、位相空間XがZの点pだとするなら、p上のYの(集合論的)ファイバー(に誘導位相を入れたもの)はX {×_Z} Yと自然に同一視される。 より一般的に、任意のπ:X→Zに対して、Xの点pにおけるX {×_Z} Y→Xのファイバーは、Zの点π(p)におけるY→Zのファイバーと自然に同一視される。 スキームの圏に戻ろう。p→Zを(閉点とは限らない)点の包含とする。正確には、剰余体Kを持つ点pを像とするSpecK→Zを考える。g:Y→Zを別の射とすると、p→Zによる底変換は、p上の(スキーム論的)ファイバー、あるいはpの(スキーム論的)逆像とか呼ばれ、g^-1(p)で表される。Zが既約なら、Zの生成点の上のファイバーをgeneric fiberと呼ぶ。 開アフィンSpecAが点pを含むなら、SpecK→Zは環準同型A→A_p/pA_pに対応する。これは局所化と閉埋め込みの合成なので、9.2の方法で計算できる。(9.2.Gによりこれはモノ射である。) 9.3.B X→Yの点p上の(スキーム論的)ファイバーの位相空間は、X→Yの点p上の(位相空間論的)ファイバーの位相空間と一致する。 9.3.C τ:X→Zとπ:Y→Zのファイバー積 X ×Z Y → X の点p∈X上のファイバーは、Y→Zの点τ(p)∈Z上のファイバーと一致する。 /* ファイバー積図式を2つくっつけた図式もファイバー積図式 (直接的には前者の普遍性を満たすものが後者の普遍性も満たすとして) */ 9.3.3 視界を開く例(3.2.Oの再訪) 放物線 y^2=xをx軸に射影することを考える。xをy^2に送る環準同型Q[x]→Q[y]に相当する。 (1) 閉点1の逆像は2つの点である SpecQ[y]@Q[x]/(x-1) = Spec Q[x,y]/(y^2-x)@Spec Q[x]/(x-1) =Spec Q[x,y]/(y^2-1) =Spec Q[x,y]/(y-1) ×Spec Q[x,y]/(y+1) (2) 閉点0の逆像は1つの被約でない点を与える Spec Q{x,y]/(y^2-x,x)=Spec Q[y]/(y^2) (3) 閉点-1の逆像は、1つの被約な点であるが、基礎体上「大きさ」が2である Spec Q{x,y]/(y^2-x,x+1)=Spec Q[y]/(y^2+1)=Spec Q(i) (4) 生成点の逆像は再び1つの被約な点であるが、 Spec Q{x,y]/(y^2-x)@Spec Q(x)=Q(y) このようにQ(y)とQ(x)は体としては同型であるが、Q(y)はQ(x)の2次拡大であると言える。 [読者は(x^3-4)や(x^2-2)の逆像を考えたくなるかもしれない] 4つの場合とも、像の数は、うまく数えれば2つであることに注目せよ。・・・ この例は9.5.E, 17.4.9でまた登場する。24章の平坦射の概念とも関係する。 ・・・ 9.3.D SpecZ[i]→SpecZの点p上のファイバーは何か?pによって振る舞いが異なり、9.3.3と同様に4つの場合があるだろう。 9.3.E 環準同型u->t^2に対応する X=Spec k[t]→Y=Spec k[u] を考えて、ファイバー積X {×_Y} X を考える。まずkの標数が2でないとする。X {×_Y} X は2つの既約成分を持つことを示せ。kの標数が2のときには何が起きるか?(9.5.A) /* Spec k[t]/(t^2-u)@Spec k[s]/(s^2-u) =Spec k[t,s]/(t^2-u, s^2-u) =Spec k[t,s]/(t^2-u, t-s) ×Spec k[t,s]/(t^2-u, t+s) =Spec k[s]/(s^2-u) ×Spec k[s]/(s^2-u) 標数2ではt+s=t-sなので被約でない結果となる */ 9.3.5 ブローアップの最初の紹介 9.3.F 重要な具体的な練習 ブローアップBl(0,0)A^2をA^2×P^1の閉部分スキームとして次のように定義する:A^2の座標をx,y, P^1の座標をu,vとする。この閉部分スキームは、方程式xv=yuによってきりだされるものとする。(次のように解釈したいだろう:P^1は原点を通る直線をパラメータ付けする。ブローアップは、原点と点pが共に直線L上に存在するようなpとLの組という順序つき集合に対応する。)P^1の閉点におけるBl(0,0)A^2→P^1のファイバーを描写せよ。Bl(0,0)A^2→A^2は(0,0)以外ではA^2と同型であることを示せ。(0,0)上のファイバーは効果的カルチエ因子(8.4.1で零因子でない1つの方程式で切り出される閉部分スキームとして定義された)である。これは、「例外因子」と呼ばれ、22章でさらに議論する。この例は22.1や20.2.Dを動機づける例となる。 /* (au+bv)上のファイバーは、生成元[(ay-bx),(au+bv)]とその特殊化たち[(x-x0,y-y0),(au+bv)] (ay0-bx0=0)からなる。 ・・・ */ 我々はまだ正則性を定義していないが、Bl(0,0)A^2は滑らかであることを主張する、手を振るような議論はできる:P^1の各標準的開集合Ui上の逆像は、A^2と同型である。こうしてブローアップは、A^2から1点を切り取って、P^1で貼り合わせ、別の滑らかな曲面を作りだすという手術である。 9.3.6 --- 9.4 底変換で保たれる性質 ・・・ --- 9.5(*) 底変換で保存されない性質と、修正の仕方 幾何的直観から、引き戻しで保たれることを期待したい性質がいくつかある。位相空間の圏ではファイバーは引き戻しについてうまく振舞う。例えばすべてのファイバーが既約な族の引き戻しはまた既約である。連結についても同様である。代数幾何では、残念ながら、どちらも成り立たない。例9.2.3で示したように SpecC→SpecR←SpecCのファイバー積はSpecC+SpecCであり、SpecC→Rのファイバーは既約で連結だが引き戻したSpecC+SpecC→SpecCのファイバーは既約でも連結でもない。整という性質についても同様に引き戻しでうまく振舞わない。9.4.1で、単射性が底変換で保たれないことを示すのにも用いた。 9.5.A 練習 kは表数pの体とする、従ってk(u)/k(u^p)は不分離拡大である。k(u) @_k(u^p) k(u)を考えることで、被約な射のファイバーは被約とは限らないことを示せ。(我々は間もなくこれが標数pのときだけに起きることを見る。それは不分離拡大の存在による。) これらの問題を次のように修正する 9.5.1 スキームXの幾何的点とは、射Spec k→X (kは代数閉体)であると定義する。恐ろしいことにこれはスキームの点と呼ばれるものの3種類目である。1つが通常の点(底空間の元)、2つがZ値点(射Z→Xのこと, 6.3.9)、そして3つ目が幾何的点である。幾何的点は明らかにZ値点の1種ではあるが、通常の点に近い性質も持つ。それらは、通常の点に、剰余体の代数閉体への埋め込みという情報を加えたものと解釈できる。 射X→Yの幾何的ファイバーは、Yの幾何的点の上のファイバーと定義できる。すなわち、幾何的点Speck→Yとのファイバー積である。スキームの射のファイバーが幾何的に連結であるとは、そのすべての幾何的点での幾何的ファイバーが連結であることと定義する。k-スキームXが幾何的に連結であるとは、構造射X→Speckが幾何的に連結であることと定義する。(既約、整、被約についても同様。)我々はすぐに、これらの状況を確認するには、k ̄への底変換だけを確認すれば良いことを見る。 9.5.B ファイバーが幾何的に連結(既約、整、被約)であるという性質は底変換で保たれる。 /* X→YのZ→Yによる底変換を考える。Zの幾何的点SpecK→Z上での幾何的ファイバーはZ→YによりYの幾何的点での幾何的ファイバーに押し出される */ 9.5.C 算術的な精神のための練習 モノ射SpecC→SpecRについてすべての幾何的ファイバーは2点からなる(cf.9.2.3)ことを示せ。こうしてSpecCは幾何的に被約だが幾何的に既約でないR-スキームである。 /* kが代数閉体、特にkがx^2+1を分解できる体であればR上のテンソル積 k(×)C は2点からなると主張する。C=R[X]/(X^2+1)と書けるからテンソル積の係数拡大の視点からこれはk[X]/(X^2+1)でありk上でX^2+1が2つの1次式に分解されればこれは2つの体の直積である。 */ 9.5.D 次のようなk-スキームの例を挙げよ 被約だが幾何的に被約でない 連結だが幾何的に連結でない 整だが幾何的に整でない 9.5.E 練習(幾何的ファイバーが有用だと確信するために) 例9.3.3:放物線y^2=xのx軸への射影を思い出そう。すなわち環準同型Q[x]→Q[y]:x->y^2 に対応するアフィンスキームの射である。この射の幾何的ファイバーは、原点[(x)]を除いて常に2点であることを示せ。 (SpecC→SpecQ[x]とSpecQ ̄→SpecQ[x]はどちらも幾何的ファイバーと呼ばれるものであることに注意せよ) /*・閉点p=(x-a)上のファイバー 9.3.3の議論の通り通常のファイバーはQ[x]上のQ[x]/(x-a)とQ[y]のテンソル積でQ[y]/(y^2-a)である。幾何的ファイバーではQ[x]/(x-a)の代わりにk[x]/(x-a),kは代数閉体と置き換える結果、k[y]/(y^2-a)となる。a≠0のとき(y^2-a)は常に2つの一次式に分解されるの剰余環のSpecは2点となる。 ・生成点p=(0)上のファイバー Q(x)とQ[y]のテンソル積である。KはQ(x)を含む代数閉体で、特に±√xという元を持つ。そうしてSpec K[y]/(y^2-x)は2点となる。*/ ・・・ --- 9.6 射影スキームの積:Segre埋め込み --- 9.7 正規化 正規化は、被約なスキームを、正規なスキームに変えることである。 具体的には正規なスキームからの射 v :X ̄→X で、 vはX ̄とXの既約成分の間に全単射を誘導し、 それぞれの既約成分で双有理な射を与えるものである。 ・・ Xが既約、従って整である場合から始める 正規化は、既約で正規なスキームからの支配的な射で最も短いものである まず、X=SpecAの場合を考える。 9.7.A Aを整域とする。AのK(A)における整閉包A ̄はAの正規化である。 /* 正規で既約で支配的な射 SpecB→SpecA を考える。 Bは整閉整域で、環準同型f:A→Bがあって、核は冪零、従って単射である。(6.5.Bの下の注意) 従って S=A-{0}として A→BをK(A)→S^-1Bに延長できる。 x∈A ̄は、x^n+a[n-1]*x^(n-1)+..+x[0]=0 を満たす f(x)は同じ方程式を満たす。 従って K(A)→S^-1B の定義域を A ̄に制限すると像はBに収まり、A ̄→Bを得る。 https://math.stackexchange.com/questions/2553201/universal-property-of-integral-closure 私のイメージは、整閉整域は水面から上に伸びている棒である。 整閉でない整域は水面より高い所にある棒であり、 全商体をとることはその棒を水面の下まで延長する操作である。 正規化は、棒の下端を水面まで伸ばしてくることに相当する。 [注意] Bの逆像⊂K(A)がA ̄に収まっているとは限らない。(A ̄=Z, B=Z[1/2]) つまり棒の一部は水面より下に沈んでいることがある。 */  9.7.B 整スキームの正規化は存在する。(9.1の発想が有効かもしれない) /* 開埋め込みに対して:局所化は正規化とよく振舞うのであった。(5.4.A) 貼り合わせに対して: 2つの場合を考察する。X=X1∪X2, X1∩X2=X12,X1,X2はアフィンとおく。 X1,X2,X12の正規化をX1 ̄,X2 ̄,X12 ̄とおく。X12 ̄は、X1 ̄→X1の、X12の逆像である。 X1 ̄とX2 ̄をX12 ̄にそって貼り合わせれば良い。 */ 9.7.C 正規化は整で全射である /* 整も全射もアフィン局所的に考えれば良い 環の準同型が整で単射ならLying overが成り立つ(スキームは全射)のであった。(7.2.6) */ 9.7.D Xが既約とは限らない被約なスキームの場合に拡張せよ。 /**/ 例 9.7.E 結節型三次曲線 9.7.F 尖点型三次曲線 9.7.G y^2=x^4 (標数≠2) 正規化は1次元では特異点を解消するが、高次元ではそうとは限らない。 z^2=x^2+y^2は正規であるが特異点を持った(5.4.I(b)) 9.7.H SpecZ[15i]の正規化 9.7.I Xを整スキームとする。 ・・・ 9.7.3 定理:整閉包の有限性 Aをネーター整域としてK=K(A),L/Kは有限次拡大、BをLにおけるAの整閉包とする。 (AKLB setup) (a)Aが整閉でL/Kが分離的なら、BはA-加群として有限生成 (b)Aがk-代数として有限生成なら、BはA-加群として有限生成 9.7.M (a)Xを整有限型k-スキームとする。Xの正規化は有限射である (b)・・・ 9.7.N 整有限型k-スキームの正規化は常に双有理である /* [Stacks]では相対的Spec(17章)を使った違うアプローチを使っている。 https://stacks.math.columbia.edu/tag/0BAK AをO_X-代数の準連接層とする。部分層A'⊂Aを A'(U) = {f∈A(U) | すべてのx∈Uに対してf_x∈A_xはOX_x上整である} で定義する。A'_xはOX_xのA_xにおける整閉包であり、 環A'(U)⊂A(U)はO_X(U)のA(U)における整閉包である。 ・・・ */ ============== 10 分離射と固有射、そして(ついに!)多様体 10.1 分離射(と準分離射の見直し) 分離性は古典的多様体におけるハウスドルフ性に相当する根本的な性質である。(10.1.Aを見よ) 古典的多様体では、ハウスドルフ性のおかげで幾何的直観がうまく定理に結びついた。 分離性の定義は最初は奇妙に見えるが、振り返れば、ただ完璧である。 10.1.1 動機 まず、我々がどうしてハウスドルフ性を好むのかを振り返ろう。 位相空間がハウスドルフであるとは、任意の異なる2点x,yに対して、 x,yの開近傍で交わらないものがとれることを意味するのであった。 実直線はハウスドルフだが、原点を二重にした実直線はハウスドルフでない。 /* 2つの実直線を、原点以外の点を同一視したもの。(cf.例4.4.5) 2つの原点の任意の開近傍は交わってしまう */ 古典的多様体の多くの証明はハウスドルフ性に本質的に依存する。 例えば、コンパクトな1次元多様体の分類は非常に簡潔であるが、 ハウスドルフ性を仮定しなければ大変複雑なことになる。 そういうわけで、一度その定義を身につければ、我々は二重原点を正当に議論から除外できる。 そして、古典的多様体に相当する、多様体(variety)という用語を定義できる。 古典的多様体の類推から、アフィンスキームや射影スキームが分離的であることに驚く余地はないだろう。 古典的多様体では、ハウスドルフ性は部分集合に自動的に引き継がれた。 (より正確には、Yが古典的多様体で、Xがその部分集合で、 ハウスドルフ性以外の古典的多様体の条件を満たせば、ハウスドルフ性は付随する。) 同様に、分離的スキームへの局所閉埋め込みはまた分離的である。(10.1.Bと10.1.13(a)を組み合わせよ) おまけに、アフィンスキームへの分離射は、定義域の2つの開アフィンの交わりがまたアフィンになるという性質を持つ(10.1.8)。このおかげで、チェックコホモロジーが準コンパクトスキームで容易にうまくいく(18章)。これは、R^nにおいては、2つの凸集合の交わりがまた凸集合になることの類推と言えるかもしれない。凸集合はコホモロジーを持たないことと同様に、アフィンスキームは準連接コホモロジーの視点からは自明だからこの類推はよく言えている。 ここで重要になってくる常套手段が「対角射」である。射π:X→Yが与えられたときに、対角射X→ X {×_Y} X (2つの恒等射X→Xの積)の振舞いから、(たいてい楽しい図式追跡を使って)しばしば有用な結果が得られる。この常套手段は幾何学の多くの分野に応用されている。(対角射のもう1つの産物には、21章で代数的な微分の定義を与えてくれる。) グロタンディークは、対象の性質ではなく射の性質を考えよ、と我々に教えた。そうすれば、対象がある性質を持つかどうかは、終対象への射がその性質を持つかどうかに帰着する。これは7章の始めでも触れた。この精神で、分離性はスキームの性質ではなく、射の性質である。 10.1.2 定義に進む前に、対角射の性質を観察しよう 10.1.3 命題 X→Yをスキームの射とする。対角射δは、局所閉埋め込みである。 対角射は、しばしばδで表記する。埋め込まれた局所閉部分スキーム(「対角線」と呼ぼう)はΔで表記する。 [証明] Xの像を覆う、X {×_Y} X の開被覆を描写する。Yの開アフィン被覆をViとおいて、逆像の開アフィン被覆をUijとおく。Uij×UijはX×Xのアフィンな開部分スキームである。ΔはUij×Uijで被覆される。 Vi=SpecB, Uij=SpecAとおくと、A @_B A→A が全射であることから、Uij→Uij×Uij閉埋め込みである。 我々が使った開集合族はX×X全体を被覆するとは限らないから、δが閉埋め込みとは限らない。 10.1.4 定義 X→Yが分離射とは、対角射が閉埋め込みであることを言う。A上のスキームXが分離的であるとは、構造射X→SpecAが分離射であることを言う。10.1.3と、像が閉集合な局所閉埋め込みは閉埋め込みであるという事実から、分離射であることはΔが閉集合であることと同値である。この条件は、次に示すように、空間がハウスドルフである条件と似ている。 10.1.A 重要でない練習 位相空間Xがハウスドルフであることは、対角Δが閉集合であることと同値である。 10.1.6 命題 U,VがA-スキームXの開集合のとき自然な同型Δ∩(U ×_A V)=U∩Vがある 10.1.7 k上の多様体とは、k上の被約で分離的で有限型なスキームと定義する。例えば被約で有限型アフィンk-スキームは多様体である。この概念は5.3.7のアフィン多様体の概念や10.1.14の射影多様体の概念の拡張である。 k上の多様体は圏をなす。射は自動的に、有限型で分離射であることを10.1.Lで見る。 多様体Xの部分多様体とは、Xの被約で局所閉部分スキームである。Xの開(閉)部分多様体とは、Xの開(閉)部分スキームである。 もし6.6.4(**)のセクションを読んだなら、群多様体の概念を得る。10.3.9で少し議論する。 ・・・ 10.1.8 命題 X→Aは分離射で、UとVはXの開アフィンとする。U∩Vも開アフィンとなる。 これを証明する前に、ここから得られる自明でない結果を紹介する。X=SpecAのとき、Aの2つの開アフィンの交わりは開アフィンということになる。これは全く自明でない。基本開集合についてはD(f)∩D(g)=D(fg)が成り立ったが、一般の開アフィンを扱う手段はほとんど持っていなかった。 (注意:この性質は分離性を特徴づけるわけではない。Xが二重原点を持つ直線(分離的でない)でも、この性質が成り立つ) [証明] 10.1.6 ・・ 10.1.16 応用 射のグラフ 10.1.17 Zスキームの射π:X→Yに対して、Γ_π:X→X{×_Z} Y (id,π)を射のグラフと呼ぶ。そうするとπは、Γ_πと第二成分の射影に分解できる。 10.1.18 命題 射のグラフは局所閉埋め込みである。Yが分離Z-スキームならばΓは閉埋め込みである。Yが準分離Z-スキームならばΓは準コンパクトである。 [デカルト図式による証明] マジック図式により、1.3.S X → X{×_Z} Y ↓  ↓ Y → Y{×_Z} Y はデカルト図式である。(局所)閉埋め込みは底変換で保たれた。これにより命題が従う。 10.1.19 キャンセル定理 --- 10.2 分離スキームへの有理写像 6.5で有理写像を紹介した時に、都合のよい状況では最大の定義域を考えることができると予告した。今、都合が良い状況を正確にできる:被約から分離への定理10.2.2である。まず、「2つの射が一致する場所」という概念を正確にする。 10.2.A π,π'をZスキームの射:X→Yとする。πとπ'が一致する場所というものを考えたい。特に、それはXの局所閉部分スキームである。 μ:W→Xを考える。πとπ'が「μで一致する」とは、合成πμとπ'μが一致することと定義する。次の意味で普遍性を満たす局所閉埋め込みi:V→Xがあることを示せ: 任意のπとπ'が「μで一致する」ようなμ:W→Xについて、μはiを経由してW→V→Xと分解する。 また、Yが分離的ならばVは閉部分スキームであることを示せ: (ヒント:X→Y×Y←Yのファイバー積を考えよ) /* */ ・・・ 10.2.B k ̄-多様体の射は閉点の行き先で定まる ・・・ 10.2.C より重要でない練習 原点を二重にした直線X(4.4.5)は分離的でない。 π,π':W→Xが一致する場所が閉部分スキームでない例を挙げよ。 (cf.10.1.3, 10.1.D, 12.7.C) このセクションの中心的な結果に進もう 10.2.2 被約から分離への定理 π,π':U→Zを被約なスキームから分離スキームへの射とする。 π,π'の一致する場所がUで稠密な開集合を含むならば、π,π'はU全体で一致する。 [証明] π,π'の一致する場所をVとおく。Vは10.2.AよりUの閉部分スキームである。 ところが既約スキームの、稠密な開集合を含む閉部分スキームは、U全体しかない。 10.2.3 結果1:U,VをXの開集合として、π:U→Yとπ':V→YがU∩Vの稠密な開集合で一致するなら、それらはU∩Vで一致する。 結果2:有理写像は「最大の定義域」を持つ。 例えば有理写像 A^2→P^1の(x,y)→[x:y]という写像の最大の定義域はA^2-{0,0}である(cf.6.5.3)。 定義域の補集合は、不定な点と呼ばれる。・・・ (22.4.Lで、射影スキームへの有理写像では、ブローアップによって不定な点を扱うことができることを見る) 10.2.D 被約から分離への定理10.2.2は、始域の被約性あるいは終域の分離性を捨てると成り立たないことを示せ。 ・・・ 10.2.4 有理写像のグラフ Xが被約でYが既約のとき有理写像π:X-->YのグラフΓ_πを定義する。 (U,π')を代表類の1つとする。Γ_πを、Γ_π'→U×Y→X×Yによって埋め込まれたΓ_π'のX×Yにおける閉包と定義する。1つ目の埋め込みは閉埋め込みで、2つ目の埋め込みは開埋め込みである。 10.2.E 上記のグラフは代表類の取り方に依らないことを示せ。 ヒント:2つの代表類をとって・・・ (指摘:Yの分離性は必要ない) ・・・ --- 10.3 固有射 位相空間の射(すなわち連続写像)は、コンパクト集合の逆像が常にコンパクトであるとき、固有射と呼ばれた。スキームの射の固有性はそれに類似する概念である。 例えば、C上の多様体は、古典的位相でコンパクトのとき、固有であった。 代わりに、射影A-スキームはA上固有である。すなわちこれは、射影スキームという都合が良いスキームで満たされる性質である。 位相空間の射π:X→Yは、閉集合を閉集合に送るとき、閉写像と呼ばれた。スキームの射は、底空間の写像が閉写像のとき、閉射と呼ばれた。(7.3.8で整射は閉射であることを指摘した。) 任意の底変換をしても閉射が保たれるとき、「絶対閉射」と呼ばれる。 具体的にはスキームの射π:X→Yは、任意のZ→Yに対して誘導されるZ {×_Y} X → Z が常に閉射であるとき絶対閉射と呼ばれる。 スキームの固有性の定義を動機づけるのに、可算個の開基を持つ局所コンパクトハウスドルフ空間の連続写像は、絶対閉写像であるとき、またそのときに限り固有であることを指摘しよう。これを練習として証明することは歓迎される。 /* */ =============== 11章 次元 11.1 次元と余次元 誰もが、曲線とは何かを知っている。十分に数学を勉強して混乱するまでは。 -- F.Klein ここまでにスキームについて多くのことを見たが、基本的な用語でまだ定義できていないものがある。次元や滑らかさに関する用語である。これらの概念はより微妙であるが、数学のどこの分野でもこれらの概念は微妙なのである。 この章ではまず次元を扱う。これは幾何的直観と合致するべきである。この概念は、いつも捉えにくい。線形代数のときでさえも次元の定義は、すぐに慣れたかもしれないが、初めてみた時にはよく分からないものであっただろう。古典的多様体の次元も同様に自明でない。例えば2次元多様体と3次元多様体が同型でないことがどう分かるだろうか。その答えはおそらく位相を使うもので、従って次元の用語はしばしば位相と密接に関連する。 複素数で考える時の注意として、我々の次元は代数的なものであり、従って実次元の半分である。例えばA_C^1を複素数の直線としてみるときその次元は1である。 11.1 次元の定義は純粋に位相的である:それは位相空間のみに依存し、構造層に依存しない:位相空間Xの次元dimXを、既約閉集合の上昇列の長さ(1点集合{0}を0番目と数える)の上限として定義する。(次元は無限のこともある。空集合好きな読者は、空集合の次元を-∞と定めるだろう。線形代数においても、次元は部分線形空間の上昇列の長さであった。) 一方で、環の次元を、包含関係にある素イデアルの下降列の長さ(極大イデアルを0番目と数える)の上限として定義する。これらの次元は、クルル次元と呼ばれる。(読者はネーター環の次元は有限と想像するかもしれないがそれは正しくない。11.1.Kを見よ。) 11.1.A 簡単な練習 dim SpecA = dimA である。(ヒント:3.7.E) 冪零元は底空間に影響しないから dim Spec A=dim Spec A/√0 である。 11.1.2 我々は既にk[t]、Z、k、k[x]/(x^2)の素イデアルをすべて知っている。 それらの次元が1,1,0,0 であることをすぐに確認できるだろう。 11.1.3 既約でない空間の次元については注意しなければならない。Z = X∪Y でX,Yが閉集合の時、dimZ = max(dimX,dimY)である。すなわち、次元は1つの局所だけでは決定されない。これはややこしいなので、次元を扱う時はできるだけ既約な場合に話を限る。位相空間が等次元的、純粋次元(次元nで等次元的、純粋にn次元)であるとは、すべての既約成分が同じ次元(n)を持つことを言う。次元1,2で等次元的な位相空間を、それぞれ曲線,曲面と呼ぶ。 11.1.B スキームが次元nを持つことは、それが高々次元nな開アフィンで被覆できて、ある開アフィンがちょうど次元nを持つことと同値である。 ヒント:次を示しておくのが有用だろう: 「開集合U⊂Xに対して、Uの既約閉集合と、Xの既約閉集合でUと交わるものの間には全単射がある」 /* これは今後実際有用である。Uの閉集合Vをとる。Xの既約閉集合WでW∩U=Vになるものが2つ存在するとしてW,W'と名付ける。Wは2つの閉集合(X-U)∩W, W∩W'の和集合になるので既約性よりW=W'である。*/ /* この状況でnより長い既約閉集合の列が存在しないことを確認するのにヒントの結果が使える。*/ この結果から、次元は「局所ごと」に考察できるとは言える。 11.1.C 次元0のネータースキームは有限個の点しか持たない /* 命題3.6.15 */ 11.1.D 整射のファイバー, 7.3.11で予告した: X→Yが整射のとき、すべての空でないファイバーは次元0である。 ヒント:整射は底変換で保たれるから、Y=Spec kの場合を考えれば良い。すなわちk→Aが整拡大ならばdimA=0を示したい。p⊂mを2つの包含関係にある素イデアルと仮定する。A/pは整域である。pが極大であることを示す。0≠x∈Aをとってxの最小多項式をF(x)とおくと定数項は0でない。ここから1/x∈Aが言える。(係数が体であることに注意せよ) 11.1.E 重要な練習 SpecA→SpecBが整拡大B→Aに対応する射ならdimSpecA=dimSpecBである。 ヒント:Going-up(7.2.F) 11.1.F スキームの正規化X~→XについてdimX~=dimXである 11.1.G Xをアフィンk-スキームとして、K/kを代数拡大とする。 (a) Xが純粋にn次元とする。X_K = X×SpecKも純粋にn次元である(24.5.Fで一般化する)。 ヒント:まずX=SpecAで、Aが整域である場合に帰着せよ。X'の既約成分はA'=A @_k Kの極小イデアルpに対応する。a∈ker(A→A'/p)と仮定する。aはA'の極小イデアルの元だから5.5.12より零因子である。これとA'はA-加群として自由であることを利用し、aによるスカラー倍が単射であることを示せ。11.1.E(整拡大なら等次元)が利用できる。 (b) 逆を示せ。 /* (a) 11.1.Bよりアフィンに帰着して良い。 次元は底集合だけで決まるから√0で割って整域にして考えて良い。 ちなみに、Aが整域でもA'は整域とは限らない。 例えば k=R, A=C[x,y]/(y-x^2) のとき A=R[t,x,y]/(y-x^2,t^2+1) A'=C[t,x,y]/(y-x^2,t^2+1)=C[x,y]/(y-x^2)×C[x,y]/(y-x^2) Aの元に対応するA'の元a@1が極小イデアルpに属するならばa=0を言いたい。 5.5.12よりa@1は零因子であることが要求される K/kの生成元をk1,..,kmとおくと A'の元は、{a1@k1 + .. + am@km|a1,..,an∈A}と一意的に表せる構造である。 特にこれが0であることはa1=..=am=0を意味し、 これにa@1との積が0であることはa*a1=..=a*am=0を意味する。 Aが整域であることからa@1は零因子でないことが納得できる。*/ 11.1.H dimZ[x]=2である。 (ヒント:Z[x]の素イデアルは3.2.Qで特定した) 11.1.4 余次元 次元は非交和に対して振る舞いが悪かったので、我々は余次元を扱う時に注意が必要である。例えばYがXの閉集合の時、Yの次元をdimX-dimYで定義すると、例えばXが1点Yと曲線Zの非交和のようなときにdimX-dimY=1となってしまいXのYの近傍での振る舞いと一致しない。 より良い定義を考える。病的な状況を避けるため、我々はYが既約のときだけ定義することにする。位相空間の既約な閉集合Y⊂Xの余次元を、Yの閉包から始まる既約閉集合の上昇列の長さ(Y ̄を0番目と数える。既約集合の閉包は既約であることを思い出そう。3.6.B(b))の上限と定義する。これを、codim[X]Yと表記する。特に、点p∈Xの余次元は{p} ̄の余次元とする。 環Aに対する素イデアルpの余次元は、pから始まる下降列の長さの上限である。これはSpecAでの{p}の余次元と一致する。整域では(0)は余次元0であり、Zでは素イデアル(23)は余次元1である。余次元codim[A]pは局所化の次元dimA_pと一致する(3.2.8)。これはしばしばイデアルの高さとも呼ばれる。・・・ 11.1.I YをXの既約閉集合としてYの生成点をηとする。codim[X]Y = dim OX_η /* 局所的に考えて X=SpecA, Y=SpecA/I とおくと OX_ηの素イデアルはXの素イデアルでIを含むものに対応した */ codim[X]Y=0であることはYがXの既約成分であることに対応する。同様にYの余次元が1であることはYを含む既約成分をY'とするとYとY'にはさまれた既約閉集合が存在しないことを意味する。すべての既約成分が余次元1な閉集合は超曲面と呼ばれる。 11.1.J codim[X]Y + dimY ≦ dimX /* Yを経由する既約閉集合の列の長さがある */ XとYが多様体のときは等号が成立する(11.2.9)が一般には成立しない。 注意:余次元はこれでもやや奇妙に振舞うことがある。例えば平面とそれに交わる直線の合併を考える。全空間は次元2である。平面の外にある直線上の点を考えると次元0、余次元1であり上記の等号が成立しない。 また、平面と直線の交点を考えるとその余次元は2である。これは余次元0(直線)に含まれる余次元2の部分集合であるが、はさまれる余次元1の部分集合が存在しないことに注目せよ。11.3.9ではより奇妙な振舞いを紹介する。 11.1.5, 11.1.6 一意分解整域では余次元1の素イデアルは単項イデアルである ・・・ 11.1.7 楽しいが重要でない反例 ネーター環では素イデアルの上昇列は有限なので、読者はネーター環の次元は有限と思うかもしれない。反例の達人である永田がそうではない例を示している。 11.1.K(**) 無限次元を持つネーター環の例 ・・・ 11.1.8 一方でネーター局所環は有限次元を持つ(11.3.I(a)) これは難しい事実、クルルの標高定理11.3.7を要する。 --- 11.2 次元、超越次元、ネーターの正規化 既約な多様体の次元の別の強力な解釈を与える。超越次元である。証明では他でも有用なネーターの正規化を利用する。読者が超越拡大の理論に親しみがない場合のためにいくつかの視界を開く導入をする。 11.2.A F拡大体Eの元がF上代数的であるとはF上整であることを意味する。E/Fが代数拡大であるとはそれが整拡大であることを意味する。2つの代数拡大の合成は代数拡大である(7.2.C)。E/Fを体の拡大としてF'とF"を2つの中間体とするとき、F'F"がF'上でもF"上でも代数的なときF'〜F"と書く。ここでF'F"はF'とF"を共に含む最小の中間体である。 (a) この〜は同値関係であることを示せ。 E/F超越基底とはF上代数的に独立な元の集合である。 (b) E/Fが2つの超越基底を持ち、片方が個数nならばもう片方も個数nであることを示せ。・・・ こうして超越基底の大きさを超越次元と呼ぶ(無限かもしれない)。記号は tr.deg E/Fで表す。有限生成な拡大は有限な超越次元を持つ。(cf.9.5.16) 11.2.1 定理(次元=超越次元)11.2.7で簡潔に証明する。 Aは体k上有限生成な整域とする。SpecA = tr.deg K(A)/k である。 Xが既約なk-多様体ならば dimX = tr.deg K(X)/k である。 先にこれを認めて、現象を観察する。 11.2.B 次元の視点からのヒルベルトの零点定理 A = k[x1,..xn]/Iとする。Aの極大イデアルの剰余体はkの有限次元拡大である。(ヒント:極大イデアルは次元0であり、k上超越次元0に対応する。) 11.2.C X→Yが既約k-多様体の支配的な射のとき、dimX≧dimYである。 (これはより一般には正しくない。例えば生成点の既約な曲線への包含を考えよ。) /* アフィンで考える。支配的という条件は環の準同型の核が冪零という条件となるのであった。 */ 11.2.D 概念の練習 wz-xy=0,wy-xx=0,xz-yy=0で切り取られるA^4の曲面Sを考える。(3.6.F, 8.2.Aでも登場した。)Sは4次元空間から3つの方程式で切り取られるから曲線だと思うかもしれない・・・。例えばD(w)=Spec k[x,w]_wを考えると良い。 1つ目の方程式は3次元多様体を切り取る。2つ目の方程式は曲面を切り取る。ところがそれは2つの成分を持っていて、3つ目の方程式はそのうち片方を切り取るという状況となっている。 11.2.2 既約多様体の有理写像の次数 11.2.3 ネーターの正規化 我々は定理11.2.1の証明のために次を使う: 11.2.4 ネーターの正規化補題 Aをk上有限生成な整域とする。tr.deg_k K(A) = n ならば、k上代数的に独立なx1,..,xn∈Aがあって、Aがk[x1,..,xn]上有限(従って7.2.2により整)となるものがある。 これは、整有限型アフィンk-スキームXを与えられたときに、全射有限射X→A_k^nを見つけられるということを意味する。前者性はLying over(7.2.5)、特に11.1.Eから従うものである。この解釈は時に幾何的ネーターの正規化と呼ばれる(射影バージョンのネーターの正規化は30.3.7を見よ)。 /*「正規化」という言葉 について https://mathoverflow.net/questions/81420/noether-normalization-vs-normalization-of-varieties */ 11.2.5 永田によるネーターの正規化補題の証明 Aの生成元をy1,..,ymとおく。A=k[y1,..,ym]/pとおける。 mに関する帰納法を使う。y1,..ymが代数的独立の場合はm=nで直ちである。 y1,..,ymが代数的独立でない場合、f(y1,..,ym)=0という関係がある。1=y1-ym^r1,z2=y2-ym^r2,..,z[m-1]=y[m-1]-ym^r[m-1]とおく。0< B, k[y1,..,yn] -> C が単射なら k[x1,..,xm,y1,..,ym] -> B@C も単射 ・地から足を浮かせた視点を考えてみた:次の「旧」型の図式 X×Y→X×A^n → X    ↓   ↓ ↓ A^m×A^n → A^m    ↓   ↓ Y → A^n → k ここで×は全部k上のファイバー積とすると 右下を頂点とする3つの四角はファイバー積図式だから、 左の四角と右上の四角もファイバー積図式であることが従う Y→A^nとX→A^mが全射有限射であることと、全射有限射は底変換で保たれることと、全射有限射は合成で保たれることにより、X×Y→A^m×A^nも全射有限射であることが示せる */ 11.2.8 次の定理は、既約多様体では余次元は次元の差である(11.1.Jの等号が成り立つ)ことを主張する。 11.2.9 定理 Xは純粋次元を持つ局所有限型k-スキーム(例えば多様体)、Yは既約閉部分集合、ηはYの生成点とする。dimY + dimOX_η = dimX が成り立つ。 (11.1.IよりdimOX_η = codim[X]Y であった) この定理を証明することで、定理11.2.12(Going-Down)のような整閉整域の有限次拡大に関する主張の記述が便利になる。 証明の前に、代数の言葉に翻訳する。 11.2.F 環Aは、任意の包含関係にある素イデアルp⊂qについて、pからqに至る素イデアルの極大な上昇列が常に同じ長さを持つとき、カテナリーと呼ぶ。(この用語は本質的にここでしか使わない。)Aが体k上有限生成な環の局所化のとき、Aはカテナリーである。 /* k[x1,..,xn]がカテナリーである所にはすぐに帰着する ・[Stacks]では RがコーエンマコーレーならばR[x1,..xn]もコーエンマコーレー コーエンマコーレーならばカテナリー、という流れであった。 */ 11.2.10 指摘 多くの自然に現れる環はカテナリーである。Z上有限生成な環、完備ネーター局所環、デデキント整域、そしてコーエンマコーレー環(26.2.13)。むしろ、カテナリーでない環の例を与えるのは難しい。 https://stacks.math.columbia.edu/tag/02JE 11.2.11 定理11.2.9の証明 /* 別の資料:http://math.stanford.edu/~conrad/145Page/handouts/codimension.pdf */ 11.2.G 次に帰着せよ Xがアフィン既約k-多様体で、ZはXの既約閉集合で極大なもの(ZとXにはさまれる既約閉集合が存在しない)ならば、dimZ=dimX-1である。 /* Xは純粋次元だから既約な場合に帰着できる。Xの素イデアルの列がすべて同じ長さdimXを持つことを示せば良い。Xの極大な既約閉集合Zをとる。Zを通過する素イデアルの列で、長さdimX-1を持つものがある。すなわち dimZ=dimX-1を示せば、これを繰り返すことに帰着する。*/ dimX=dとおくとネーターの正規化(11.2.4)から環の拡大に対応する有限射π:X→A^dがある。そうすると、定義域をZに制限したときの像π(Z)は、A^dの既約閉部分集合となるのであった(7.3.M,有限射は閉射)。 11.2.H π(Z)が超曲面であることを示すことに帰着せよ。 ヒント:次元と超越次数に関する定理11.2.1より超曲面の次元はd-1である。練習11.1.Eよりdimπ^-1(π(Z)) = dimπ(Z)である。だが注意せよ。Zとπ^-1(π(Z))は等しいとは限らない。 /* 有限射は次元を保つ。dimZ=dimπ(Z)*/ π(Z)が超曲面でないと仮定すると、超曲面Hに包含される。次のGoing-Down定理11.2.12により、Z⊂Z'⊂XとなるXの閉集合Z'が存在し、Zの極大性に矛盾する。 11.2.12 整閉整域の有限拡大におけるGoing-Down定理 単射φ:B→Aは有限拡大(AはB加群として有限)とする。Bは整閉整域で、Aは整域とする。Bの素イデアルの包含列q⊂q'と、q'の上にあるAの素イデアルp'が存在するなら、qの上にあるAの素イデアルpでp⊂p'となるものが存在する ・・・ (24.5.Eでは平坦射におけるGoing-Down定理が登場する) 証明では以下の事実を利用する。これは難しくないが、11.3.Cと同じ技巧を使うため、11.3.Dまで待つことにする。 11.2.13 命題「素イデアルの回避」 p1,..,pnを環Aの素イデアルとして、Iをp1,..,pnのどれにも包含されない別のイデアルとする。このときIは∪piに含まれていない。つまりどのpiにも属さないf∈Iが存在する。 11.2.14 定理11.2.12の証明 ・・・ --- 11.3 余次元1の奇跡:クルルの単項イデアル定理とハルトークスの補題 このセクションでは、余次元1に関する数々の結果を探求する。まず、一般的な状況にあてはまる2つの結果:クルルの単項イデアル定理11.3.3とハルトークスの補題11.3.11:を紹介し、それから関数とそれが消える余次元1の点を結びつける。この2つの結果はとても有用であることが分かる。例えば、クルルの単項イデアル定理は、余次元を計算するのに役に立ち、余次元が奇妙に振舞うことを示す。ハルトークスの補題は一意分解整域であることの有用な特徴づけを与える(11.3.5)。このセクションの結果は、(局所)ネーターという仮定を要する。証明は技術的に難しく、結果の使い方にはそれほど視野を与えない。なので、証明に親しむよりも、どのように結果を使うかを理解するのが重要である。 11.3.1 クルルの単項イデアル定理 ベクトル空間では、単一の線形方程式は常に余次元0または1の部分空間を切り出した(方程式が0のときに余次元0となる)。クルルによる単項イデアル定理は、この線形代数の事実を一般化したものである。 11.3.2 クルルの単項イデアル定理(幾何的バージョン) Xを局所ネータースキームとしてfを関数とする。V(f)の既約成分は余次元0または1である。 これは次の代数的な主張の結果である。多様体において(最も適用される場合である)、これの証明(11.3.H)をするための知識は読者には十分あるはずだ。一般の場合の証明は技術的で、それがめちゃくちゃ長いわけではないことを示すためだけに11.5に書いた。 11.3.3 クルルの単項イデアル定理(代数的バージョン) Aはネーター環でf∈Aとする。fが属する素イデアルで極小なものは、余次元が高々1である。さらに、fが零因子でなければ、そのような素イデアルは余次元がちょうど1である。 例えば、局所単項閉部分スキームは余次元0か1であり、効果的カルチエ因子は純粋余次元1である。読者がきっと直接証明できるであろう別の例がある: 11.3.A 体k上の既約な(n+1)変数斉次多項式はP^nの次元(n-1)の整スキームを表すことを示せ。 /* */ 11.3.B (A,m)をネーター局所環としてf∈mとする。dim A/(f)≧dimA - 1である。 /* A/m=k */ 11.3.C 重要な練習(繰り返し使われる) (a) 「射影空間では超曲面は任意の次元1以上のものと交わる」 XをP_k^nの次元が1以上の閉集合、Hを空でないP_k^nの超曲面とする。HとXは交わることを示せ。(ヒント:Hのアフィン錘を考える。X上の錘に11.3.3を利用せよ。) (b) X→P^nは次元rの閉集合とする。余次元rの線形空間はXと交わる。 ヒント:(a)の議論を修正せよ。11.3.Fはこれを一般化する。 (c) r+1個の空でない超曲面の交わりでXと交わらないものが存在する (d) kを無限体とする。r個の超平面の交わりで、Xと有限個の点で交わるものが存在する。 /* 私には難しかった・・・ https://math.stackexchange.com/questions/1105333/trouble-with-vakils-foag-exercise-11-3-c */ 11.3.D 「素イデアル回避」 命題11.2.13を証明せよ。ヒント:nに関する帰納法を利用せよ。文献を参照するな -- より長い議論を見ることになる。 11.3.E A^nの余次元の上限 kを体とする。A^dのX,Yを等次元的閉部分多様体(特異かもしれない)として余次元をそれぞれm,nとする。X∩Yのすべての成分は余次元が高々m+nであることを次のように示せ: ・対角Δ⊂A^d×A^dが余次元dの正則埋め込みであることを示せ ・・・ --- 11.4 ============== 12章 正則と滑らかさ 12.1 ザリスキ接空間 まず、スキームの点における接空間の定義から始める。それは「滑らかな」点での読者が知っている接空間と似たように振る舞うが、そうでない点でも定義される。「滑らかな」点での幾何的な直観は定義の道しるべになり、定義が今度は幾何的な直観をより正確にする。定義は短く、そして意外である。主な困難は、これが実際に接空間と呼ぶのにふさわしいと確信することである。これを説明するのは厄介である。定義が直観と一致することを示したいのだが、我々の直観は我々が認識しているほど良くないからである。なので読者はまずは定義を受け入れ、後でこれが理にかなっていることを確信するのが良い。 局所環(A,m)の「ザリスキ余接空間」は、m/m^2と定義される。これは、剰余体A/m上のベクトル空間である。これの双対ベクトル空間が、「ザリスキ接空間」である。Xがスキームのとき、p∈Xにおけるザリスキ余接空間TX_p^は、局所環OX_pとして定義される。そしてその双対ベクトル空間がザリスキ接空間TX_pである。ザリスキ余接空間の元は、余接ベクトルあるいは微分(differential)と呼ばれる。ザリスキ接空間の元は、接ベクトルと呼ばれる。 余接空間は定義がより短い点で、代数的には接空間より自然である。また、余接空間は空間上の関数の言葉でより自然に定義され、我々は実際しばしばスキームをその上の関数の言葉で考える。これについては後で登場する。 この定義が理にかなっていることを示す2つの議論がある。 [1] 微分幾何では、点での接空間を、その点における微分(derivation)のベクトル空間として定義することがある。古典的多様体の点pでの微分とは、pの近くの関数f(つまりO_pの元)を入力し、f'(p)∈Rを出力するR-線形写像であり、ライプニッツ則 (fg)'=f'g+g'f を満たすものである。(我々は後で微分を21.2.17で一般的に定義する。) 微分を与えることは、O_pの極大イデアルmからRへの写像を与えることと同等である。(m→Rを定めれば、O_p→m→R の合成でO_p→Rが定まる。ここでO_p→mは、f → f-f(p)という写像) この状況で、m^2は0に移る。なぜならf,g∈m ならば f(p)=g(p)=0 ということであり (fg)'=f'g+fg'=0 である。そういうわけで、微分は、m/m^2をRに送る写像を誘導する。m/m^2をRに送る写像とはすなわち m/m^2の双対ベクトル空間である。 /* 例えば通常の2次元平面の原点での微分とは、f(x,y) を A*∂f/∂x+B*∂f/∂y|(x,y)=0 に送るようなものであり、∂f/∂x, ∂f/∂y を基底とする2次元ベクトル空間である。この写像を与えることは、A,Bを与えることと同等で、それは f=x と f=y をそれぞれ何に送るかで決定される。言い換えると、極大イデアル(x,y)の行き先で決定される。 */ 12.1.A 逆に線形写像φ:m/m^2→Rが与えられた時にこれが微分を与えることを確認せよ。 (ライプニッツ則が成り立つことを確認せよ) /* 大雑把にm1,m2∈mによって f=m1+r1, g=m2+r2 とおくと fg=m1m2 + (r2m1+r1m2) + r1r2 で f'=φ(m1), g'=φ(m2), (fg)'=φ(r2m1+r1m2) ということだと思う */ [2](私はこちらをより好む) A^nにおける関数を接空間に「制限」したものは、線形な関数になるべきである。例えばx^2+xy+x+yを原点の近くに制限したものはどのような関数か?読者はx+yと答えられるだろう。 このようにして、線形な項を取り出すことができる。そういうわけで、m/m^2は接空間上の線形な関数であり、すなわち余接空間である。特に、ある点で消える関数(mに属する元)をその点において接空間に居る関数として描写できるべきである。 12.1.2 古い様式での例 ザリスキ接空間の実際の計算は読者が過去に多変数解析で習った通りである。A^3においてx+y+z^2+xyz=0とx-2y+z+x^2y^2z^3=0で切り取られる曲線があるとする。(読者はクルルの単項イデアル定理11.3.3を使ってこれが曲線であることを確認しても良いがそうすることは重要ではない)原点での接線は何であるか?(そもそもそこで滑らかであるか?) 答え:原点の近くで1つ目の曲面はx+y=0の挙動で、2つ目の曲面はx-2y+z=0の挙動をする。 従って曲線はこれらの曲面で切り取られる直線を接線に持ち、(古典的な意味で)滑らかである。 ・・・ 我々が使った事実をもっと明らかにしよう。Aを環、mを極大イデアル、f∈mを点[m]∈SpecAで消える関数とする。SpecA/(f)の[m]におけるザリスキ接空間は、SpecAのザリスキ接空間の中で、f (mod m^2)による方程式で切り取られるものである。次の練習は読者にこのことについて良く考えることを強いるだろう。 12.1.B 重要な練習(接空間におけるクルルの単項イデアル定理―しかしそれよりずっと簡単) 上記のことを示せ。 (我々はfで割ることとmで局所化することの順番を入れ替えることができる[4.3.Gの後の指摘]) 従って、[m]におけるSpecA/(f)のザリスキ接空間の次元は、[m]におけるSpecAのザリスキ接空間の次元に等しいか、あるいは1だけ小さい。(次元が無限の場合は然るべき意味だと解釈する) /* B = A/(f) とおく。環Bで m mod fが生成するイデアルを区別のためnと書くことにする。 環 N = n/n^2 と M = m/m^2 を比べたい。 環 M = m/m^2 で f mod m^2 が生成するイデアルを f'とおく。 N = M/f' が最初の主張である。 (どちらもmの元を mod f, mod m^2 の違いで割ったもの) f mod m^2 が 0 の場合は次元が等しい。 そうでない場合は、Mはベクトル空間だから、f mod m^2を含む基底をとれて、 残りの基底が、Nの基底となる。 */ 別の例を見よう。x+y+z^2=0 と x+y+x^2+y^4+z^5=0 は原点を通る曲線を切り取る。この曲線の原点での接線を考えよう。1次の項を取り出すと x+y=0 と x+y=0 を得る。これでは直線ではなく平面だ。これは、この曲線が原点で滑らかでないからである。・・・。我々はしばらく後(定理12.2.1)でこの現象を、点が特異点であることの定義に使う。 12.1.C 閉部分スキームY,Z⊂Xを考える。どちらもp∈Xを含むとする。 (a) TZ_p は TX_p の部分ベクトル空間である (b) T[Y∩Z]_p = TY_p ∩ TZ_p である。(左辺の)∩はスキーム論的な意味である。 (c) T[Y∪Z]_p は TY_pとTZ_pが貼る空間を包含する。∪はスキーム論的な意味である。 (d) T[Y∪Z]_p は TY_pとTZ_pが貼る空間より真に大きくなり得る(ヒント:4.2.1) /* 点pの近くで考える。X=SpecA, Y=SpecA/a, Z=SpecA/b とおいて良い。[a⊂p, b⊂p] (a) κ=K(A/p)とおく。Z 環Zのp/p^2→κから環Xのp/p^2→κへの単射が自然に定義できる: (b) Y∩Z = Spec A/(a+b) である。[このa+bはa,bで生成されるAのイデアル] 右辺は Hom(p/p^2,κ) の部分空間としての共通部分である: p1-p2∈a∩p mod p^2 のとき p1,p2をκの同じ元に送る(aを0に送る) かつ p1-p2∈b∩p mod p^2 のとき p1,p2をκの同じ元に送る(bを0に送る) という条件となる。 これは p1-p2∈(a+b)∩p mod p^2 のとき p1,p2をκの同じ元に送る((a+b)∩p mod p^2を0に送る)ことと同値である。 (c) Y∪Z = Spec A/(a∩b) である。 従って左辺はHom(p/p^2,κ)のうち、a∩b∩p mod p^2を0に送るものであるから、 a∩p mod p^2を0に送るものやb∩p mod p^2を0に送るものを含む。 (d) A=k[x,y],p=(x,y),a=(y-x^2),b=(y),a∩b=(y(y-x^2))とすると a∩p mod p^2=(y) mod p^2, b∩p=(y) mod p^2, a∩b∩p mod p^2=0 TY_p,TZ_pは ax+by mod p^2 を c=ua+vbに送るもののうちu=0なもの TX_pは何でも良いのでu≠aなものも含む。 */ ここに喜ばしい応用がある。 12.1.3 問題。環A=k[x,y,z]/(xy-z^2)で(x,z)は単項イデアルではないことを示せ。 dim A=2であり(なぜか?)、A/(x,z)はk[y]と同型で次元1だから、このイデアルは余次元1である(定理11.2.9)。我々の持つ幾何的な絵ではSpecAは円錐であり(x,y)は母線である。そうすると円錐の頂点に注目することが提案されよう。 12.1.4 解答。m=(x,y,z)とおく。SpecAは原点で3次元のザリスキ接空間を持つ。しかしSpecA/(x,z)は原点で1次元のザリスキ接空間である。SpecA/(f)の形で書ける環の原点でのザリスキ接空間の次元は12.1.Bより2以上だから、(x,y)は(f)の形で書けない。 12.1.5 指摘。別の視点もある。通常の意味での次数付き環として考える。(x,z)は同次イデアルで、x,zは基礎環上で線形独立である。このことから単項イデアルでないことが言える。(読者はこれをじっくり考えて楽しんでも良い。) /* Z[i]の(3+4i,5)は単項イデアルとなることを観察する。iは1次として次数付けできない(i^2=-1だから)。なので多項式環ではxやzは掛け算が常に"新しい"所に属するというところが鍵のようである。 */ 接空間の視点の利点は、k[x,y,z]/(xy-z^2-z^3)や局所化(k[x,y,z]/(xy-z^2))_(x,y,z)のような次数によるフィルタが使えない場合も使えることである。しかし"随伴する次数付き環"(22.3の議論で暗示的に現れる)や完備化(29章で記述される)を使うとこれらの場合にも次数付き環の方法を使えるようにできる。 12.1.D (x,z)⊂A=k[w,x,y,z]/(wz-xy)は単項イデアルでない。(5.4.Dの後で予告した) /*Aの原点でのザリスキ接空間は4次元、A/(x,z)の原点でのザリスキ接空間yとwで生成される2次元。*/ 12.1.E A=k[w,x,y,z]/(wz-xy)は分解整域ではない 12.1.F 高次の場合(この練習は楽しいが今後使われない) (a) (xy,yz,zx)⊂k[x,y,z]は2つ以下の生成元では表せない。 ヒントmod m=(x,y,z)では情報は得られない。mの高次で割った剰余を考えよ。 (b) Spec A/(xy,yz,zx)→SpecA は正則な埋め込みではない(8.4で予告した) /* (a) m^2をm^3で割った剰余を考えるかな・・・ */ 12.1.6 射と接空間 π:X→Y, π(p)=qとする。微分可能多様体の圏では、多様体の射から接空間の射が誘導された。ザリスキ接空間でもこれが成り立つ:茎準同型OY_q→OX_pが誘導される。p,qでの局所環の極大イデアルをそれぞれm,nとおくとn,n^2はそれぞれm,m^2に移る。こうして余接空間の写像n/n^2→m/m^2を得る。さらに剰余体がともにκであれば、これはκ-ベクトル空間の写像となるので、接空間の写像Hom(m/m^2,κ)→Hom(n/n^2,κ)も誘導される。余接空間の写像はいつでも存在するが、接空間の写像は同じ剰余体を持つ要求が必要である。 ザリスキ接空間になじむためのいくつかの練習がある。読者が微分可能多様体の知識をいくらか持っているなら、我々の定義が接空間の概念を適切に捉えていることをさらに確信できるだろう。 12.1.G 重要な練習 Xを有限型k-スキームとする。局所的にSpec k[x1,..,xn]/(f1,..fr)の形をしている。k-値点(剰余体がkとなる閉点)のザリスキ接空間は、次のヤコビ行列による写像k^r→k^nの余核で与えられる: J = (∂f1/∂x1(p).. ∂fr/∂x1(p) ; ... ; ∂f1/∂xn(p) ..∂fr/∂xn(p) ) ヒント:"平行移動"によりpが原点の場合を考えると、1次の項を考えることになる /* 幾何的なイメージ: 例えば X=k[x,y,z]/(x+y,y+2z) の場合、Xは図形として2つの平面の交わりのなす直線である。 余接空間の元は、1次式{ax+by+cz|a,b,c∈k}を、Xで定数なものを同一視した商群である。(例えば x+y+zとzは同じ余接空間の元) 接空間の元(p,q,r)は、そのax+by+czをスカラーap+bq+crに送るもので、上記の同一視に対してwell-definedなものである。つまり x+y や y+2z を 0におくるようなものである。要するに(1,1,0)にも(0,1,2)にも垂直なベクトルである。 解釈: ax+by+czに相当するものを縦ベクトル(a;b;c)と解釈する。接空間は、これに左から掛かることでスカラーを得る横ベクトルと解釈される。 k^rの行列Jによるk^nへの像は、r個の列ベクトルで貼られる。r個の列ベクトルはf1,..frの1次の項を取り出したものに相当する。求める接空間はこの像に居る縦ベクトルに左から掛けた時に0となるような横ベクトルだから、k^nをこの像で割った商空間と同一視される。*/ 12.1.7 指摘 この結果は剰余体がX上分離的な場合に一般化される(21.3.10)。そこまで待つのは証明が複雑だからではない。より初等的な証明が可能となるからである。 12.1.8 警告 ヤコビ行列は縦横を反対に定義されることが多い。ここでは便宜的に習慣と逆にしている。 12.1.H ヤコビ行列の階数は表現の仕方に依存しない Aを有限生成k-代数, A= k[x1,..,xn]/I, I=(f1,..fr)とする。p∈SpecAとする。 (a) g∈Iとする。ヤコビ行列にgの偏微分係数のなす列ベクトルを加えても(pでの)階数は変わらない。 従ってヤコビ行列の階数はIの生成元の選び方に依存しない (b) 変数の添加。q∈k[x1,..xn]とする。h=y-q∈k[y,x1,..,xn]とおく。(f1,..,fr,h)のなすヤコビ行列は元のヤコビ行列と同じ階数を持つ。従ってヤコビ行列の階数はAの生成元の選び方に依存しない。 /* (a) gはf1,..,frの線形結合で書けるから新しい列ベクトルは他の列ベクトルの線形結合で書ける。 (b) f1..frはyでの偏微分が0だから、hはf1,..frから線形独立である。*/ 12.1.I Xをk-スキームとする。Mor_k(Spec k[ε]/(ε^2),X)と、剰余体がkであるXの点と接ベクトルの組という集合の間の全単射を描写せよ。(これは例えば変形理論で重要になる。) /* 前者は、Spec k[ε]/(ε^2)の1点をXのどの点に送るかと、局所環(A,m)からk[ε]/(ε^2)への環準同型でmをεに送るものという情報を持つが、これは接ベクトルが持つ情報と同じである。*/ 12.1.J Z[2i]=Z[x]/(x^2+4)の点(2,2i)におけるザリスキ接空間の次元はいくつか。Z[√-2]=Z[x]/(x^2+2)の点(2,x)におけるザリスキ接空間の次元はいくつか。 (読者が幾何的なものを好むならZをCに、2をyに置き換えて、C[x,y]/(x^2+y^2)やC[x,y]/(x^2+y)の原点での次元を考えよ。) /* 前者は2次元、後者は1次元だと思う */ --- 12.2 体上の正則性と滑らかさ 正則性の定義の鍵となる考えは次の結果である:ザリスキ接空間は最低でも局所環の次元を持つ: 12.2.1 定理 (A,m,k)をネーター局所環とする。dimA≦dim_k m/m^2 が成り立つ。 [12.2.2 証明] Aがネーターなのでmは有限生成であり、従ってm/m^2も有限生成である。 右辺をnとおく。m/m^2の生成元をmの元f1,..,fnに持ち上げると中山の補題によりこれらはmを生成する。次にクルルの標高定理11.3.7の結果である11.3.IによりdimA≦nが言える。 12.2.3 定義 12.2.1における等号が成り立つ時、Aは正則局所環であると言う。特にそれは定義によりネーター環である。(ネーター環Aがすべての素イデアルpに対してでA_pが正則局所環であるときにAは正則環と言うが、我々はこの用語を基本的に使わない。安心できることに、正則局所環は正則環である(12.8.2)。)局所ネータースキームXがp∈Xにおいて正則であるとは、局所環OX_pが正則であることを言う。(非特異と言う用語もよく使われるが、我々は"正則"で統一する。)OX_pが正則でないとき、点pは特異点と言う。(非正則という用語も使われるが、我々は"特異"で統一する。これは習慣による。)スキームは、すべての点で正則のときに正則と言う。どこかで特異であるときに特異と言う。 12.2.A 次元0のネーター局所環が正則であることは、体であることと同値である。 読者はこれがスキームの"滑らかさ"を記述する適切な方法であることを徐々に確信していくだろう。驚くべきことにクルルは正則局所環という概念を純粋に代数的な理由で導入した。しばらくしてからザリスキが1947年に幾何的に根本的な概念であることを認識した。 12.2.B Xを純粋n次元な有限型k-スキームとしてpを正則な閉点とする。従ってOX_pは次元nの正則局所環である。f∈OX_pとする。OX_p/(f)はf∈m-m^2の時に限り次元(n-1)の正則局所環である。 (ヒント:12.1.B) /* そこに書いた考察の通りである */ 12.2.C 正則性の切り取りによる基準 Xを有限型k-スキームとしてDをXの効果的カルチエ因子(8.4.1)とする。pがDにおいて正則であるならばpはXにおいて正則である。 (ヒント:12.1.B) /* イデアル層が局所的に零因子でない関数によって生成されるという定義だった。局所で考える。 */ --- 12.3 例 12.4 ベルチニの定理 --- 12.5 もう1つの(余)次元1の奇跡:離散付値環 次元1の場合は重要であった。素イデアルが次元1で分離されることを理解すれば、任意の次元での事実を証明するのに帰納法が使えるからである。これが、クルルの単項イデアル定理(11.3.3) --- 12.6 滑らか(でエタール)な射(最初の定義) 12.7(*) 分離性と固有性の付値による基準 12.8(*) 正則局所環に関するより高度な事実 12.9(*) フィルター付き環と加群、Artin-Reesの補題 ============== 13章 準連接層と連接層 準連接層と連接層はベクトル束の概念の一般化である。まず、ベクトル束について議論し、それを局所自由層として解釈するところから始める。 略式には、幾何的な空間(例えば古典的多様体)X上のベクトル束VはXの点で連続的にパラメータ付けされたベクトル空間の族である。言い換えると、X上の各点pに対してベクトル空間があって、pを動かした時のこれらのベクトル空間は連続的に貼り合わさってVをなす。心にとめておく自明でない例は、古典的多様体の接ベクトル束と、円上のメビウス束(直線束として解釈)である。我々は13.1でこれを精密にする。読者はこの概念が初めてでも、心配しなくて良い。この概念は動機のためだけに使うのである。 環つき空間X上の自由層とは、ある添え字集合Iによる直和O_X^+Iに同型なO_X-加群である。X上の局所自由層とは、局所的に自由層と同型なO_X-加群である。これが、ベクトル束に対応する。準連接層は、局所自由層を含み、O_X-加群の圏より小さい便利なアーベル圏をなす。加群が自由加群の一般化であるような様子で、準連接層は局所自由層の一般化である。連接層は、おおまかにいうと、準連接層の有限次元バージョンで、有限次元の局所自由層を含む都合のよいアーベル圏をなす。・・・ 13.1 ベクトル束と局所自由層 古典的多様体でのベクトル束をより正確に思い出そう。数論的な読者も避ける必要はない。例えば代数体の整数環の分数イデアルは「滑らかな曲線上の直線束」として解釈できる(13.1.M)。 古典的多様体M上の階数nのベクトル束は、写像π:V→Mであって、Mの各点xに対してπ^-1(x)がn次元の実ベクトル空間であって、近傍Uによって位相同型φ:U×R^n→π^-1(U)を構成できるようなものである。この位相同型はUでの「自明化」と呼ばれる。 階数1のベクトル束を直線束と呼ぶ。・・・ 13.1.1 変換関数 U1,U2での自明化を与えられた時、U1∩U2においてそれらの自明化は、U1∩U2の関数を成分とする線形変換T12∈GL_nによる変換で結びついている。族UiがMを被覆し、各Uiでの自明化が与えられている時、変換関数Tijたちは次のコサイクル条件を満たす: Tjk・Tij = Tik (これはTij=Tji^-1を含意する。)Tijのデータを自明化の変換関数と呼ぶ。 逆に、被覆Uiと変換関数Tijが与えられれば、それらを貼り合わせることによってベクトル束を(同型を除いて)決定できる。 /* 一度実行しておこう。 ・球面の接ベクトル束 例えば単位球面上の開集合U1:x>0と開集合U2:y>0の間の貼り合わせを考える。接ベクトル束は階数2の接ベクトル束Vで、集合{((a;b;c),(d;e;f))|(a;b;c)は単に球面上の点、(d;e;f)は点(a;b;c)で球面に接するベクトル⇔ad+be+cf=0}と書ける。[セミコロンはプレーンテキストで縦ベクトルを表す訳者の習慣である。] U1での自明化φ1:U1×R^2→π^-1(U1)を考える:((a;b;c),(Y;Z))→{(a;b;c),(d;e;f)}。e=Y, f=Z と指定すると、接ベクトルの条件から d=(be+cf)/(-a) と一意的に決めることができる(U1ではa≠0)。これでφ1を定義できた。 次にU2での自明化φ2:U2×R^2→π^-1(U2)を考える:((a;b;c),(X';Z'))→{(a;b;c),(d;e;f)}。同様にd=X',f=Z'と指定して、e=(ad+cf)/(-b)で定めることでφ2を定義できる。 このときの変換写像T12を考察すると、(Y;Z)を(X';Z')=((bY+cZ)/(-a);Z) に移す写像であることが分かる(Y,Zについて1次式である)。逆変換は (Y;Z) = ((aX'+cZ')/(-b);Z')である。 */ 13.1.2 切断の層 階数nのベクトル束V→Mを固定する。Vの切断Fの層(2.2.G)は、OM-加群をなす -- Uの開集合上の切断に対して、U上の関数によるスカラー倍は別の切断を与える。 さらに、U上の自明化が与えられた時、U上の切断はU上のn個の関数の組と同一視できる。 こうして、自明化が与えられれば、F|UiとO_Ui^+nの同型を得る。このようなFを階数nの局所自由層と言う。FがO^+nと同型なら、Fは自由層と言う。 13.1.3 切断の層に対する変換関数 M上のベクトル束と、被覆Ui上の自明化が与えられたとする。その切断が与えられたとする。そうすると、各Uiに対して、n個の関数の組si~が対応する。 /* ベクトル束の切断は、つまりベクトル場である。 先の球面の接ベクトル束の記号を続ける。U1での切断として例えばx軸周りに回転するベクトル場を考える。 切断とは、(a;b;c)から((a;b;c),(d;e;f))∈Vを与えることである。 U1上では、(a;b;c)から((a;b;c),(Y;Z))を与えても良い。Y=-c, Z=b と定めるのが1つの切断を与えるわけである。 (-c;b)というのがここでいう2個の関数の組s1~ということになる。 */ 13.1.A 交わりUi∩Ujでのベクトル値関数si~とsj~は切断によらない(自明化の変数変換と同じ)変数変換Tij(si)=sjで結びつけられる。 /* 同様に(a;b;c)から((a;b;c),(X';Z'))を与えることを考える。(X';Z')=((bY+cZ)/(-a);Z)の関係があったから、U1での切断の定め方と合わせるには(X';Z')=(b(-c)+c(b)/(-a);b) = (0;b) と定めなければいけない。すなわちs2~=(0;b)である。これは「x軸周りに回転するベクトル場」として合っている。s1~とs2~の関係は計算から分かるように自明化の変換と同じ変換の関係である。 一般な場合に対する状況を言葉で書いてみた。 Ui→Ui×R^n=π^-1(Ui)←π^-1(Ui∩Uj) Uj→Uj×R^n=π^-1(Uj)←π^-1(Ui∩Uj) Ui×R^n=π^-1(Ui)は自明化写像。両者のπ^-1(Ui∩Uj)は一致する。Ui×R^nとUj×R^nを変換関数Tijでつなぐと可換図式になるわけである。 切断とはUiからπ^-1(Ui)への写像であり、自明化写像を使ってUiからUi×R^nへの写像としたものが「n個の関数の組」si~に相当する。si~とsj~はπ^-1(Ui∩Uj)に移すと一致する必要があるから、同じ変換関数Tijで結び付けられる。 */ 階数nの局所自由層Fを与えられたとする。つまり近傍Uiでの自明化F|Ui→O_Ui^+n(同型)と、コサイクル条件を満たす変換関数Tij∈GL_n(O(Ui∩Uj))が与えられている。このように、階数nの局所自由層とは、階数nのベクトル束と"同等"である。この視点変えは、以前に環つき空間を考える時の視点変えと同様である:空間を理解することは空間上の関数を理解するのと同じであり、ベクトル束(定義からはM上の"空間"であった)を理解することは、その切断(関数)を理解するのと同じである。 13.1.4 定義 階数1の局所自由層は可逆層と呼ばれる。(重要でない補足:"可逆層"とは本質的には直線束であるものを指し示す憎い言葉である。動機としてはXが局所環つき空間でF,GがO_X加群でF@G→O_X(同型)となるときF,Gを可逆層と呼ぶ。すなわちO_X加群の集合にテンソル積で積構造を入れたモノイドにおいて、可逆層は可逆元である。我々はこの事実は今後一度も使わない。人々はしばしば"可逆層"と呼ぶ代わりに非正式に"直線束"と呼ぶ。"直線層"という用語はより合理的であるが、普及していない。) 13.1.5 スキーム上の局所自由層 スキームX上の階数nの局所自由層とは、O_X加群で、階数nの局所自由層である。正確には、開被覆U_iがあって、それぞれのU_i上でのFの切断がO_Ui^+nとなっている。この開被覆は、変換関数を定める:つまりコサイクル条件を満たすTij∈GL_n(O(Ui∩Uj))の集まりである。 逆に、開被覆と変換関数の組で、コサイクル条件Tjk・Tij = Tikを満たすものを定めれば局所自由層が定まる。つまり開集合Uに対して切断が定まる。略式には、それはそれぞれのU∩Ui上で自由層の切断であり、交わりで一致するようなものである。正式には、S^i = (s^i_1;..;s^i_n)∈Γ(U∩Ui, OX)^nの集まりで、U∩Ui∩UjにおいてTij S^i = S^jが成り立つものである。 読者はこれらをベクトル束として考えると良いが、ベクトル束と"同等"であるが"同じ"ではないことを心に留めておこう。(我々は後で(17.1.4)スキームX上のベクトル束:V→Xの"全空間"というものを相対的SPECを使って定義する。)しかし局所自由層の視点はより有用である:例えば、ベクトル束より定義がずっと短い。 13.1.6 有用な事実を与える重要な一連の練習 以下の練習は有用で、重要で、そして意外と自明でない。一般の環つき空間に成り立つので読者はスキームの性質を使わないで証明するべきである。また、それらはすべてXについて局所的な性質なので読者は自由層の場合に帰着できる。 13.1.B F,Gをそれぞれ階数m,nの局所自由層とする。Hom_OX(F,G)は階数mnの局所自由層である。 /* Hom_OX(F,G)はn行m列の行列のなす加群みたいなものである。 その表記で開集合を移る時の変換関数について考えておく: xをyに移す Hom_OX(F,G)の元:A.x=y を満たす行列Aは、 x'=Tx12.x を y'=Ty12.yに移す:A'.x'=y' を満たす:A'.Tx12.x=Ty12.y を満たす そういうわけで A' = Ty12.A.Tx12^-1 という関係である。 */ 13.1.C Gを階数nの局所自由層とする。このとき G^=Hom(G,O_X)も階数nの局所自由層でありGの双対層と呼ぶ。 Gの変換関数を使ってG^の変換関数を描写せよ。(特にn=1の場合は逆写像となる) GとG^^が同型であることを示せ。 (注意:GとG^が同型であることを示そうとしないこと。 例えば後で出てくるP^n上の局所自由層O(1)はその双対と同型でない。) /* 上記の議論より、開集合を移る時に 縦ベクトル xがx'=Txに移るなら y∈G^は横ベクトルみたいなもので、yはy'x' = yxとなるようなy'に移る。つまり yは y'T^-1に移る。 さらにz∈G^^について zは y'z'=yz となるように変換されると考えることができるのでzはxと同じ変換関数に従う。 局所自由層は変数変換によって決定されることからG^^とGは同型であると言えると思う。 */ 13.1.D O_X加群としてのテンソル積F@G [2.6.J]も局所自由層である。 特にFが可逆層ならば F@F^=O_Xである。 /* 局所で階数mの自由層と階数nの自由層のテンソル積は階数mnの自由層である。 */ 13.1.E テンソル積は一般には右完全性だけを保存するのであった。一方で、局所自由層によるテンソル積は完全である。 (ヒント:完全性は茎で確認することができる。また、テンソル積は茎をとることと可換であった。2.6.J(b)) /* 局所で自由加群とのテンソル積であり、そのような局所は開集合の基をなすというのが1つの視点だと思う(2.5.1)。 */ --- 13.2 準連接層 スキームが環を貼り合わせて定義されたように、準連接層は環上の加群を貼り合わせて定義される。 /* O_X加群が準連接層であることは、環つき空間がスキームであるようなものだと思う */ はるか以前(4.1.D)A加群Mに対してO-加群M~を定義した。D(f)での切断はM_fであった。 13.2.1 定理 Xをスキーム、FをO_X-加群とする。SpecA⊂Xに対して、 「FのSpecAへの制限が、A-加群MによるM~に同型である」という性質をPとおく。 この性質は5.3.2のアフィン交通補題の2つの条件を満たす。 (この証明は後回しにする) 13.2.2 定義 すべてのSpecA⊂Xに対して上記の性質を満たす時、Fは準連接層であるという。 13.2.A O_X-加群は準連接層とは限らない (a) X=Spec k[t]とする。原点(t)を台とする摩天楼層k(t)は準連接層ではない  [通常のk[t]加群としての構造を入れる] (b) 一方で、生成点を台とする摩天楼層は準連接層である。 /* (a) 原点を含む開集合での切断がk(t)、他の開集合での切断は0という層である (b) すべての開集合は生成点を含む。すべての開集合での切断はk(t)である。 *一方で、構造層をイデアル層で割った層は準連接層になる。 具体的には集合としては原点(t)を台とする摩天楼層k=k[t]/(t)であるが、 k[t]加群としてのt倍写像によって消える構造をしていることがポイントである。 つまり(t)は零化イデアルとなっているので(t)による局所化は消えてしまう。 これは原点を除いた開集合での切断が0という層であることと合致している。 つまりこの層Fの切断について F(D(t)) = F(X)_t の関係が成り立っている ちなみにこの層は、一点からの閉埋め込みによる構造層の押し出しでもある。 (という描写からも準連接層でなければならない) */ 13.2.B 準連接層は局所自由とは限らない (上記13.2.A(b)) 13.2.C A_k上の階数nのベクトル束は自明である。 (体上有限生成加群の構造) 13.2.3 定理13.2.1の証明 /* アフィン射がアフィン局所的な性質であることの証明と類似している */ アフィン交通補題による。アフィン交通補題の2つの条件のうち、アフィン射の時のように、2つ目の貼り合わせ条件がより自明でない。 FをO_X加群としてSpecA⊂Xとする。SpecAがSpecA_fi(i=1..n)で被覆されていて、FのSpecA_fiへの制限が、A_fi-加群Miとなっていて、交わりでの同型φij:(Mi)_fj→(Mj)_fiがあって、コサイクル条件φik=φjk・φijを満たしていることを仮定する。このときにA-加群Mがあって、付随する層M~のD(fi)での切断M~_fiが与えられたMiに同型になるようなものの存在を示したい: 13.2.D なぜこれを示せば十分なのか?つまりなぜF|SpecAがM~だと言えるのか。 /* 2.5の視点:層は開被覆から復元される;つまり部分層F|SpecAiが与えられたものに一致するような層は一意的な同型を除いて一意的だから、M~|SpecAiが与えられたものに一致するならFはM~に同型でなければならない。 ・現実味を得るために考察した例: Spec k[x,y]を考える。D(x)上でM1=k[x,y,1/x]、D(y)上でM2=k[x,y,1/y], 交わりではM12=k[x,y,1/x,1/y]という加群を考えて制限写像は自然な埋め込みとする。 Spec k[x,y]上の加群でD(x),D(y)への制限がM1,M2になるようなものを考える。 k[x,y]という加群はこれを満たすが、(x,y)というイデアルな加群つまりx*k[x,y]∪y*k[x,y]もこれを満たし、同型でない。これはもちろん、D(x)とD(y)がSpec k[x,y]を被覆していないからである。原点(x,y)を含む開集合への制限も指定すれば、それに合う大域的な加群は一意的に決まる。 */ MはΓ(SpecA,F)であるべきだということを我々は知っている。そこで「直積M1×..×Mnの元で、貼り合わさる組」という集合を考えてこれをMとおく。すなわち: 0→M→M1×..×Mn→M12×..×M(n-1)n が完全列である(右側の矢印は"差分写像"である)。そうするとMはA-加群の射の核なのでA-加群である。あとは、Mi=M_fiを確認すれば良い(この確認はいろいろな方法があり読者は読み進める代わりに自分の方法を探しても良い)。 特にM1=M_f1を確認する。局所化は完全(1.6.F(a))だから 0→M_f1→(M1)_f1×..×(Mn)_f1→(M12)_f1×..×(M1n)_f1×(M23)_f1×..×(M(n-1)n)_f1 は完全である。f1はA_fで可逆だから(M1)_f1はM1自体である。(Mj)_fiは貼り合わせより(Mi)_fjと同型である。よってi,jが1でないとき(Mij)_f1は(M1)_fifjと同型で(ここでコサイクル条件を使った)、(M1i)_fiは(M1)_fiと同型だから、先の完全列は 0→M_f1→M1×..×(M1)_fn→(M1)_f2×..×(M1)_fn×(M1)_f2f3×..×(M1)_fn-1fn となる。仮定より、F|SpecA_f1はM1~と同型だから、Spec A_f1の被覆: Spec A_f1 = Spec A_f1∪SpecA_f1f2∪SpecA_f1f3∪..∪SpecA_fifn において、A_f1-加群としての完全列: 0→M1→M1×..×(M1)_fn→(M1)_f2×..×(M1)_fn×(M1)_f2f3×..×(M1)_fn-1fn を得る。右側の矢印は、先の写像と同一である。こうしてM_f1とM1は、共通する写像の核として表せたので同型であることが言えた。 /* Mi=M_fiに関する、訳者による、魔法の言葉を使わない考察: ・M = M1×..×Mn → M1 という射影pがある。 ・局所化の普遍性によりpは準同型 M→M_f1→M1 とM_f1を経由する。 ・M_f1→M1の逆写像qを作りたい。 ・任意のm1∈M1に対して、M1とM2がM12で貼り合わさるということは、m1と貼り合わさるM2_f1の元がある。それをm2/f1^r2とおく。より正確にはM1からM12への局所化写像をφ1, M2からM12への局所化写像をφ2とおくとφ1(m1*f1^r2)=φ2(m2) となるようなm2∈M2が存在する。 m2の取り方は一般に一意的でないが、f1で局所化した像φ2(m2)は一意的である。 M2以外についても同様に行い、被覆が有限個であることから、r2からrnの最大値をrとおくことで、m2*f1^(r-r2)をm2'等とおくと、(m1*f1^r,m2', ..,mn')∈(M1×..×Mn)が貼り合わさる組をなし、Mの元となる・・というにはまだ早かった!m1*f1^rとm2'が貼り合わさって、m1*f1^rと3'が貼り合わさるからと言って、m2'とm3'がM23=D(f2)∩D(f3)で貼り合わさるとはまだ言えない。コサイクル条件からは言えることは、m2'とm3'はM123=D(f1)∩D(f2)∩D(f3)では貼り合わさることである。これは、m2'とm3'にf1を何回か掛ければD(f2)∩D(f3)で貼り合わさることを意味する。またもや、これをすべての組で行って、現れるf1の指数の最大値をRとおいて、m2'*f1^Rをm2"等とおくと、今度こそ、(m1*f1^(r+R),m2", ..,mn")∈(M1×..×Mn)が貼り合わさる組をなし、Mの元となる。 q(m1)をπ(m1*f1^(r+R),m2", ..,mn")/f1^(r+R)∈M_f1と定める。πはMからM_f1への局所化準同型である。m2の取り方は一般には一意的でなかったがf1による局所化は一意的なのでそのπの像は一意的であるので、q(m1)はwell-definedであり、これが射影pから誘導される準同型M_f1→M1の逆写像となっていることを確認できる。 */ 13.2.E 最後に証明を完結させるために、 M→ M_fi→Mi→(Mi)_fj ↑ ↓ M→ M_fj→Mj→(Mj)_fi の右端を同型φijでつなぐと可換になることを示せ。 (両端の右矢印は局所化で、中央の右矢印は上記の議論で示した同型である) /* i=1,j=2で説明すると、定義に沿って追った(m1,m2,..,mn)∈M のそれぞれの行き先はちょうど上記の記号でφ1(m1)とφ2(m2)となり仮定により一致する。 */ --- 13.3 基本開アフィンを使った準連接層の特徴づけ 準連接層は局所的にとても特殊な形をしているので、準連接層を知るには、開アフィンでの切断を知り、「基本開アフィン」への制限の仕方を知っていれば十分である。これを「基本アフィン基」と名付けるものによって詳しく描写する。(これは通常の意味での開集合の基ではない) ここの議論は準連接層を特徴づける有用な視点であるが、読者によっては13.3.3に飛んでも良い。 「基本アフィン基」は、Xの開アフィンである。我々は既にこれが開集合の基をなすことを知っている。しかし一旦そのことは忘れよう。我々は特にSpec A_f→Spec Aの形をした基本開アフィンに興味がある。 13.3.1 スキームX上の「基本開アフィン基」とは、アフィン開集合と、基本開集合の埋め込みで構成された情報である。 言い換えると、我々はいくつかの開集合(アフィン開集合)と、その間のいくつかの射(基本開埋め込み)だけを覚えておくのである。 (熟練者へ:位相空間の開集合の埋め込みがなす圏でいうと部分圏を描写した) /* 例えばSpecA∩SpecB の形でアフィンでないような開集合を無視する */ /*ザリスキ位相よりも粗い「グロタンディーク位相」考えていると解釈できる。この視点は13.3.4で説明される */ 我々はこの「基本開アフィン基」を使って層を定義できる。我々はそれぞれの開アフィンでの切断を知っていて、それを基本開埋め込みへどう制限するかを知っている。 (読者は一致の公理と貼り合わせ公理について良く考える必要がある) 茎は、Xの開集合を渡る余極限であった。これを、基本開アフィン基を渡るときだけの場合と比較する。読者は、片方の元がもう片方の元と対応することを確認できるだろう(慎重に考察せよ) /* 任意の開集合Uに対して、SpecA⊂Uが存在することが根底にある。これは開アフィンが開集合の基をなすことよる事実である。*/ 13.3.A 練習 基本開アフィン基による層の切断は、切断の芽によって決まる。 /* 一貫性のある芽は切断を一意的に定めるのであった。ここで、芽に関して前の段落に書いてある事実が根拠として使われる。 */ 13.3.2 定理 (a) 基本開アフィン基での層F^bは、対応する通常の層Fを一意的に定める。 (b) 基本開アフィン基での層の射は、対応する通常の層の射を一意的に定める。 (c) 基本開アフィン基でのO_X加群は、対応する通常の層Fを一意的に定める。 証明は2.5と同様である。一貫性のある芽を使った構成を利用して示す。ここで繰り返す目的は、開集合がフィルター付き集合をなせば十分であるのを見るためである。 熟練者へ:(a)と(b)は、2つのグロタンディーク位相による層の圏が圏同値であることを言っている。 [証明] F^bを基本開アフィン基での層とする。我々は芽を定義できる。 F(U)を一貫性のある芽の集まりとして定義する: {各点の芽の集まりで、任意のpに対して開近傍U_p上の切断s(p)があって、s(p)の芽はU_p全体で与えられた芽の集まりと合致するようなもの} これは実際に層をなす。制限写像、一致の公理、貼り合わせ公理は本当に容易に確認される。 次に、Uが基本開アフィン基の元のときに、F(U)がF^b(U)に一致することを示す。これは2.4.Eによる。 13.3.B (b)を証明せよ(2.5.C) 13.3.C (c)を証明せよ(2.5.3) 13.3.3 準連接層の特徴づけ FをO_X-加群とする。Spec A_f ⊂ Spec A ⊂ X を基本開埋め込みとする。 φ:Γ(SpecA,F)→Γ(SpecA_f,F)を制限写像とする。 φはA加群からA_f加群への写像なので局所化の普遍性により、 Γ(SpecA,F) → Γ(SpecA,F)_f → Γ(SpecA_f,F) と分解する。 (1つ目の矢印は、A_f加群とみなすための ・{@_A} A_f というテンソル積である) 13.3.D とても重要な練習 Fが準連接層であることは、すべての基本開集合埋め込みSpecA_f→SpecAに対して、上記のΓ(SpecA,F)_f→Γ(SpecA_f,F)が同型であることと同値である。 /* 13.2.1を使うにはFをSpecAに制限した部分層が、A-加群Mに付随する層M~に同型であることを示せば良い。層の同型を言うには、開集合の基での切断の同型で、制限写像と可換なものの存在を示せば良かった(2.5の視点)。 f1,f2∈Aとする。Spec(A),D(f1),D(f2),D(f1)∩D(f2)上の切断をそれぞれM,M1,M2,M12とおく。Γ(SpecA,F)_f→Γ(SpecA_f,F)の同型を仮定しているので切断の同型はある。制限写像との可換性が自明でない(人によっては自明かもしれないけど・・): M = M ↓ ↓ M_f1→M1 ↓ ↓ M_f1f2→M12 を可換図式にするような2行目と3行目の同型(τ1,τ12と名付ける)がある。(左の列は局所化写像で、右の列は制限写像である。)同様に、 M = M ↓ ↓ M_f2→M2 ↓ ↓ M_f1f2→M12 の上の四角を同型にする2行目の同型τ2がある。このときに3行目の同型に先のτ12を使ったときにこの下の四角が可換図式になることを確認したい。 ・M_f2の元は、f2を何回か掛ければM→M_f2の像に居る。Mの元に持ち上げてしまえば、制限写像のなす四角形 M→M1 ↓ ↓ M2→M12 が可換図式であることから、2つの経路の行き先が同じになることが言える。ところが、目的の図式に現れる加群はM_f2あるいはM_f1f2と同型で、従ってf2を掛ける写像は自己同型写像である。従ってf2を何回か掛けた元の行き先が同じなら、それ自体の行き先が同じだと言える。 */ 13.3.E 重要な練習 (cf.qcqs補題7.3.5) Xをqcqsスキームとする。FをXの準連接層としてf∈Γ(X,O_X)を関数とする。 制限写像 Γ(X,F) → Γ(X_f,F) は局所化である。 すなわちΓ(X,F)_f → Γ(X_f,F) は同型である。 (ヒント:完全列 0 → Γ(X,F)→ +Γ(U_i,F) → +Γ(U_ijk,F) に次の関手 ・@_F(X) {F(X)_f} を適用せよ。ここでU_iはXの有限被覆、UijkはUi∩Ujの有限被覆。) /* Xが構造層の場合が、qcqs補題の内容であった。同様の議論で示されるものと思う。 (訳者の理解はまだ十分でない)*/ 13.3.F 重要な練習:13.3.Eの系 X上の準連接層Fのqcqs射:X→Yによる押し出しは準連接層である。 /* SpecBの逆像はqcqsなので13.3.Eを使える(2行目と3行目の間で使う): Γ(SpecB,π_*(F))_f = Γ(π^-1(SpecB),F)_π#(f) = Γ(π^-1(SpecB)_π#(f),F) = Γ(SpecB_f,π_*(F)) によりπ_*(F)が準連接層と言えた。 */ 13.3.G 良い練習:冪零元の層 Aを環、f∈Aとする。Nil(A_f)=Nil(A)_fであることを示し、任意のスキームの冪零元の準連接層を定義せよ。 これはイデアル層の例である。 /* 射の流れは Nil(A)_f←Nil(A)→A→A_f←Nil(A_f) となっている。 良く観察すればどちらも {a/f^n | a^m=0} と書ける。 a/fが冪零ならaが冪零という所が鍵だと思った。 */ 13.3.H 繰り返し使われる重要な練習 Xをqcqsスキーム、LをXの可逆層、sをLの大域切断、FをX上の準連接層とする。 Γ(X_s,F)は、F{@_O_X}L^n を s^nで割った商として解釈できる。 言い換えると、Γ(X_s,F)の元は、十分大きなsの冪を掛ければX全体に延長できる。 より正確には:+[n≧0] Γ(X,L^@n)が次数付き環であることに注意し、sをその1次の元と解釈する。さらに+[n≧0] Γ(X,F@L^@n)が次数付き加群であることに注意し、Γ(X,F@L^@n)_s_0 から Γ(X_s,F)への自然な写像を描写し、同型であることを示せ。 (ヒント:存在を示した後はアフィンの場合で同型を示すと良い) /* 可逆層とは階数1の局所自由層のことであった。 qc性よりXを有限個のアフィンSpecAiで被覆する。sのSpecAiへの制限をsi∈Aiとすると、X_s∩SpecAiはSpecAiの開集合D(si)上の切断であり、X_s上の切断uのD(si)上への制限をai/si^nとおける。すべてのAiでこれを行い、現れた分母に指数の最大値をNとすれば、u*s^nのそれぞれのD(si)への制限は、SpecAiに延長できる(延長の結果は一意的ではない)。 これが貼り合わさるようにできることはqcqs補題の訳者による考察と同様である: すべての組AiとAjに対してAi∩Ajを有限個のSpecBkで被覆すると、上記の方法によるD(si)からAiへの延長とD(sj)からAjへの延長は、sを何回か掛ければSpecBkで一致する。すべてのBkでこれを行い、再び現れたsを掛ける回数の最大値をsに掛ければ良い。 */ 13.3.I より重要でない練習 qc性を外すと仮定しない場合13.3.Hの主張は成り立たない例を挙げよ (ヒント:無限個のアフィンの直和を考えよ。無限個の直和は局所化と可換でないことが鍵である。) /* X=+Spec k[x], s=x, 第n成分が1/x^n とか */ 13.3.4(**) グロタンディーク位相 基本開アフィン基は、通常の意味で位相をなさない。例えば、2つのアフィンの和集合はアフィンとは限らない。しかしこれは位相の一般化:サイトあるいはグロタンディーク位相の最初の例である。好奇心のために定義を書いておくが、我々はこの概念は使わない。(より詳細は[Stacks]を見よ。) 考え方としては、位相の概念から、層を定義するのに必要な所だけを抜き出してくるのである。我々は開集合の概念は必要だが底空間は必要ない:つまり点すら必要ない。我々が必要とするものの少なさをよく考えよう。層を考えるためには、開集合が何で、包含写像が何であるかを知っている必要があり、それらが"うまく振舞う"必要がある:特に、開集合と包含による射は圏をなす必要がある。(例えば、包含と包含の合成は包含であるべきだ。基本開アフィン基ではこれは成り立つ:基本開集合の基本開集合は基本開集合であった。)すなわち我々はこの圏の情報を要求するのである。この時点で、我々は前層を定義できる:この"開集合"の圏からの反変関手である。この考えは2.2.Aの一般化である。 この定義を進めて層を定義するには、より多くの情報が必要となる。我々は2つの開集合の交わりが開集合であることを望む:すなわちこの"開集合"の圏がファイバー積で閉じていることを望む(開集合の交わりはファイバー積であった:1.3.O)。一致の公理と貼り合わせ公理を記述するためには、いくつかの開集合が別の開集合を被覆する条件を知る必要がある。そこで、この情報をグロタンディーク位相の情報として覚えておくのである。 被覆の情報はいくつかの明らかな性質を満たす:すべての開集合は自身を被覆する、被覆は引き戻される(射Y→XがあるならXの被覆を引き戻してYの被覆を得られる)、被覆の被覆は被覆である。このような公理を満たす"開集合"の圏と被覆の情報を、グロタンディーク位相あるいはサイトと呼ぶ([Stacks]にはいくつか異なる有用な定義もある)。グロタンディーク位相上の層を、通常の層と同じように定義することができる。トポスとはグロタンディーク位相上の集合の層を表す怖い用語である。 グロタンディーク位相は、代数幾何で、あるいは周辺の分野で広く用いられる。エタール位相を使うエタールコホモロジーはガロアコホモロジーの一般化であり、重要な道具である。より一般的な滑らかな位相、平坦な位相も同様である。Deligne-Mumfordあるいはアルティンスタックはそれぞれエタール位相と滑らかな位相を使う。テイトはまた別のグロタンディーク位相を使って、Qpのような全不連結な体での非アルキメデス解析幾何の理論を構成した。K-理論の仕事(例えばVoevodskyの仕事)はエキゾチック位相を使う。 --- 13.4 準連接層はアーベル圏をなす スキームX上の準連接層の間の射は、単に、O_X-加群としての射として定義される。 こうして、X上の準連接層の圏QCoh_Xが定義される。 (定義からこれはO_X-加群の圏の充満部分圏である。) 我々はこれからこの層がアーベル圏をなすことを示す。 Cがアーベル圏だと示すには、確認すべきことは多い。 しかしアーベル圏の部分圏だと分かれば、より簡単になる。・・具体的には: (1) 0はCに属する (2) Cは有限個の和で閉じている (3) Cは核と余核で閉じている を示せば十分である。 今回のQCoh⊂Mod_OXの場合では、最初の2つは易しい。 0は確かに準連接層であり、F,Gが準連接層ならSpecA上でF=M~,G=N~とおけて、F+Gは(M+N)~であるから、それは準連接層である。(3)を確認しよう。これには練習13.3.Dを使う。α:F→Gを準連接層の準同型とする。任意の開アフィンUで考えると加群の準同型β:M→Nを考えることになる。(kerα)(U)=kerβ,(cokerα)(U)=cokerβと定義する。これらは基本開集合への制限、すなわちある元を可逆にする局所化操作に対して都合良く振舞う: 0→K→M→N→P→0 が完全なら、 0→K_f→M_f→N_f→P_f→0 は完全である。このことからker,cokerは準連接層を定義する。さらに、茎を考えることによってこれらが確かに核や余核となっていることが確認できる。こうして、準連接層の圏は確かにアーベル圏をなす。 13.4.A スキームXの開アフィンSpec Aiによる被覆をとる。X上準連接層の準同型の列F→G→Hが完全であることは、すべてのSpec Ai上での切断の列が完全であることと同値である。 (特に開アフィンでの切断をとる関手は完全関手である。切断を取る関手は一般には左完全しか言えなかった(2.6.F)。)特に、準連接層の単射や全射はアフィン被覆によって確認できる。 (注意:左完全なものを見たとき、読者はやがてこれを長完全列の始まりだと解釈するだろう。すなわち0→F(U)→G(U)→H(U)には、H^1の列が続く。そこで我々はH^1(Spec A,F)=0であることを期待するわけで、それは実際正しいことを後で見るだろう) /* 前⇒後:加群の列M→N→Pがあって、付随する層の準同型M~→N~→P~が層の準同型として完全ならばM→N→Pが加群の列として完全であることを示すのが本質である。 im(M~→N~)はker(cokerM~→N~)で定義される。層のcokerは、前層のcokerの層化として定義されるのであった。前層のcokerは、基本開集合D(f)ではcoker[M_f→N_f]と定まる。そこでcoker[M→N]をLとおくと、coker[M_f→N_f]がL_fと自然に同型であった(1.6.F(a))。基本開集合D(f)上でL_fの形をしている前層は、加群L~に付随する層そのものである。 [注意:基本開アフィン基で層を決定する視点を使っている。基本開集合でない開集合Uでは、coker[M~(U)→N~(U)]とL~(U)が一致しないことがあり得る。これは、凸集合の部分集合は凸とは限らなく、コホモロジーが証明するとは限らないことの類似である。] ということは層のアフィンでの切断(im(M~→N~))(SpecA)は前層の切断im(M→N)と自然に同型で、同様に(ker(N~→P~))(SpecA)はker(N→P)に自然に同型だから、仮定から結論が従う。 後⇒前:茎を見れば良い。 */ 13.4.1 ベクトル束に馴染みがある読者への警告 ベクトル束の射は、局所自由な準連接層の射よりも、制限的である。局所自由層の局所自由部分層は、ベクトル束の部分ベクトル束をなすとは限らない。典型的な例は、Spec k[t]上の 0→tk[t]→k[t]→k→0 という完全列である。 tk[t]に付随する層はk[t]に付随する層の部分層であるが、部分ベクトル束に対応しない:余核はベクトル束でない(この例は13.1.9に現れたものである)。 13.4.B より重要でない練習:別の定義との同値性 O_X-加群がFが準連接層であることは、開被覆Uiがあって、それぞれのUiに制限した部分層F|Uiが、O_X^+I → O_X^+J → F|Ui → 0のように表現できることと同値である。これは多くの状況で有用である:例えば複素解析幾何など。 --- 13.5 加群様の構成 核や余核と同様に、加群に関する都合の良い構成は、準連接層に拡張される。(1つ例外がある。A-加群の間のHomはA-加群をなすが、準連接層の間のHOMは、良い状況でしか準連接層にならない(13.7.A)。これは一般にはHomが局所化と可換ではないことによる。) 13.5.1 自由加群による局所自由層 13.5.A (a) 0→F'→F→F"→0 をX上の準連接層の完全列とする。U=SpecAを開アフィンとする。F',F"がSpecAで自由だとする:F'|SpecA=A~^+a, F"|SpecA=A~^+bとする。FもSpecAで自由であり、完全列0→A^+a→A^+(a+b)→A^b→0が対応する。(F',F"が局所自由ならばFは局所自由であることが従う) (b) 階数が有限な場合、変換関数(行列)は、左上a×aブロックがF'の変換行列で、右下b×bブロックがF"の変換行列な、ブロック的上三角行列(つまり左下b行a列がすべて0)として解釈できる。 /* (a) 13.4.AよりΓ(SpecA,F)→Γ(SpecA,F")は全射である。Γ(SpecA,F")の生成元の全射による逆像を1つずつとってm1",..,mb"として、Γ(SpecA,F')の生成元の単射による像をm1',..,ma'とする。Γ(SpecA,F)はm1",..,mb",m1',..,ma'で生成される。 自由であること:Σai"*mi"+Σaj'*nj'=0 とおく。全射で0に移ることとΓ(SpecA,F")が自由であることからai"=0, Γ(SpecA,F')からの写像が単射であることからaj'=0が言える。 (b) U=SpecA,V=SpecBの変換を考える。FのUとVでの生成元をそれぞれm1",..,mb",m1',..,ma'とn1",..,nb",n1',..,na'とおいて、F',F"上で添え字で対応しているとおく。F',F,F"の変換関数をT',T,T"とおく。T(mi')はT(ni')と同一となる。nj"の全射による像はT"(mj"の全射による像)に移ることが要求されるから、T(mj")はmj"+(全射の核)の形をしている。 */ 13.5.B 0→F'→F→F"→0 をX上の準連接層の完全列とする。 (a) F,F"が階数有限な局所自由層ならばF'もそうである。 ヒント:せよ。F,F"の階数をm,nとしてF→F"をA成分のn行m列行列Mと解釈せよ。Xの点pに対して、Mのn個の列ベクトルが線形独立(n×nの行列式が消えない)近傍が存在することを示せ。・・・座標変換によって最初のn列が単位行列で残りが0になるようにできる。F'をA^+(m-n)と解釈せよ。 (b) F'とFが局所自由でもF"は局所自由とは限らない。 /* (a) ヒントに従ってX=SpecAでF,F"が自由な場合に帰着する。指摘:F'が自由とは限らない。 例えば A=Z^2として A^2→A を ((x,y),(z,w)) → (x,w) で定めると核は自由A-加群ではない。 (しかし"局所"ごとにみれば (x,z)→x と (y,w)→w でありそれぞれの核は自由である。) ヒントに従って地に足がついた考察をしていく。 ・pでの局所化も全射になる ・M_p成分の行列として、n個の列ベクトルで線形独立なものが存在する。 ・M_p成分の行列としてそのn×nの行列式は0でない ・もう少し考えるとその行列式は冪零でない -- F_pの選んだn個の列ベクトル成分を射影した商空間(A_p^nと同型)からF"_p(A_p^nと同型)への全射がある。 同型写像によって戻せば何回繰り返しても全射であることが分かる。 つまりそのn×nの行列式の冪乗の行列式は0でない -- ・そうすると局所化の性質によりその行列式をfとすれば、D(f)はpを含む開集合となる ・M_fではn×n行列は可逆となるから、n個の列ベクトルの線形結合により任意の列ベクトルを表せる。 ・基本変形により、残りの列ベクトルが0となるような基底変換ができる。 ・選んだn個の列ベクトルに相当する成分が0なA^mの元からなるA^(m-n)に同型な部分集合が核となる (b) 0→Z→Z→Z/2Z→0 とか */ 13.5.2 テンソル積 13.5.C F,Gが準連接層のとき、F@Gは次のように記述される準連接層となる:Γ(SpecA,F)=M, Γ(SpecA,G)=Nとおくと、Γ(SpecA,F@G)=M@Nであり、制限写像Γ(SpecA,F@G)→Γ(SpecA_f,F@G)は局所化M@N→(M@N)_f=M_f@N_fである。(後半の同型は読者が望むなら普遍性によって示せるし、明示的な構成を使っても示せるだろう。) /* (2.6.J) */ 13.3の基本開アフィン基という道具によって、層化がうまく扱われていることに注意せよ。これは我々が頻繁に使う特徴である。準連接層に関する構成では、一般の層では層化を必要とするものも、基本開アフィン基で考えると層化をしなくて良いのである。これが準連接層を開アフィン基で考えることが便利であることの理由である。 Fの切断s,Gの切断tからF@Gの切断s@tを得る。Fが可逆層のとき、これはしばしばstと表記される。 13.5.3 テンソル代数の構成 MをA-加群とする。テンソル代数T●(M)は次のようなZ≧0で次数付けられた可換とは限らない-代数である。T^0(M)=A, T^n(M)=M@..@M (n個)で乗法演算は読者が期待する通りである。 対称代数Sym●M 外積代数Λ●M ・・・ 13.5.D Fを準連接層とする。準連接層T^n F, Sym^n F, Λ^n Fを定義せよ。 (基本開アフィンで定義し13.3.Dを利用するとよい) Fが局所自由で階数mのとき、それらも局所自由であることを示し、それらの階数を求めよ。 特に Λ^m Fは det F と表記され、行列式束あるいは行列式局所自由層と呼ばれる。 /*・・階数はm^n, H(m,n), C(m,n)*/ 読者は非可換代数T●F、可換代数Sym●F、擬可換代数Λ●Fも構成できる。 13.5.E 0→F'→F→F"→0 を局所自由層の完全列とする。 Sym^r F=G^0⊃G1⊃...⊃G^r⊃G^r+1=0 というフィルター付けで、部分商がG^p/G^p+1=Sym^pF'@Sym^(r-p)F"となるものができる。 (この練習と次の練習のための2つの方針のヒント: (1) 変換行列を13.5.Aのブロック的上三角行列と解釈し、Sym^r Fの変換行列をr+1個のブロックを持つ変換行列と解釈せよ。 (2) 十分に小さい開アフィンSpecAをとればF',F,F"は自由である。具体的な構成がf∈Aによる局所化でうまく振る舞うことを示せばよい。完全列は0→A^+p→A^+(p+q)→A^+q→0と書ける。e1,..,epをA^+pの標準基底、f1,..fqをA^+qの標準基底として、e1',..ep'をA^+(p+q)への像、f1',..fq'をA^+(p+q)への任意な持ち上げとする。 Sym^r F|SpecA = +[i=0..r] Sym^i F'|SpecA @ Sym^(r-i) F"|SpecA であることに注意せよ。 G^p := +[i=p..r] Sym^i F'|SpecA @ Sym^(r-i) F"|SpecA はうまく定義された部分層であり、標準基底の取り方と持ち上げの取り方に依存しないことを示せ。 /**/ 13.5.F 0→F'→F→F"→0 を局所自由層の完全列とする。 Λ^r F=G^0⊃G1⊃...⊃G^r⊃G^r+1=0 というフィルター付けで、部分商がG^p/G^p+1=Λ^pF'@Λ^(r-p)F"となるものができる。特に階数有限の場合はdetF=detF'@detF"である。 /**/ 13.5.G Fを局所自由層で階数nとする。写像Λ^r F × Λ^(n-r) F → Λ^n Fで、同型Λ^r F→Λ^(n-r)F^[双対] @ Λ^n Fを誘導するものを描写せよ。 これはベクトル束の完全対合と呼ばれるものである。 読者はこれを後の21.5.Iで代数閉体上の正則な多様体でのホッジ数の双対性を示すのに使うかもしれない。 13.5.H 0→F1→..→Fn→0を階数有限の局所自由層の完全列とする。 行列式束の"交代積"は自明であることを示せ: detF1@detF2^[双対]@detF3@detF4^[双対]@...@detFn^(-1)^n=O_X (ヒント:短完全列に分解し、13.5.Bを使い階数有限の局所自由層の短完全列であることを示し、13.5.Fを利用せよ。) /**/ 13.5.4 重要:閉部分スキームに対応するイデアル層 X→Yが閉埋め込みのとき、O_Y→i_*O_Xという全射なY上の層準同型が存在するのであった(8.1)。(X上の層をY上の層とみなすことで、i_*はしばしば省略される。)その核I_[X/Y]はY上の"イデアル層"であった:任意の開集合Uで層の切断は、U上の関数環のイデアルをなすのであった。 難しい練習8.1.H(イデアル層がいつ閉部分スキームに由来するか)と、難しいかもしれなかった練習13.3.D(準連接層の特徴づけ)を比較せよ。イデアル層が閉部分スキームに由来することと、準連接層であることが同値であることが分かるだろう。(準連接層でない例は8.1.F, 13.2.Aにあった。) 0→I_[X/Y]→O_Y→i_*O_X→0 という完全列を、X→Yに対応する閉部分スキーム完全列と呼ぶ。 13.5.5 ねじれ自由層とねじれ層 A-加群Mは、am=0ならばaが零因子またはm=0が成り立つときねじれ自由と言う。 この概念を使うのはたいていAが整域のときであり、そのときこの条件はam=0ならばa=0またはm=0と書き直される。この場合、加群Mのねじれ部分加群M_torsは、Mの元でAの元によって零化されるもので構成される(Aが整域でない場合は、これはA-加群になるとは限らない)。Mがねじれ自由であることはM_tors=0と同値である。 一方、M=M_torsのとき、Mをねじれ加群と言う。これはM@K(A)=0と同値である。 Xがスキームのとき、すべてのp∈Xに対してF_pがねじれ自由加群のときFはねじれ自由と言う。(注意:[Gr-EGA]では"strictly torsion free"と呼んでいる。) 13.5.6 被約なスキーム上のねじれ準連接層 被約なスキーム上の準連接層がねじれ層であるとは、すべての既約成分の生成元での茎が0であることを言う。我々はこれを主に正則な曲線上の連接層に対して使う(13.7.G(b))。 ---- 13.6 有限型層と連接層 A-加群Mについて3つの性質がある。 [1] 有限生成:全射A^+p→M→0がある [2] 有限表現を持つ:完全列A^q→A^p→M→0がある [3] 13.6.1 連接である:有限生成かつ、全射と限らない任意のA^p→Mに対して核が有限生成 13.6.A Aが有限表現を持つならばAへの任意の全射φの核kerφは有限生成である ヒント:次の2つの完全列をどのように結び付けられるか見つけ、蛇の補題を利用せよ。 0→K→A^p→M→0 0→K'→A^p'→M→0 /* このKとは[A^q→A^p]の像であるから有限生成である。 A^pの生成元a1,..,apの行き先をm1,..,mpとする。下の行の全射による持ち上げをb1,..,bpととる。aiをbiに写すことで、右側の四角を可換にする縦の準同型を得る。KはA^pの部分加群とみなせるからこの準同型でA^p'の元に写る。それはK'の像に属し、従ってKからK'への準同型も得る。 蛇の補題よりcoker[K→K']とcoker[A^p→A^p']は同型で、A^p'が有限生成なのでこれらの余核は有限生成。Kとcoker[K→K']が有限生成だからK'が有限生成と言えた。 */ 13.6.2 Aがネーター環の場合には[1][2][3]は同等となる。 [証明] [1]⇒[3]が自明でない。任意のA^p→Mをとる。核はA^pの部分加群であるから、3.6.Wにより有限生成である。 13.6.3 連接なA-加群は、A-加群の圏の部分圏をなす(13.8で示される) [Aがネーターな場合の証明] 13.4の議論を思い出そう。我々は次の3つを確認する必要がある: (1) 0は連接である (2) 連接性は有限個の和で閉じている (3) 連接性は核と余核で閉じている 最初の2つは明らかである。(3)について、f:M→Nを有限生成加群の写像とする。fの余核は有限生成加群の全射による像なので有限生成であり、核も有限生成加群の部分加群なのでネーターな場合は有限生成である(3.6.X)。 13.6.B (*) 簡単な練習(ネーターでない人向け) A自身が連接A-加群であることは、A-加群が有限表現を持つことと連接であることが一致することと同値である。 /* A自身が連接で、完全列A^q→A^p→M→0があるとする。A^p'→Mを与えられたとする。 0→K'→A^p'→M→C 0→K→A^p→M→0 13.6.AのようにA^p'からA^pへの準同型が像の持ち上げによって定まり、K'→Kも誘導される。ker[K'→K]とker[A^p'→A^p]は同型となる。連接性が有限個の和で閉じていることを使えば、A自身が連接であることによりker[K'→K]=ker[A^p'→A^p]の有限生成性が言えた。 */ 次の2つの練習は、[1][2][3]がアフィン局所的な性質であることを示すものである: 13.6.C MがA-加群として[1][2][3]のどれかを満たす。f∈Aとする。A_f-加群としてのM_fもそれを満たす。 /* [1] 同じ生成元を使えば良い [2] A^q→A^p→M→0を仮定する。fによる局所化は完全性を保つのであった。 よってA-加群として A_f^q→A_f^p→M_f→0 は完全なので、A_f-加群としても完全である。 [3] α:A_f^p→M_f の核を考える。標準基底e1,..,epがm1,..mpに写るとする。m1,..,mpにfを何回か掛ければ局所化写像M→M_fの像に居るようにできる。必要な回数をNとしてf^N*e1,..,f^N*epを改めてe1,..,epとおく。β:A^p→M を、標準基底e1',..ep'をm1,..,mpに写すものとして定める。βの核Kは有限生成である。局所化は核と可換だから、αの核はK_fであり従って有限生成である。 */ 13.6.D 有限個のD(fi)がSpecAを被覆し、それぞれのM_fiがA_fi加群として[1][2][3]のどれかを満たす。A加群としてのMもそれを満たす。 /* 2つの開集合D(f),D(g)で被覆される場合を考えれば十分である。 [1] M_fの生成元をm1,m2,..mk、M_gの生成元をn1,n2,..,nj とおく。13.6.C[3]と同様の議論でmiが局所化写像の像にいるようにできる。仮定より任意のm∈Mはm=Σai'*mi(ai'∈A_f) と書ける。これは、あるNが存在してf^N*m=Σai*mi (ai∈A) と書けることを意味する。同様に g^N'*m=Σbi*ni (bi∈A) と書ける。「貼り合わせに関する重要なテクニック」により目的を達成する。 [2] [1]の方法で全射A^(p+p')→Mが定まる。核をKとおく。 局所化して全射 K_f→A_f^(p+p')→M_f を得て、13.6.AよりK_fは有限生成。同様に K_g→A_g^(p+p')→M_gの核も有限生成。[1]の結果よりKは有限生成。 [3] A^p→Mの核をKとおく。f,gでそれぞれ局所化すれば[2]と同様の議論で目的を達成する。 */ 13.6.4 定義 準連接層は、各アフィンで[1][2][3]を満たすとき、それぞれ有限型層、有限表現層、連接層と呼ぶ。連接層は常に有限型層であり、局所ネータースキームの場合はこれらの概念は一致する。13.6.3より連接層はアーベル圏をなすのであった。これはCoh_Xと表記される。 13.6.C,Dのおかげで[1][2][3]はアフィン交通補題によりアフィン局所的なので、特定のアフィン被覆でこの性質を確認することができる。有限階数の局所自由層は常に有限型層で、O_Xが連接ならば連接層である。 13.6.5 連接層の随伴点 ネーターなアフィンスキームの場合については5.5で議論した。 局所ネータースキーム上の随伴点とは、O_Xの随伴点のことを言う。 13.6.6 なぜ連接か? 命題13.6.3は連接層を考える良い動機である。これはベクトル束について考えることができる(専門用語でない意味で)小さな圏を与える。 ネーターな人には連接層と有限型層が同じものを意味するが、次のような人はこの定義について気にするべきである:1つは複素幾何学者である。複素解析空間を古典的な位相で考えると、連接O_X-加群の概念を定義することができる。そうすると岡の定理はC^n(従って任意の複素多様体)の構造層が連接層であることを主張するがこれはとても難しい。 もう1つは数論の人である。ネーターでない環を扱うときや、非アルキメデス解析幾何をする必要があるかもしれない。 警告:文献によっては連接層を有限生成として誤って定義しているものが稀ではない。これは混乱を来たすので正しい定義を使いなさい。これは誠実さという理由だけでなく、ものごとを証明するのにどのような仮定が実際に必要なのかを知るのに役に立ち、証明、すなわちものごとがなぜ正しいのかを覚えておくのに役に立つ。 --- 13.7 有限型層と連接層の喜ばしい性質たち まずはHOM(hom sheaf)から始めよう。これは始域が連接層の時にうまく振る舞う。 13.7.A (a) Fが連接層で、Gが準連接層のとき、HOM(F,G)は準連接層である [実際には連接層である必要はなく、有限表現層であることだけが必要となる] (b) さらにGが連接層でO_Xが連接層ならばHOM(F,G)は連接層である (c) Fが連接層、Gが準連接層のときHOM(F,・)はQCoh_XからQCoh_Xへの左完全関手であり、HOM(・,G)はCoh_XからQCoh_Xへの左完全反変関手である。 /* (a) 定義の復習:HOM(F,G)のUでの切断は、層準同型F|U→G|Uで定義されるのであり、これは一般にはF(U)→G(U)とは異なるが、Uがアフィンの時は全単射が存在し同一視できるのであった。Γ(SpecA,F)=M, Γ(SpecA,G)=N とおくと Γ(SpecA,HOM(F,G))はHom_A (M,N)ということになる。準連接層であることを言うには、Hom_A (M,N)_f → Hom_A_f (M_f,N_f)が同型であることを示すことになる。Mが有限表現A加群のとき、Hom_A (M,N)_f = Hom_A_f (M_f,N_f) が成り立つのであった(1.6.G)。 (b) 前のセクションの議論より開アフィンで確認すれば良いのだった。 A^p→Hom(M,N)の核を考える。 (c) 準連接層の完全性は開アフィンの切断で確認できるので、1.6.Fに帰着する。 */ 13.7.1 連接層の双対 F,O_Xが連接層なら双対F^[双対]:=HOM(F,O_X)も連接層である。これは13.1.Cのベクトル束での描写を一般化したものである。そこで、自然な準同型F→F^^ があったという議論は連接層についても成り立つ。しかしベクトル束のときと違ってこれは同型とは限らない。例えばFをSpeck[t]上のk[t]/(t)に付随する連接層とするとF^=0である。 F^^が自身に戻るものは反射的層と呼ばれるが我々はこの用語は使わない。標準的な写像F@F^→O_Xはトレース写像と呼ばれる。(なぜだか分かるだろうか?) 13.7.B Fが階数有限な局所自由層でGが準連接層ならばHom(F,G)=F^@G /* どちらもFの生成元をGのどの元に送るかによって記述できる */ 13.7.C 0→F→G→H→0 が準連接層の完全列でHが局所自由層とする。別の準連接層Iに対して 0→HOM(H,I)→HOM(G,I)→HOM(F,I)→0 は完全である。(右完全性が自明でない) /* アフィンで考える。F,G,H,Iを加群を表す記号として濫用する。 Hom(F,I)からHom(G,I)が誘導されて、F→Gとの合成が与えられたものに一致することを示したい。 Hの元の生成元をh1,h2,..とするとGの元は、g=f+Σak*hk [fはF→Gの像に居る元]と一意的に書ける。 Hom(G,I)を、この一意的な分解で得られるfの行き先として定めれば良い。*/ 13.7.D 練習:有限型準連接層の台は閉集合 /* 開アフィンSpecAへの制限で考える。F(SpecA)は有限生成A加群Mに伴う層である。有限生成だから生成元x1,x2,..,xnをとれる。 それぞれの位数; a1*x1=0 となる最小の自然数a1を(あれば)とる。Mの芽が消えるのはD(a1*a2*..*an)においてであり、台=その補集合は閉集合である。 */ 有限型とは限らないと反例がある:A=C[t], M=+[a∈C] C[t]/(t-a) この例は直和+を直積Πに置き換えると成立しないことに注意せよ。 /* 直和の場合は任意の元はAの元を何回か掛けると0になるので生成点での局所化が消える。 直積の場合は"通分"できないので生成点での局所化が消えない。 */ 13.7.E 幾何的中山 Fを有限型準連接層とするpを含む開集合Uでの切断a1,a2,..,an∈F(U)に対して、それらのpでの芽が「pでのファイバー」 = F_p@κ(p) を生成するならpを含む開アフィンSpecA⊂Uがあって、 ・a1,a2,...,anはF(SpecA)を生成する ・a1,a2,...,anの各q∈SpecAでの芽はqでのファイバーを生成する /* 中山の補題より F_pの生成元に持ちあがる。F_pの生成元は、必要に応じてpを何回か掛ければ(pは可逆だから生成元のままで居られる)局所化写像F(SpecA)→F_pの像に居るようにできる。 A→F(SpecA)→C→0 ↓ ↓ A_p^n→F_p を考える。局所化は完全性を保つからC_p=0となる、すなわちCの元は積閉集合S=A-pの元を掛けると0になる。 ここでF(SpecA)は有限生成なのでCも有限生成なので、それぞれの生成元ciを零化するSの元siの総積s=ΠsiをとればD(si)でciは全部消える。これによるD(si)=Spec A_siが問題文にあるSpecAとして適する。*/ 13.7.F 有用な練習 Fを有限生成を持つ層とする。F_pが自由ならばFはpのある近傍で自由である。従ってFが局所自由であることはすべての点での茎が自由であることと同値である。 /* K→A→F(SpecA)→C→0  ↓ ↓ A_p^n=F_p を考える。前の練習よりSpecAをうまくとればC=0にできるのであった。局所化は完全性を保つからK_p=0である。13.6.AによりKが有限生成であるから前の練習と同じ議論で解決する。 */ これはいくつもの視界を開く。有限表現を持つ層では、局所自由性は茎局所的である。さらに、整スキームでは、連接層は生成点で自由だから/*環の局所化が体だから*/、稠密な開集合で局所自由であることが分かる。また、既約スキームの生成点で0な連接層は稠密な開集合で0であることが分かる。これらは生成点の有用性が分かる例である。 13.7.G 練習 (a) 正則な曲線上の、ねじれ自由連接層は局所自由である (b) 正則で整な曲線上の、ねじれ連接層の台は有限個の点である (c) F /* (a) 前の練習よりF_pが自由であることを示せば良い。 ねじれ自由層の定義13.5.5よりF_pはねじれ自由である。 正則な曲線とは、 */ 13.7.4 準連接層のrank FをXの準連接層とする。Fの点p∈Xでのファイバーは、κ(p)上のベクトル空間となる。pを含む開集合での切断は 、点pでそのベクトル空間上に「値をとる」と言える。Fのpでのrankはこのベクトル空間の次元(無限かもしれな い)で定義される。より具体的にはpが属する開アフィンSpecAをとってpをAの素イデアルとみなしたとき、F (SpecA)=M とすると dim_K(A_p) M_p/pM_p である。この定義は、局所自由層のrankの定義と同じ結果を与える。 局所自由層では、rankは局所定数である。時に、逆が成り立つ(13.7.K) Xが既約で、Fが準連接層のとき、Fの(点pを指定しない)rankは、習慣的に、生成点でのrankを意味する。例えば 、整スキームのrank 0の準連接層は、ねじれ準連接層である(13.5.5) 13.7.H 練習 A^1_k = Spec k[t] 上の、加群k[t]/(t)に伴う連接層Fを考える 各点におけるFのrankを調べよ(生成点を忘れないように) /* この加群は、準連接層の例で登場した。加群M=k[t]/(t)とは実体としてはkであり、k[t]加群としてのスカラ ー倍を考えるときにtは0として作用することを意味する。 点(t)でのファイバーはkであり、他の点(生成点を含む)でのファイバーは0である。実際、開集合D(t)ではtは可 逆化されるのであらゆる元はm=tm/t=0となる。 従って、点(t)でrank 1, 他の点では rank 0 である。 */ 13.7.I 練習 任意の点でrank(F+G) = rank(F)+rank(G), rank(F@G) = rank(F)rank(G)を示せ。 /* 点pを含むSpecAをとる。F,GがそれぞれA加群M,Nに伴う準連接層だとおく。 [ M+N ]_p / p[ M+N ]_p = M_p/pM_p + N_p/pN_p [ M@N ]_p / p[ M@N ]_p = M_p/pM_p @ N_p/pN_p を示せば良い --- A-加群M,N,Lと素イデアルpに対して成り立つ種々の性質による ・M@(A/p) = M/pM ・[M+N]_p = M_p+N_p ・[M+N]@L = [M@L]+[N@L] ・[M@N]_p = M_p@N_p ・[M@N]@L = [M@L] @_L [N@L] http://www.math.s.chiba-u.ac.jp/~matsu/math/module.pdf --- */ もしFが有限型ならrankは有限で、中山の補題より、A_p-加群としてのM_pの生成元の個数と等しい。 13.7.J 重要な練習 FはX上の有限型準連接層とする。rankFは上半連続である。 --- 13.8 ・・・ 13.8.A A-加群Mが連接ならば有限表現を持ち、有限表現を持つならば有限生成 /* Mが連接なら定義より有限生成なので全射A^p→M→0があり、A^p→Mの核Kが有限生成なのでKへの全射A^q→K→ 0を使えばA^q→A^p→M→0を得る。A^q→A^p→M→0があれば当然全射A^p→M→0がある。*/ 13.8.B 0は連接である /* A^p→0の核はA^pなので有限生成。 */ 以下13.8.Lまでの一連の練習では0→M→N→P→0を完全列とする。 一連の練習で、M,N,Pのうち2つが連接ならもう1つも連接であることを示す。 13.8.1 ヒント 0→A^p→A^(p+q)→A^q→0  ↓ ↓ ↓ 0→M→N→P→0 と書いて蛇の補題を使うのがしばしば有用となるだろう。 13.8.C Nが有限生成ならばPは有限生成である(右完全性だけが必要) /* 同じ生成元を使えば良い */ 13.8.D M,Pが有限生成ならNは有限生成である(右完全性だけが必要) /* 全射A^p→M, A^q→Pをとれば A^p+A^q→Nを構成できる */ 13.8.E N,Pが有限生成でもMは有限生成とは限らない /* A=N=k[x1,x2,...], M=(x1,x2,...) */ 13.8.F Nが連接で、Mが有限生成ならばMは連接(左完全性だけが必要) /* A^p→Mの核を考える。M→Nは単射だから、A^p→M→Nと合成したものの核と等しいからNの連接性よりその核は 有限生成と言える。*/ 13.8.G N,Pが連接ならMは連接 /* 全射A^p→Nをとる。A^p→N→Pと合成したものの核は有限生成なので完全列A^q→A^p→P→0がある。 A^q→A^p→N→P→0と核の普遍性からA^q→M→N→Pという分解を得てMの有限生成性が言える。13.8.Fから結論が 言える */ 13.8.H Mが有限生成でNが連接ならばPは連接 /* A^q→Pの核を考える。全射A^p→M→0をとる。A^(p+q)→Nが誘導される。蛇の補題よりK1→K2→K3→0が完全で Nの連接性よりK2が有限生成なのでK3は有限生成である(13.8.C) */ 13.8.I M,Pが連接ならNは連接 /* 全射A^p→0, A^q→Pをとり、A^(p+q)→Nを誘導する。蛇の補題より K1→K2→K3→0 が完全でK1,K3が有限生成 ならばK2は有限生成(13.8.D)に帰着する */ 13.8.J 連接加群の有限個の直和は連接 /* 完全列 0→M→M+P→P→0 があるので13.8.Iより従う */ 13.8.K Mが有限生成でNが連接とする。任意のφ:M→Nに対してim(φ)は連接 /* M→im(φ)は全射だからim(φ)は有限生成。im(φ)→Nは単射でNが連接だから、13.8.Fの内容に帰着する。*/ 13.8.L 連接加群の写像の核と余核は連接 /* 核は13.8.Gによる(左完全性しか使っていない)。 余核は、13.8.Hによる(右完全性しか使っていない)。 あるいは 0→im[M→N]→N→coker[M→N]→0 と 13.8.K による。 */ この時点で、連接なA-加群の圏が、A-加群の圏の部分アーベル圏をなすことを検証できた。 13.8.M M,NをPの部分加群として、M,N,Pは連接とする。M+N[MとNで生成される部分加群]と、M∩Nも連接である。 (ヒント:M+N[直和]→Pを考えよ。) /* M+Nは有限生成で、M+N→Pは単射だから、13.8.FよりM+Nは連接。 これと完全列0→M∩N→M+N→M+N→0 と M+Nが連接であることによりM∩Nも連接 */ 13.8.N Aが連接ならば、有限表現を持つ加群Mは連接である /* 0→K→A^r→M から A^q→A^p→M→0 への縦の写像を誘導する ker[K→A^q] と ker[A^r→A^p] が同型でありA^pが連接だからこれらは連接。 従ってKも連接*/ 13.8.O Mが有限表現を持ち、Nが連接ならばHom(M,N)は連接 /* A^p→A^p→M→0 より N^p→N^p→Hom(M,N)→0 を得るのでHom(M,N)は連接 */ 13.8.P Mが有限表現を持ち、Nが連接ならばM@Nは連接 /* A^p→A^p→M→0 より N^p→N^p→M@N→0 を得るのでM@Nは連接 */ 13.8.Q A-加群Mが有限生成(あるいは有限表現を持つ、連接)ならばA_f-加群M_fもそうである (このセクションを自己完結したものにするための13.6.Cの繰り返し) 13.8.R (f1,..,fn)=AとしてM_fiがすべて有限生成ならMもそうである。 13.8.S (f1,..,fn)=AとしてM_fiがすべて連接ならMもそうである。 (13.6.D) =============== 14章 直線束:可逆層と因子 次に、直線束(可逆層)の考察や分類に便利で強力な方法を述べる。まず射影空間上の直線束O(n)という根本的な例から始める(14.1)。次にヴェイユ因子を紹介し、それを使ってPicXを様々な状況で決定する(14.2)。最後に、イデアル層が可逆層、特に効果的カルチエ因子となることを議論する(14.3)。中心的なテーマは、直線束は「余次元1の情報」と関係するということである。 /* O(n)はセールの捩り層Serre's twisting sheafと呼ばれる */ 14.1 射影空間上のいくつかの直線束 体k上の射影空間上の重要な可逆層の族を述べる。 準備として、まずPk^1=Proj k[x0,x1]上の可逆層O[P_k^1](1)から始める。添え字P_k^1はその層が住んでいる空間を示し、しばしば省略される。O(1)を、変換関数を使って述べよう。それは開アフィンU0=D(x0)=Spec k[x10]と、U1=D(x1)=Spec k[x01]では自明である。(x10等は4.4.9の記法を引き継いでいる。)すなわちU0での切断は、x10の多項式である。U0からU1の変換関数は、x01倍=1/x10倍であり、従ってU1からU0への変数変換はx10=1/x01倍である。 /* 例えばU0上の切断5x10と、U1上の切断5x01が、貼り合わされる。*/ 理解を確認するために、O(1)の大域切断を計算しよう。これは(4.4.6)でP^1の構造層の大域切断が定数に限ることを計算したのと同様である。少し考察すれば、O(1)の大域切断は1次関数であることが分かる。U0ではax10+b, U1ではa+bx01である。特にO(1)の大域切断の次元は2であり、構造層の大域切断の次元は1であったことから、両者は同型でない。自明でない直線束の初めての例である! 次に、O(n)を考える。これは変換関数がO(1)のn乗となったものである。変換関数の明示により、O(n)はn個のO(1)のテンソルである。O(m)@O(n)=O(m+n)もすぐに分かる。 14.1.A 重要な練習 dim Γ(P^1, O(n)) = n+1 (n≧0), 0 (n<0) である 14.1.1 例 はるか以前(2.6.J(a))、O_X加群のテンソル積をとるときには層化が必須だと注意した。 すなわちF,GがO_X-加群のとき、F(X)@G(X) = (F@G)(X)とは限らない。 我々はその例を手にした。X=P^1,F=O(1),G=O(-1)とする。Gは大域切断を持たない。 14.1.B m≠n ならば O(m)≠O(n)である。従って単射Z→Pic P^1:n->O(n)がある。 O(n)の大域切断を、n次の斉次多項式と同一視するのが有用である。例えば、O(n)の定義がアフィン被覆の選択に依らないことが分かるし、切断の多項式による記述がアフィン被覆の選択に依らないことも分かる(cf.4.5.12, 28.1.M)。この視点の有用性は次のような問いで明らかである: O(3)の切断x0^3-x0*x1^2の零点集合は何か? O(1)の元x0+x1とO2の元x0^2の積はO(3)のどの元か? O(-2)の有理切断x0^4(x1+x0)/x1^7はどこで何位の零と極を持つか? (零と極の位数は12.5.7で定義し、14.2でまた会うが、この時点でこの質問の答えは明らかなはずである) P_k^mでの可逆層O[P_k^m](n)を定義する。開アフィンUiではそれは自明であり、UiからUjへの変換関数は、xij^n倍である。 14.1.C 欠かせない練習(cf.8.2.K) dim_k Γ(P_k^m,O[P_k^m](n)) = C(m+n,m)である。 14.1.2 Γ(P_k^m,O[P_k^m](n))の元は自然にm+1変数のn次の多項式と同一視される。例えばx+y+2zはO[P^2](1)の元である。それは関数でないが、零点集合は意味を持つ。 また、固定されたmに対して、C(m+n,m)はnのm次多項式であることに注目せよ。後で、h^0(P^m,O(n)):=dim(P^m,O(n))と定義し、もっと後で高次コホモロジー群を定義し、オイラー特性数χ(P^m,O(n)):=Σ(-1)^i*h^i(P^m,O(n))を定義する。オイラー特性数はh^0より都合良く振舞うことを知るだろう。そしてこの多項式C(m+n,n)はオイラー特性数であることを後で(18.1.3)示していく。 14.1.3 これらがP_k^mの直線束のすべてである。 kを体とする。14.2.9でこれらのO(n)がP_k^mの可逆層のすべてであることを見る。m=1の場合を次の練習で扱う。(この方法は、すぐに他の方法に負けてしまう。) 14.1.D m=1の場合 P_k^1のすべての可逆層はO(n)の形をしていることを示せ。 ヒント:単項イデアル整域上の有限生成加群の分類(12.5.14)より、A_k^1上の可逆層はすべて自明である(13.2.Cの特殊な場合)。P_k^1の標準開集合の間の変換関数の決定に帰着せよ。 注意:一般の環AではP_A^1にはO(n)の形をしていない可逆層がある。読者はあとで例を考えることができるだろう。(SpecAが連結でなければ?SpecAが自明でない可逆層を持てば?) ・・・ 14.2 直線束とヴェイユ因子 14.3(*) 効果的カルチエ因子"="可逆イデアル層 ============ 15章 準連接層と射影A-スキーム 前半の2つの節は素直であるが、後半の2つの節は技巧的である。 15.1 次数付き環に対応する準連接層 射影A-スキーム上の準連接層を記述したい。 射影A-スキームとは、Z≧0-次数付き環S●で、S_0=Aで、S+がA上有限生成であることを言うのであった。 M●をZ-次数付き次数付きS●-加群とする。(・・・) 準連接層M~●を次のように定義する: (注意:M~●は次数付きなものではない) fを正の斉次元とする。 基本開集合D(f)={p:f(p)≠0}におけるM●(f)を、(((M●)_f)_0)~ で定義する 15.1.A M●~の斉次イデアルp⊂S●における茎がM●_pの0次成分と同型であることを示せ (これにより「一貫性のある芽」の言葉でM●~を定義することもできる) 次数付き加群の写像M●→N●があるとき、層の射M●~→N●~が誘導される。 明示的には、D(f)の上では ((M●)_f)_0→((N●)_f)_0 が誘導される。 このようにして、~は次数付きS●加群の圏から、ProjS●の準連接層の圏への関手である。 15.1.B 関手~は完全関手である 15.1.C M●とM'●が十分高い次数で一致するならM●~とM'●~は同型である。  従って関手~は圏同値を与えるわけではない この現象は前層と層の関係の類似である。 15.1.D S_0-加群の写像 M_0→Γ(ProjS●,M●~) を描写せよ。 15.1.1 例:S●の斉次イデアルはProjS●の閉部分スキームを与える 15.1.E 完全関手~を I●→S●→S●/I●に適用することで、  ProjS●/I●→ProjS●を得る ProjS●のすべての閉部分集合はこの方法で得られる。 このことは8.2.Cで見たし、15.4.Hで再び違う視点で見る。 ----- 15.2 射影A-スキーム上の可逆層(直線束) M●を次数付きS●加群とする。 次数付き加群 M(n)● を、M(n)_m = M_{n+m}で定義する。 すなわち Γ(D(f),M(n)●~) = ((M●)_f)_n である。 ・・・ ---- 15.3 大域的に生成された層、底点自由な直線束 この話題はより難しいが重要になるものである。 S●はA上の有限生成な次数付き環で、次数1で生成されているとする。 15.3.1 定理 任意のProjS●上の有限型準連接層Fは、 +[有限個]O(-n) → F → 0 の形で表現できる ・・ この定理により、例えば射影スキーム上のすべての連接層の大域切断が有限次元であることを示せる。(18.1.5) この事実は連接層の固有射による押し出しが再び連接であることに至る[グロタンディークの連接定理(18.9.1)] 証明に進む前に、古典的な重要な結果に言及する: 15.3.2 ヒルベルトのシチジー定理 ・・・ 15.3.3 大域的に生成された層 Xをスキーム、FをO-加群とする。この章で最も重要な定義: Fが大域的に生成された層、あるいは大域切断により生成された層であるとは、 X上の自由層からの全射 +O^I → F が存在することを言う。 |I|個の直和因子の1に対応する切断が、大域切断という用語に対応する: それらの像が、Fの茎を生成する。 Iが有限集合にとれるとき、Fは有限に大域的に生成された層であると呼ぶ。 追加の定義: 点pにおいてFが大域的に生成されるとは、 pの茎において +O^I_p → F_p が全射であるようなものが存在することを言う。 鍵となる洞察はpにおいて大域的な生成されているということは、 pでのすべての芽が、大域切断の芽の線形結合で表されるということである。 Fが大域的に生成されることは、すべての点pにおいて大域的に生成されていることと同値であることに注意する。(2.4.E) 15.3.A (現実味の確認) 任意のアフィンスキーム上の任意の準連接層は大域的に生成される。 任意のアフィンスキーム上の有限型準連接層は有限に大域的に生成される。 ・・ 15.3.C 易しいが重要な練習 FはX上の有限型準連接層とする。 (a) Fが点pで大域的に生成されることは、 pにおいてFのファイバーが大域切断で生成される、 すなわち大域切断たちから F_p/m*f_pへの写像が全射であることと同値である (ヒント:幾何的中山:13.7.E) (b) Fが点pで大域的に生成されるならば、Fはpの近傍で大域的に生成される。 (c) さらにXが準コンパクトとする。  FがXのすべての閉点で大域的に生成されているなら、Fは大域的に生成されている。 (空でない準コンパクトスキームは閉点を持つ[5.1.E]。従って閉点は稠密である。) 15.3.D FはX上の有限型準連接層としてXは準コンパクトとする。 Fが大域的に生成されるならば、有限に大域的に生成される。 15.3.E X上の可逆層Lが大域的に生成されることは、 任意のp∈Xに対してpで消えないLの大域切断が存在することと同値である。 (このことは定理16.4.1で重要になる) 15.3.4 定義 LをX上の可逆層とする。 Lの大域切断がすべて消える点をLの底点と呼ぶ。これはXの閉集合となる。 (スキーム論的に考えて、底点を閉部分スキームとして定義することもできる。) Lが底点を持たないとき、Lを底点自由と呼ぶ。 後で18.2.Iで扱うことには、Xがkスキームのとき、Lが底点自由であることは、 K/kを体の拡大として、Kへの底変換で得られる層が底点自由であることと同値である。 15.3.5 「底点自由な直線束と、射影空間への写像」 6.3.M(a)を思い出す:X上のn+1個の関数は共通零点を持たなければP^nへの写像を与える。 これを一般化することができる: 15.3.F s0,..,snを可逆層Lの(n+1)個の大域切断で共通零点を持たないものとする XからP^nへの写像が定義できる。 後で定理16.4.1で、射影空間へのすべての写像がこの形で書けることを見る。 ・・・ 15.3.6 定義 kスキームX上の「線形系」とは、 kベクトル空間V(通常有限次元)、可逆層L、線形写像λ:V→Γ(X,L)の組である。 しばしば単にVで呼び、残りの情報を省略する。 λが同型であるとき、「完全線形系」であると呼び、しばしば|L|と表記される。 そうすると先の定義は、Vに関するものとして延長される: Vのすべての元が消える点を、Vの底点と呼ぶ。 Vが底点を持たない時、Vを底点自由と呼ぶ。 15.3.8 セールの定理A S●を次数1で生成され、A=S0上有限生成とする。 FをProjS●上の任意な有限型準連接層とする。 あるn_0が存在し、n≧n_0 ならば、F(n)は有限に大域的に生成される。 ---- 15.4 準連接層と次数付き加群 ============ 16章 準連接層の押し出しと引き戻し 16.1 導入 この章は準連接層の押し出しと引き戻し、それらの性質といくつかの応用を扱う。 B→Aを環準同型とする。練習1.5.Eのように、Bによるテンソル積は、MをB加群M_Bとして見なす忘却関手と随伴の関係にある。 Hom_A(N {@_B} A, M) = Hom_B(N,M_B) この構成は、(適切な意味で)局所化と都合良く振舞い、スキーム上の準連接層の圏で(しばしば)うまくいく。(実は環つき空間上のO-加群の圏で常にうまくいく。16.3.5)この簡単なほうの構成M→M_Bは我々の旧友の押し出しに相当することが判明する。もう一方は相対的な引き戻しで、これは旧友とは呼びたくない。/*ファイバー積としてよく現れた*/ 16.2 準連接層の押し出し 押し出しから始めよう。この章の精神は、「理にかなった」状況では準連接層の押し出しは準連接層であり、これは加群の構成から理解されるというものである。まず典型的な例から始めよう。 16.2.A π:SpecA→SpecBをアフィンスキームの射として、MをA-加群とする。M~はSpecA上の準連接層である。同型 π_*(M~)→M_B~を与えよ。 (ヒント:方針は1つしかない。基本開集合を見よ。) /* 環準同型π#:B→Aによって、A加群MはB加群とみなせ、M_Bはこれによって定義される。 f∈Bとおく Bの開集合 D(f)でのM_Bの切断は M_B のfによる局所化 Bの開集合 D(f)でのπ_*(M~)の切断は Mのπ(f)による局所化 どちらも集合としては で書けて、 B加群としての構造は、π(b)∈Aのスカラー積定まる。 */ このことから、準連接層のアフィン射による押し出しは準連接層である。 16.2.B π:X→Yがアフィン射ならば、π*はXの準連接層からYの準連接層への完全関手である。 /* 集合としては一緒だから単射も全射も保たれるんだと思う */ 次の事実は、以前に証明した事実であるが、上記のさらなる一般化である。 16.2.1 定理 (練習13.3.F) π:X→YがqcqsでFがXの準連接層ならば、押し出しはYの準連接層である。 連接層の押し出しは、連接層とは限らない。 例えば、構造射:A_k^1→Spec (k→k[t]に対応する)を考える。 π*O(A_k^1)はk[t]という加群であるが、これはk加群として有限生成でない。 しかし良い状況に限れば、連接層の押し出しは連接層となる: 16.2.C π:X→Yが局所ネータースキームの有限射ならば、 Xの連接層の押し出しはYの連接層である。 (ヒント:まず構造層の押し出しが有限型であることを示すと良い) /* AがB加群として有限で、MがA加群として有限生成ならば、MはB加群としても有限生成 ということだと思う */ のちに準連接層のコホモロジーを定義した際に示すことには: FがP_k^n上の連接層ならば、Fの大域切断は有限次元のk加群であり、 より一般に、FがProjS●上の連接層ならばS_0=Aとして大域切断は連接なA加群である。 これは「射影射による連接層の押し出しは連接層である」という事実の特殊な場合である。(18.8.1) もっと一般的には、ネーター性の仮定のもとで、連接層の固有射による押し出しは連接層である。(グロタンディークの連接定理18.9.1) ---- 16.3 準連接層の引き戻し π:X→Yに対して、引き戻しπ^*はY上の準連接層の圏からX上の準連接層の圏への共変関手である。準連接層の引き戻しは、初見では、(そして二度目ぐらいでは、)混乱しやすいものである。(それは層の逆像と関係あるけれど、異なる。) 理解に役に立つ3つの文脈がある。 GをY上の準連接層とする。 (i) 開集合への制限  開埋め込みi:U→Yによる引き戻しは、GのUへの制限である。(2.2.8) (ii) 点への制限  閉埋め込みi:p→Yによる引き戻しは、Gのpでのファイバーである。(4.3.7) /* G_p と 剰余体κ(p) を OX_p-加群としてテンソルしたもの*/ (iii) ベクトル束の引き戻し  GはY上局所自由とする。U_iをGの自明化、T_ijをGの変換行列とすると  π^-1 U_i は π^*(g)の自明化であり、π^*(T_ij)はπ^*(g)の変換行列である。 (これは16.3.7(3)で示される) 戦略 引き戻しに関する3つの視点を見る。それぞれ欠点があるが理解に役に立つ。 (a) 準連接層をアフィン開集合の対応する環の上での加群として理解していることから、まずはこの文脈での解釈から始める。これは事実を証明したり理解するのに実用的に役に立つ。欠点は、特に終域がアフィンでない時に、貼り合わせの議論が大変で、定義を書き下すのが煩雑であるということである。 (b) ファイバー積でみたように、貼り合わせの定義は、普遍性を使うと簡潔に議論できる。これが2つ目の視点である。これは上品であるが、構成を要する欠点がある:これはO-加群というより大きい圏で考えるので、準連接層を準連接層に写すことは明らかではない。 しかし終域がアフィンな場合は、(a)の構成が普遍性を満たすことは容易に分かる。さらに、普遍性は終域上アフィン局所的な性質である: π:X→Yをスキームの射、i:U→Yを開埋め込み、GをYの準連接層とする。 gの引き戻しが存在するならば、そのπ^-1(U)への制限は、GをUに制限したものを引き戻したものと標準的に同一視できる。こうして、終域がアフィンの場合に帰着するので、引き戻しは一般的に存在し、準連接層である。 (c) 3つ目の視点は一般の環つき空間を使うものである。 これは短く、普遍性を満たすことがすぐに分かる。 (少なくとも我々の定義では)準連接層を準連接層に写すことはすぐには分からない:定義からは準連接層をO-加群に写すことしか分からない。しかし(b)の議論の後半に書いた(a)を使う方法で、引き戻しが実際に準連接層であることが分かる。 16.3.2 (a)の視点 π:X→YとY上の準連接層Gを考える。 π(SpecA)⊂SpecBをとる。G|_SpecB = N~ とおく。 引き戻しはテンソル積であることが期待される: すなわち引き戻しのSpecAでの大域切断は、N {@_B} A であるべきだ。より正確には、それらは同一視されるべきだ。これを使って引き戻しを定義することができる。(それがwell-definedであることを示す必要がある。) あるいは、別の方法で引き戻しを定義して、上記の同一視を得る方法を見つけるのでも良いだろう。 16.3.3 ・・ SpecB⊂Yの逆像における引き戻しをテンソル積で定義する場合、 SpecBの取り方を変えた場合にちゃんと貼り合わさるかどうかが自明でない。 16.3.4 普遍性の視点による引き戻し π:X→Yを、「環つき空間の射」とする。 (準連接とは限らない)O-加群としての引き戻しを、随伴性によって"定義"する: Hom_OX(π^*(G),F) ←→ Hom_OY(G,π_*(F)) 普遍性により、π^*(g)は同型を除いて一意的に決まる。 我々はそれが存在することを確認すれば良い。 (この作業が必要という意図で定義という言葉に引用符をつけた) 押し出しのときにはより大きな圏で考えることでこの問題を解決したことを指摘しておく。 16.3.A テンソル積で構成した引き戻しが、この普遍性を満たすことを確認せよ。 (ヒント: 環上の加群のバージョンでの随伴性が現れる) /* 環準同型B→Aがあり、MをA加群、NをB加群とする。 A加群としての Hom(N{@_B} A, M) と B加群としての Hom(N,M)が対応する。 左から右 nの行き先を n@1の行き先で決める 右から左 n@aの行き先をnの行き先のa倍で決める これは「A加群をB加群として見なす忘却関手」とテンソル積の随伴性である。*/ 16.3.B i:U→Xを開埋め込みとして、FをO_X加群とする。 F|Uは上記の普遍性を満たすことを確認せよ。 すなわち Hom_OU(F|U,G) ←→Hom_OX(F,i_*G) の間の全単射を示せ。 /* Uの外でi_*Gは茎を持たないからFの茎は0に移るしかない */ 次に、π:X→Yに対するπ^*(G)が普遍性を満たすときに、 j:V→Yを任意のYの開部分スキームとして πによる逆像となるXの開部分スキーム i:U=π^-1(V)→Xを考える。 π^*(G)|U は U→Vに関して普遍性を満たす。 ・・ (これらを利用して引き戻しが存在することの議論) ・・ 16.3.5 3つ目の定義: π^*(G) = π^-1(G) @_π^-1(O_Y) O_X で定義する。 ・・・ 16.3.7 定理 π:X→Yがpをqに送るとする。 (1) 構造層の引き戻しは構造層である (2) 有限型準連接層の引き戻しは有限型準連接層である (3) 局所自由層の引き戻しは局所自由層で、階数や変換関数は自然に対応する (4) 引き戻しの引き戻しは、合成の引き戻し (5) π*はQCoh_YからQCoh_Xへの関手である (6) 切断の引き戻し  s:G←O_Y に対して π^*(s):π^*(G)←O_X が存在する。  sが消える点の引き戻しは、π^*(s)が消える点である。 (7) 茎の引き戻し  π^*(G) のpでの茎は、Gのqでの茎とOX_pのOY_q加群としてのテンソル積 (8) 準連接層のファイバーの引き戻し:  π^*(G) のpでのファイバーは、Gのqでのファイバーとκ(p)のκ(q)加群としてのテンソル積 (9) 引き戻しはテンソル積を保つ (10) 引き戻しは右完全関手である 16.3.E 上記の定理16.3.7を証明せよ /*・・*/ 16.3.F 引き戻しは左完全とは限らない例 X=A^1=Spec k[t] として、i:P→Xを原点の埋め込みとする。 閉部分スキームの完全列 0→O_X(-P)→O_X→i_*O|p→0 に対して iによる引き戻しを考えよ /*これは加群としては 0→tk[t]→k[t]→k[t]/(t)→0 である。 k[t]_t 加群として k[t]/t でテンソルするのである。 (私は「その加群の他の元のt倍」で表せる元を消去する、と考えた。) テンソル積は右完全であるが左完全を保つとは限らないことに帰着する できあがる加群の列は tk→k→k→0 で写像はそれぞれ零写像、恒等写像である。 この左の項は、conormal sheafとして21章でまた現れる。 また、これが完全でないことは、ホモロジー代数的に考えると、 Tor(k[t]/(t),k[t]/(t))が自明でないことと結び付けられる。 */ 16.3.G 押し引き公式 W→X ↓↓ Z→Y の可換図式があって縦の矢印がqcqsとする。 FをX上の準連接層とする。 X→Y←Zの順に押し出し、引き戻したZ上の層 F1から X←W→Zの順に引き戻し、押し出したZ上の層 F2への自然な準同型がある。(逆はない) /* 全部アフィンだったら(アルファベット順にSpecA,B,C,Dとした。) C {@_B} M から D {@_A} M への C加群としての自然な準同型 大雑把な茎の描写を試みた: F1の点zにおける茎は、像となるy∈Yでの茎={xiの茎| xiの像がy}の直和みたいなもの F2の点zにおける茎は、逆像となるwiの茎の直和みたいなもの wiはxiのどれかに移る。異なるwi,wjが同じxkに移ることもある。 F1の茎を与えられた時には、wi,wjの芽を共にxkでの芽と定めることができるが、 F2の茎を与えられて、wi,wjでの芽が異なる時に、xkでの芽を自然に定めることができない */ 16.3.H 射影公式 π:X→Yがqcqsとする。F,GをそれぞれX,Y上の準連接層とする。 (a) 自然な準同型 π_*(F) @ G → π_*( F @ π^*(G) ) がある。 (b) Gが局所自由なら(a)は同型である (c) πがアフィン射なら、(a)は同型である。 /* π(SpecA)⊂SpecBとして、A-加群M、B-加群NがF,Gに対応するとする。 M {@_B} N → M {@_A} (A {@_B} N) というB加群としての準同型である。 (b)もしN=B^kならこれは M^k → M^k である (c)分からない */ /* */ 16.4 直線束と射影スキームへの写像 16.5 曲線から射影の拡張定理/**/ 16.6 豊富な直線束、非常に豊富な直線束 16.7(*) モジュライ空間としてのグラスマン多様体 ================ 17章 相対的Spec、Proj、射影射 この章では、・・ 17.1 相対的Spec A-代数Bを与えられたとき、A-スキーム SpecB→SpecA を定義できた。 これから普遍性を使って、この大域化を描写する。 スキームXと、-代数の準連接層B'を考える。 この-代数のSpecをとって、X上アフィンなスキームSpecB'→Xをどうやって定めるか説明する。 2つの考え方ができる。 17.1.1 任意のSpecA⊂Xに対して、Γ(SpecA,B')はA-代数である。これを張り合わせる。 17.1.2 普遍性を使う。 SpecB→SpecAの状況では「A-スキームWからSpecBへの射は、A-代数の準同型 B→Γ(W,OW)に対応する」という事実を使ってSpecB→SpecAを定義できた。(我々の旧友6.3.Fである) 任意の射μ:W→Xに対して、W→SpecB'は、Wについて関手的で、 環つき空間としての射O_X→μ_*O_Wが合成 O_X→B'→μ_*O_W に一致する。 O_X→B'はB'のO_X-代数としての構造に由来する。 「β;Spec B'→X は 関手 μ(W→X)--->B'→μ_*O_W を表現する」 /**/ 17.1.A X=SpecAでB'がA-代数のときSpecB→SpecAはこの普遍性をみたす(6.3.F) 17.1.3 --- 17.2 相対的Proj 17.3 射影射 --- 17.4 曲線への応用 17.4.1 定理:すべての整な曲線は、正則で射影的な双有理モデルを持つ /**/ ==================== 18章 準連接層のチェックコホモロジー この話題は驚くほど単純でうまくできている。コホモロジーがやっと登場したので、読者はコホモロジーはややこしいものに違いないと思ったかもしれない。しかし実際にはほとんど背景知識が必要ない。スキームを定義してすぐに準連接層を定義してコホモロジーを定義することも可能であった。 18.1 望まれる性質 すぐに定義する前に、準連接層の持つべき性質を議論する。 大域切断関手は左完全関手であったので、 層の短完全列 0→F→G→H→0 に対して、0→F(X)→G(X)→H(X)は完全である。 0→F(X)→G(X)→H(X) →H^1(X,F)→H^1(X,G)→H^1(X,H)→ と延長して長完全列を作るということを期待する・・・ (実は23章で扱う導来関手によってこれは機械的に可能である) X,Yは分離的で準コンパクトなA-スキームとする。H^i(X,F)はA加群となる。特にAが体kのときはk-ベクトル空間となる。その場合にはH^i(X,F)の次元をh^i(X,F)と表記する。 (i)それぞれのH^iはX上の準連接層の圏からA加群の圏への共変関手である (ii) H^0(X,F)=Γ(X,F) である。 (iii) 上記のように短完全列から長完全列を作ることができる。連結準同型H^i(X,H)→H^(i+1)(X,F)は後で定義される。 (iv) 準コンパクトで分離なAスキームの射π:X→YとX上の準連接層Fに対して、H^i(Y,π_*F)→H^i(X,F)が存在する。 (キャンセル定理10.1.19によりπは準コンパクトで分離的な射であり、13.3.Fにより押し出しπ_*Fは確かに準連接層である。) (v) アフィン射π:X→YとX上の準連接層Fに対して(iv)の写像は同型である。 (vi) Xがn個のアフィン集合で被覆されるならi≧nに対して常にH^i(X,F)=0  特にX自体がアフィンスキームならi>0に対してコホモロジーはすべて消える。  これは(iii)によりアフィンスキームではΓは完全関手であることと結びつく(13.4.A)  また、dimX=nならば、i>nに対してH^i(X,F)が消えることも言える。(次元に関する消滅) (18.2.6, 22.4.T, 18.2.7) (vii) 関手H^iは直和に関して都合良く振舞う:余極限を保存する (viii) X=P_A^n 上のO(m)のコホモロジーについて次のように決定される: ・m≧0 ならば H^0(X,O(m)) は rankが C(n+m,m)の自由A加群 ・m≦-n-1 ならば H^n(X,O(m)) は rankが C(-m-1,-n-m-1)の自由A加群 ・それ以外の場合 H^i(X,O(m)) = 0 (viii)の最初の場合は14.1.Cですでに示した。 この事実は、後で証明する事実で今初めて出会うものを多く含意している: ・これらの直線束のコホモロジーは確かにnより大きい次数では消えている ・これらのコホモロジー群は常に有限生成A加群である  これは射影Aスキーム上の連接層では常に成り立ち(18.1.4)  実は固有Aスキーム上でも成り立つ(グロタンディークの連接定理18.9.1) ・m>-n-1のとき最高次コホモロジー群が消滅する(小平消滅21.5.7) ・m=-n-1のとき最高次コホモロジー群が1次元となる(双対層) ・セールの双対 H^i(P^n,O(m))×H^{n-i}(P^n,O(-n-1-m)) → H^n(P^n,O(-n-1)) ・交代和Σ(-1)^i*h^i(P^n,O(m))はmの多項式である(ヒルベルト多項式18.6.1) ・・・ n=1, m=-2 18.1.1 指摘: (vi)の逆に相当するのが、セールのアフィン性に関するコホモロジーによる特徴づけである: Xがqcqsなスキームで、任意のi>0と任意の準連接層Fに対してH^i(X,F)=0ならば、Xはアフィンである。 我々はこの事実を使わない。従って証明も与えない。 18.3 射影空間の直線束のコホモロジー 18.3.A H^i(A^2-{(0,0)},O)を計算せよ =================== 22.2.1 Xが空集合ならばBl_X(Y)=Y  Xが効果的なカルチエ因子ならばブローアップは同型 22.2.A 開集合でのブローアップは、ブローアップの開集合への制限 ブローアップの局所での様子は局所でのブローアップとして計算できる 22.2.B YkがYの開被覆で、X→X∩Ykのブローアップが存在するならば、X→Yのブローアップが存在する 22.2.2 ブローアップは中心以外では同型 22.2.3 X=YならばBl_X(Y)は空集合 22.2.C Yが既約でXが生成点を含まなければ、ブローアップは既約  Yが被約ならばブローアップも被約 22.2.4 X→Yが局所的に1つの方程式で切り取られるならばブローアップは存在する  22.2.BよりSpec A/(t) → Spec A の形を考えれば良い I = ker(A→A_t) とおく。 Spec A/I→SpecA がブローアップとなる 22.2.D 22.2.E 22.2.F Spec A/I は D(t)のスキーム論的閉包である ネーターな場合は、V(t)の上にある随伴点を取り除くと解釈できる。 この表現はわざと曖昧にしている。どうやったら正確になるかは、読者が望まなければ考えなくても良い。 22.2.5 ブローアップ閉包補題 ファイバー図式 W→Z ↓↓ X→Y でX→Yが有限型イデアル層に対応する閉埋め込みを考える(従ってW→Zもそうである) 最初に読むときはZ→Yも閉埋め込みな特殊な場合だけを考えると良いかもしれない。 ・・・ ==================== 29.5 ザリスキの連結補題とシュタイン分解 ここでは28.1.8で言及したザリスキの連結補題を述べて証明する。 29.5.1 ザリスキの連結補題 局所ネータースキームの間の固有射π:X→YがO-連結(28.1.7)ならば、すべてのq∈Yに対してπ^-1(q)は連結である。